中考数学天利38套难题集及原题和答案

申明敬告: 本站不保证该用户上传的文档完整性,不预览、不比对内容而直接下载产生的反悔问题本站不予受理。

文档介绍

中考数学天利38套难题集及原题和答案

2012 年暑假,我买了一本《天利 38 套 –中考数学》,全做完了,又到网上把每一套附带 答案的试卷下载下来,为的是我在今后教学中好用。时间虽然进去了,但理科这东西思维 方法和题型永不过时。现将每套的难题集中采集如下,并把每套的试题及答案附后,便于 查阅。现献给慧眼有缘人。 2012 北京 8. 小翔在如图 1 所示的场地上匀速跑步,他从点 A 出发,沿箭头所示方向经过点 B 跑到 点 C ,共用时 30 秒.他的教练选择了一个固定的位置观察小翔的跑步过程.设小翔跑步 的时间为 t (单位:秒),他与教练的距离为 y (单位:米),表示 y 与t 的函数关系的图 象大致如图 2 所示,则这个固定位置可能是图 1 中的 A.点 M B.点 N C.点 P D.点 Q 12.在平面直角坐标系 xOy 中,我们把横 、纵坐标都 是整数的点叫做整点.已知点  0 4A , ,点 B 是 x 轴正 半轴上的整点,记 AOB△ 内部(不包括边界)的整点 个数为 m .当 3m  时,点 B 的横坐标的所有可能值 是 ;当点 B 的横坐标为 4n ( n 为正整数)时, m  (用含 n 的代数式表示.) 19.如图,在四边形 ABCD 中,对角线 AC BD, 交于点 E , 90 45 30 2BAC CED DCE DE         , , , , 2 2BE  . 求 CD 的长和四边形 ABCD 的面积 20.已知:如图, AB 是 O⊙ 的直径, C 是 O⊙ 上一点, OD BC⊥ 于 点 D ,过点 C 作 O⊙ 的切线,交OD 的延长线于点 E ,连结 BE . (1)求证: BE 与 O⊙ 相切; (2)连结 AD 并延长交 BE 于点 F ,若 9OB  , 2sin 3ABC  ,求 BF 的长. 22.操作与探究: (1)对数轴上的点 P 进行如下操作:先把点 P 表示的数乘以 1 3 ,再把所得数对应的点 向右平移 1 个单位,得到点 P 的对应点 P . 点 A B, 在数轴上,对线段 AB 上的每个点进行上述操作后得到线段 A B  ,其中点 A B, 的对应点分别为 A B , .如图 1,若点 A 表示的数是 3 ,则点 A 表示的数 是 ;若点 B表示的数是 2,则点 B 表示的数是 ;已知线段 AB 上的点 E 经过上 述操作后得到的对应点 E 与点 E 重合,则点 E 表示的数是 ; (2)如图 2,在平面直角坐标系 xOy 中,对正方形 ABCD 及其 内部的每个点进行如下操作:把每 个点的横、纵坐标都乘以同一 种实数 a ,将得到的点先向右平移 m 个单位,再向上平移 n 个单位 ( 0 0m n , ),得到正方形 A B C D    及其内部的点,其中点 A B, 的对应点分别为 A B , 。已知正方形 ABCD 内部的一个点 F 经过上述操作后得到的对应点 F 与点 F 重合,求点 F 的坐标。 23.已知二次函数 2 3( 1) 2( 2) 2y t x t x     在 0x  和 2x  时的函数值相等。 (1) 求二次函数的解析式; (2) 若一次函数 6y kx  的图象与二次函数的 图象都经过点 ( 3 )A m , ,求 m 和 k 的值; (3) 设二次函数的图象与 x 轴交于点 B C, (点 B 在点 C 的左侧),将二次函数的图象在点 B C, 间的 部分(含点 B 和点 C )向左平移 ( 0)n n  个单位后得到 的图象记为 G ,同时将(2)中得到的直线 6y kx  向上平移 n 个单位。请结合图象回 答:当平移后的直线与图象 G 有公共点时, n 的取值范围。 24.在 ABC△ 中, BA BC BAC   , , M 是 AC 的中点, P 是线段 BM 上的动点,将 线段 PA 绕点 P 顺时针旋转 2 得到线段 PQ 。 (1) 若    且点 P 与点 M 重合(如图 1),线段 CQ 的延长线交射线 BM 于点 D ,请补全图形,并写出 CDB 的度数; (2) 在图 2 中,点 P 不与点 B M, 重合,线段 CQ 的延长线与射线 BM 交于点 D ,猜 想 CDB 的大小(用含  的代数式表示),并加以证明; (3) 对于适当大小的  ,当点 P 在线段 BM 上运动到某一位置(不与点 B , M 重 合)时,能使得线段 CQ 的延长线与射线 BM 交于点 D ,且 PQ QD ,请直接写出  的范 围。 25.在平面直角坐标系 xOy 中,对于任意两点 1 1 1( )P x y, 与 2 2 2( )P x y, 的“非常距离”,给出如下定义: 若 1 2 1 2| | | |x x y y ≥ ,则点 1P 与点 2P 的“非常距离”为 1 2| |x x ; 若 1 2 1 2| | | |x x y y   ,则点 1P 与点 2P 的“非常距离”为 1 2| |y y . 例如:点 1(1 2)P , ,点 2 (3 5)P , ,因为|1 3| | 2 5|   ,所以点 1P 与点 2P 的“非常距离”为| 2 5| 3  ,也就是图 1 中线段 1PQ 与线段 2P Q 长度 的较大值(点 Q 为垂直于 y 轴的直线 1PQ 与垂直于 x 轴的直线 2P Q 的交点)。 (1)已知点 1( 0)2A  , , B 为 y 轴上的一个动点, ①若点 A 与点 B 的“非常距离”为 2,写出一个满足条件的点 B 的坐标; ②直接写出点 A 与点 B 的“非常距离”的最小值; (2)已知C 是直线 3 34y x  上的一个动点, ①如图 2,点 D 的坐标是(0,1),求点 C 与点 D 的“非常距离”的最小值及相应 的点 C 的坐标; ②如图 3, E 是以原点 O 为圆心,1 为半径的圆上的一个动点,求点 C 与点 E 的 “非常距离”的最小值及相应的点 E 和点 C 的坐标。 2012 上海 18、如图所示, Rt ABC 中, 90C   , 1BC  , 30A   , 点 D 为边 AC 上的一动点,将 ABD 沿直线 BD 翻折,点 A 落 在点 E 处,如果 DE AD 时,那么 DE  . 21、如图所示,在 Rt ABC , 90ACB   , D 是边 AB 的中点, BE CD ,垂足为 E ,已知 15AC  , 3 5cosA  . ①求线段CD 的长; ②求 sin DBE 的值. 23、如图所示,在菱形 ABCD 中,点 E 、 F 分别在 BC 、 CD 上, BAF DAE   , AE 与 BD 相交于点G . ①求证: BE DF ; 2 %2 当 DF AD FC DF  时,求证:四边形 BEFG 是平行四边形. 24、如图,在平面直角坐标系中,二次函数 2 6y ax x c   过点 (4A , 0) 和 ( 1B  , 0) ,并与 y 轴交于点 C ,点 D 在线段 OC 上,设 DO t ,点 E 在第二象限,且 90ADE   , 1 2tan DAE  , EF OD 于 F . C B D A E D BC A E D CB A FG x D FE C y ①求二次函数的解析式; ②用含 t 的代数式表示 EF 和OF 的长; 3 %2 当 ECA CAO   时,求 t 的值. 25、已知扇形 AOB 中, 90AOB   , 2OA OB  ,C 为弧 AB 上的动点,且不与 A 、 B 重合,OE AC 于 E ,OD BC 于 D . ①若 1BC  ,求OD 的长; ②在 DOE 中,是否存在长度保持不变的边,若存在,求出该 边的长; 若不存在,请说明理由; ③设 BD x , DOE 的面积为 y ,求 y 与 x 的函数关系式及 定义域. AO B CD E A B C 第(17)题 D E F 2012 天津 (10)若关于 x 的一元二次方程 ( 2)( 3)x x m   有实数根 1x 、 2x ,且 1 2x x ,有下列结论: ① 1 2x  , 2 3x  ; ② 1 4m   ; ③二次函数 1 2( )( )y x x x x m    的图象与 x 轴交点的坐标为 2 0( ,)和 3 0( ,). 其中,正确结论的个数是 (A)0 (B)1 (C)2 (D)3 (17)如图,已知正方形 ABCD 的边长为 1,以顶点 A 、 B 为圆心,1 为半径的两弧交于 点 E ,以顶点 C 、 D 为圆心,1 为半径的两弧交于点 F ,则 EF 的长为 . (18)“三等分任意角”是数学史上一个著名问题.已知一个角 MAN ,设 1 3 MAN   . (Ⅰ)当 69MAN   时,  的大小为 (度); (Ⅱ)如图,将 MAN 放置在每个小正方形的边长为 1 cm 的 网格中,角的一边 AM 与水平方向的网格线平行,另 一边 AN 经过格点 B ,且 2.5AB  cm.现要求只能 使用带刻度...的直尺,请你在图中作出  ,并简要说 明作法(不要求证明) . B MA N 第(18)题 (25)(本小题 10 分) 已知一个矩形纸片 OACB ,将该纸片放置在平面直角坐标系中,点 11 0A( ,),点 0 6B( ,),点 P 为 BC 边上的动点(点 P 不与点 B 、 C 重合),经过点O 、 P 折叠该纸 片,得点 B 和折痕 OP .设 BP t . (Ⅰ)如图①,当 30BOP   时,求点 P 的坐标; O xA CB P B O x y A CB P B C Q 第(25)题 图① 图② y (Ⅱ)如图②,经过点 P 再次折叠纸片,使点C 落在直线 PB 上,得点 C 和折痕 PQ ,若 AQ m ,试用含有 t 的式子表示 m ; (Ⅲ)在(Ⅱ)的条件下,当点 C 恰好落在边 OA上时,求点 P 的坐标(直接写出结 果即可). (26)(本小题 10 分) 已知抛物线 2y ax bx c   ( 0 2a b  )的顶点为 0 0( )P x y, ,点 (1 )AA y, 、 (0 )BB y, 、 ( 1 )CC y , 在该抛物线上. (Ⅰ)当 1a  , 4b  , 10c  时,①求顶点 P 的坐标;②求 A B C y y y 的值; (Ⅱ)当 0 0y ≥ 恒成立时,求 A B C y y y 的最小值. 2012 重庆 10.(2012 重庆)已知二次函数 )0(2  acbxaxy 的图象如图所 示对称轴为 2 1x .下列结论中,正确的是( ) A. 0abc  B. 0a b  C. 2 0b c  D. 4 2a c b  24.(2012 重庆)已知:如图,在菱形 ABCD 中,F 为边 BC 的中点,DF 与对角线 AC 交 于点 M,过 M 作 ME⊥CD 于点 E,∠1=∠2. (1)若 CE=1,求 BC 的长; (2)求证:AM=DF+ME. 25.(2012 重庆)企业的污水处理有两种方式,一种是输送到污水厂进行集中处理,另一 种是通过企业的自身设备进行处理.某企业去年每月的污水量均为 12000 吨,由于污水厂 处于调试阶段,污水处理能力有限,该企业投资自建设备处理污水,两种处理方式同时进 行.1 至 6 月,该企业向污水厂输送的污水量 y1(吨)与月份 x(1≤x≤6,且 x 取整数)之 间满足的函数关系如下表: 7 至 12 月,该企业自身处理的污水量 y2(吨)与月份 x(7≤x≤12,且 x 取整数)之间满足 二次函数关系式为 )0(2 2  acaxy .其图象如图所示.1 至 6 月,污水厂处理每吨污 水的费用: 1z (元)与月份 x 之间满足函数关系式: xz 2 1 1  ,该企业自身处理每吨污水 的费用: 2z (元)与月份 x 之间满足函数关系式: 2 2 12 1 4 3 xxz  ;7 至 12 月,污水厂 处理每吨污水的费用均为 2 元,该企业自身处理每吨污水的费用均为 1.5 元. (1)请观察题中的表格和图象,用所学过的一次函数、反比例函数或二次函数的有关知 识,分别直接写出 21,yy 与 x 之间的函数关系式; (2)请你求出该企业去年哪个月用于污水处理的费用 W(元)最多,并求出这个最多费 用; (3)今年以来,由于自建污水处理设备的全面运行,该企业决定扩大产能并将所有污水全 部自身处理,估计扩大产能后今年每月的污水量都将在去年每月的基础上增加 a%,同时每 吨污水处理的费用将在去年 12 月份的基础上增加(a﹣30)%,为鼓励节能降耗,减轻企 业负担,财政对企业处理污水的费用进行 50%的补助.若该企业每月的污水处理费用为 18000 元,请计算出 a 的整数值. (参考数据: ≈15.2, ≈20.5, ≈28.4) 26.(2012 重庆)已知:如图,在直角梯形 ABCD 中,AD∥BC,∠B=90°,AD=2, BC=6,AB=3.E 为 BC 边上一点,以 BE 为边作正方形 BEFG,使正方形 BEFG 和梯形 ABCD 在 BC 的同侧. (1)当正方形的顶点 F 恰好落在对角线 AC 上时,求 BE 的长; (2)将(1)问中的正方形 BEFG 沿 BC 向右平移,记平移中的正方形 BEFC 为正方形 B′EFG,当点 E 与点 C 重合时停止平移.设平移的距离为 t,正方形 B′EFG 的边 EF 与 AC 交于点 M,连接 B′D,B′M,DM,是否存在这样的 t,使△B′DM 是直角三角形?若存 在,求出 t 的值;若不存在,请说明理由; (3)在(2)问的平移过程中,设正方形 B′EFG 与△ADC 重叠部分的面积为 S,请直接写 出 S 与 t 之间的函数关系式以及自变量 t 的取值范围. 2012 南京 5、(2012 江苏南京 2 分)若反比例函数 ky x  与一次函数 y x 2  的图像没有..交点, 则 k 的值可以是【 】 A. -2 B. -1 C. 1 D. 2 6、(2012 江苏南京 2 分)如图,菱形纸片 ABCD 中,∠A=600,将纸片折叠,点 A、D 分别落在 A’、D’处,且 A’D’经过 B,EF 为折痕,当 D’F  CD 时, CF FD 的值为【 】 A. 3 1 2  B. 3 6 C. 2 3 1 6  D. 3 1 8  16、(2012 江苏南京 2 分)在平面直角坐标系中,规定把一个三角形先沿 x 轴翻折,再 向右平移两个单位称为一次变换,如图,已知等边三角形 ABC 的顶点 B、C 的坐标分别 是,(-1,-1),(-3,-1),把三角形 ABC 经过连续 9 次这样的变换得到三角形 A’B’C’,则点 A 的对应点 A’的坐标是 ▲ 27、(2012 江苏南京 10 分)如图,A、B 为⊙O 上的两个定点,P 是⊙O 上的动点(P 不 与 A、B 重合),我们称∠APB 为⊙O 上关于 A、B 的滑动角。 (1)已知∠APB 是 O 上关于点 A、B 的滑动角。 ① 若 AB 为⊙O 的直径,则∠APB= ② 若⊙O 半径为 1,AB= 2 ,求∠APB 的度数 (2)已知 2O 为 1O 外一点,以 2O 为圆心作一个圆与 1O 相交于 A、B 两点,∠APB 为 1O 上关于点 A、B 的滑动角,直线 PA、PB 分别交 2O 于点 M、N(点 M 与点 A、点 N 与点 B 均不重合),连接 AN,试探索∠APB 与∠MAN、∠ANB 之间的数量关系。 2012 广州 24. (本小题满分 14 分) 如图 9,抛物线 与 x 轴交于 A、B 两点 (点 A 在点 B 的左侧)。与 y 轴交于点 C. (1)、求点 A、B 的坐标; (2)、设 D 为已知抛物线的对称轴上的任意一点。当△ACD 的面积等于△ACB 的面积时, 求点 D 的坐标; (3)、若直线l 经过点 E(4,0),M 为直线l 上的动点,当以 A、B、M 为顶点所作的直角 三角形有且只有三个时,求直线l 的解析式。 34 3 8 3 2  xxy 25. (本小题满分 14 分)www .xk b1 .co m 如图 10,在平行四边形 ABCD 中,AB=5,BC=10,F 为 AD 的中点。CE⊥AB 于点 E,设∠ ABC=α(600≤<α<900). (1)、当α=600 时,求 CE 的长。 (2)、当 600≤<α<900 时, ①是否存在正整数 k ,使得∠EFD= k ∠AEF?若存 在,求出 k 的值;若不存在,请说明理由。 ②连接 CF,当 CE2-CF2 取最大值时,求 tan ∠DCF 的值。 2012 武汉 11.(2012 武汉)甲、乙两人在直线跑道上同起点、同终 点、同方向匀速跑步 500 米,先到终点的人原地休息.已知 甲先出发 2 秒.在跑步过程中,甲、乙两人的距离 y(米) 与乙出发的时间 t(秒)之间的关系如图所示,给出以下结 论:①a=8;②b=92;③c=123.其中正确的是( ) A. ①②③ B.仅有①② C.仅有①③ D.仅有②③ 12.(2012 武汉)在面积为 15 的平行四边形 ABCD 中,过点 A 作 AE 垂直于直线 BC 于点 E,作 AF 垂直于直线 CD 于点 F,若 AB=5,BC=6,则 CE+CF 的值为( ) A. 11+ B. 11﹣ C. 11+ 或 11﹣ D. 11﹣ 或 1+ 15.(2012 武汉)如图,点 A 在双曲线 y= 的第一象限的那一支上,AB 垂直于 x 轴与点 B,点 C 在 x 轴正半轴上,且 OC=2AB,点 E 在线段 AC 上,且 AE=3EC,点 D 为 OB 的 中点,若△ADE 的面积为 3,则 k 的值为 . 16.(2012 武汉)在平面直角坐标系中,点 A 的坐标为(3.0),点 B 为 y 轴正半轴上的一 点,点 C 是第一象限内一点,且 AC=2.设 tan∠BOC=m,则 m 的取值范围是 . 22.(2012 武汉)在锐角三角形 ABC 中,BC=5,sinA= , (1)如图 1,求三角形 ABC 外接圆的直径; (2)如图 2,点 I 为三角形 ABC 的内心,BA=BC,求 AI 的长. 23.(2012 武汉)如图,小河上有一拱桥,拱桥及河道的截面轮廓线由抛物线的一部分 ACB 和矩形的三边 AE,ED,DB 组成,已知河底 ED 是水平的,ED=16 米,AE=8 米,抛 物线的顶点 C 到 ED 的距离是 11 米,以 ED 所在的直线为 x 轴,抛物线的对称轴为 y 轴建 立平面直角坐标系. (1)求抛物线的解析式; (2)已知从某时刻开始的 40 小时内,水面与河底 ED 的距离 h(单位:米)随时间 t(单 位:时)的变化满足函数关系 h=﹣ (t﹣19)2+8(0≤t≤40),且当水面到顶点 C 的距离 不大于 5 米时,需禁止船只通行,请通过计算说明:在这一时段内,需多少小时禁止船只 通行? 24.(2012 武汉)已知△ABC 中,AB= ,AC= ,BC=6 (1)如图 1,点 M 为 AB 的中点,在线段 AC 上取点 N,使△AMN 与△ABC 相似,求线 段 MN 的长; (2)如图 2,是由 100 个边长为 1 的小正方形组成的 10×10 的正方形网格,设顶点在这些 小正方形顶点的三角形为格点三角形. ①请你在所给的网格中画出格点△A1B1C1 与△ABC 全等(画出一个即可,不需证明) ②试直接写出所给的网格中与△ABC 相似且面积最大的格点三角形的个数,并画出其中 一个(不需证明). 25.(2012 武汉)如图 1,点 A 为抛物线 C1:y= x2﹣2 的顶点,点 B 的坐标为(1,0)直 线 AB 交抛物线 C1 于另一点 C (1)求点 C 的坐标; (2)如图 1,平行于 y 轴的直线 x=3 交直线 AB 于点 D,交抛物线 C1 于点 E,平行于 y 轴 的直线 x=a 交直线 AB 于 F,交抛物线 C1 于 G,若 FG:DE=4:3,求 a 的值; (3)如图 2,将抛物线 C1 向下平移 m(m>0)个单位得到抛物线 C2,且抛物线 C2 的顶点 为点 P,交 x 轴于点 M,交射线 BC 于点 N.NQ⊥x 轴于点 Q,当 NP 平分∠MNQ 时,求 m 的值. (2012 成都) 20. (本小题满分 10 分) 如图,△ABC 和△DEF 是两个全等的等腰直角三角形,∠BAC=∠EDF=90°,△DEF 的顶点 E 与 △ABC 的斜边 BC 的中点重合.将△DEF 绕点 E 旋转,旋转过程中,线段 DE 与线段 AB 相交于 点 P,线段 EF 与射线 CA 相交于点 Q. (1)如图①,当点 Q 在线段 AC 上,且 AP=AQ 时,求证:△BPE≌△CQE; (2)如图②,当点 Q 在线段 CA 的延长线上时,求证:△BPE∽△CEQ;并求当 BP= a , CQ= 9 2 a 时,P、Q 两点间的距离 (用含 a 的代数式表示). 23.(2012 成都)有七张正面分别标有数字 3 , 2 , 1 ,0,l,2,3 的卡片,它们除 数字不同外其余全部相同.现将它们背面朝上,洗匀后从中随机抽取一张,记卡片上的数 字为 a ,则使关于 x 的一元二次方程 2 2( 1) ( 3) 0x a x a a     有两个不相等的实数 根,且以 x 为自变量的二次函数 2 2( 1) 2y x a x a     的图象不经过...点(1,O)的概率 是________. 24.(2012 成都)如图,在平面直角坐标系 xOy 中,直线 AB 与 x 轴、y 轴分别交于点 A, B,与反比例函数 ky x  ( k 为常数,且 0k  )在第一象限的图象交于点 E,F.过点 E 作 EM ⊥y 轴于 M,过点 F 作 FN⊥x 轴于 N,直线 EM 与 FN 交于点 C.若 BE 1 BF m  ( m 为大于 l 的 常数).记△CEF 的面积为 1S ,△OEF 的面积为 2S ,则 1 2 S S =________. (用含 m 的代数式 表示) 25.(2012 成都)如图,长方形纸片 ABCD 中,AB=8cm,AD=6cm,按下列步骤进行裁剪和 拼图: 第一步:如图①,在线段 AD 上任意取一点 E,沿 EB,EC 剪下一个三角形纸片 EBC(余 下部分不再使用); 第二步:如图②,沿三角形 EBC 的中位线 GH 将纸片剪成两部分,并在线段 GH 上任意 取一点 M,线段 BC 上任意取一点 N,沿 MN 将梯形纸片 GBCH 剪成两部分; 第三步:如图③,将 MN 左侧纸片绕 G 点按顺时针方向旋转 180°,使线段 GB 与 GE 重 合,将 MN 右侧纸片绕 H 点按逆时针方向旋转 180°,使线段 HC 与 HE 重合,拼成一个与三 角形纸片 EBC 面积相等的四边形纸片. (注:裁剪和拼图过程均无缝且不重叠) 则拼成的这个四边形纸片的周长的最小值为________cm,最大值为________cm. 26.(2012 成都)(本小题满分 8 分) “城市发展 交通先行”,成都市今年在中心城区启动了缓堵保畅的二环路高架桥快速 通道建设工程,建成后将大大提升二环路的通行能力.研究表明,某种情况下,高架桥上 的车流速度 V(单位:千米/时)是车流密度 x (单位:辆/千米)的函数,且当 0< x ≤28 时,V=80;当 28< x ≤188 时,V 是 x 的一次函数. 函数关系如图所示. (1)求当 28< x ≤188 时,V 关于 x 的函数表达式; (2)若车流速度 V 不低于 50 千米/时,求当车流密度 x 为多少时,车流量 P(单位: 辆/时)达到最大,并求出这一最大值. (注:车流量是单位时间内通过观测点的车辆数,计算公式为:车流量=车流速度×车 流密度) 27.(2012 成都)(本小题满分 I0 分) 如图,AB 是⊙O 的直径,弦 CD⊥AB 于 H,过 CD 延长线上一点 E 作⊙O 的切线交 AB 的延长线于 F.切点为 G,连接 AG 交 CD 于 K. (1)求证:KE=GE; (2)若 2KG =KD·GE,试判断 AC 与 EF 的位置关系,并说明理由; (3) 在(2)的条件下,若 sinE= 3 5 ,AK= 2 3 ,求 FG 的长. 28.(2012 成都)(本小题满分 l2 分) 如图,在平面直角坐标系 xOy 中,一次函数 5 4y x m  ( m 为常数)的图象与 x 轴交 于点 A( 3 ,0),与 y 轴交于点 C.以直线 x=1 为对称轴的抛物线 2y ax bx c   ( a b c, , 为常数,且 a ≠0)经过 A,C 两点,并与 x 轴的正半轴交于点 B. (1)求 m 的值及抛物线的函数表达式; (2)设 E 是 y 轴右侧抛物线上一点,过点 E 作直线 AC 的平行线交 x 轴于点 F.是否存 在这样的点 E,使得以 A,C,E,F 为顶点的四边形是平行四边形?若存在,求出点 E 的坐 标及相应的平行四边形的面积;若不存在,请说明理由; (3)若 P 是抛物线对称轴上使△ACP 的周长取得最小值的点,过点 P 任意作一条与 y 轴 不平行的直线交抛物线于 1 1 1M ( )x y, , 2 2 2M ( )x y, 两点,试探究 21 1 2 P PM M M M  是否为定 值,并写出探究过程. 2012 长沙市 24.如图,已知正方形 ABCD 中,BE 平分∠DBC 且交 CD 边于点 E,将△BCE 绕点 C 顺 时针旋转到△DCF 的位置,并延长 BE 交 DF 于点 G. (1)求证:△BDG∽△DEG; (2)若 EG•BG=4,求 BE 的长. 25.在长株潭建设两型社会的过程中,为推进节能减排,发展低碳经济,我市某公司以 25 万元购得某项节能产品的生产技术后,再投入 100 万元购买生产设备,进行该产品的生产 加工.已知生产这种产品的成本价为每件 20 元.经过市场调研发现,该产品的销售单价定 在 25 元到 30 元之间较为合理,并且该产品的年销售量 y(万件)与销售单价 x(元)之间 的函数关系式为: (年获利=年销售收入﹣生产成本﹣投资成本) (1)当销售单价定为 28 元时,该产品的年销售量为多少万件? (2)求该公司第一年的年获利 W(万元)与销售单价 x(元)之间的函数关系式,并说明 投资的第一年,该公司是盈利还是亏损?若盈利,最大利润是多少?若亏损,最小亏损是 多少? (3)第二年,该公司决定给希望工程捐款 Z 万元,该项捐款由两部分组成:一部分为 10 万元的固定捐款;另一部分则为每销售一件产品,就抽出一元钱作为捐款.若除去第一年 的最大获利(或最小亏损)以及第二年的捐款后,到第二年年底,两年的总盈利不低于 67.5 万元,请你确定此时销售单价的范围. 26.如图半径分别为 m,n(0<m<n)的两圆⊙O1 和⊙O2 相交于 P,Q 两点,且点 P (4,1),两圆同时与两坐标轴相切,⊙O1 与 x 轴,y 轴分别切于点 M,点 N,⊙O2 与 x 轴,y 轴分别切于点 R,点 H. (1)求两圆的圆心 O1,O2 所在直线的解析式; (2)求两圆的圆心 O1,O2 之间的距离 d; (3)令四边形 PO1QO2 的面积为 S1,四边形 RMO1O2 的面积为 S2. 试探究:是否存在一条经过 P,Q 两点、开口向下,且在 x 轴上截得的线段长为 的抛物线?若存在,请求出此抛物线的解析式;若不存在,请说明理由. 2012 福州市 10.如图,过点 C(1,2)分别作 x 轴、y 轴的平行线,交直线 y= -x+6 于 A、B 两点,若反比例函数 y=k x(x>0)的图像与△ ABC 有公共点,则 k 的取值范围是 A.2≤k≤9 B.2≤k≤8 C.2≤k≤5 D.5≤k≤8 15.如图,已知△ABC,AB=AC=1,∠A=36°,∠ABC 的平分线 BD 交 AC 于点 D,则 AD 的长是______,cosA 的值是 ______________.(结果保留根号) 20.(满分 12 分)如图,AB 为⊙O 的直径,C 为⊙O 上一点, AD 和过 C 点的切线互相垂直,垂足为 D,AD 交⊙O 于点 E. (1) 求证:AC 平分∠DAB; (2) 若∠B=60º,CD=2 3,求 AE 的长. A B C D 第 15 题图 A B C O x y 第 10 题图 A B C D E O 第 20 题图 21.(满分 13 分)如图①,在 Rt△ABC 中,∠C=90º,AC=6,BC=8,动点 P 从点 A 开始 沿边 AC 向点 C 以每秒 1 个单位长度的速度运动,动点 Q 从点 C 开始沿边 CB 向点 B 以每 秒 2 个单位长度的速度运动,过点 P 作 PD∥BC,交 AB 于点 D,连接 PQ.点 P、Q 分别从 点 A、C 同时出发,当其中一点到达端点时,另一点也随之停止运动,设运动时间为 t 秒(t ≥0). (1) 直接用含 t 的代数式分别表示:QB=______,PD=______. (2) 是否存在 t 的值,使四边形 PDBQ 为菱形?若存在,求出 t 的值;若不存在,说明 理由.并探究如何改变点 Q 的速度(匀速运动),使四边形 PDBQ 在某一时刻为菱 形,求点 Q 的速度; (3) 如图②,在整个运动过程中,求出线段 PQ 中点 M 所经过的路径长. 第 21 题图① A B C D P Q 第 21 题图② A B C D P Q 22.(满分 14 分)如图①,已知抛物线 y=ax2+bx(a≠0)经过 A(3,0)、B(4,4)两点. (1) 求抛物线的解析式; (2) 将直线 OB 向下平移 m 个单位长度后,得到的直线与抛物线只有一个公共点 D, 求 m 的值及点 D 的坐标; (3) 如图②,若点 N 在抛物线上,且∠NBO=∠ABO,则在(2)的条件下,求出所有满足 △POD∽△NOB 的点 P 的坐标(点 P、O、D 分别与点 N、O、B 对应). A B D O x y 第 22 题图① A B D O x y 第 22 题图② N 2012 年沈阳市 24.已知,如图①,∠MON=60°,点 A,B 为射线 OM,ON 上的动点(点 A,B 不与 点 O 重合),且 AB= 34 ,在∠MON 的内部、△AOB 的外部有一点 P,且 AP=BP,∠ APB=120°. (1)求 AP 的长; (2)求证:点 P 在∠MON 的平分线上; (3) 如图②,点 C,D,E,F 分别是四边形 AOBP 的边 AO,OB,BP,PA 的中点, 连接 CD,DE,EF,FC,OP. ①当 AB⊥OP 时,请直接..写出四边形 CDEF 的周长的值; ②若四边形 CDEF 的周长用 t 表示,请直接..写出 t 的取值范围. 25.已知,如图,在平面直角坐标系中,点 A 坐标为(-2,0),点 B 坐标为 (0,2 ),点 E 为线段 AB 上的动点(点 E 不与点 A,B 重合),以 E 为顶点作∠OET=45°,射线 ET 交线段 OB 于点 F,C 为 y 轴正半轴上一点,且 OC=AB,抛物线 y= 2 x2+mx+n 的图象经过 A,C 两点. (1) 求此抛物线的函数表达式; (2) 求证:∠BEF=∠AOE; (3) 当△EOF 为等腰三角形时,求此时点 E 的坐标; (4) 在(3)的条件下,当直线 EF 交 x 轴于点 D,P 为(1) 中抛物线上一动点,直 线 PE 交 x 轴于点 G,在直线 EF 上方的抛物线上是否存在一点 P,使得△EPF 的面积是△ EDG 面积的( 122  ) 倍.若存在,请直接..写出点 P 的坐标;若不存在,请说明理由. 吉林省 2012 年 14.如图,在 等边 ABC 中, D 是边 AC 上的一点,连接 BD ,将 BCD 绕点 B 逆时针旋转 60 ,得到 BAE , 连接 ED ,若 10BC  , 9BD  ,则 AED 的周长是 ______. 24.如图 1, , ,A B C 为三个超市,在 A 通往 C 的道路(粗 实线部分)上有一 D 点, D 与 B 有道路(细实线部 分)相通. A 与 D , D 与C , D 与 B 之间的路程分别为 25km ,10km ,5km .现计 划在 A 通往 C 的道路上建一个配货中心 H ,每天有一辆货车只为这三个超市送货.该 货车每天从 H 出发,单独为 A 送货1次,为 B 送货1次,为C 送货 2 次.货车每次仅能给一家 超市送货,每次送货后均返回配货中心 H .设 H 到 A 的路程为 xkm .这辆货车每天行驶的路 程为 ykm . (1)用含 x 的代数式填空: 当 0 x≤ ≤25 时,货车从 H 到 A 往返1次的路程为 2xkm . 货车 从 H 到 B 往返1次的路程为_______ km . 货车从 H 到C 往返 2 次的路程为_______ km . 这辆货车每天行驶的路程 y  __________. 当 25 x ≤35 时, 这辆货车每天行驶的路程 y  _________; (2)请在图 2 中画出 y 与 x ( 0 x≤ ≤35 )的函数图象; (3)配货中心 H 建在哪段,这辆货车每天行驶的路程最短? 25.如图,在 ABC 中, 90A   , 2AB cm , 4AC cm ,动点 P 从点 A 出发,沿 AB 方向以1 /cm s 的速度向点 B 运动,动点 Q 从 点 B 同 时 出 发 , 沿 BA 方 向 以 1 /cm s 的速度向点 A 运动.当点 P 到达 点 B 时, P , Q 两 点同时停止运动.以 AP 为一边向上作正方形 APDE ,过点 Q 作 QF BC∥ ,交 AC 于点 F .设点 P 的运动时间为 ts ,正方形 APDE 和梯 形 BCFQ 重合部分的面积为 2Scm . (1)当t  _____s 时,点 P 与点Q 重合; (2)当t  _____s 时,点 D 在QF 上; (3)当点 P 在Q , B 两点之间(不包括Q , B 两点)时,求 S 与t 之间的函数关系式. 26.问题情境[ 如图,在 x 轴上有两点 ( ,0)A m , ( ,0)B n ( 0n m  ).分别过点 A ,点 B 作 x 轴 的垂线,交抛物线 2y x 于点 C 、点 D .直线 OC 交直线 BD 于点 E ,直线 OD 交直 线 AC 于点 F ,点 E 、点 F 的纵坐标分别记为 .Ey 、 Fy . 特例探究 填 空 : 当 1m  , 2n  时 , .Ey =____, Fy =______. 当 3m  , 5n  时 , .Ey =____, Fy =______.[来源:21 世纪教育网] 归纳证明 对任意 m , n ( 0n m  ),猜想 .Ey 与 Fy 的大小关系,并证明你的猜想 拓展应用. (1) 若将“抛物线 2y x ”改为“抛物线 2 ( 0)y ax a  ”,其它条件不变,请直接 写出 .Ey 与 Fy 的大小关系. (2) 连接 EF , AE .当 . 3 OFEOFEBS S △四边形 时,直接写出 m 和 n 的关系及四边形 OFEA 的形状. 哈尔滨市 2012 19.如图,平行四边形 ABCD 绕点 A 逆时针旋转 300,得到平行四边形 AB1C1D1(点 B1 与点 B 是对应点,点 C1 与点 C 是对应点,点 D1 与点 D 是对应点),点 B1 恰好落在 BC 边上,则∠C= 度. 20.如图。四边形 ABCD 是矩形,点 E 在线段 CB 的延长线上,连接 DE 交 AB 于点 F,∠AED=2 ∠CED,点 G 是 DF 的中点,若 BE=1,AG=4,则 AB 的长为 27. 如图,在平面直角坐标系中,点 0 为坐标原点,直线 y=2x+4 交 x 轴于点 A,交 y 轴于点 B,四边形 ABC0 是平行四边形,直线 y=_x+m 经过点 C,交 x 轴于点 D. (1)求 m 的值; (2)点 P(0,t)是线段 OB 上的一个动点(点 P 不与 0,B 两点重合),过点 P 作 x 轴 的平行线,分别交 AB,0c,DC 于点 E,F,G.设线段 EG 的长为 d,求 d 与 t 之间的函数关 系式(直接写出自变量 t 的取值范围); (3)在(2)的条件下,点 H 是线段 OB 上一点,连接 BG 交 OC 于点 M,当以 OG 为直 径的圆经过点 M 时,恰好使∠BFH=∠AB0.求此时 t 的值及点 H 的坐标. 28. 已知:在△ABC 中,∠ACB=900,点 P 是线段 AC 上一点,过点 A 作 AB 的垂线,交 BP 的延长线于点 M,MN⊥AC 于点 N,PQ⊥AB 于点 Q,AQ=MN. (1)如图 l,求证:PC=AN; (2)如图 2,点 E 是 MN 上一点,连接 EP 并延长交 BC 于点 K,点 D 是 AB 上一点, 连接 DK,∠DKE=∠ABC,EF⊥PM 于点 H,交 BC 延长线于点 F,若 NP=2,PC=3,CK:CF=2: 3,求DQ 的长. 2012 年江西省南昌市 27.(2012 南昌)如图,已知二次函数 L1:y=x2﹣4x+3 与 x 轴交于 A.B 两点(点 A 在点 B 左边),与 y 轴交于点 C. (1)写出二次函数 L1 的开口方向、对称轴和顶点坐标; (2)研究二次函数 L2:y=kx2﹣4kx+3k(k≠0). ①写出二次函数 L2 与二次函数 L1 有关图象的两条相同的性质; ②若直线 y=8k 与抛物线 L2 交于 E、F 两点,问线段 EF 的长度是否发生变化?如果不会, 请求出 EF 的长度;如果会,请说明理由. 28.(2012 南昌)已知,纸片⊙O 的半径为 2,如图 1,沿弦 AB 折叠操作. (1)①折叠后的 所在圆的圆心为 O′时,求 O′A 的长度; ②如图 2,当折叠后的 经过圆心为 O 时,求 的长度; ③如图 3,当弦 AB=2 时,求圆心 O 到弦 AB 的距离; (2)在图 1 中,再将纸片⊙O 沿弦 CD 折叠操作. ①如图 4,当 AB∥CD,折叠后的 与 所在圆外切于点 P 时,设点 O 到弦 AB.CD 的 距离之和为 d,求 d 的值; ②如图 5,当 AB 与 CD 不平行,折叠后的 与 所在圆外切于点 P 时,设点 M 为 AB 的 中点,点 N 为 CD 的中点,试探究四边形 OMPN 的形状,并证明你的结论. 2012 年广东省 20.(2012•广东)有三张正面分别写有数字﹣2,﹣1,1 的卡片,它们的背面完全相同, 将这三张卡片北背面朝上洗匀后随机抽取一张,以其正面的数字作为 x 的值,放回卡片洗 匀,再从三张卡片中随机抽取一张,以其正面的数字作为 y 的值,两次结果记为(x,y). (1)用树状图或列表法表示(x,y)所有可能出现的结果; (2)求使分式 + 有意义的(x,y)出现的概率; (3)化简分式 + ,并求使分式的值为整数的(x,y)出现的概率. 21.(2012•广东)如图,在矩形纸片 ABCD 中,AB=6,BC=8.把△BCD 沿对角线 BD 折叠,使点 C 落在 C′处,BC′交 AD 于点 G;E、F 分别是 C′D 和 BD 上的点,线段 EF 交 AD 于点 H,把△FDE 沿 EF 折叠,使点 D 落在 D′处,点 D′恰好与点 A 重合. (1)求证:△ABG≌△C′DG; (2)求 tan∠ABG 的值; (3)求 EF 的长. 22.(2012•广东)如图,抛物线 y= x2﹣ x﹣9 与 x 轴交于 A、B 两点,与 y 轴交于点 C, 连接 BC、AC. (1)求 AB 和 OC 的长; (2)点 E 从点 A 出发,沿 x 轴向点 B 运动(点 E 与点 A、B 不重合),过点 E 作直线 l 平 行 BC,交 AC 于点 D.设 AE 的长为 m,△ADE 的面积为 s,求 s 关于 m 的函数关系式, 并写出自变量 m 的取值范围; (3)在(2)的条件下,连接 CE,求△CDE 面积的最大值;此时,求出以点 E 为圆心, 与 BC 相切的圆的面积(结果保留π). 2012 年河北省 12.如图 6,抛物线 2 1 ( 2) 3y a x   与 2 2 1 ( 3) 12y x   交于点 (13)A , ,过点 A 作 x 轴 的平行线,分别交两条抛物线于点 B C, .则以下结论: ①无论 x 取何值, 2y 的值总是正数. ② 1a  . ③当 0x  时, 2 1 4y y  . ④ 2 3AB AC . 其中正确结论是( ) A.①② B.②③ C.③④ D.①④ 22.如图 12,四边形 ABCD 是平行四边形,点 (1 0) (31) (3 3)A B C,, ,, , .反比例函数 ( 0)my xx   的图象经过点 D ,点 P 是一次函数 3 3 ( 0)y kx k k    的图象与该反比例函数图象的一个公 共点. (1)求反比例函数的解析式; (2)通过计算,说明一次函数 3 3 ( 0)y kx k k    的图 象一定过点C ; (3)对于一次函数 3 3 ( 0)y kx k k    ,当 y x随 的增大而增大时,确定点 P 横坐 标的取值范围(不必写出过程). 23.如图13 1 ,点 E 是线段 BC 的中点,分别以 B C, 为直角顶点的 EAB EDC△ 和△ 均 是等腰直角三角形,且在 BC 的同侧. (1) AE ED和 的数量关系为___________, AE ED和 的位置关系为___________; (2)在图13 1 中,以点 E 为位似中心,作 EGF△ 与 EAB△ 位似,点 H 是 BC 所 在直线上的一点,连接GH HD, ,分别得到了图13 2 和图13 3 ; ①在图13 2 中,点 F 在 BE 上, EGF EAB△ 与△ 的相似比是1:2 , H 是 EC 的中点.求证: .GH HD GH HD , ②在图13 3 中,点 F 在 BE 的延长线上, EGF EAB△ 与△ 的相似比是 k:1,若 2BC  ,请直接写出CH 的长为多少时,恰好使得GH HD GH HD 且 (用含 k 的代数式表示). 25.(本小题满分 10 分) 如图 14, ( 5 0) ( 3 0).A B ,, , 点C 在 y 轴的正半轴上, CBO ∠ = 45 ,CD AB∥ , 90CDA  ∠ .点 P 从点 (4 0)Q , 出发,沿 x 轴向左以每秒 1 个单位长的速度运动,运 动时间为t 秒. (1) 求点C 的坐标; (2) 当 15BCP  ∠ 时,求t 的值; (3) 以点 P 为圆心, PC 为半径的 P⊙ 随点 P 的运动而变化,当 P⊙ 与四边形 ABCD 的边(或边所在的直线)相 切时,求t 的值. 26.如图15 1 和图15 2 ,在 ABC△ 中, 513 14 cos .13AB BC ABC  , , ∠ 探究 在如图15 1 , AH BC 于点 H ,则 AH  _______, AC  _______, ABC△ 的面积 ABCS△ =___________. 拓展 如图15 2 ,点 D 在 AC 上(可与点 A C, 重合),分别过点 A C, 作直线 BD 的垂线,垂足为 E F, .设 .BD x AE m CF n  , , (当点 D 与点 A 重合时,我们 认为三角形 ABD 的面积为 0. (1)用含 x m, 或 n 的代数式表示 ABDS△ 及 CBDS△ ; (2)求 ( )m n 与 x 的函数关系式,并求 ( )m n 的最大值和最小值. (3)对给定的一个 x 值,有时只能确定唯一的点 D ,指出这样的 x 的取值范围. 发现 请你确定一条直线,使得 A B C, , 三点到这条直线的距离之和最小(不必写 出过程),并写出这个最小值. 2012 陕西省 23.(本题满分 8 分) 如图, PA PB、 分别与 O 相切于点 A B、 ,点 M 在 PB 上,且 //OM AP , MN AP ,垂足为 N . (1)求证: =OM AN ; (2)若 O 的半径 =3R , =9PA ,求OM 的长. 24.(本题满分 10 分)[来源:学科网] 如果一条抛物线  2= + + 0y ax bx c a  与 x 轴有两个交点,那么以该抛物线的顶点和这两个 交点为顶点的三角形称为这条抛物线的“抛物线三角形”. (1)“抛物线三角形”一定是 三角形; (2)若抛物线  2=- + >0y x bx b 的“抛物线三角形”是等腰直角三角形,求b 的值; (3)如图,△OAB 是抛物线  2=- + ' '>0y x bx b 的“抛物线三角形”,是否存在以原点O 为 对称中心的矩形 ABCD ?若存在,求出过 O C D、 、 三点的抛物线的表达式;若不存在,说 明理由. [来源:学#科#网 Z#X#X#K] 25.(本题满分 12 分) 如图,正三角形 ABC 的边长为3+ 3 . (1)如图①,正方形 EFPN 的顶点 E F、 在边 AB 上,顶点 N 在边 AC 上.在正三角形 ABC 及其内部,以 A 为位似中心,作正方形 EFPN 的位似正方形 ' ' ' 'EFPN ,且使正方形 ' ' ' 'EFPN 的面积最大(不要求写作法); (2)求(1)中作出的正方形 ' ' ' 'EFPN 的边长; (3)如图②,在正三角形 ABC 中放入正方形 DEMN 和正方形 EFPH ,使得 DE EF、 在边 AB 上,点 P N、 分别在边CB CA、 上,求这两个正方形面积和的最大值及最小值, 并说明理由. 2012 年安徽省 9.如图,A 点在半径为 2 的⊙O 上,过线段 OA 上的一点 P 作直线  ,与⊙O 过 A 点的切线 交于点 B,且∠APB=60°,设 OP= x ,则△PAB 的面积 y 关于 x 的函数图像大致是 ( ) 10.在一张直角三角形纸片的两直角边上各取一点,分别沿 斜边中点与这两点的连线剪去两个三角形,剩下的部分是 如图所示的直角梯形,其中三边长分别为 2、4、3,则原 直角三角形纸片的斜边长是( ) A.10 B. 54 C. 10 或 54 D.10 或 172 13.如图,点 A、B、C、D 在⊙O 上,O 点在∠D 的内部,四边形 OABC 为平行四边形,则 ∠OAD+∠OCD=_______________°. 14.如 图,P 是 矩 形 ABCD 内的任意一点,连接 PA、 PB、PC、PD,得到△PAB、△PBC、△PCD、△PDA,设它们的面积分别是 S1、S2、 S3、S4,给出如下结论: ①S1+S2=S3+S4 ② S2+S4= S1+ S3 ③若 S3=2 S1,则 S4=2 S2 ④若 S1= S2,则 P 点在矩形的对角线上 其中正确的结论的序号是_________________(把所有正确结论的序号都填在横线上). 23.如图,排球运动员站在点 O 处练习发球,将球从 O 点正上方 2m 的 A 处发出,把球看 成点,其运行的高度 y(m)与运行的水平距离 x(m)满足关系式 y=a(x-6)2+h.已知球 网与 O 点的水平距离为 9m,高度为 2.43m,球场的边界距 O 点的水平距离为 18m。 (1)当 h=2.6 时,求 y 与 x 的关系式(不要求写出自变量 x 的取值范围) (2)当 h=2.6 时,球能否越过球网?球会不会出界?请说明理由; (3)若球一定能越过球网,又不出边界,求 h 的取值范围。 第 23 题 图 2012 年河南 8.如图,已知 AB 是⊙O 的直径,且⊙O 于点 A, EC = CB .则下列结论中不一定正确的是( ) A. BA⊥DA B. OC//AE C. ∠COE=2∠ECA D. OD⊥AC 14.如图,在 Rt△ABC 中,∠C=90°,AC=6,BC=8.把△ABC 绕 AB 边上的点 D 顺时 针旋转 90°得到△A′B′C′,A′C′交 AB 于点 E。若 AD=BE,则△A′DE 的面积是_________. 15.如图,在 Rt△ABC 中,∠ACB=90°,∠B=30°,BC=3,点 D 是 BC 边上一动点 (不与点 B、C 重合),过点 D 作 DE⊥BC 交 AB 边于点 E,将∠B 沿直线 DE 翻折,点 B 落在射线 BC 上的点 F 处,当△AEF 为直角三角形时,BD 的长为__________. 22.(10 分)类比转化、从特殊到一般等思想方法,在数学学习和研究中经常用到, 如下是一个案例,请补充完整。 原题:如图 1,在□ABCD 中,点 E 是 BC 边的中点,点 F 是线段 AE 上一点,BF 的延长线 交射线 CD 于点 G。若 3 EF AF ,求 CG CD 的值。 (1)尝试探究 在图 1 中,过点 E 作 EH//AB 交 BG 于点 H,则 AB 和 EH 的数量关系 是_____________,CG 和 EH 的数量关系是______________, CG CD 的值是__________. (2)类比延伸 如图 2,在原题的条件下,若 )0(  mmEF AF , 则 CG CD 的值是_____________(用含 m 的代数式表示),试写出解答 过程。 (3)拓展迁移 如图 3,梯形 ABCD 中,DC//AB,点 E 是 BC 的 E O C D BA 第 8 题 E C D B A 第 14 题 A′ B′ C′ E F CDB A 第 15 题 E F CD BA 图 3 延长线上一点,AE 和 BD 相交于点 F。若 aCD AB  , )0,0(  babBE BC ,则 EF AF 的值是 __________(用含 a,b 的代数式表示)。 E F C D B G A 图 1 E F C D B G A 图 2 23.(11 分)如图,在平面直角坐标系中,直线 12 1  xy 与抛物线 32  bxaxy 交于 A、B 两点,点 A 在 x 轴上,点 B 的纵坐标为 3。点 P 是直线 AB 下方的抛物线上一动点 (不与点 A、B 重合),过点 P 作 x 轴的垂线交直线 AB 于点 C,作 PD⊥AB 于点 C,作 PD⊥AB 于点 D。 (1)求 a、b 及 sin∠ACP 的值; (2)设点 P 的横坐标为 m. ① 用含 m 的代数式表示线段 PD 的长,并求出线段 PD 长的最大值; ②连接 PB,线段 PC 把△PDB 分成两个三角形,是否存在合适的 m 值,使这两个三 角形的面积之比为 9:10?若存在,直接写 m 的值;若不存在,说明理由。 第 23 题 x y A BC D P O 黄冈市 2012 8. 如图,在Rt △ ABC 中,∠C=90° ,AC=BC=6cm,点P 从点A 出发,沿AB 方向以每秒 cm的速度向终点B 运动;同时,动点Q 从点B 出发沿BC 方 向以每秒1cm 的速度向终点C 运动,将△PQC 沿BC 翻折,点P 的对应点为点P′.设Q点运动的时间t 秒,若四边形QPCP′为菱 形,则t 的值为 16.某物流公司的快递车和货车同时从甲地出发,以各自的 速度匀速向乙地行驶,快 递车到达乙地后卸完物品再另装货物共用45 分钟,立即按原路 以另一速度匀速返回,直至与货车相遇.已知货车的速度为60 千米/ 时,两车之间的距离y(千米)与货车行驶时间x(小时)之 间的函数图象如图所示,现有以下4 个结论: ①快递车从甲地到乙地的速度为100 千米/时;②甲、乙两地 之间的距离为120 千米;③图中点B 的坐标为(3 ,75);④快递车从乙地返回时的速度为 90千米/时. 以上4 个结论中正确的是____________(填序号) 24.(12 分)某科技开发公司研制出一种新型产品,每件产品的成本为2400 元,销售单价 定为3000 元.在该产品的试销期间,为了促销,鼓励商家购买该新型产品,公司决定商家 一次购买这种新型产品不超过10 件时,每件按3000 元销售;若一次购买该种产品超过10 件时,每多购买一件,所购买的全部产品的销售单价均降低10 元,但销售单价均不低于 2600 元. (1)商家一次购买这种产品多少件时,销售单价恰好为2600 元? (2)设商家一次购买这种产品x 件,开发公司所获的利润为y 元,求y(元)与x(件)之间的函 数关系式,并写出自变量x 的取值范围. (3)该公司的销售人员发现:当商家一次购买产品的件数超过某一数量时,会出现随着一次 购买的数量的增多,公司所获的利润反而减少这一情况.为使商家一次购买的数量越多,公 司所获的利润越大,公司应将最低销售单价调整为多少元?(其它销售条件 不变) 25.(14 分)如图,已知抛物线的方程C1:y=- (x+2)(x-m)(m>0)与x 轴相交于 点B、C,与y 轴相交于点E,且点B 在点C 的左侧. (1)若抛物线C1过点M(2,2),求实数m 的值. (2)在(1)的条件下,求△BCE 的面积. (3)在(1)的条件下,在抛物线的对称轴上找一点H,使BH+EH 最小,并求出点H 的坐标. (4)在第四象限内,抛物线C1上是否存在点F,使得以点B、C、F 为顶点的三角形与△BCE 相 似?若存在,求m 的值;若不存在,请说明理由. 2012 年潍坊市 11.若直线 y=-2x-4 与直线 y=4x+b 的交点在第三象限,则 b 的取值范围是( ). A. -48 D.-4≤6≤8 12.下图是某月的日历表,在此日历表上可以用 一个矩形圈出 3×3 个位置相邻的 9 个数(如 6, 7,8,l3,14,l5,20,21,22).若圈出的 9 个数中,最大数与最小数的积为 192,则这 9 个 数的和为( ). A.32 B.126 C.135 D.144 22.(本题满分 l0 分)如图,已知平行四边形 ABCD,过 A 作 AM⊥BC 于 M,交 BD 于 E,过 C 作 CN⊥AD 于 N,交 BD 于 F,连结 AF、CE. (1)求证:四边形 AECF 为平行四边形; (2)当 AECF 为菱形,M 点为 BC 的中点时,求 AB:AE 的值. 24.(本题满分 11 分)如图,已知抛物线与坐标轴分别交于 A(-2,O)、B(2,0)、C(0,-l) 三点,过坐标原点 0 的直线 y=kx 与抛物线交于 M、N 两点.分别过点 C,D(0,-2)作平行于 x 轴的直线 21 ll 、 . (1)求抛物线对应二次函数的解析式; (2)求证以 ON 为直径的圆与直线 1l 相切; (3)求线段 MN 的长(用 k 表示),并证明 M、N 两点到直线 2l 的距离之和等于线段 MN 的 长. 2012 年内蒙古呼和浩特市 8.已知:在等腰梯形 ABCD 中,AD∥BC,AC⊥BD,AD=3,BC=7,则梯形的面积是 ( ) A . 25 B . 50 C . D . 9.已知:M,N 两点关于 y 轴对称,且点 M 在双曲线 上,点 N 在直线 y=x+3 上,设 点 M 的坐标为(a,b),则二次函数 y=﹣abx2+(a+b)x( ) A . 有最大值,最大值为 B.有最大值,最大值为 C.有最小值,最小值为 D . 有最小值,最小值为 10.下列命题中,真命题的个数有( ) ①一个图形无论经过平移还是旋转,变换后的图形与原来图形的对应线段一定平行 ②函数 图象上的点 P(x,y)一定在第二象限 ③正投影的投影线彼此平行且垂直于投影面 ④使得|x|﹣y=3 和 y+x2=0 同时成立的 x 的取值为 . A . 3 个 B . 1 个 C . 4 个 D . 2 个 25.如图,抛物线 y=ax2+bx+c(a<0)与双曲线 相交于点 A,B,且抛物线经过坐标 原点,点 A 的坐标为(﹣2,2),点 B 在第四象限内,过点 B 作直线 BC∥x 轴,点 C为直 线 BC 与抛物线的另一交点,已知直线 BC 与 x 轴之间的距离是点 B 到 y 轴的距离的 4 倍, 记抛物线顶点为 E. (1)求双曲线和抛物线的解析式; (2)计算△ABC 与△ABE 的面积; (3)在抛物线上是否存在点 D,使△ABD 的面积等于△ABE 的面积的 8 倍?若存在,请 求出点 D 的坐标;若不存在,请说明理由. 2012 年贵州省贵阳市 24.(12 分)(2012•贵阳)如果一条直线把一个平面图形的面积分成相等的两部分,我们 把这条直线称为这个平面图形的一条面积等分线. (1)三角形有 _________ 条面积等分线,平行四边形有 _________ 条面积等分线; (2)如图①所示,在矩形中剪去一个小正方形,请画出这个图形的一条面积等分线; (3)如图②,四边形 ABCD 中,AB 与 CD 不平行,AB≠CD,且 S△ABC<S△ACD,过点 A 画出四边形 ABCD 的面积等分线,并写出理由. 25.(12 分)(2012•贵阳)如图,二次函数 y= x2﹣x+c 的图象与 x 轴分别交于 A、B 两 点,顶点 M 关于 x 轴的对称点是 M′. (1)若 A(﹣4,0),求二次函数的关系式; (2)在(1)的条件下,求四边形 AMBM′的面积; (3)是否存在抛物线 y= x2﹣x+c,使得四边形 AMBM′为正方形?若存在,请求出此抛物 线的函数关系式;若不存在,请说明理由. 2012 年乌鲁木齐 7、(2012 新疆乌鲁木齐,7,4 分)为使我市冬季“天更蓝、房更暖”、政府决定实施 “煤改气”供暖改造工程,现甲、乙两工程队分别同时开挖两条 600 米长的管道,所挖 管道长度 y(米)与挖掘时间 x(天)之间的关系如图所示,则下列说法中:①甲队每天 挖 100 米;②乙队开挖两天后,每天挖 50 米;③当 x=4 时,甲、乙两队所挖管道长度 相同;④甲队比乙队提前 2 天完成任务. 正确 的个数有( ) A、1 个 B、2 个 C、3 个 D、4 个 8、(2012 新疆乌鲁木齐,8,4 分)如图是一张足够长的矩形纸条 ABCD,以点 A 所在直 线为折痕,折叠纸条,使点 B 落在边 AD 上,折痕与边 BC 交于点 E;然后将其展平,再 以点 E 所在直线为折痕,使点 A 落在边 BC 上,折痕 EF 交边 AD 于点 F.则∠AFE 的大小 是( ) A、22.50 B、450 C、600 D、67.50 9、(2012 新疆乌鲁木齐,9,4 分)古希腊数学家把 1,3,6,10,15,……叫做三角形 数,则第 16 个三角形数与第 14 个三角形数的差是( ) A、30 B、31 C、32 D、33 10、(2012 新疆乌鲁木齐,10,4 分)如图,AD∥BC,∠D=900,AD=2,BC=5,DC=8. 若在边 DC 上有点 P,使△PAD 与△PBC 相似,则这样的 点 P 有( ) A、1 个 B、2 个 C、3 个 D、4 个 15、(2012 新疆乌鲁木齐,15,4 分)等腰△ABC 内接于半径为 5 的⊙O,点 O 到底边 BC 的距离为 3,则 AB 的长为 . 22、(2012 新疆乌鲁木齐,22,10 分)如图,AB 是⊙O 的直径,C 为圆周上的一点,过 点 C 的直线 MN 满足∠MCA=∠CBA. (1)求证:直线 MN 是⊙O 的切线; (2)过点 A 作 AD⊥MN 于点 D,交⊙O 于点 E,已知 AB=6,BC=3,求阴影部分的面积. 24、(2012 新疆乌鲁木齐,24,12 分)如图,已知点 A(-12,0),B(3,0),点 C 在 y 轴的正半轴上,且∠ACB=900. (1)求点 C 的坐标; (2)求 Rt△ACB 的角平分线 CD 所在直线 l 的 解析式; (3)在 l 上求出满足 S△PBC= 2 1 S△ABC (4)已知点 M 在 l 上,在平面内是否存在点 N,使以 O、C、M、N 为顶点的四边形是 菱形.若存在,请直接写出点 N 的坐标;若不存在.请说明理由. 2012 年兰州市 12.如图,AB 是⊙O 的直径,弦 BC=2cm,F 是弦 BC 的中点,∠ABC=60°.若动点 E 以 2cm/s 的速度从 A 点出发沿着 A→B→A 方向运动,设运动时间为 t(s)(0≤t<3),连接 EF,当△BEF 是直角三角形时,t(s)的值为【 】 A. 7 4 B.1 C. 7 4 或 1 D. 7 4 或 1 或 9 4 13.如图,四边形 ABCD 中,∠BAD=120°,∠B=∠D=90°,在 BC、CD 上分别找一点 M、N,使△AMN 周长最小时,则∠AMN+∠ANM 的度数为【 】 A.130° B.120° C.110° D.100° 14.二次函数 y=ax2+bx+c(a≠0)的图象如图所示,若|ax2+bx+c|=k(k≠0)有两个不相 等的实数根,则 k 的取值范围是【 】 A.k<-3 B.k>-3 C.k<3 D.k>3 19.如图,已知⊙O 是以坐标原点 O 为圆心,1 为半径的圆,∠AOB=45°,点 P 在 x 轴上 运动,若过点 P 且与 OA 平行的直线与⊙O 有公共点,设 P(x,0),则 x 的取值范围 是 . 20.如图,M 为双曲线 y= 3 x 上的一点,过点 M 作 x 轴、y 轴的垂线,分别交直线 y=-x +m 于点 D、C 两点,若直线 y=-x+m 与 y 轴交于点 A,与 x 轴相交于点 B,则 AD•BC 的值为 . 26.如图,Rt△ABC 中,∠ABC=90°,以 AB 为直径的⊙O 交 AC 于点 D,E 是 BC 的中 点,连接 DE、OE. (1)判断 DE 与⊙O 的位置关系并说明理由; (2)若 tanC= 5 2 ,DE=2,求 AD 的长. 27.若 x1、x2 是关于一元二次方程 ax2+bx+c(a≠0)的两个根,则方程的两个根 x1、x2 和 系数 a、b、c 有如下关系:x1+x2=- b a ,x1•x2= c a .把它称为一元二次方程根与系 数关系定理.如果设二次函数 y=ax2+bx+c(a≠0)的图象与 x 轴的两个交点为 A(x1,0),B(x2,0).利用根与系数关系定理可以得到 A、B 连个交点间的距离为: AB=|x1-x2|= 21 2 21 4)( xxxx  = a c a b 42      = 2 2 4 a acb  = || 42 a acb  . 参考以上定理和结论,解答下列问题: 设二次函数 y=ax2+bx+c(a>0)的图象与 x 轴的两个交点 A(x1,0)、B(x2,0),抛物 线的顶点为 C,显然△ABC 为等腰三角形. (1)当△ABC 为直角三角形时,求 b2-4ac 的值; (2)当△ABC 为等边三角形时,求 b2-4ac 的值. 28.如图,Rt△ABO 的两直角边 OA、OB 分别在 x 轴的负半轴和 y 轴的正半轴上,O 为坐 标原点,A、B 两点的坐标分别为(-3,0)、(0,4),抛物线 y= 2 3 x2+bx+c 经过点 B,且顶点在直线 x= 5 2 上. (1)求抛物线对应的函数关系式; (2)若把△ABO 沿 x 轴向右平移得到△DCE,点 A、B、O 的对应点分别是 D、C、E, 当四边形 ABCD 是菱形时,试判断点 C 和点 D 是否在该抛物线上,并说明理由; (3)在(2)的条件下,连接 BD,已知对称轴上存在一点 P 使得△PBD 的周长最小,求 出 P 点的坐标; (4)在(2)、(3)的条件下,若点 M 是线段 OB 上的一个动点(点 M 与点 O、B 不重 合),过点 M 作∥BD 交 x 轴于点 N,连接 PM、PN,设 OM 的长为 t,△PMN 的面 积为 S,求 S 和 t 的函数关系式,并写出自变量 t 的取值范围,S 是否存在最大值? 若存在,求出最大值和此时 M 点的坐标;若不存在,说明理由. 2012 年昆明 23.(本小题 9 分)如图,在平面直角坐标系中,直线 1 23y x   交 x 轴于点 P ,交 y 轴于点 A ,抛物线 21 2y x bx c    的图象过点 ( 1,0)E  ,并与直线相交于 A 、 B 两点. ⑴ 求抛物线的解析式(关系式); ⑵ 过点 A 作 AC AB 交 x 轴于点C ,求点C 的坐标; ⑶ 除点C 外,在坐标轴上是否存在点 M ,使得 MAB 是直角三角形?若存 在,请求出点 M 的坐标,若不存在,请说明理由. 2012 年新疆 24.(12 分)(2012•新疆)如图 1,在直角坐标系中,已知△AOC 的两个顶点坐标分别为 A(2,0),C(0,2). (1)请你以 AC 的中点为对称中心,画出△AOC 的中心对称图形△ABC,此图与原图组 成的四边形 OABC 的形状是 ,请说明理由; (2)如图 2,已知 D( ,0),过 A,C,D 的抛物线与(1)所得的四边形 OABC 的边 BC 交于点 E,求抛物线的解析式及点 E 的坐标; (3)在问题(2)的图形中,一动点 P 由抛物线上的点 A 开始,沿四边形 OABC 的边从 A ﹣B﹣C 向终点 C 运动,连接 OP 交 AC 于 N,若 P 运动所经过的路程为 x,试问:当 x 为 何值时,△AON 为等腰三角形(只写出判断的条件与对应的结果)? 2012 年临沂市 13.(2012 临沂)如图,AB 是⊙O 的直径,点 E 为 BC 的中点,AB=4,∠BED=120°,则 图中阴影部分的面积之和为( ) A.1 B. 3 2 C. 3 D. 2 3 14.(2012 临沂)如图,正方形 ABCD 的边长为 4cm,动点 P、Q 同时从点 A 出发,以 1cm/s 的速度分别沿 A→B→C 和 A→D→C 的路径向点 C 运动,设运动时间为 x(单位: s),四边形 PBDQ 的面积为 y(单位:cm2),则 y 与 x(0≤x≤8)之间函数关系可以用图象 表示为( ) A. B. C. D. 19.(2012 临沂)读一读:式子“1+2+3+4+···+100”表示从 1 开始的 100 个连续自然数的和, 由于式子比较长,书写不方便,为了简便起见,我们将其表示为 100 1n n   ,这里“∑”是求和符 号通过对以上材料的阅读,计算   2012 1 1 1n n n  =__________. 26.(2012 临沂)如图,点 A 在 x 轴上,OA=4,将线段 OA 绕点 O 顺时针旋转 120°至 OB 的位置. (1)求点 B 的坐标; (2)求经过点 A.O、B 的抛物线的解析式; (3)在此抛物线的对称轴上,是否存在点 P,使得以点 P、O、B 为顶点的三角形是等腰 三角形?若存在,求点 P 的坐标;若不存在,说明理由. 2012 年内蒙古包头 12.关于 x 的一元二次方程  2x mx+5 m 5 =0  的两个正实数根分别为 x1,x2,且 2x1+x2=7,则 m 的值是【 】 A.2 B. 6 C. 2 或 6 D . 7。 18.如图,在平面直角坐标系中,点 A 在 x 上,△ABO 是直角三角形,∠ABO=900,点 B 的 坐标为(-1,2),将△ABO 绕原点 O 顺时针旋转 900,得到△Al BlO,则过 A1, B 两点的直 线解析式为 。[来源:Zxxk.Com] 19.如图,直线 1y= x 22  与 x 轴、y 轴分别交于点 A 和点 B ,点 C 在直线 AB 上,且点 C 的纵坐标为一 1 ,点 D 在反比例函数 ky= x 的图象 上 ,CD 平行于 y 轴, OCD 5S 2  则 k 的值为 。 20.如图,将△ABC 纸片的一角沿 DE 向下翻折,使点 A 落在 BC 边上的 A ′点处,且 DE∥ BC ,下列结论: ① ∠AED=∠C;② A D A E DB EC   ;③ BC= 2DE ;④ BD A E A CAD A ES S S     四 形边 。 其中正确结论的个数是 个。 24 .如图,已知 AB 为⊙O 的直径,过⊙O 上的点 C 的切线交 AB 的延长线于点 E , AD⊥EC 于点 D 且交⊙O 于点 F ,连接 BC , CF , AC 。 (1)求证:BC=CF; (2)若 AD=6 , DE=8 ,求 BE 的长; (3)求证:AF + 2DF = AB。 25 .如图,在 Rt△ABC 中,∠C =900,AC = 4cm , BC = 5 cm,点 D 在 BC 上,且 CD = 3 cm ,现有两个动点 P,Q 分别从点 A 和点 B 同时出发,其中点 P 以 1 厘米/秒的速度沿 AC 向终点 C 运动;点 Q 以 1 . 25 厘米/秒的速度沿 BC 向终点 C 运动.过点 P 作 PE∥ BC 交 AD 于点 E ,连接 EQ。设动点运动时间为 t 秒(t > 0 )。 (1)连接 DP ,经过 1 秒后,四边形 EQDP 能够成为平行四边形吗?请说明理由; (2)连接 PQ ,在运动过程中,不论 t 取何值时,总有线段 PQ 与线段 AB 平行。为什 么? (3)当 t 为何值时,△EDQ为直角三角形。 26.已知直线 y = 2x + 4 与 x 轴、y 轴分别交于 A , D 两点,抛物线 21y= x +bx+c2  经过点 A , D ,点 B 是抛物线与 x 轴的另一个交点。 (1)求这条抛物线的解析式及点 B 的坐标; (2)设点 M 是直线 AD 上一点,且 AOM OMDS : S 1 : 3   ,求点 M 的坐标; (3)如果点 C(2,y)在这条抛物线上,在 y 轴的正半轴上是否存在点 P,使△BCP 为 等腰三角形?若存在,请求出点 P 的坐标;若不存在,请说明理由。 2012 年黄石卷 10. (2012 湖北黄石 3 分)如图所示,已知 A 1 1( , y )2 ,B 2(2, y ) 为反比例函数 1y x  图 像上的两点,动点 P (x,0) 在 x 正半轴上运动,当线段 AP 与线段 BP 之差达到最大时,点 P 的坐标是【 】 A. 1( ,0)2 B. (1,0) C. 3( ,0)2 D. 5( ,0)2 16. (2012 湖北黄石 3 分)如图所示,已知 A 点从点(1,0)出发,以每秒1个单位 长的速度沿着 x 轴的正方向运动,经过 t 秒后,以 O、A 为顶点作菱形 OABC,使 B、C 点都在第一象限内,且∠AOC=600,又以 P(0,4)为圆心,PC 为半径的圆恰好与 OA 所在直线相切,则 t= . 23. (2012 湖北黄石 8 分)某楼盘一楼是车库(暂不销售),二楼至二十三楼均为商品 房(对外销售).商品房售价方案如下:第八层售价为 3000 元/米 2,从第八层起每上 升一层,每平方米的售价增加 40 元;反之,楼层每下降一层,每平方米的售价减少 20 元.已知商品房每套面积均为 120 平方米.开发商为购买者制定了两种购房方案: 方案一:购买者先交纳首付金额(商品房总价的 30%),再办理分期付款(即贷款). 方案二:购买者若一次付清所有房款,则享受 8%的优惠,并免收五年物业管理费 (已知每月物业管理费为 a 元) (1)请写出每平方米售价 y(元/米 2)与楼层 x(2≤x≤23,x 是正整数)之间的函数解析 式; (2)小张已筹到 120000 元,若用方案一购房,他可以购买哪些楼层的商品房呢? (3)有人建议老王使用方案二购买第十六层,但他认为此方案还不如不免收物业管理费而 直接享受 9%的优惠划算.你认为老王的说法一定正确吗?请用具体的数据阐明你的看法。 24. (2012 湖北黄石 9 分)如图 1 所示:等边△ABC 中,线段 AD 为其内角平分线,过 D 点的直线 B1C1⊥AC 于 C1 交 AB 的延长线于 B1. (1)请你探究: AC CD AB DB  , 1 1 1 1 AC C D AB DB  是否成立? (2)请你继续探究:若△ABC 为任意三角形,线段 AD 为其内角平分线,请问 AC CD AB DB  一定成立吗?并证明你的判断. (3)如图 2 所示 Rt△ABC 中,∠ACB=900,AC=8, AB 40 3  ,E 为 AB 上一点且 AE=5,CE 交其内角角平分线 AD 与 F.试求 DF FA 的值. 25. (2012 湖北黄石 10 分)已知抛物线 C1 的函数解析式为 2y ax bx 3a(b 0)    , 若抛物线 C1 经过点 (0, 3) ,方程 2ax bx 3a 0   的两根为 1x , 2x ,且 1 2x x 4  。 (1)求抛物线 C1 的顶点坐标. (2)已知实数 x 0 ,请证明: 1x x  ≥ 2 ,并说明 x 为何值时才会有 1x 2x   . (3)若抛物线先向上平移 4 个单位,再向左平移 1 个单位后得到抛物线 C2,设 1A(m, y ) , 2B(n, y ) 是 C2 上的两个不同点,且满足: 00AOB 9  , m 0 , n 0 .请你用含有 m 的表达式表示出△AOB 的面积 S,并求出 S 的最小值及 S 取最小 值时一次函数 OA 的函数解析式。 (参考公式:在平面直角坐标系中,若 1 1P(x , y ) , 2 2Q(x , y ) ,则 P,Q 两点间的距离 2 2 2 1 2 1(x x ) (y y )   ) 2012 年北京市高级中等学校招生考试 数 学 试 卷 学校 姓名 准考证号 一、选择题(本题共 32 分,每小题 4 分) 下面各题均有四个选项,其中只有一个是符合题意的. 1. 9 的相反数是 A. 1 9  B. 1 9 C. 9 D.9 2. 首届中国(北京)国际服务贸易交易会(京交会)于 2012 年 6 月 1 日闭幕,本届 京交会期间签订的项目成交总金额达 60 110 000 000 美元,将 60 110 000 000 用科学记数法表示应为 A. 96.011 10 B. 960.11 10 C. 106.011 10 D. 110.6011 10 3. 正十边形的每个外角等于 A.18 B.36 C. 45 D. 60 4. 右图是某个几何体的三视图,该几何体是 A.长方体 B.正方体 C.圆柱 D.三棱柱 5. 班主任王老师将 6 份奖品分别放在 6 个完全相同的不透明礼盒中,准备将它们奖 给小英等 6 位获“爱集体标兵”称号的同学.这些奖品中 3 份是学习文具,2 份 是科普读物,1 份是科技馆通票.小英同学从中随机取一份奖品,恰好取到科普 读物的概率是 A. 1 6 B. 1 3 C. 1 2 D. 2 3 6. 如图,直线 AB ,CD 交于点O ,射线OM 平分 AOC ,若 76BOD   , 则 BOM 等于 A.38 B.104 C.142 D.144 7. 某课外小组的同学们在社会实践活动中调查了 20 户家庭某月的用电量,如下表所 示: 用电量 120 140 160 180 200 (度) 户数 2 3 6 7 2 则这 20 户家庭该月用电量的众数和中位数分别是 A.180,160 B.160,180 C.160,160 D.180,180 8. 小翔在如图 1 所示的场地上匀速跑步,他从点 A 出发,沿箭头所示方向经过点 B 跑到点C ,共用时 30 秒.他的教练选择了一个固定的位置观察小翔的跑步过 程.设小翔跑步的时间为t (单位:秒),他与教练的距离为 y (单位:米),表 示 y 与t 的函数关系的图象大致如图 2 所示,则这个固定位置可能是图 1 中的 A.点 M B.点 N C.点 P D.点Q 二、填空题(本题共 16 分,每小题 4 分) 9. 分解因式: 2 6 9mn mn m   . 10.若关于 x 的方程 2 2 0x x m   有两个相等的实数根,则 m 的值是 . 11.如图,小明同学用自制的直角三角形纸板 DEF 测量树的 高度 AB ,他调整自己的位置,设法使斜边 DF 保持水 平,并且边 DE 与点 B 在同一直线上.已知纸板的两条直 角边 40cmDE  , 20cmEF  ,测得边 DF 离地面的高度 1.5mAC  , 8mCD  , 则树高 AB  m . 12.在平面直角坐标系 xOy 中,我们把横 、纵坐标 都是整数的点叫做整点.已知点  0 4A , ,点 B 是 x 轴正半轴上的整点,记 AOB△ 内部(不包括 边界)的整点个数为 m .当 3m  时,点 B 的横 坐标的所有可能值是 ;当点 B 的横坐标为 4n ( n 为正整数)时, m  (用含 n 的代数式表示.) 三、解答题(本题共 30 分,每小题 5 分) 13.计算:   1 0 1π 3 18 2sin 45 8          . 14.解不等式组: 4 3 4 2 1. x x x x       , 15.已知 02 3 a b ≠ ,求代数式  2 2 5 2 24 a b a ba b    的值. 16.已知:如图,点 E A C, , 在同一条直线上, AB CD∥ , AB CE AC CD , . 求证: BC ED . 17.如图,在平面直角坐标系 xOy 中,函数  4 0y xx   的图象与一次函 数 (1)求一次函数的解析式; (2)设一次函数 y kx k  的图象与 y 轴交于点 B ,若 P 是 x 轴上一点, 且满足 PAB△ 的面积是 4,直接写出点 P 的坐标. 18.列方程或方程组解应用题: 据林业专家分析,树叶在光合作用后产生的分泌物能够吸附空气中的一些悬浮颗 粒物,具有滞尘净化空气的作用.已知一片银杏树叶一年的平均滞尘量比一片国槐树 叶一年的平均滞尘量的 2 倍少 4 毫克,若一年滞尘 1000 毫克所需的银杏树叶的片数与 一年滞尘 550 毫克所需的国槐树叶的片数相同,求一片国槐树叶一年的平均滞尘量. 四、解答题(本题共 20 分,每小题 5 分) 19.如图,在四边形 ABCD 中,对角线 AC BD, 交于点 E , 90 45 30 2BAC CED DCE DE         , , , , 2 2BE  . 求CD 的长和四边形 ABCD 的面积. 20.已知:如图, AB 是 O⊙ 的直径,C 是 O⊙ 上一点,OD BC⊥ 于点 D ,过点C 作 O⊙ 的切线,交OD 的延长线于点 E ,连 结 BE . (1)求证: BE 与 O⊙ 相切; (2)连结 AD 并延长交 BE 于点 F ,若 9OB  , 2sin 3ABC  ,求 BF 的长. 21.近年来,北京市大力发展轨道交通,轨道运营里程大幅增加,2011 年北京市又调 整修订了 2010 至 2020 年轨道交通线网的发展规划.以下是根据北京市轨道交通 指挥中心发布的有关数据制作的统计图表的一部分. 北京市轨道交通已开通线路 相关数据统计表(截至 2010 年底) 开通时间 开通线路 运营里程 (千米) 1971 1 号线 31 1984 2 号线 23 2003 13 号线 41 八通线 19 2007 5 号线 28 2008 8 号线 5 10 号线 25 请根据以上信息解答下列问题: (1)补全条形统计图并在图中标明相应数据; (2)按照 2011 年规划方案,预计 2020 年北京市轨道交通运营里程将达到多少千 米? (3)要按时完成截至 2015 年的轨道交通规划任务,从 2011 到 2015 这 4 年中,平 均每年需新增运营里程多少千米? 22.操作与探究: (1)对数轴上的点 P 进行如下操作:先把点 P 表示的数乘以 1 3 ,再把所得数对应的 点向右平移 1 个单位,得到点 P 的对应点 P . 点 A B, 在数轴上,对线段 AB 上的每个点进行上述操作后得到线段 A B  ,其中 点 A B, 的对应点分别为 A B , .如图 1,若点 A 表示的数是 3 ,则点 A 表 示的数是 ;若点 B 表示的数是 2,则点 B 表示的数是 ;已知线段 AB 上的点 E 经过上述操作后得到的对应点 E 与点 E 重合,则点 E 表示的数 是 ; (2)如图 2,在平面直角坐标系 xOy 中,对正方形 ABCD 及其内部的 每个点进行如下操作:把每 个点的横、纵坐标都乘以同一种实数 a ,将得到 的点先向右平移 m 个单位,再向上平移 n 个单位( 0 0m n , ),得到正方 形 A B C D    及其内部的点,其中点 A B, 的对应点分别为 A B , 。已知正方 形 ABCD 内部的一个点 F 经过上述操作后得到的对应点 F 与点 F 重合,求 点 F 的坐标。 五、解答题(本题共 22 分,第 23 题 7 分,第 24 题 7 分,第 25 题 8 分) 23.已知二次函数 2 3( 1) 2( 2) 2y t x t x     在 0x  和 2x  时的函数值相等。 (4) 求二次函数的解析式; (5) 若一次函数 6y kx  的图象与二次函数的 图象都经过点 ( 3 )A m , ,求 m 和 k 的值; (6) 设二次函数的图象与 x 轴交于点 B C, (点 B 在点 C 的左侧),将二次函数的图象在点 B C, 间的部分(含点 B 和点 C )向左平移 ( 0)n n  个单位后得到的图象记为G ,同时将(2)中得 到的直线 6y kx  向上平移 n 个单位。请结合图象回答:当平移后的直线 与图象G 有公共点时, n 的取值范围。 24.在 ABC△ 中, BA BC BAC   , , M 是 AC 的中点, P 是线段 BM 上的动点, 将线段 PA 绕点 P 顺时针旋转 2 得到线段 PQ 。 (1) 若    且点 P 与点 M 重合(如图 1),线段CQ 的延长线交射线 BM 于点 D ,请补全图形,并写出 CDB 的度数; (2) 在图 2 中,点 P 不与点 B M, 重合,线段CQ 的延长线与射线 BM 交于点 D , 猜想 CDB 的大小(用含  的代数式表示),并加以证明; (3) 对于适当大小的  ,当点 P 在线段 BM 上运动到某一位置(不与点 B , M 重 合)时,能使得线段CQ 的延长线与射线 BM 交于点 D ,且 PQ QD ,请直 接写出  的范围。 25.在平面直角坐标系 xOy 中,对于任意两点 1 1 1( )P x y, 与 2 2 2( )P x y, 的“非常距离”,给出如下定义: 若 1 2 1 2| | | |x x y y ≥ ,则点 1P 与点 2P 的“非常距离”为 1 2| |x x ; 若 1 2 1 2| | | |x x y y   ,则点 1P 与点 2P 的“非常距离”为 1 2| |y y . 例如:点 1(1 2)P , ,点 2 (3 5)P , ,因为|1 3| | 2 5|   ,所以点 1P 与 点 2P 的“非常距离”为| 2 5| 3  ,也就是图 1 中线段 1PQ 与线段 2P Q 长度的较大值 (点Q 为垂直于 y 轴的直线 1PQ 与垂直于 x 轴的直线 2P Q 的交点)。 (1)已知点 1( 0)2A  , , B 为 y 轴上的一个动点, ①若点 A 与点 B 的“非常距离”为 2,写出一个满足条件的点 B 的坐标; ②直接写出点 A 与点 B 的“非常距离”的最小值; (2)已知C 是直线 3 34y x  上的一个动点, ①如图 2,点 D 的坐标是(0,1),求点C 与点 D 的“非常距离”的最小值及 相应的点C 的坐标; ②如图 3, E 是以原点O 为圆心,1 为半径的圆上的一个动点,求点C 与点 E 的 “非常距离” 的最小值及相应的点 E 和点C 的坐标。 1. 在下列代数式中,次数为三的单项式是( ) A. 2xy B. 3 3x y C. 3x y D. 3xy 2. 数据 5 , 7 , 5 ,8 , 6,13 , 5 的中位数是( ) A. 5 B. 6 C. 7 D.8 3. 不等式组 2 6 2 0 x x      的解集是( ) A. 3x   B. 3x   C. 2x  D. 2x  4. 在下列根式中,二次根式 a b 的有理化因式是( ) A. a b B. a b C. a b D. a b 5. 在下列图形中,为中心对称图形的是( ) A.等腰梯形 B.平行四边形 C.正五边形 D.等腰三角形 6. 如果两圆的半径分别为 6 和 2 ,圆心距为 3 ,那么这两圆的位置关系是( ) A.外离 B.相切 C.相交 D.内含 7. 计算: 1 12   . 8. 因式分解: xy x  . 9. 已知正比例函数 ( 0)y kx k  ,点 (2 , 3) 在函数上,则 y 随 x 的增大而 (选填“增 大”或“减小”). 10. 方程 1 2x   的根是 . 11. 如果关于 x 的方程 2 6 0x x c   ( c 为常数)没有实数根,那么 c 的取值范围是 . 12. 将抛物线 2y x x  向下平移 2 个单位,所得的新抛物线的解析式为 . 13. 布袋中装有个 3 红球和 6 个白球,它们除颜色外其他都相同,如果从布袋中随机摸出一个球,那么 所摸到的球恰好为红球的概率是 . 14. 某校 500 名学生参加生命安全知识测试,测试分数均大于或等于 60 且小于100 ,分数段的频率分 布情况如表所示,其中每个分数段可包括最小值,不包括最大值,结合表格的信息,可得测试分数 在80 90 分数段的学生有 名. 15. 如图,已知梯形 ABCD , AD // BC , 2BC AD ,若 AD a  , AB b  ,那么 AC  (用 a  , b  表示). 16. 在 ABC 中,点 D , E 分别在 AB , AC 上, AED B   ,如 果 2AE  , ADE 的面积为 4 ,四边形 BCED 的面积为 5 ,那么 边 AB 的长为 . 分数段 60~70 70~80 80~90 90~100 频率 0.2 0.25 0.25 D CB A A B D C E 17. 我们把两个三角形的中心之间的距离叫做重心距,在同一平面内有两个边长相等的等边三角形,如 果当它们的一边重合时重心距为 2 ,那么当它们的一对角成对顶角时重心距为 . 18. 如图所示, Rt ABC 中, 90C   , 1BC  , 30A   , 点 D 为边 AC 上的一动点,将 ABD 沿直线 BD 翻折,点 A 落 在点 E 处,如果 DE AD 时,那么 DE  . 19. 计算: 11 2 21 1 2( 3 1) 32 22 1           20. 解方程: 2 6 1 3 9 3 x x x x     21. 如图所示,在 Rt ABC , 90ACB   , D 是边 AB 的中点, BE CD ,垂足为 E ,已知 15AC  , 3 5cosA  . ①求线段 CD 的长; ②求 sin DBE 的值. C B D A E D BC A 22. 某工厂生产一种产品,当生产数量至少为10 吨,但不超过 50 吨时,每吨的成本 y 万元与生产数量 x 吨的函数关系式如图所示. ①求 y 与 x 的函数关系式,并写出其定义域; ②当生产这种产品的总成本为 280 万元时,求该产品的生产数量. (注:总成本=每吨的成本×生产数量) 23. 如图所示,在菱形 ABCD 中,点 E 、 F 分别在 BC 、 CD 上, BAF DAE   , AE 与 BD 相交于点 G . ①求证: BE DF ; ②当 DF AD FC DF  时,求证:四边形 BEFG 是平行四边形. 24. 如图,在平面直角坐标系中,二次函数 2 6y ax x c   过点 (4A , 0) 和 ( 1B  , 0) ,并 与 y 轴交于点 C ,点 D 在线段OC 上,设 DO t ,点 E 在第二象限,且 90ADE   , 1 2tan DAE  , EF OD 于 F . ①求二次函数的解析式; ②用含 t 的代数式表示 EF 和OF 的长; ③当 ECA CAO   时,求 t 的值. x D FE OB A C y 10 50 10 x O y 6 E D CB A FG 25. 已知扇形 AOB 中, 90AOB   , 2OA OB  , C 为 AB 上的动点,且不与 A 、 B 重 合,OE AC 于 E ,OD BC 于 D . ①若 1BC  ,求 OD 的长; ②在 DOE 中,是否存在长度保持不变的边,若存在,求出该边的长; 若不存在,请说明理由; ③设 BD x , DOE 的面积为 y ,求 y 与 x 的函数关系式及定义域. AO B CD E 参考答案 (7) A (8) B (9) C (10) C (11) B (12) D (13) 1 2 (14) ( 1)x y  (15) 减小 (16) 3x  (17) 9c  (18) 2 2y x x   (19) 1 3 (20) 150 (21) 2a b  (22) 3 (23) 4 (24) 3 1 第一小问: 第二小问: 第三小问: (24) 第一小问解析: 第二小问解析: 第三小问解析: 2012 年中考数学精析系列——南京卷 (本试卷满分 120 分,考试时间 120 分钟) 一、选择题(本大题共 6 小题,每小题 2 分,共 12 分) 1、(2012 江苏南京 2 分)下列四个数中,负数是【 】 A. -2 B.  2-2 C. - 2 D.  2-2 【答案】C。 【考点】实数的运算,正数和负数,绝对值的性质,有理数的乘方的定义,算术平方 根。 【分析】根据绝对值的性质,有理数的乘方的定义,算术平方根对各选项分析判断后 利用排除法求解: A、|-2|=2,是正数,故本选项错误;B、 2-2 =4,是正数,故本选项错 误; C、- 2 <0,是负数,故本选项正确;D、  2-2 = 4 =2,是正数,故本 选项错误。 故选 C。 2、(2012 江苏南京 2 分)PM 2.5 是指大气中直径小于或等于 0.0000025 m 的颗粒 物,将 0.0000025 用科学记数法表示为【 】 A. -50.25 10 B. -60.25 10 C. -52.5 10 D. -62.5 10 【答案】C。 【考点】科学记数法。 【分析】根据科学记数法的定义,科学记数法的表示形式为 a×10n,其中 1≤|a|<10,n 为 整数,表示时关键要正确确定 a 的值以及 n 的值。在确定 n 的值时,看该数是大于或等于 1 还是小于 1。当该数大于或等于 1 时,n 为它的整数位数减 1;当该数小于 1 时,-n 为它 第一个有效数字前 0 的个数(含小数点前的 1 个 0)。0.0000025 第一个有效数字前有 6 个 0,从而 0.0000025= -52.5 10 。故选 C。 3、(2012 江苏南京 2 分)计算   3 22 2a a 的结果是【 】 A. a B. 2a C. 3a D. 4a 【答案】B。 【考点】整式的除法,幂的乘方,同底幂的除法。 【分析】根据幂的乘方首先进行化简,再利用同底数幂的除法的运算法则计算后直接 选取答案:    3 22 2 6 4 2= =a a a a a  ,故选 B。 4、(2012 江苏南京 2 分)12 的负的平方根介于【 】 A. -5 和-4 之间 B. -4 与-3 之间 C. -3 与-2 之间 D. -2 与-1 之间 【答案】B。 【考点】估算无理数的大小,不等式的性质。 【分析】∵9 < 12 < 16,∴ 9 12 16< < 。∴- 16 - 12 - 9< < ,即 -4 - 12 -3< < 。故选 B。 5、(2012 江苏南京 2 分)若反比例函数 ky x  与一次函数 y x 2  的图像没有..交 点,则 k 的值可以是【 】 A. -2 B. -1 C. 1 D. 2 【答案】A。 【考点】反比例函数与一次函数的交点问题,一元二次方程的判别式。 【分析】把两函数的解析式组成方程组,再转化为求一元二次方程解答问题,求出 k 的取值范围,找出符合条件的 k 的值即可: ∵反比例函数 ky x  与一次函数 y=x+2 的图象没有交点, ∴ k y x y x 2      ① ② 无解,即 k =x 2x  无解,整理得 x2+2x-k=0, ∴△=4+4k<0,解得 k<-1。 四个选项中只有-2<-1,所以只有 A 符合条件。故选 A。 6、(2012 江苏南京 2 分)如图,菱形纸片 ABCD 中,∠A=600,将纸片折叠,点 A、D 分别落在 A’、D’处,且 A’D’经过 B,EF 为折痕,当 D’F  CD 时, CF FD 的值为 【 】 A. 3 1 2  B. 3 6 C. 2 3 1 6  D. 3 1 8  【答案】A。 【考点】翻折变换(折叠问题),菱形的性质,平行的性质,折叠的性质,锐角三角函数 定义,特殊角的三角函数值。 二、填空题(本大题共 10 小题,每小题 2 分,共 20 分) 7、(2012 江苏南京 2 分)使 1 x 有意义的 x 的取值范围是 ▲ 【答案】 x 1 。 【考点】二次根式有意义的条件。 【分析】根据二次根式被开方数必须是非负数的条件,要使 1 x 在实数范围内有意义, 必须1 x 0  ,即 x 1 。 8、(2012 江苏南京 2 分)计算 2 2 2  的结果是 ▲ 【答案】 2+1。 【考点】分母有理化。 【分析】分子分母同时乘以 2 即可进行分母有理化: 2 2 2 2+2= = 2+122  。 9、(2012 江苏南京 2 分)方程 3 2 0x x 2   的解是 ▲ 【答案】x=6。 【考点】解分式方程。 【分析】方程最简公分母为:  x x 2 。故方程两边乘以  x x 2 ,化为整式方程后求 解,并代入检验即可得出方程的根: 去分母得:3(x-2)-2x=0, 去括号得:3x-6-2x=0, 整理得:x=6, 经检验得 x=6 是方程的根。 10、(2012 江苏南京 2 分)如图, 1 、 2 、 3 、 4 是五边形 ABCDE 的 4 个外 角,若 2A 1 0   ,则 1 2 3 4        ▲ 【答案】300。 【考点】多边形外角性质,补角定义。 【分析】由题意得,∠A 的外角=180°-∠A=60°, 又∵多边形的外角和为 360°,∴∠1+∠2+∠3+∠4=360°-∠A 的外角 =300°。 11、(2012 江苏南京 2 分)已知一次函数 y kx k 3   的图像经过点(2,3),则 k 的值为 ▲ 【答案】2。 【考点】直线上点的坐标与方程的关系。 【分析】根据点在直线上,点的坐标满足方程的关系,将(2,3)代入 y kx k 3   ,得 3 2k k 3   ,解得,k=2。 12、(2012 江苏南京 2 分)已知下列函数 ① 2y x ② 2y x  ③  2y x 1 2   , 其中,图象通过平移可以得到函数 2y x 2x 3   的图像的有 ▲ (填写所有正确选 项的序号) 【答案】①③。 【考点】二次函数图象与平移变换。 【分析】把原式化为顶点式的形式,根据函数图象平移的法则进行解答: ∵  22y x 2x 3= x+1 4    ∴由函数图象平移的法则可知,进行如下平移变换 ①    2 21 42y x y x+1 y= x+1 4    向左平移 个单位 向下平移 个单位 ,故①正 确。 ② 2y x 2x 3   的图象开口向上, 2y x  的图象开口向下,不能通过平移 得到,故②错误。 ③      2 2 22 6y x 1 2 y x+1 2 y= x+1 4       向左平移 个单位 向下平移 个单位 ,,故③ 正确。 ∴图象通过平移可以得到函数 2y x 2x 3   的图像的有① 2y x , ③  2y x 1 2   。 13、(2012 江苏南京 2 分)某公司全体员工年薪的具体情况如下表: 年薪/万元 30 14 9 6 4 3.5 3 员工数/人 1 1 1 2 7 6 2 则所有员工的年薪的平均数比中位数多 ▲ 万元。 【答案】2。 【考点】中位数,加权平均数。 【分析】根据加权平均数的定义求出员工的工资平均数: (30+14+9+6×2+4×7+3.5×6+3×2)÷20 =120÷20 =6。 中位数是一组数据从小到大(或从大到小)重新排列后,最中间的那个数(最中 间两个数的平均数)。因此这 20 个员工的年薪的中位数是第 10 和 11 人的工资的平均数, 工资均为 4,∴中位数为:4。 ∴该公司全体员工年薪的平均数比中位数多 6-4=2 万元。 14、(2012 江苏南京 2 分)如图,将 45的∠AOB 按图摆放在一把刻度尺上,顶点 O 与尺下沿的端点重合,OA 与尺下沿重合,OB 与尺上沿的交点 B 在尺上的读数为 2cm,若按相同的方式将37 的∠AOC 放置在该尺上,则 OC 与尺上沿的交点 C 在尺上 的读数约为 ▲ cm (结果精确到 0.1 cm,参考数据:sin37 0.60  ,cos37 0.80  , tan37 0.75  ) 【答案】2.7。 【考点】解直角三角形的应用,等腰直角三角形的性质,矩形的性质,锐角三角函数定 义,特殊角的三角函数值。 【分析】过点 B 作 BD⊥OA 于 D,过点 C 作 CE⊥OA 于 E。 在△BOD 中,∠BDO=90°,∠DOB=45°, ∴BD=OD=2cm。 ∴CE=BD=2cm。 在△COE 中,∠CEO=90°,∠COE=37°, ∵ CEtan37 0.75OE    ,∴OE≈2.7cm。 ∴OC 与尺上沿的交点 C 在尺上的读数约为 2.7cm。 15、(2012 江苏南京 2 分)如图,在平行四边形 ABCD 中,AD=10cm,CD=6cm, E 为 AD 上一点,且 BE=BC,CE=CD,则 DE= ▲ cm 【答案】2.5。 【考点】平行四边形的性质,平行的性质,等腰三角形的性质,相似三角形的判定和 性质。 【分析】∵四边形 ABCD 是平行四边形,AD=10cm,CD=5cm, ∴BC=AD=10cm,AD∥BC,∴∠2=∠3。 ∵BE=BC,CE=CD, ∴BE=BC=10cm,CE=CD=5cm,∠1=∠2,∠3=∠D。 ∴∠1=∠2=∠3=∠D。∴△BCE∽△CDE。∴ BC CE CD DE  ,即10 5 5 DE  ,解得 DE=2.5cm。 16、(2012 江苏南京 2 分)在平面直角坐标系中,规定把一个三角形先沿 x 轴翻 折,再向右平移两个单位称为一次变换,如图,已知等边三角形 ABC 的顶点 B、C 的 坐标分别是,(-1,-1),(-3,-1),把三角形 ABC 经过连续 9 次这样的变换得到 三角形 A’B’C’,则点 A 的对应点 A’的坐标是 ▲ 【答案】(16,1+ 3 )。 【考点】分类归纳(图形的变化类),翻折变换(折叠问题),坐标与图形性质,等边三 角形的性质,锐角三角函数定义,特殊角的三角函数值。 【分析】先由△ABC 是等边三角形,点 B、C 的坐标分别是(-1,1)、(-3,- 1),求得点 A 的坐标;再寻找规律,求出点 A 的对应点 A′的坐标: 如图,作 BC 的中垂线交 BC 于点 D,则 ∵△ABC 是等边三角形,点 B、C 的坐标分别是(-1,1)、(-3,-1), ∴BD=1, 0AD BD tan60 3   。∴A(—2, 1 3  )。 根据题意,可得规律:第 n 次变换后的点 A 的对应点的坐标:当 n 为奇数时为 (2n-2,1+ 3 ),当 n 为偶数时为(2n-2, 1 3  )。 ∴把△ABC 经过连续 9 次这样的变换得到△A′B′C′,则点 A 的对应点 A′的坐标 是:(16,1+ 3 )。 三、解答题(本大题共 11 题,共 88 分) 17、(2012 江苏南京 6 分)解方程组 x 3y 1 3x 2y 8       【答案】解: x 3y 1 3x 2y 8       ① ② , 由①得 x=-3y-1③, 将③代入②,得 3(-3y-1)-2y=8,解得:y=-1。 将 y=-1 代入③,得 x=2。 ∴原方程组的解是 x 2 y 1     。 【考点】解二元一次方程组。 【分析】解二元一次方程组的解题思想是用代入法或加减法消元,化为一元一次方程 求解。本题易用代入法求解。先由①表示出 x,然后将 x 的值代入②,可得出 y 的值, 再代入①可得出 x 的值,继而得出了方程组的解。 18、(2012 江苏南京 9 分)化简代数式 2 2 x 1 x 1 x 2x x   ,并判断当 x 满足不等式组   x 2 1 2 x 1 6      时该代数式的符号。 19、(2012 江苏南京 8 分)如图,在直角三角形 ABC 中,∠ABC=90°,点 D 在 BC 的延长线上,且 BD=AB,过 B 作 BE  AC,与 BD 的垂线 DE 交于点 E, (1)求证:△ABC≌△BDE (2)三角形 BDE 可由三角形 ABC 旋转得到,利用尺规作出旋转中心 O(保留作图痕 迹,不写作法) 20、(2012 江苏南京 8 分)某中学七年级学生共 450 人,其中男生 250 人,女生 200 人。该校对七年级所有学生进行了一次体育测试,并随即抽取了 50 名男生和 40 名女生的测试成绩作为样本进行分析,绘制成如下的统计表: 成绩 划记 频数 百分比 不及格 9 10% 及格 18 20% 良好 36 40% 优秀 27 30% 合计 90 90 100% (1)请解释“随即抽取了 50 名男生和 40 名女生”的合理性; (2)从上表的“频数”、“百分比”两列数据中选择一列,用适当的统计图表示; (3)估计该校七年级学生体育测试成绩不合格的人数。 【答案】解:(1)∵ 90250 50450   (人), 90200 40450   (人), ∴该校从七年级学生中随机抽取 90 名学生,应当抽取 50 名男生和 40 名女生。 (2)选择扇形统计图,表示各种情况的百分比,图形如下: (3)450×10%=45(人)。 答:估计该校七年级学生体育测试成绩不及格 45 人. 【考点】频数(率)分布表,抽样调查的可靠性,频数、频率和总量的关系,用样本 估计总体,扇形统计图或条形统计图。 【分析】(1)所抽取男生和女生的数量应该按照比例进行,根据这一点进行说明即 可。 (2)可选择扇形统计图,表示出各种情况的百分比,也可选择条形统计图, 答案不唯一。 (3)根据用样本估计总体的方法即可得出答案。 21、(2012 江苏南京 7 分)甲、乙、丙、丁 4 名同学进行一次羽毛球单打比赛,要 从中选 2 名同学打第一场比赛,求下列事件的概率。 (1)已确定甲打第一场,再从其余 3 名同学中随机选取 1 名,恰好选中乙同学; (2)随机选取 2 名同学,其中有乙同学. 【答案】解:(1)已确定甲打第一场,再从其余 3 名同学中随机选取 1 名,恰好选中 乙同学的概率是 1 3 。 (2)从甲、乙、丙、丁 4 名同学中随机选取 2 名同学,所有等可能出现 的结果有:(甲、乙)、(甲、丙)、(甲、丁)、(乙、丙)、(乙、丁)、 (丙、丁),共有 6 种, 所有的结果中,满足“随机选取 2 名同学,其中有乙同学”(记为事 件 A)的结果有 3 种:(甲、乙)、(乙、丙)、(乙、丁)。 ∴P(A)= 3 1=6 2 。 【考点】列举法,概率。 【分析】(1)由一共有 3 种等可能性的结果,其中恰好选中乙同学的有 1 种,即可求 得答案。 (2)先用列举法求出全部情况的总数,再求出符合条件的情况数目,二者的 比值就是其发生的概率。 22、(2012 江苏南京 8 分)如图,梯形 ABCD 中,AD//BC,AB=CD,对角线 AC、 BD 交于点 O,AC  BD,E、F、G、H 分别为 AB、BC、CD、DA 的中点 (1)求证:四边形 EFGH 为正方形; (2)若 AD=2,BC=4,求四边形 EFGH 的面积。 【答案】(1)证明:在△ABC 中,E、F 分别是 AB、BC 的中点,EF= 1 2 AC。 同理 FG= 1 2 BD,GH= 1 2 AC,HE= 1 2 BD。 ∵在梯形 ABCD 中,AB=DC,∴AC=BD。 ∴EF=FG=GH=HE,∴四边形 EFGH 是菱形。 设 AC 与 EH 交于点 M, 在△ABD 中,E、H 分别是 AB、AD 的中点,则 EH∥BD,同理 GH∥AC。 又∵AC⊥BD,∴∠BOC=90°。∴∠EHG=∠EMC=90°。 ∴四边形 EFGH 是正方形。 (2)解:连接 EG。 在梯形 ABCD 中,∵E、F 分别是 AB、DC 的中点, ∴ 1EG AD BC 32   ( ) 。 在 Rt△EHG 中,∵EH2+GH2=EG2,EH=GH, ∴ 2 9EH 2  ,即四边形 EFGH 的面积为 9 2 。 【考点】三角形中位线定理,等腰梯形的性质,正方形的判定,梯形中位线定理,勾 股定理。 【分析】(1)先由三角形的中位线定理求出四边相等,然后由 AC⊥BD 入手,进行正 方形的判断。 (2)连接 EG,利用梯形的中位线定理求出 EG 的长,然后结合(1)的结论 求出 2 9EH 2  ,也即得出了正方形 EHGF 的面积。 23、(2012 江苏南京 7 分)看图说故事。 请你编一个故事,使故事情境中出现的一对变量 x、y 满足图示的函数关系式,要求: ①指出 x 和 y 的含义;②利用图中数据说明这对变量变化过程的实际意义,其中需设 计“速度”这个量 【答案】解: ①该函数图象表示小明骑车离出发地的路程 y(单位:km)与他所用的 时间 x(单位:min)的关系。 ②小明以 400m/min 的速度匀速骑了 5min,在原地休息了 6min,然后 以 500m/min 的速度匀速骑车回出发地。(本题答案不唯一) 【考点】开放型问题,函数的图象。 【分析】①结合实际意义得到变量 x 和 y 的含义;②由于函数须涉及“速度”这个量,只 要叙述清楚时间及相应的路程,体现出函数的变化即可。 24、(2012 江苏南京 8 分)某玩具由一个圆形区域和一个扇形区域组成,如图,在 1O 和扇形 2O CD 中, 1O 与 2O C 、 2O D 分别相切于 A、B, 2CO D 60   ,E、F 事直线 1 2O O 与 1O 、扇形 2O CD 的两个交点,EF=24cm,设 1O 的半径为 x cm, ① 用含 x 的代数式表示扇形 2O CD 的半径; ② 若 1O 和扇形 2O CD 两个区域的制作成本分别为 0.45 元 2/cm 和 0.06 元 2/cm ,当 1O 的半径为多少时,该玩具成本最小? 【答案】解:(1)连接 O1A。 ∵⊙O1 与 O2C、O2D 分别切一点 A、B, ∴O1A⊥O2C,O2E 平分∠CO2D。 ∵ 2CO D 60   ,∴∠AO2O1= 1 2 ∠CO2D=30°。 在 Rt△O1AO2 中, 1 2 1 1 2 AOsin AO O O O   ,∴O1O2=A O1 sin∠AO2O1 =x sin30° =2x。 ∵EF=24cm,∴FO2=EF-EO1-O1O2=24-3x,即扇形 O2CD 的半 径为(24-3x)cm。 (2)设该玩具的制作成本为 y 元,则    2 2 2360 60 24 3xy 0.45 x 0.06 0.9 x 7.2 x 28.8360              20.9 x 4 14.4   ( ) 。 ∴当 x=4 时,y 的值最小。 答:当⊙O1 的半径为 4cm 时,该玩具的制作成本最小。 【考点】切线的性质,锐角三角函数定义,扇形面积的计算,二次函数的最值。 【分析】(1)连接 O1A.由切线的性质知∠AO2O1= 1 2 ∠CO2D=30°;然后在 Rt△O1AO2 中利用锐角三角函数的定义求得 O1O2=2x;最后由图形中线段间的和差关 系求得扇形 O2CD 的半径 FO2。 (2)设该玩具的制作成本为 y 元,则根据圆形的面积公式和扇形的面积公式 列出 y 与 x 间的函数关系,然后利用二次函数的最值即可求得该玩具的最小制作成 本。 25、(2012 江苏南京 8 分)某汽车销售公司 6 月份销售某厂家的汽车,在一定范围 内,每部汽车的进价与销售有如下关系,若当月仅售出 1 部汽车,则该部汽车的进价 为 27 万元,每多售一部,所有出售的汽车的进价均降低 0.1 万元/部。月底厂家根据 销售量一次性返利给销售公司,销售量在 10 部以内,含 10 部,每部返利 0.5 万元, 销售量在 10 部以上,每部返利 1 万元。 ① 若该公司当月卖出 3 部汽车,则每部汽车的进价为 万元; ② 如果汽车的销售价位 28 万元/部,该公司计划当月盈利 12 万元,那么要卖出多少 部汽车?(盈利=销售利润+返利) 【答案】解:(1)26.8。 (2)设需要售出 x 部汽车, 由题意可知,每部汽车的销售利润为:28-[27-0.1(x-1)]= (0.1x+0.9)(万元), 当 0≤x≤10,根据题意,得 x·(0.1x+0.9)+0.5x=12,整理,得 x2+14x-120=0, 解这个方程,得 x1=-20(不合题意,舍去),x2=6。 当 x>10 时,根据题意,得 x·(0.1x+0.9)+x=12,整理,得 x2 +19x-120=0, 解这个方程,得 x1=-24(不合题意,舍去),x2=5。 ∵5<10,∴x2=5 舍去。 答:要卖出 6 部汽车。 【考点】一元二次方程的应用。 【分析】(1)根据若当月仅售出 1 部汽车,则该部汽车的进价为 27 万元,每多售出 1 部,所有售出的汽车的进价均降低 0.1 万元/部,得出该公司当月售出 3 部汽车时, 则每部汽车的进价为:27-0.1×2=26.8。, (2)利用设需要售出 x 部汽车,由题意可知,每部汽车的销售利润,根据当 0≤x≤10,以及当 x>10 时,分别讨论得出即可。 26、((2012 江苏南京 9 分)“?”的思考 下框中是小明对一道题目的解答以及老师的批阅。 我的结果也正确 小明发现他解答的结果是正确的,但是老师却在他的解答中划了一条横线,并打开了 一个“?” 结果为何正确呢? (1)请指出小明解答中存在的问题,并补充缺少的过程: 变化一下会怎样…… (2)如图,矩形 A′B′C′D′在矩形 ABCD 的内部,AB∥A′B′,AD∥A′D′,且 AD: AB=2:1,设 AB 与 A′B′、BC 与 B′C′、CD 与 C′D′、DA 与 D′A′之间的距离分别为 a、 b、c、d,要使矩形 A′B′C′D′∽矩形 ABCD,a、b、c、d 应满足什么条件?请说明理 由. 【答案】解:(1)小明没有说明矩形蔬菜种植区域的长与宽之比为 2:1 的理由。 题目:某村计划建造如图所示的矩形蔬菜温室,要求长与宽的比为 2:1,在温室内,沿前侧内墙 保留 3m 的空地,其他三侧内墙各保留 1m 的通道,当温室的长与宽各为多少时,矩形蔬菜种植区 域的面积是 288m2? 解:设矩形蔬菜种植区域的宽为 xm,则长为 2xm, 根据题意,得 x•2x=288. 解这个方程,得 x1=-12(不合题意,舍去),x2=12 所以温室的长为 2×12+3+1=28(m),宽为 12+1+1=14(m) 答:当温室的长为 28m,宽为 14m 时,矩形蔬菜种植区域的面积是 288m2. ? 在“设矩形蔬菜种植区域的宽为 xm,则长为 2xm.”前补充以下过 程: 设温室的宽为 ym,则长为 2ym。 则矩形蔬菜种植区域的宽为(y-1-1)m,长为(2y-3-1) m。 ∵ 2y 3 1 2y 4 2y 1 1 y 2       ,∴矩形蔬菜种植区域的长与宽之比为 2: 1。 (2)a+c b+d =2。理由如下: 要使矩形 A′B′C′D′∽矩形 ABCD,就要 A D AD A B AB     ,即     AD a c 2 AB b d 1     , 即     2AB a c 2 AB b d 1     ,即 a+c b+d =2。 【考点】一元二次方程的应用(几何问题),相似多边形的性质,比例的性质。 【分析】(1)根据题意可得小明没有说明矩形蔬菜种植区域的长与宽之比为 2:1 的 理由,所以由已知条件求出矩形蔬菜种植区域的长与宽的关系即可。 (2)由使矩形 A′B′C′D′∽矩形 ABCD,利用相似多边形的性质,可得 A D AD A B AB     ,然后利用比例的性质。 27、(2012 江苏南京 10 分)如图,A、B 为⊙O 上的两个定点,P 是⊙O 上的动点 (P 不与 A、B 重合),我们称∠APB 为⊙O 上关于 A、B 的滑动角。 (1)已知∠APB 是 O 上关于点 A、B 的滑动角。 ① 若 AB 为⊙O 的直径,则∠APB= ② 若⊙O 半径为 1,AB= 2 ,求∠APB 的度数 (2)已知 2O 为 1O 外一点,以 2O 为圆心作一个圆与 1O 相交于 A、B 两点,∠APB 为 1O 上关于点 A、B 的滑动角,直线 PA、PB 分别交 2O 于点 M、N(点 M 与点 A、 点 N 与点 B 均不重合),连接 AN,试探索∠APB 与∠MAN、∠ANB 之间的数 量关系。 【答案】解:(1)①900。 ②如图,连接 AB、OA、OB. 在△AOB 中,∵OA=OB=1.AB= 2 ,∴OA2+OB2=AB2。 ∴∠AOB=90°。 当点 P 在优弧 AB 上时(如图 1),∠APB= 1 2 ∠AOB=45°; 当点 P 在劣弧 AB 上时(如图 2), ∠APB= 1 2 (360°-∠AOB)=135°。 (2)根据点 P 在⊙O1 上的位置分为以下四种情况. 第一种情况:点 P 在⊙O2 外,且点 A 在点 P 与点 M 之间,点 B 在点 P 与点 N 之间,如图 3, ∵∠MAN=∠APB+∠ANB, ∴∠APB=∠MAN-∠ANB。 第二种情况:点 P 在⊙O2 外,且点 A 在点 P 与点 M 之间,点 N 在点 P 与点 B 之间,如图 4, ∵∠MAN=∠APB+∠ANP=∠APB+(180°-∠ANB), ∴∠APB=∠MAN+∠ANB-180°。 第三种情况:点 P 在⊙O2 外,且点 M 在点 P 与点 A 之间,点 B 在点 P 与点 N 之间,如图 5, ∵∠APB+∠ANB+∠MAN=180°, ∴∠APB=180°-∠MAN-∠ANB。 第四种情况:点 P 在⊙O2 内,如图 6, ∠APB=∠MAN+∠ANB。 【考点】圆周角定理,勾股定理逆定理,三角形内角和定理和外角性质。 【分析】(1)①根据直径所对的圆周角等于 90°即可得∠APB=900。 ②根据勾股定理的逆定理可得∠AOB=90°,再分点 P 在优弧 AB 上;点 P 在劣弧 AB 上两种情况讨论即可。 (2)根据点 P 在⊙O1 上的位置分为四种情况得到∠APB 与∠MAN、∠ANB 之间的数量关系。 2012 年中考数学精析系列——南京卷 (本试卷满分 120 分,考试时间 120 分钟) 一、选择题(本大题共 6 小题,每小题 2 分,共 12 分) 1、(2012 江苏南京 2 分)下列四个数中,负数是【 】 A. -2 B.  2-2 C. - 2 D.  2-2 【答案】C。 【考点】实数的运算,正数和负数,绝对值的性质,有理数的乘方的定义,算术平方 根。 【分析】根据绝对值的性质,有理数的乘方的定义,算术平方根对各选项分析判断后 利用排除法求解: A、|-2|=2,是正数,故本选项错误;B、 2-2 =4,是正数,故本选项错 误; C、- 2 <0,是负数,故本选项正确;D、  2-2 = 4 =2,是正数,故本 选项错误。 故选 C。 2、(2012 江苏南京 2 分)PM 2.5 是指大气中直径小于或等于 0.0000025 m 的颗粒 物,将 0.0000025 用科学记数法表示为【 】 A. -50.25 10 B. -60.25 10 C. -52.5 10 D. -62.5 10 【答案】C。 【考点】科学记数法。 【分析】根据科学记数法的定义,科学记数法的表示形式为 a×10n,其中 1≤|a|<10,n 为 整数,表示时关键要正确确定 a 的值以及 n 的值。在确定 n 的值时,看该数是大于或等于 1 还是小于 1。当该数大于或等于 1 时,n 为它的整数位数减 1;当该数小于 1 时,-n 为它 第一个有效数字前 0 的个数(含小数点前的 1 个 0)。0.0000025 第一个有效数字前有 6 个 0,从而 0.0000025= -52.5 10 。故选 C。 3、(2012 江苏南京 2 分)计算   3 22 2a a 的结果是【 】 A. a B. 2a C. 3a D. 4a 【答案】B。 【考点】整式的除法,幂的乘方,同底幂的除法。 【分析】根据幂的乘方首先进行化简,再利用同底数幂的除法的运算法则计算后直接 选取答案:    3 22 2 6 4 2= =a a a a a  ,故选 B。 4、(2012 江苏南京 2 分)12 的负的平方根介于【 】 A. -5 和-4 之间 B. -4 与-3 之间 C. -3 与-2 之间 D. -2 与-1 之间 【答案】B。 【考点】估算无理数的大小,不等式的性质。 【分析】∵9 < 12 < 16,∴ 9 12 16< < 。∴- 16 - 12 - 9< < ,即 -4 - 12 -3< < 。故选 B。 5、(2012 江苏南京 2 分)若反比例函数 ky x  与一次函数 y x 2  的图像没有..交 点,则 k 的值可以是【 】 A. -2 B. -1 C. 1 D. 2 【答案】A。 【考点】反比例函数与一次函数的交点问题,一元二次方程的判别式。 【分析】把两函数的解析式组成方程组,再转化为求一元二次方程解答问题,求出 k 的取值范围,找出符合条件的 k 的值即可: ∵反比例函数 ky x  与一次函数 y=x+2 的图象没有交点, ∴ k y x y x 2      ① ② 无解,即 k =x 2x  无解,整理得 x2+2x-k=0, ∴△=4+4k<0,解得 k<-1。 四个选项中只有-2<-1,所以只有 A 符合条件。故选 A。 6、(2012 江苏南京 2 分)如图,菱形纸片 ABCD 中,∠A=600,将纸片折叠,点 A、D 分别落在 A’、D’处,且 A’D’经过 B,EF 为折痕,当 D’F  CD 时, CF FD 的值为 【 】 A. 3 1 2  B. 3 6 C. 2 3 1 6  D. 3 1 8  【答案】A。 【考点】翻折变换(折叠问题),菱形的性质,平行的性质,折叠的性质,锐角三角函数 定义,特殊角的三角函数值。 二、填空题(本大题共 10 小题,每小题 2 分,共 20 分) 7、(2012 江苏南京 2 分)使 1 x 有意义的 x 的取值范围是 ▲ 【答案】 x 1 。 【考点】二次根式有意义的条件。 【分析】根据二次根式被开方数必须是非负数的条件,要使 1 x 在实数范围内有意义, 必须1 x 0  ,即 x 1 。 8、(2012 江苏南京 2 分)计算 2 2 2  的结果是 ▲ 【答案】 2+1。 【考点】分母有理化。 【分析】分子分母同时乘以 2 即可进行分母有理化: 2 2 2 2+2= = 2+122  。 9、(2012 江苏南京 2 分)方程 3 2 0x x 2   的解是 ▲ 【答案】x=6。 【考点】解分式方程。 【分析】方程最简公分母为:  x x 2 。故方程两边乘以  x x 2 ,化为整式方程后求 解,并代入检验即可得出方程的根: 去分母得:3(x-2)-2x=0, 去括号得:3x-6-2x=0, 整理得:x=6, 经检验得 x=6 是方程的根。 10、(2012 江苏南京 2 分)如图, 1 、 2 、 3 、 4 是五边形 ABCDE 的 4 个外 角,若 2A 1 0   ,则 1 2 3 4        ▲ 【答案】300。 【考点】多边形外角性质,补角定义。 【分析】由题意得,∠A 的外角=180°-∠A=60°, 又∵多边形的外角和为 360°,∴∠1+∠2+∠3+∠4=360°-∠A 的外角 =300°。 11、(2012 江苏南京 2 分)已知一次函数 y kx k 3   的图像经过点(2,3),则 k 的值为 ▲ 【答案】2。 【考点】直线上点的坐标与方程的关系。 【分析】根据点在直线上,点的坐标满足方程的关系,将(2,3)代入 y kx k 3   ,得 3 2k k 3   ,解得,k=2。 12、(2012 江苏南京 2 分)已知下列函数 ① 2y x ② 2y x  ③  2y x 1 2   , 其中,图象通过平移可以得到函数 2y x 2x 3   的图像的有 ▲ (填写所有正确选 项的序号) 【答案】①③。 【考点】二次函数图象与平移变换。 【分析】把原式化为顶点式的形式,根据函数图象平移的法则进行解答: ∵  22y x 2x 3= x+1 4    ∴由函数图象平移的法则可知,进行如下平移变换 ①    2 21 42y x y x+1 y= x+1 4    向左平移 个单位 向下平移 个单位 ,故①正 确。 ② 2y x 2x 3   的图象开口向上, 2y x  的图象开口向下,不能通过平移 得到,故②错误。 ③      2 2 22 6y x 1 2 y x+1 2 y= x+1 4       向左平移 个单位 向下平移 个单位 ,,故③ 正确。 ∴图象通过平移可以得到函数 2y x 2x 3   的图像的有① 2y x , ③  2y x 1 2   。 13、(2012 江苏南京 2 分)某公司全体员工年薪的具体情况如下表: 年薪/万元 30 14 9 6 4 3.5 3 员工数/人 1 1 1 2 7 6 2 则所有员工的年薪的平均数比中位数多 ▲ 万元。 【答案】2。 【考点】中位数,加权平均数。 【分析】根据加权平均数的定义求出员工的工资平均数: (30+14+9+6×2+4×7+3.5×6+3×2)÷20 =120÷20 =6。 中位数是一组数据从小到大(或从大到小)重新排列后,最中间的那个数(最中 间两个数的平均数)。因此这 20 个员工的年薪的中位数是第 10 和 11 人的工资的平均数, 工资均为 4,∴中位数为:4。 ∴该公司全体员工年薪的平均数比中位数多 6-4=2 万元。 14、(2012 江苏南京 2 分)如图,将 45的∠AOB 按图摆放在一把刻度尺上,顶点 O 与尺下沿的端点重合,OA 与尺下沿重合,OB 与尺上沿的交点 B 在尺上的读数为 2cm,若按相同的方式将37 的∠AOC 放置在该尺上,则 OC 与尺上沿的交点 C 在尺上 的读数约为 ▲ cm (结果精确到 0.1 cm,参考数据:sin37 0.60  ,cos37 0.80  , tan37 0.75  ) 【答案】2.7。 【考点】解直角三角形的应用,等腰直角三角形的性质,矩形的性质,锐角三角函数定 义,特殊角的三角函数值。 【分析】过点 B 作 BD⊥OA 于 D,过点 C 作 CE⊥OA 于 E。 在△BOD 中,∠BDO=90°,∠DOB=45°, ∴BD=OD=2cm。 ∴CE=BD=2cm。 在△COE 中,∠CEO=90°,∠COE=37°, ∵ CEtan37 0.75OE    ,∴OE≈2.7cm。 ∴OC 与尺上沿的交点 C 在尺上的读数约为 2.7cm。 15、(2012 江苏南京 2 分)如图,在平行四边形 ABCD 中,AD=10cm,CD=6cm, E 为 AD 上一点,且 BE=BC,CE=CD,则 DE= ▲ cm 【答案】2.5。 【考点】平行四边形的性质,平行的性质,等腰三角形的性质,相似三角形的判定和 性质。 【分析】∵四边形 ABCD 是平行四边形,AD=10cm,CD=5cm, ∴BC=AD=10cm,AD∥BC,∴∠2=∠3。 ∵BE=BC,CE=CD, ∴BE=BC=10cm,CE=CD=5cm,∠1=∠2,∠3=∠D。 ∴∠1=∠2=∠3=∠D。∴△BCE∽△CDE。∴ BC CE CD DE  ,即10 5 5 DE  ,解得 DE=2.5cm。 16、(2012 江苏南京 2 分)在平面直角坐标系中,规定把一个三角形先沿 x 轴翻 折,再向右平移两个单位称为一次变换,如图,已知等边三角形 ABC 的顶点 B、C 的 坐标分别是,(-1,-1),(-3,-1),把三角形 ABC 经过连续 9 次这样的变换得到 三角形 A’B’C’,则点 A 的对应点 A’的坐标是 ▲ 【答案】(16,1+ 3 )。 【考点】分类归纳(图形的变化类),翻折变换(折叠问题),坐标与图形性质,等边三 角形的性质,锐角三角函数定义,特殊角的三角函数值。 【分析】先由△ABC 是等边三角形,点 B、C 的坐标分别是(-1,1)、(-3,- 1),求得点 A 的坐标;再寻找规律,求出点 A 的对应点 A′的坐标: 如图,作 BC 的中垂线交 BC 于点 D,则 ∵△ABC 是等边三角形,点 B、C 的坐标分别是(-1,1)、(-3,-1), ∴BD=1, 0AD BD tan60 3   。∴A(—2, 1 3  )。 根据题意,可得规律:第 n 次变换后的点 A 的对应点的坐标:当 n 为奇数时为 (2n-2,1+ 3 ),当 n 为偶数时为(2n-2, 1 3  )。 ∴把△ABC 经过连续 9 次这样的变换得到△A′B′C′,则点 A 的对应点 A′的坐标 是:(16,1+ 3 )。 三、解答题(本大题共 11 题,共 88 分) 17、(2012 江苏南京 6 分)解方程组 x 3y 1 3x 2y 8       【答案】解: x 3y 1 3x 2y 8       ① ② , 由①得 x=-3y-1③, 将③代入②,得 3(-3y-1)-2y=8,解得:y=-1。 将 y=-1 代入③,得 x=2。 ∴原方程组的解是 x 2 y 1     。 【考点】解二元一次方程组。 【分析】解二元一次方程组的解题思想是用代入法或加减法消元,化为一元一次方程 求解。本题易用代入法求解。先由①表示出 x,然后将 x 的值代入②,可得出 y 的值, 再代入①可得出 x 的值,继而得出了方程组的解。 18、(2012 江苏南京 9 分)化简代数式 2 2 x 1 x 1 x 2x x   ,并判断当 x 满足不等式组   x 2 1 2 x 1 6      时该代数式的符号。 19、(2012 江苏南京 8 分)如图,在直角三角形 ABC 中,∠ABC=90°,点 D 在 BC 的延长线上,且 BD=AB,过 B 作 BE  AC,与 BD 的垂线 DE 交于点 E, (1)求证:△ABC≌△BDE (2)三角形 BDE 可由三角形 ABC 旋转得到,利用尺规作出旋转中心 O(保留作图痕 迹,不写作法) 20、(2012 江苏南京 8 分)某中学七年级学生共 450 人,其中男生 250 人,女生 200 人。该校对七年级所有学生进行了一次体育测试,并随即抽取了 50 名男生和 40 名女生的测试成绩作为样本进行分析,绘制成如下的统计表: 成绩 划记 频数 百分比 不及格 9 10% 及格 18 20% 良好 36 40% 优秀 27 30% 合计 90 90 100% (1)请解释“随即抽取了 50 名男生和 40 名女生”的合理性; (2)从上表的“频数”、“百分比”两列数据中选择一列,用适当的统计图表示; (3)估计该校七年级学生体育测试成绩不合格的人数。 【答案】解:(1)∵ 90250 50450   (人), 90200 40450   (人), ∴该校从七年级学生中随机抽取 90 名学生,应当抽取 50 名男生和 40 名女生。 (2)选择扇形统计图,表示各种情况的百分比,图形如下: (3)450×10%=45(人)。 答:估计该校七年级学生体育测试成绩不及格 45 人. 【考点】频数(率)分布表,抽样调查的可靠性,频数、频率和总量的关系,用样本 估计总体,扇形统计图或条形统计图。 【分析】(1)所抽取男生和女生的数量应该按照比例进行,根据这一点进行说明即 可。 (2)可选择扇形统计图,表示出各种情况的百分比,也可选择条形统计图, 答案不唯一。 (3)根据用样本估计总体的方法即可得出答案。 21、(2012 江苏南京 7 分)甲、乙、丙、丁 4 名同学进行一次羽毛球单打比赛,要 从中选 2 名同学打第一场比赛,求下列事件的概率。 (1)已确定甲打第一场,再从其余 3 名同学中随机选取 1 名,恰好选中乙同学; (2)随机选取 2 名同学,其中有乙同学. 【答案】解:(1)已确定甲打第一场,再从其余 3 名同学中随机选取 1 名,恰好选中 乙同学的概率是 1 3 。 (2)从甲、乙、丙、丁 4 名同学中随机选取 2 名同学,所有等可能出现 的结果有:(甲、乙)、(甲、丙)、(甲、丁)、(乙、丙)、(乙、丁)、 (丙、丁),共有 6 种, 所有的结果中,满足“随机选取 2 名同学,其中有乙同学”(记为事 件 A)的结果有 3 种:(甲、乙)、(乙、丙)、(乙、丁)。 ∴P(A)= 3 1=6 2 。 【考点】列举法,概率。 【分析】(1)由一共有 3 种等可能性的结果,其中恰好选中乙同学的有 1 种,即可求 得答案。 (2)先用列举法求出全部情况的总数,再求出符合条件的情况数目,二者的 比值就是其发生的概率。 22、(2012 江苏南京 8 分)如图,梯形 ABCD 中,AD//BC,AB=CD,对角线 AC、 BD 交于点 O,AC  BD,E、F、G、H 分别为 AB、BC、CD、DA 的中点 (1)求证:四边形 EFGH 为正方形; (2)若 AD=2,BC=4,求四边形 EFGH 的面积。 【答案】(1)证明:在△ABC 中,E、F 分别是 AB、BC 的中点,EF= 1 2 AC。 同理 FG= 1 2 BD,GH= 1 2 AC,HE= 1 2 BD。 ∵在梯形 ABCD 中,AB=DC,∴AC=BD。 ∴EF=FG=GH=HE,∴四边形 EFGH 是菱形。 设 AC 与 EH 交于点 M, 在△ABD 中,E、H 分别是 AB、AD 的中点,则 EH∥BD,同理 GH∥AC。 又∵AC⊥BD,∴∠BOC=90°。∴∠EHG=∠EMC=90°。 ∴四边形 EFGH 是正方形。 (2)解:连接 EG。 在梯形 ABCD 中,∵E、F 分别是 AB、DC 的中点, ∴ 1EG AD BC 32   ( ) 。 在 Rt△EHG 中,∵EH2+GH2=EG2,EH=GH, ∴ 2 9EH 2  ,即四边形 EFGH 的面积为 9 2 。 【考点】三角形中位线定理,等腰梯形的性质,正方形的判定,梯形中位线定理,勾 股定理。 【分析】(1)先由三角形的中位线定理求出四边相等,然后由 AC⊥BD 入手,进行正 方形的判断。 (2)连接 EG,利用梯形的中位线定理求出 EG 的长,然后结合(1)的结论 求出 2 9EH 2  ,也即得出了正方形 EHGF 的面积。 23、(2012 江苏南京 7 分)看图说故事。 请你编一个故事,使故事情境中出现的一对变量 x、y 满足图示的函数关系式,要求: ①指出 x 和 y 的含义;②利用图中数据说明这对变量变化过程的实际意义,其中需设 计“速度”这个量 【答案】解: ①该函数图象表示小明骑车离出发地的路程 y(单位:km)与他所用的 时间 x(单位:min)的关系。 ②小明以 400m/min 的速度匀速骑了 5min,在原地休息了 6min,然后 以 500m/min 的速度匀速骑车回出发地。(本题答案不唯一) 【考点】开放型问题,函数的图象。 【分析】①结合实际意义得到变量 x 和 y 的含义;②由于函数须涉及“速度”这个量,只 要叙述清楚时间及相应的路程,体现出函数的变化即可。 24、(2012 江苏南京 8 分)某玩具由一个圆形区域和一个扇形区域组成,如图,在 1O 和扇形 2O CD 中, 1O 与 2O C 、 2O D 分别相切于 A、B, 2CO D 60   ,E、F 事直线 1 2O O 与 1O 、扇形 2O CD 的两个交点,EF=24cm,设 1O 的半径为 x cm, ① 用含 x 的代数式表示扇形 2O CD 的半径; ② 若 1O 和扇形 2O CD 两个区域的制作成本分别为 0.45 元 2/cm 和 0.06 元 2/cm ,当 1O 的半径为多少时,该玩具成本最小? 【答案】解:(1)连接 O1A。 ∵⊙O1 与 O2C、O2D 分别切一点 A、B, ∴O1A⊥O2C,O2E 平分∠CO2D。 ∵ 2CO D 60   ,∴∠AO2O1= 1 2 ∠CO2D=30°。 在 Rt△O1AO2 中, 1 2 1 1 2 AOsin AO O O O   ,∴O1O2=A O1 sin∠AO2O1 =x sin30° =2x。 ∵EF=24cm,∴FO2=EF-EO1-O1O2=24-3x,即扇形 O2CD 的半 径为(24-3x)cm。 (2)设该玩具的制作成本为 y 元,则    2 2 2360 60 24 3xy 0.45 x 0.06 0.9 x 7.2 x 28.8360              20.9 x 4 14.4   ( ) 。 ∴当 x=4 时,y 的值最小。 答:当⊙O1 的半径为 4cm 时,该玩具的制作成本最小。 【考点】切线的性质,锐角三角函数定义,扇形面积的计算,二次函数的最值。 【分析】(1)连接 O1A.由切线的性质知∠AO2O1= 1 2 ∠CO2D=30°;然后在 Rt△O1AO2 中利用锐角三角函数的定义求得 O1O2=2x;最后由图形中线段间的和差关 系求得扇形 O2CD 的半径 FO2。 (2)设该玩具的制作成本为 y 元,则根据圆形的面积公式和扇形的面积公式 列出 y 与 x 间的函数关系,然后利用二次函数的最值即可求得该玩具的最小制作成 本。 25、(2012 江苏南京 8 分)某汽车销售公司 6 月份销售某厂家的汽车,在一定范围 内,每部汽车的进价与销售有如下关系,若当月仅售出 1 部汽车,则该部汽车的进价 为 27 万元,每多售一部,所有出售的汽车的进价均降低 0.1 万元/部。月底厂家根据 销售量一次性返利给销售公司,销售量在 10 部以内,含 10 部,每部返利 0.5 万元, 销售量在 10 部以上,每部返利 1 万元。 ① 若该公司当月卖出 3 部汽车,则每部汽车的进价为 万元; ② 如果汽车的销售价位 28 万元/部,该公司计划当月盈利 12 万元,那么要卖出多少 部汽车?(盈利=销售利润+返利) 【答案】解:(1)26.8。 (2)设需要售出 x 部汽车, 由题意可知,每部汽车的销售利润为:28-[27-0.1(x-1)]= (0.1x+0.9)(万元), 当 0≤x≤10,根据题意,得 x·(0.1x+0.9)+0.5x=12,整理,得 x2+14x-120=0, 解这个方程,得 x1=-20(不合题意,舍去),x2=6。 当 x>10 时,根据题意,得 x·(0.1x+0.9)+x=12,整理,得 x2 +19x-120=0, 解这个方程,得 x1=-24(不合题意,舍去),x2=5。 ∵5<10,∴x2=5 舍去。 答:要卖出 6 部汽车。 【考点】一元二次方程的应用。 【分析】(1)根据若当月仅售出 1 部汽车,则该部汽车的进价为 27 万元,每多售出 1 部,所有售出的汽车的进价均降低 0.1 万元/部,得出该公司当月售出 3 部汽车时, 则每部汽车的进价为:27-0.1×2=26.8。, (2)利用设需要售出 x 部汽车,由题意可知,每部汽车的销售利润,根据当 0≤x≤10,以及当 x>10 时,分别讨论得出即可。 26、((2012 江苏南京 9 分)“?”的思考 下框中是小明对一道题目的解答以及老师的批阅。 我的结果也正确 小明发现他解答的结果是正确的,但是老师却在他的解答中划了一条横线,并打开了 一个“?” 结果为何正确呢? (1)请指出小明解答中存在的问题,并补充缺少的过程: 变化一下会怎样…… (2)如图,矩形 A′B′C′D′在矩形 ABCD 的内部,AB∥A′B′,AD∥A′D′,且 AD: AB=2:1,设 AB 与 A′B′、BC 与 B′C′、CD 与 C′D′、DA 与 D′A′之间的距离分别为 a、 b、c、d,要使矩形 A′B′C′D′∽矩形 ABCD,a、b、c、d 应满足什么条件?请说明理 由. 【答案】解:(1)小明没有说明矩形蔬菜种植区域的长与宽之比为 2:1 的理由。 题目:某村计划建造如图所示的矩形蔬菜温室,要求长与宽的比为 2:1,在温室内,沿前侧内墙 保留 3m 的空地,其他三侧内墙各保留 1m 的通道,当温室的长与宽各为多少时,矩形蔬菜种植区 域的面积是 288m2? 解:设矩形蔬菜种植区域的宽为 xm,则长为 2xm, 根据题意,得 x•2x=288. 解这个方程,得 x1=-12(不合题意,舍去),x2=12 所以温室的长为 2×12+3+1=28(m),宽为 12+1+1=14(m) 答:当温室的长为 28m,宽为 14m 时,矩形蔬菜种植区域的面积是 288m2. ? 在“设矩形蔬菜种植区域的宽为 xm,则长为 2xm.”前补充以下过 程: 设温室的宽为 ym,则长为 2ym。 则矩形蔬菜种植区域的宽为(y-1-1)m,长为(2y-3-1) m。 ∵ 2y 3 1 2y 4 2y 1 1 y 2       ,∴矩形蔬菜种植区域的长与宽之比为 2: 1。 (2)a+c b+d =2。理由如下: 要使矩形 A′B′C′D′∽矩形 ABCD,就要 A D AD A B AB     ,即     AD a c 2 AB b d 1     , 即     2AB a c 2 AB b d 1     ,即 a+c b+d =2。 【考点】一元二次方程的应用(几何问题),相似多边形的性质,比例的性质。 【分析】(1)根据题意可得小明没有说明矩形蔬菜种植区域的长与宽之比为 2:1 的 理由,所以由已知条件求出矩形蔬菜种植区域的长与宽的关系即可。 (2)由使矩形 A′B′C′D′∽矩形 ABCD,利用相似多边形的性质,可得 A D AD A B AB     ,然后利用比例的性质。 27、(2012 江苏南京 10 分)如图,A、B 为⊙O 上的两个定点,P 是⊙O 上的动点 (P 不与 A、B 重合),我们称∠APB 为⊙O 上关于 A、B 的滑动角。 (1)已知∠APB 是 O 上关于点 A、B 的滑动角。 ① 若 AB 为⊙O 的直径,则∠APB= ② 若⊙O 半径为 1,AB= 2 ,求∠APB 的度数 (2)已知 2O 为 1O 外一点,以 2O 为圆心作一个圆与 1O 相交于 A、B 两点,∠APB 为 1O 上关于点 A、B 的滑动角,直线 PA、PB 分别交 2O 于点 M、N(点 M 与点 A、 点 N 与点 B 均不重合),连接 AN,试探索∠APB 与∠MAN、∠ANB 之间的数 量关系。 【答案】解:(1)①900。 ②如图,连接 AB、OA、OB. 在△AOB 中,∵OA=OB=1.AB= 2 ,∴OA2+OB2=AB2。 ∴∠AOB=90°。 当点 P 在优弧 AB 上时(如图 1),∠APB= 1 2 ∠AOB=45°; 当点 P 在劣弧 AB 上时(如图 2), ∠APB= 1 2 (360°-∠AOB)=135°。 (2)根据点 P 在⊙O1 上的位置分为以下四种情况. 第一种情况:点 P 在⊙O2 外,且点 A 在点 P 与点 M 之间,点 B 在点 P 与点 N 之间,如图 3, ∵∠MAN=∠APB+∠ANB, ∴∠APB=∠MAN-∠ANB。 第二种情况:点 P 在⊙O2 外,且点 A 在点 P 与点 M 之间,点 N 在点 P 与点 B 之间,如图 4, ∵∠MAN=∠APB+∠ANP=∠APB+(180°-∠ANB), ∴∠APB=∠MAN+∠ANB-180°。 第三种情况:点 P 在⊙O2 外,且点 M 在点 P 与点 A 之间,点 B 在点 P 与点 N 之间,如图 5, ∵∠APB+∠ANB+∠MAN=180°, ∴∠APB=180°-∠MAN-∠ANB。 第四种情况:点 P 在⊙O2 内,如图 6, ∠APB=∠MAN+∠ANB。 【考点】圆周角定理,勾股定理逆定理,三角形内角和定理和外角性质。 【分析】(1)①根据直径所对的圆周角等于 90°即可得∠APB=900。 ②根据勾股定理的逆定理可得∠AOB=90°,再分点 P 在优弧 AB 上;点 P 在劣弧 AB 上两种情况讨论即可。 (2)根据点 P 在⊙O1 上的位置分为四种情况得到∠APB 与∠MAN、∠ANB 之间的数量关系。 机密★启用前 2012 年天津市初中毕业生学业考试试卷 数 学 本试卷分为第Ⅰ卷(选择题)、第Ⅱ卷(非选择题)两部分。第Ⅰ卷为第 1 页至 第 3 页,第Ⅱ卷为第 4 页至第 8 页。试卷满分 120 分。考试时间 100 分钟。 答卷前,请你务必将自己的姓名、考生号、考点校、考场号、座位号填写在“答 题卡”上,并在规定位置粘贴考试用条形码。答题时,务必将答案涂写在“答题卡” 上,答案答在试卷上无效。考试结束后,将本试卷和“答题卡”一并交回。 祝你考试顺利! 第Ⅰ卷 注意事项: 1.每题选出答案后,用 2B 铅笔把“答题卡”上对应题目的答案标号的信息点涂 黑。如需改动,用橡皮擦干净后,再选涂其他答案标号的信息点。 2.本卷共 10 题,共 30 分。 一、选择题(本大题共 10 小题,每小题 3 分,共 30 分.在每小题给出的四个选项 中,只有一项是符合题目要求的) (1) 2cos60 的值等于 (A)1 (B) 2 (C) 3 (D) 2 (2)下列标志中,可以看作是中心对称图形的是 (A) (B) (C) (D) (3)据某域名统计机构公布的数据显示,截至 2012 年 5 月 21 日,我国“.NET”域 名注册量约为 560 000 个,居全球第三位.将 560 000 用科学记数法表示应为 (A) 3560 10 (B) 456 10 (C) 55.6 10 (D) 60.56 10 (4)估计 6 1 的值在 (A) 2 到3之间 (B)3到 4 之间 (C) 4 到5 之间 (D)5 到 6 之间 (5)为调查某校 2000 名学生对新闻、体育、动画、娱乐、戏曲五类电 视节目的喜爱情况,随机抽取部分学生进行调查,并结合调查数据 作出如图所示的扇形统计图. 根据统计图提供的信息,可估算出该校喜爱体育节目的学生共有 (A)300 名 (B)400 名 (C)500 名 (D)600 名 (6)将下列图形绕其对角线的交点逆时针旋转90 ,所得图形一定与原图形重合的是 (A)平行四边形 (B)矩形 (C)菱形 (D)正方形 (7)右图是一个由 4 个相同的正方体组成的立体图形,它的三视图是 5% 10% 35% 30% 体育 动画 娱乐 新闻 戏曲 第(5)题 (A) (B) 第(7)题 (D)(C) (8)如图,在边长为 2 的正方形 ABCD 中, M 为边 AD 的 中点,延长 MD 至点 E ,使 ME MC ,以 DE 为边作 正方形 DEFG ,点G 在边CD 上,则 DG 的长为 (A) 3 1 (B)3 5 (C) 5 1 (D) 5 1 (9)某电视台“走基层”栏目的一位记者乘汽车赴360 km 外的农村采访,全程的前 一部分为高速公路,后一部分为乡村公路.若汽车在高速公路和乡村公路上分别 以某一速度匀速行驶,汽车行驶的路程 y (单位:km)与 时间 x (单位:h)之间的关系如图所示,则下列结论正确 的是 (A)汽车在高速公路上的行驶速度为100 km/h (B)乡村公路总长为90 km (C)汽车在乡村公路上的行驶速度为 60 km/h (D)该记者在出发后 4.5 h 到达采访地 (10)若关于 x 的一元二次方程 ( 2)( 3)x x m   有实数根 1x 、 2x ,且 1 2x x ,有下列结 论: ① 1 2x  , 2 3x  ; ② 1 4m   ; ③二次函数 1 2( )( )y x x x x m    的图象与 x 轴交点的坐标为 2 0( ,)和 3 0( ,). 其中,正确结论的个数是 第(9)题 2 3.5 / kmy 270 180 O / hx 第(8)题 A B D C M E F G (A)0 (B)1 (C)2 (D)3 机密★启用前 2012 年天津市初中毕业生学业考试试卷 数 学 第Ⅱ卷 注意事项: 1.用黑色墨水的钢笔或签字笔将答案写在“答题卡”上。 2.本卷共 16 题,共 90 分。 二、填空题(本大题共 8 小题,每小题 3 分,共 24 分) (11)| 3 |  . (12)化简 2 2 1 ( 1) ( 1) x x x   的结果是 . (13)袋子中装有 5 个红球和 3 个黑球,这些球除了颜色外都相同.从袋子中随机地 摸出 1 个球,则它是红球的概率是 . (14)将正比例函数 6y x  的图象向上平移,则平移后所得图象对应的函 数解析式可以是 (写出一个即可). (15)如图, ABC△ 是 O⊙ 的内接三角形, AB 为 O⊙ 的直径,点 D 为 O⊙ 上一点,若 55CAB   ,则 ADC 的大小为 (度). (16)若一个正六边形的周长为 24,则该正六边形的面积为 . (17)如图,已知正方形 ABCD 的边长为 1,以顶点 A 、 B 为圆心,1 为 半径的两弧交于点 E ,以顶点C 、 D 为圆心,1 为半径的两弧交于 第(15)题 A BO C D A B C 第(17)题 D E F 点 F ,则 EF 的长为 . (18)“三等分任意角”是数学史上一个著名问题.已知一个角 MAN ,设 1 3 MAN   . (Ⅰ)当 69MAN   时,  的大小为 (度); (Ⅱ)如图,将 MAN 放置在每个小正方形的边长为 1 cm 的网格中,角的一边 AM 与水平方向的网格线平行,另一边 AN 经过格点 B ,且 2.5AB  cm.现要求只能使用带刻度...的直尺,请你在图中作出  ,并简 要说明作法(不要求证明) . B MA N 第(18)题 三、解答题(本大题共 8 小题,共 66 分.解答应写出文字说明、演算步骤或推理过 程) (19)(本小题 6 分) 解不等式组 3 1 3 2 1 1 x x x x        , . (20)(本小题 8 分) 已知反比例函数 1ky x  ( k 为常数, 1k  ). (Ⅰ)其图象与正比例函数 y x 的图象的一个交点为 P ,若点 P 的纵坐标是 2 , 求 k 的值; (Ⅱ)若在其图象的每一支上, y 随 x 的增大而减小,求 k 的取值范围; (Ⅲ)若其图象的一支位于第二象限,在这一支上任取两点 1 1( )A x y, 、 2 2( )B x y, ,当 1 2y y 时,试比较 1x 与 2x 的大小. (21)(本小题 8 分) 在开展“学雷锋社会实践”活动中,某校为了解全校 1200 名学生参加活动的情 况,随机调查了 50 名学生每人参加活动的次数,并根据数据绘成条形统计图如下: 1 2 3 4 5 5 10 15 20 0 7 17 18 53 人数 第(21)题 次数 (Ⅰ)求这 50 个样本数据的平均数、众数和中位数; (Ⅱ)根据样本数据,估算该校 1200 名学生共参加了多少次活动. (22)(本小题 8 分) 已知⊙O 中, AC 为直径, MA 、 MB 分别切⊙O 于点 A 、 B . (Ⅰ)如图①,若 25BAC   ,求 AMB 的大小; (Ⅱ)如图②,过点 B 作 BD AC 于点 E ,交⊙O 于点 D ,若 BD MA ,求 AMB 的大小. O AM C B O AM C B 图① 图② D 第(22)题 E (23)(本小题 8 分) 如图,甲楼 AB 的高度为123 m,自甲楼楼顶 A 处,测得乙楼顶端 C 处的仰角为 45,测得乙楼底部 D 处的俯角为30 ,求乙楼CD 的高 度(结果精确到 0.1 m, 3 取1.73 ). (24)(本小题 8 分) 某通讯公司推出了移动电话的两种计费方式(详情见下表). 月使用 费/元 主叫限 定时间/ 分 主叫超时 费/(元/ 分) 被叫 方式一 58 150 0.25 免费 方式二 88 350 0.19 免费 设一个月内使用移动电话主叫的时间为t 分(t 为正整数), 请根据表中提供的信息回答下列问题: (Ⅰ)用含有t 的式子填写下表: 150t ≤ 150 350t  350t  350t  方式一计费/元 58 108 方式二计费/元 88 88 88 (Ⅱ)当t 为何值时,两种计费方式的费用相等; 第(23)题 C B D A 45 30 (甲 (乙 温馨提示: 若选用方式一, 每月固定交费 58 元,当主动打出电话 月累计时间不超过 150 分,不再额外交 费;当超过 150 分, 超过部分每分加收 0.25 元. (Ⅲ)当330 360t  时,你认为选用哪种计费方式省钱(直接写出结果即可). (25)(本小题 10 分) 已知一个矩形纸片OACB ,将该纸片放置在平面直角坐标系中,点 11 0A( ,),点 0 6B( ,),点 P 为 BC 边上的动点(点 P 不与点 B 、C 重合),经过点O 、 P 折叠该纸 片,得点 B 和折痕OP .设 BP t . (Ⅰ)如图①,当 30BOP   时,求点 P 的坐标; O xA CB P B O x y A CB P B C Q 第(25)题 图① 图② y (Ⅱ)如图②,经过点 P 再次折叠纸片,使点C 落在直线 PB 上,得点C 和折痕 PQ ,若 AQ m ,试用含有t 的式子表示 m ; (Ⅲ)在(Ⅱ)的条件下,当点C 恰好落在边OA 上时,求点 P 的坐标(直接写 出结果即可). (26)(本小题 10 分) 已知抛物线 2y ax bx c   ( 0 2a b  )的顶点为 0 0( )P x y, ,点 (1 )AA y, 、 (0 )BB y, 、 ( 1 )CC y , 在该抛物线上. (Ⅰ)当 1a  , 4b  , 10c  时,①求顶点 P 的坐标;②求 A B C y y y 的值; (Ⅱ)当 0 0y ≥ 恒成立时,求 A B C y y y 的最小值. 机密★启用前 2012 年天津市初中毕业生学业考试 数学参考答案 一、选择题(本大题共 10 小题,每小题 3 分,共 30 分) (1)A (2)B (3)C (4)B (5) B (6)D (7)A (8)D (9)C (10)C 二、填空题(本大题共 8 小题,每小题 3 分,共 24 分) (11)3 (12) 1 1x  (13) 5 8 (14) 6 1y x   (答案不惟一,可以是形如 6y x b   , 0b  的一次函数) (15)35 (16) 24 3 (17) 3 1 (18)(Ⅰ) 23;(Ⅱ)如图,让直尺有刻度一边过点 A ,设该边与过点 B 的竖直方 向的网格线交于点C ,与过点 B 的水平方向的 网格线交于点 D ;保持直尺有刻度的一边过点 A ,调整点C 、 D 的位置,使 5 cmCD  ,画 射线 AD ,此时 MAD 即为所求的  . 三、解答题(本大题共 8 小题,共 66 分) (19)(本小题 6 分) 解: ∵ 3 1 3 2 1 1 x x x x        , ① ,  ② 解不等式①,得 1x  . 解不等式②,得 2x  . ∴ 不等式组的解集为 1 2x  . B MA N C D (20)(本小题 8 分) 解:(Ⅰ)由题意,设点 P 的坐标为 ( 2)m, , ∵ 点 P 在正比例函数 y x 的图象上, ∴ 2 m ,即 2m  . ∴ 点 P 的坐标为 (2 2), . ∵ 点 P 在反比例函数 1ky x  的图象上, ∴ 12 2 k  ,解得 5k  . (Ⅱ)∵ 在反比例函数 1ky x  图象的每一支上, y 随 x 的增大而减小, ∴ 1 0k   ,解得 1k  . (Ⅲ)∵ 反比例函数 1ky x  图象的一支位于第二象限, ∴ 在该函数图象的每一支上, y 随 x 的增大而增大. ∵ 点 1 1( )A x y, 与点 2 2( )B x y, 在该函数的第二象限的图象上,且 1 2y y , ∴ 1 2x x . (21)(本小题 8 分) 解:(Ⅰ)观察条形统计图,可知这组样本数据的平均数是 1 3 2 7 3 17 4 18 5 5 3.350x           , ∴ 这组样本数据的平均数是 3.3. ∵ 在这组样本数据中,4 出现了 18 次,出现的次数最多, ∴ 这组数据的众数是 4. ∵ 将这组样本数据按从小到大的顺序排列,其中处在中间的两个数都是 3, 有 3 3 32   , ∴ 这组数据的中位数是 3. (Ⅱ)∵ 这组样本数据的平均数是 3.3, ∴ 估计全校 1200 人参加活动次数的总体平均数是 3.3, 有 3.3 1200 3960  . ∴ 该校学生共参加活动约 3960 次. (22)(本小题 8 分) 解:(Ⅰ)∵ MA 切⊙O 于点 A ,有 90MAC   . 又 25BAC   , ∴ 65MAB MAC BAC       . ∵ MA 、 MB 分别切⊙O 于点 A 、 B , ∴ MA MB ,有 MAB MBA   . ∴ 180 ( ) 50AMB MAB MBA         . (Ⅱ)如图,连接 AD 、 AB . ∵ MA AC ,又 BD AC , ∴ //BD MA . 又 BD MA , ∴ 四边形 MADB 是平行四边形. ∵ MA MB , ∴ 四边形 MADB 是菱形,有 AD BD . 又 AC 为直径, AC BD ,得  AB AD ,有 AB AD . ∴ ABD△ 是等边三角形,有 60D   . ∴ 在菱形 MADB 中, 60AMB D     . (23)(本小题 8 分) 解: 如图,过点 A 作 AE CD 于点 E , 根据题意, 45CAE   , 30DAE   . ∵ AB BD ,CD BD , ∴ 四边形 ABDE 为矩形. ∴ 123DE AB  . A C D 30 45 E B O AM C B O AM C B DE 在 Rt ADE△ 中, tan DEDAE AE   , ∴ 123 123 123 3tan tan30 3 3 DEAE DAE      . 在 Rt ACE△ 中,由 45CAE   , 得 123 3CE AE  . ∴ 123( 3 1) 335.8CD CE DE     . 答:乙楼CD 的高度约为335.8 m. (24)(本小题 8 分) 解:(Ⅰ)当150 350t  时,方式一: 0.25 20.5t  ; 当 350t  时,方式一: 0.25 20.5t  ;方式二:0.19 21.5t  . (Ⅱ)∵ 当 350t  时, (0.25 20.5) (0.19 21.5) 0.06 1 0t t t      , ∴ 当两种计费方式的费用相等时,t 的值在150 350t  取得. ∴ 列方程 0.25 20.5 88t   ,解得 270t  . 答:当主叫时间为 270 分时,两种计费方式的费用相等. (Ⅲ)方式二. (25)(本小题 10 分) 解:(Ⅰ)根据题意, 90OBP   , 6OB  , 在 Rt OBP△ 中,由 30BOP   , BP t ,得 2OP t . 根据勾股定理, 2 2 2OP OB BP  , 即 2 2 2(2 ) 6t t  ,解得 2 3t  ( 2 3t   舍去). ∴ 点 P 的坐标为 (2 3 6), . (Ⅱ)∵ OB P△ 、 QC P△ 分别是由 OBP△ 、 QCP△ 折叠得到的, 有 OB P△ ≌ OBP△ , QC P△ ≌ QCP△ . ∴ OPB OPB   , QPC QPC   . ∵ 180OPB OPB QPC QPC          , ∴ 90OPB QPC     . ∵ 90BOP OPB     , ∴ BOP CPQ   . 又 90OBP C     , O xA CB P B C Q y ∴ OBP△ ∽ PCQ△ ,有 OB BP PC CQ  . 由题设 BP t , AQ m , 11BC  , 6AC  ,则 11PC t  , 6CQ m  . ∴ 6 11 6 t t m   . ∴ 21 11 66 6m t t   ( 0 11t< < )即为所求. (Ⅲ)点 P 的坐标为 11 13( 6)3  , 或 11 13( 6)3  , . (26)(本小题 10 分) 解:(Ⅰ)若 1a  , 4b  , 10c  , 此时抛物线的解析式为 2 4 10y x x   . ① ∵ 2 24 10 ( 2) 6y x x x      , ∴ 抛物线的顶点坐标为 ( 2 6)P  , ; ② ∵点 (1 )AA y, 、 (0 )BB y, 、 ( 1 )CC y , 在抛物线 2 4 10y x x   上, ∴ 15Ay  , 10By  , 7Cy  . ∴ 15 510 7 A B C y y y    . (Ⅱ)由0 2a b  ,得 0 12 bx a     . 由题意,如图,过点 A 作 1AA x 轴于点 1A ,则 1 AAA y , 1 1OA  . 连接 BC ,过点C 作CD y 轴于点 D ,则 B CBD y y  , 1CD  . 过点 A 作 //AF BC ,交抛物线于点 1( )EE x y, ,交 x 轴于点 2( 0)F x , , 则 1FAA CBD   . 于是 1Rt AFA△ ∽ Rt BCD△ . 有 1 1AA FA BD CD  ,即 2 2 1 11 A B C y x xy y    . 过点 E 作 1EG AA 于点G , 易得 AEG BCD△ ∽△ . 有 AG EG BD CD  ,即 11A E B C y y xy y    . ∵ 点 (1 )AA y, 、 (0 )BB y, 、 ( 1 )CC y , 、 1( )EE x y, 在抛物线 2y ax bx c   上, 得 Ay a b c   , By c , Cy a b c   , 2 1 1Ey ax bx c   , ∴ 2 1 1 1 ( ) ( ) 1( ) a b c ax bx c xc a b c          . x y O A 1A B C 1 E F 1x GD 12x 化简,得 2 1 1 2 0x x   ,解得 1 2x   ( 1 1x  舍去). ∵ 0 0y ≥ 恒成立,根据题意,有 2 1 1x x ≤ < , 则 2 11 1x x ≥ ,即 21 3x ≥ . ∴ A B C y y y 的最小值为3. 2012 年广州市初中毕业生学业考试 数 学 第一部分 选择题(共 30 分) 一、选择题(本大题共 10 小题,每小题 3 分,满分 30 分。在每小题给出的 4 个选项 中只有一项是符合题目要求的) 1.实数 3 的倒数是( )。 (A)、 3 1 (B)、 3 1 (C)、 3 (D)、3 2.将二次函数 2xy  的图象向下平移 1 个单位,则平移后的二次函数的解析式为 ( )。 (A)、 12  xy (B)、 12  xy (C)、 2)1(  xy (D)、 2)1(  xy 3.一个几何体的三视图如图 1 所示,则这个几何体是( )。 (A)、四棱锥 (B)、 四棱柱 (C)、三棱锥 (D)、三棱柱 4.下面的计算正确的是( ) 。 (A)、 156  aa (B)、 22 3aaa  (C)、 baba  )( (D)、 baba  2)(2 5.如图 2,在等腰梯形 ABCD 中,BC∥AD,AD=5,DC=4,DE∥AB 交 BC 于点 E,且 EC=3,则梯形 ABCD 的周长是( ) (A)、26 (B)、 25 (C)、21 (D)、20 6..已知 ,071  ba 则  ba ( ) 。 (A)、-8 (B)、 -6 (C)、6 (D)、8 7. Rt ABC△ 中,∠C=900,AC=9,BC=12,则点 C 到 AB 的距离是( )。 (A)、 5 36 (B)、 25 12 (C)、 4 9 (D)、 4 33 8.已知 a >b.若 c 是任意实数,则下列不等式中总是成立的是 ( )。 (A)、a+cb-c (C)、acbc 9.在平面中,下列命题为真命题的是( )。 (A)、四边相等的四边形是正方形 (B)、对角线相等的四边形是菱形 (C)、四个角相等的四边形是矩形 (D)、对角线互相垂直的四边形是平行四边形 10.如图 3,正比例函数 xky 11  和反比例函数 x ky 2 2  的图象交于 A(-1,2)、B (1,-2)两点。若 y1-1 (B)、 x<-1 或 01 第二部分 非选择题(共 120 分) 二、填空题(本大题共 6 小题,每小题 3 分,满分 18 分) 11.已知∠ABC=300,BD 是∠ABC 的平分线,则∠ABD= 度。 12.不等式 1x ≤10 的解集是 . 13.分解因式:  aa 82 . 14.如图 4,在等边△ABC 中,AB=6,D 是 BC 上一点,且 BC=3BD, △ABD 绕点 A 旋转后得到△ACE,则 CE 的长度为 . 15.已知关于 x 的一元两次方程 0322  kxx 有两个不相等的根,则 k 的值 为 . 16.如图 5,在标有刻度的直线l 上,从点 A 开始, 以 AB=1 为直径画半圆,记为第 1 个半 圆; 以 BC=2 为直径画半圆,记为第 2 个半 圆; 以 CD=4 为直径画半圆,记为第 3 个半圆; 以 DE=8 为直径画半圆,记为第 4 个半圆. ……,按此规律,连续画半圆,则第 4 个 半圆的面积是第 3 个半圆面积的 倍。第 n 个半圆的面积 为 .(结果保留 ) 三、解答题(本大题共 9 小题,满分 102 分,解答应写出文字说明、证明过程或演算 步骤) 17.(本小题满分 9 分) 解方程 组: 18. (本小题满分 9 分) 如图 6,点 D 在 AB 上,点 E 在 AC 上,AB=AC, ∠B=∠C. 求证:BE=CD. 19. (本小题满分 10 分) 广州市努力改善空气质量,近年来空气质量 明显好转。根据广州市环境保护局公布的 2006-2010 这五年各年的全年空气质量优良 的天数。绘制拆线图如图 7,根据图中的信息 回答: (1)、这五年的全年空气质量优良的天数的 中位数是 .极差 是 . (2)、这五年的全年空气质量优良的天数与它前一年相比较,增加最多的是 年。(填写年份) (3)、求这五年的全年空气质量优良的天数的平均数。 20. (本小题满分 10 分) 新 课标 第一网 21. (本小题满分 12 分)      123 8 yx yx   的值。,求已知: )()(511 baa b bab ababa  甲已两个袋中均装有三张除所标的数值外完全相同的卡片,甲袋中的三张卡片上所标 的数值分别为 317 、、 ,乙袋中的三 张卡片上所标的数值分别为 ,、、612 先从甲袋中随机取出一张卡片,用 x 表示取出的 卡片上标的数值,再从乙袋中随机取出一张卡片,用 y 表示取出的卡片上标的数值。 把 x 、 y 分别作为点 A 的横坐标与纵坐标。 (1)用适当的方法写出点 A( x 、 y )的所有情况。 (2)求点 A 落在第三象限的概率。 22. (本小题满分 12 分) 如图 8,⊙P 的圆心为 P(-3,2),半径为 3,直线 MN 过点 M(5,0)且平行于 y 轴, 点 M 在点 N 的上方。 (1)、在图中作出⊙P 关于 y 轴对称的⊙P',根据作图直接写出⊙P'与直线 MN 的 位置关系;w ww. X kb 1.c om (2)、若点 N 在(1)⊙P'上,求 PN 的长。 23. (本小题满分 12 分) 某城市居民用水实行阶梯收费,每户每月用水量如果未超过 20 吨,按每吨 1.9 元收 费;每户每月用水量如果超过 20 吨,未超过的部分仍按每吨 1.9 元收费,超过部分则 按每吨 2.8 元收费。设某户每月用水量为 x 吨,应收水费为 y 元。 (1) 分别写每月用水量未超过 20 吨和超过 20 吨时, 的函数关系式。与xy (2) 若该城市某户 5 月份水费平均为每吨 2.2 元,求该户 5 月份用水多少吨? 24. (本小题满分 14 分) 如图 9,抛物 线与 x 轴交于 A、B 两点(点 A 在点 B 的左侧)。与 y 轴交于点 C. (1)、求点 A、B 的坐标; (2)、设 D 为已知抛物线的对称轴上的任意一点。当△ACD 的面积等于△ACB 的面积 时,求点 D 的坐标; 34 3 8 3 2  xxy (3)、若直线l 经过点 E(4,0),M 为直线l 上的动点,当以 A、B、M 为顶点所作的 直角三角形有且只有三个时,求直线l 的解析式。 25. (本小题满分 14 分)www .xk b1 .co m 如图 10,在平行四边形 ABCD 中,AB=5,BC=10,F 为 AD 的中点。CE⊥AB 于点 E,设∠ ABC=α(600≤<α<900). (1)、当α=600 时,求 CE 的长。 (2)、当 600≤<α<900 时, ①是否存在正整数 k ,使得∠EFD= k ∠AEF?若存 在,求出 k 的值;若不存在,请说明理由。 ②连接 CF,当 CE2-CF2 取最大值时,求 tan ∠DCF 的值。 2012 年湖北省武汉市中考数学试卷 一.选择题(共 12 小题) 1.(2012 武汉)在 2.5,﹣2.5,0,3 这四个数种,最小的数是( ) A. 2.5 B. ﹣2.5 C. 0 D. 3 考点:有理数大小比较。 解答:解:∵﹣2.5<0<2.5<3, ∴最小的数是﹣2.5, 故选 B. 2.(2012 武汉)若 在实数范围内有意义,则 x 的取值范围是( ) A. x<3 B. x≤3 C. x>3 D. x≥3 考点:二次根式有意义的条件。 解答:解:根据题意得,x﹣3≥0, 解得 x≥3. 故选 D. 3.(2012 武汉)在数轴上表示不等式 x﹣1<0 的解集,正确的是( ) A. B. C. D. 考点:在数轴上表示不等式的解集;解一元一次不等式。 解答:解:x﹣1<0, ∴x<1, 在数轴上表示不等式的解集为: , 故选 B. 4.(2012 武汉)从标号分别为 1,2,3,4,5 的 5 张卡片中,随机抽取 1 张.下列事件 中,必然事件是( ) A. 标号小于 6 B. 标号大于 6 C. 标号是奇数 D. 标号是 3 考点:随机事件。 解答:解:A.是一定发生的事件,是必然事件,故选项正确; B.是不可能发生的事件,故选项错误; C.是随机事件,故选项错误; D.是随机事件,故选项错误. 故选 A. 5.(2012 武汉)若 x1,x2 是一元二次方程 x2﹣3x+2=0 的两根,则 x1+x2 的值是( ) A. ﹣2 B. 2 C. 3 D. 1 考点:根与系数的关系。 解答:解:由一元二次方程 x2﹣3x+2=0, ∴x1+x2=3, 故选 C. 6.(2012 武汉)某市 2012 年在校初中生的人数约为 23 万.数 230000 用科学记数法表示为 ( ) A. 23×104 B. 2.3×105 C. 0.23×103 D. 0.023×106 考点:科学记数法—表示较大的数。 解答:解:23 万=230 000=2.3×105. 故选 B. 7.(2012 武汉)如图,矩形 ABCD 中,点 E 在边 AB 上,将矩形 ABCD 沿直线 DE 折叠, 点 A 恰好落在边 BC 的点 F 处.若 AE=5,BF=3,则 CD 的长是( ) A. 7 B. 8 C. 9 D. 10 考点:翻折变换(折叠问题)。 解答:解:∵△DEF 由△DEA 翻折而成, ∴EF=AE=5, 在 Rt△BEF 中, ∵EF=5,BF=3, ∴BE= = =4, ∴AB=AE+BE=5+4=9, ∵四边形 ABCD 是矩形, ∴CD=AB=9. 故选 C. 8.(2012 武汉)如图,是由 4 个相同小正方体组合而成的几何体,它的左视图是( ) A. B. C. D. 考点:简单组合体的三视图。 解答:解:从左边看得到的是两个叠在一起的正方形. 故选 D. 9.(2012 武汉)一列数 a1,a2,a3,…,其中 a1= ,an= (n 为不小于 2 的整数), 则 a4 的值为( ) A. B. C. D. 考点:规律型:数字的变化类。 解答:解:将 a1= 代入 an= 得到 a2= = , 将 a2= 代入 an= 得到 a3= = , 将 a3= 代入 an= 得到 a4= = . 故选 A. 10.(2012 武汉)对某校八年级随机抽取若干名学生进行体能测试,成绩记为 1 分,2 分, 3 分,4 分 4 个等级,将调查结果绘制成如下条形统计图和扇形统计图.根据图中信息,这 些学生的平均分数是( ) A. 2.25 B. 2.5 C. 2.95 D. 3 考点:加权平均数;扇形统计图;条形统计图。 解答:解:总人数为 12÷30%=40 人, ∴3 分的有 40×42.5%=17 人 2 分的有 8 人 ∴平均分为: =2.95 故选 C. 11.(2012 武汉)甲、乙两人在直线跑道上同起点、同终点、同方向匀速跑步 500 米,先 到终点的人原地休息.已知甲先出发 2 秒.在跑步过程中,甲、乙两人的距离 y(米)与 乙出发的时间 t(秒)之间的关系如图所示,给出以下结论:①a=8;②b=92; ③c=123.其中正确的是( ) A. ①②③ B. 仅有①② C. 仅有①③ D. 仅有②③ 考点:一次函数的应用。 解答:解:甲的速度为:8÷2=4 米/秒; 乙的速度为:500÷100=5 米/秒; b=5×100﹣4×(100+2)=92 米; 5a﹣4×(a+2)=0, 解得 a=8, c=100+92÷4=123, ∴正确的有①②③. 故选 A. 12.(2012 武汉)在面积为 15 的平行四边形 ABCD 中,过点 A 作 AE 垂直于直线 BC 于点 E,作 AF 垂直于直线 CD 于点 F,若 AB=5,BC=6,则 CE+CF 的值为( ) A. 11+ B. 11﹣ C. 11+ 或 11﹣ D. 11﹣ 或 1+ 考点:平行四边形的性质;勾股定理;相似三角形的判定与性质。 解答:解:∵四边形 ABCD 是平行四边形, ∴AB=CD=5,BC=AD=6, ①如图: 由平行四边形面积公式地:BC×AE=CD×AF=15, 求出 AE= ,AF=3, 在 Rt△ABE 和 Rt△ADF 中,由勾股定理得:AB2=AE2+BE2, 把 AB=5,AE= 代入求出 BE= , 同理 DF=3 , ∴CE=6﹣ ,CF=5﹣3 , 即 CE+CF=11﹣ , ②如图: ∵AB=5,AE= ,在△ABE 中,由勾股定理得:BE= , 同理 DF=3 , 由①知:CE=6+ ,CF=5+3 , ∴CE+CF=11+ , 故选 C. 二.填空题(共 4 小题) 13.tan60°= . 考点:特殊角的三角函数值。 解答:解:tan60°的值为 . 故答案为: . 14.(2012 武汉)某校九(1)班 8 名学生的体重(单位:kg)分别是 39,40,43,43, 43,45,45,46.这组数据的众数是 . 考点:众数。 解答:解:在这一组数据中 43 是出现了 3 次,次数最多, 故众数是 43. 故答案为:43. 15.(2012 武汉)如图,点 A 在双曲线 y= 的第一象限的那一支上,AB 垂直于 x 轴与点 B,点 C 在 x 轴正半轴上,且 OC=2AB,点 E 在线段 AC 上,且 AE=3EC,点 D 为 OB 的 中点,若△ADE 的面积为 3,则 k 的值为 . 考点:反比例函数综合题。 解答:解:连 DC,如图, ∵AE=3EC,△ADE 的面积为 3, ∴△CDE 的面积为 1, ∴△ADC 的面积为 4, 设 A 点坐标为(a,b),则 AB=a,OC=2AB=2a, 而点 D 为 OB 的中点, ∴BD=OD= b, ∵S 梯形 OBAC=S△ABO+S△ADC+S△ODC, ∴ (a+2a)×b= a× b+4+ ×2a× b, ∴ab= , 把 A(a,b)代入双曲线 y= , ∴k=ab= . 故答案为 . 16.(2012 武汉)在平面直角坐标系中,点 A 的坐标为(3.0),点 B 为 y 轴正半轴上的一 点,点 C 是第一象限内一点,且 AC=2.设 tan∠BOC=m,则 m 的取值范围是 . 考点:切线的性质;坐标与图形性质;勾股定理;锐角三角函数的定义。 解答:解: 当 OC 与圆 A 相切(即到 C′点)时,∠BOC 最小, AC′=2,OA=3,由勾股定理得:OC′= , ∵∠BOA=∠AC′O=90°, ∴∠BOC′+∠AOC′=90°,∠C′AO+∠AOC′=90°, ∴∠BOC′=∠OAC′, tan∠BOC= = , 随着 C 的移动,∠BOC 越来越大,但不到 E 点,即∠BOC<90°, ∴tan∠BOC≥ , 故答案为:≥ . 三.解答题(共 9 小题) 17.(2012 武汉)解方程: . 考点:解分式方程。 解答:解:方程两边都乘以 3x(x+5)得, 6x=x+5, 解得 x=1, 检验:当 x=1 时,3x(x+5)=3×1×(1+5)=18≠0, 所以 x=1 是方程的根, 因此,原分式方程的解是 x=1. 18.(2012 武汉)在平面直角坐标系中,直线 y=kx+3 经过点(﹣1,1),求不等式 kx+3< 0 的解集. 考点:一次函数与一元一次不等式。 解答:解:如图,∵将(﹣1,1)代入 y=kx+3 得 1=﹣k+3, ∴k=2, 即 y=2x+3, 当 y=0 时,x=﹣ , 即与 x 轴的交点坐标是(﹣ ,0), 由图象可知:不等式 kx+3<0 的解集是 x<﹣ . 19.(2012 武汉)如图 CE=CB,CD=CA,∠DCA=∠ECB,求证:DE=AB. 考点:全等三角形的判定与性质。 解答:证明:∵∠DCA=∠ECB, ∴∠DCA+∠ACE=∠BCE+∠ACE, ∴∠DCE=∠ACB, ∵在△DCE 和△ACB 中 , ∴△DCE≌△ACB, ∴DE=AB. 20.(2012 武汉)一个口袋中有 4 个相同的小球,分别与写有字母 A,B,C,D,随机地 抽出一个小球后放回,再随机地抽出一个小球. (1)使用列表法或树形法中的一种,列举出两次抽出的球上字母的所有可能结果; (2)求两次抽出的球上字母相同的概率. 考点:列表法与树状图法。 解答:解:(1)如图所示: 则共有 16 种等可能的结果; (2)由树形图可以看出两次字母相同的概率为 = . 21.(2012 武汉)如图,在平面直角坐标系中,点 A,B 的坐标分别为(﹣1,3),(﹣4, 1),先将线段 AB 沿一确定方向平移得到线段 A1B1,点 A 的对应点为 A1,点 B1 的坐标为 (0,2),在将线段 A1B1 绕远点 O 顺时针旋转 90°得到线段 A2B2,点 A1 的对应点为点 A2. (1)画出线段 A1B1,A2B2; (2)直接写出在这两次变换过程中,点 A 经过 A1 到达 A2 的路径长. 考点:作图-旋转变换;弧长的计算。 解答:解:(1)所作图形如下: (2)由图形可得:AA1= , = = , 故点 A 经过 A1 到达 A2 的路径长为: + . 22.(2012 武汉)在锐角三角形 ABC 中,BC=4,sinA= , (1)如图 1,求三角形 ABC 外接圆的直径; (2)如图 2,点 I 为三角形 ABC 的内心,BA=BC,求 AI 的长. 考点:三角形的内切圆与内心;三角形的面积;勾股定理;圆周角定理;解直角三角形。 解答:(1)解:作直径 CD,连接 BD, ∵CD 是直径, ∴∠DBC=90°,∠A=∠D, ∵BC=4,sin ∠A= , ∴sin∠D= = , ∴CD=5, 答:三角形 ABC 外接圆的直径是 5. (2)解:连接 IC.BI,且延长 BI 交 AC 于 F,过 I 作 IE⊥AB 于 E, ∵AB=BC=4,I 为△ABC 内心, ∴BF⊥AC,AF=CF, ∵sin∠A= = , ∴BF= , 在 Rt△ABF 中,由勾股定理得:AF=CF= , AC=2AF= , ∵I 是△ABC 内心,IE⊥AB,IF⊥AC,IG⊥BC, ∴IE=IF=IG, 设 IE=IF=IG=R, ∵△ABI、△ACI、△BCI 的面积之和等于△ABC 的面积, ∴ AB×R+ BC×R+ AC×R= AC×BF, 即 4×R+4×R+ ×R= × , ∴R= , 在△AIF 中,AF= ,IF= ,由勾股定理得:AI= . 答:AI 的长是 . 23.(2012 武汉)如图,小河上有一拱桥,拱桥及河道的截面轮廓线由抛物线的一部分 ACB 和矩形的三边 AE,ED,DB 组成,已知河底 ED 是水平的,ED=16 米,AE=8 米,抛 物线的顶点 C 到 ED 的距离是 11 米,以 ED 所在的直线为 x 轴,抛物线的对称轴为 y 轴建 立平面直角坐标系. (1)求抛物线的解析式; (2)已知从某时刻开始的 40 小时内,水面与河底 ED 的距离 h(单位:米)随时间 t(单 位:时)的变化满足函数关系 h=﹣ (t﹣19)2+8(0≤t≤40),且当水面到顶点 C 的距离 不大于 5 米时,需禁止船只通行,请通过计算说明:在这一时段内,需多少小时禁止船只 通行? 考点:二次函数的应用。 解答:解:(1)设抛物线的为 y=ax2+11,由题意得 B(8,8), ∴64a+11=8, 解得 a=﹣ , ∴y=﹣ x2+11; (2)水面到顶点 C 的距离不大于 5 米时,即水面与河底 ED 的距离 h 至多为 6, ∴6=﹣ (t﹣19)2+8, 解得 t1=35,t2=3, ∴35﹣3=32(小时). 答:需 32 小时禁止船只通行. 24.(2012 武汉)已知△ABC 中,AB= ,AC= ,BC=6 (1)如图 1,点 M 为 AB 的中点,在线段 AC 上取点 M,使△AMN 与△ABC 相似,求线 段 MN 的长; (2)如图 2,是由 100 个边长为 1 的小正方形组成的 10×10 的正方形网格,设顶点在这些 小正方形顶点的三角形为格点三角形. ①请你在所给的网格中画出格点△A1B1C1 与△ABC 全等(画出一个即可,不需证明) ②试直接写出所给的网格中与△ABC 相似且面积最大的格点三角形的个数,并画出其中 一个(不需证明). 考点:作图—相似变换。 解答:解:(1)①△AMN∽△ABC, ∴ = ∵M 为 AB 中点,AB=2 , ∴AM= , ∵BC=6, ∴MN=3; ②△AMN∽△ACB, = , ∵BC=6,AC=4 ,AM= , ∴MN=1.5; (2)①如图所示: ②每条对角线处可作 4 个三角形与原三角形相似,那么共有 8 个. 25.(2012 武汉)如图 1,点 A 为抛物线 C1:y= x2﹣2 的顶点,点 B 的坐标为(1,0)直 线 AB 交抛物线 C1 于另一点 C (1)求点 C 的坐标; (2)如图 1,平行于 y 轴的直线 x=3 交直线 AB 于点 D,交抛物线 C1 于点 E,平行于 y 轴 的直线 x=a 交直线 AB 于 F,交抛物线 C1 于 G,若 FG:DE=4:3,求 a 的值; (3)如图 2,将抛物线 C1 向下平移 m(m>0)个单位得到抛物线 C2,且抛物线 C2 的顶 点为点 P,交 x 轴于点 M,交射线 BC 于点 N.NQ⊥x 轴于点 Q,当 NP 平分∠MNQ 时, 求 m 的值. 考点:二次函数综合题。 解答:解:(1)当 x=0 时,y=﹣2;∴A(0,﹣2). 设直线 AB 的解析式为 y=kx+b,则: ,解得 ∴直线 AB 解析式为 y=2x﹣2. ∵点 C 为直线 y=2x﹣2 与抛物线 y= x2﹣2 的交点,则点 C 的横、纵坐标满足: ,解得 、 (舍) ∴点 C 的坐标为(4,6). (2)直线 x=3 分别交直线 AB 和抛物线 C1 于 D.E 两点. ∴yD=4,yE= ,∴DE= . ∵FG=DE=4:3,∴FG=2. ∵直线 x=a 分别交直线 AB 和抛物线 C1 于 F、G 两点. ∴yF=2a﹣2,yG= a2﹣2 ∴FG=|2a﹣ a2|=2, 解得:a1=2,a2=﹣2+2 ,a3=2﹣2 . (3)设直线 MN 交 y 轴于 T,过点 N 做 NH⊥y 轴于点 H; 设点 M 的坐标为(t,0),抛物线 C2 的解析式为 y= x2﹣2﹣m; ∴0=﹣ t2﹣2﹣m,∴﹣2﹣m=﹣ t2. ∴y= x2﹣ t2,∴点 P 坐标为(0,﹣ t 2). ∵点 N 是直线 AB 与抛物线 y= x2﹣ t2 的交点,则点 N 的横、纵坐标满足: ,解得 、 (舍) ∴N(2﹣t,2﹣2t). NQ=2﹣2t,MQ=2﹣2t, ∴MQ=NQ,∴∠MNQ=45°. ∴△MOT、△NHT 均为等腰直角三角形, ∴MO=OT,HT=HN ∴OT=4,NT=﹣ ,NH= (2﹣t),PT=﹣t+ t2. ∵PN 平分∠MNQ, ∴PT=NT, ∴﹣t+ t2= (2﹣t), ∴t1=﹣2 ,t2=2(舍) ﹣2﹣m=﹣ t2=﹣ (﹣2 )2,∴m=2. 成都市二 0 一二年高中阶段教育学校统一招生考试试卷 (含成都市初三毕业会考) 数 学 A 卷(共 100 分) 第 1 卷(选择题.共 30 分) 一、选择题(本大题共 l0 个小题,每小题 3 分,共 30 分.每小题均有四个选项,其中 只有一项符合题目要求) 1.(2012 成都) 3 的绝对值是( ) A.3 B. 3 C. 1 3 D. 1 3  考点:绝对值。 解答:解:|﹣3|=﹣(﹣3)=3. 故选 A. 2.(2012 成都)函数 1 2y x   中,自变量 x 的取值范围是( ) A. 2x  B. 2x  C. 2x  D. 2x   考点:函数自变量的取值范围。 解答:解:根据题意得,x﹣2≠0, 解得 x≠2. 故选 C. 3.(2012 成都)如图所示的几何体是由 4 个相同的小正方体组成.其主视图为 ( ) A. B. C. D. 考点:简单组合体的三视图。 解答:解:从正面看得到 2 列正方形的个数依次为 2,1, 故选:D. 4.(2012 成都)下列计算正确的是( ) A. 22 3a a a  B. 2 3 5a a a  C. 3 3a a  D. 3 3( )a a  考点:同底数幂的除法;合并同类项;同底数幂的乘法;幂的乘方与积的乘方。 解答:解:A、a+2a=3a,故本选项错误; B、a2a3=a2+3=a5,故本选项正确; C、a3÷a=a3﹣1=a2,故本选项错误; D、(﹣a)3=﹣a3,故本选项错误. 故选 B 5.(2012 成都)成都地铁二号线工程即将竣工,通车后与地铁一号线呈“十”字交 叉,城市交通通行和转换能力将成倍增长.该工程投资预算约为 930 000 万元,这一 数据用科学记数法表示为( ) A. 59.3 10 万元 B. 69.3 10 万元 C. 493 10 万元 D. 60.93 10 万元 考点:科学记数法—表示较大的数。 解答:解:930 000=9.3×105. 故选 A. 6.(2012 成都)如图,在平面直角坐标系 xOy 中,点 P( 3 ,5)关于 y 轴的对称点的 坐标为( ) A.( 3 , 5 ) B.(3,5) C.(3. 5 ) D.(5, 3 ) 考点:关于 x 轴、y 轴对称的点的坐标。 解答:解:点 P(﹣3,5)关于 y 轴的对称点的坐标为(3,5). 故选 B. 7.(2012 成都)已知两圆外切,圆心距为 5cm,若其中一个圆的半径是 3cm,则另一 个圆的半径是( ) A. 8cm B.5cm C.3cm D.2cm 考点:圆与圆的位置关系。 解答:解:另一个圆的半径=5﹣3=2cm. 故选 D. 8.(2012 成都)分式方程 3 1 2 1x x   的解为( ) A. 1x  B. 2x  C. 3x  D. 4x  考点:解分式方程。 解答:解: 3 1 2 1x x   , 去分母得:3x﹣3=2x, 移项得:3x﹣2x=3, 合并同类项得:x=3, 检验:把 x=3 代入最简公分母 2x(x﹣1)=12≠0,故 x=3 是原方程的解, 故原方程的解为: 3x  , 故选:C. 9.(2012 成都)如图.在菱形 ABCD 中,对角线 AC,BD 交于点 O,下列说法错误..的是 ( ) A.AB∥DC B.AC=BD C.AC⊥BD D.OA=OC 考点:菱形的性质。 解答:解:A、菱形的对边平行且相等,所以 AB∥DC,故本选项正确; B、菱形的对角线不一定相等,故本选项错误; C、菱形的对角线一定垂直,AC⊥BD,故本选项正确; D、菱形的对角线互相平分,OA=OC,故本选项正确. 故选 B. 10.(2012 成都)一件商品的原价是 100 元,经过两次提价后的价格为 121 元,如果 每次提价的百分率都 是 x ,根据题意,下面列出的方程正确的是( ) A.100(1 ) 121x  B. 100(1 ) 121x  C. 2100(1 ) 121x  D. 2100(1 ) 121x  考点:由实际问题抽象出一元二次方程。 解答:解:设平均每次提价的百分率为 x, 根据题意得: 2100(1 ) 121x  , 故选 C. 第Ⅱ卷(非选择题,共 70 分) 二、填空题(本大题共 4 个小题,每小题 4 分,共 16 分) 1l.(2012 成都)分解因式: 2 5x x =________. 考点:因式分解-提公因式法。 解答:解:x2﹣5x=x(x﹣5). 故答案为:x(x﹣5). 12.(2012 成都)如图,将 ABCD 的一边 BC 延长至 E,若∠A=110°,则∠ 1=________. 考点:平行四边形的性质。 解答:解:∵平行四边形 ABCD 的∠A=110°, ∴∠BCD=∠A=110°, ∴∠1=180°﹣∠BCD=180°﹣110°=70°. 故答案为:70°. 13.(2012 成都)商店某天销售了 ll 件衬衫,其领口尺寸统计如下表: 则这 ll 件衬衫领口尺寸的众数是________cm,中位数是________cm. 考点:众数;中位数。 解答:解:同一尺寸最多的是 39cm,共有 4 件, 所以,众数是 39cm, 11 件衬衫按照尺寸从小到大排列,第 6 件的尺寸是 40cm, 所以中位数是 40cm. 故答案为:39,40. 14.(2012 成都)如图,AB 是⊙O 的弦,OC⊥AB 于 C.若 AB= 2 3 ,0C=1,则半径 OB 的长为________. 考点:垂径定理;勾股定理。 解答:解:∵AB 是⊙O 的弦,OC⊥AB 于 C,AB= , ∴BC= AB= ∵0C=1, ∴在 Rt△OBC 中, OB= = =2. 故答案为:2. 三、解答题(本大题共 6 个小题,共 54 分) 15.(1)(2012 成都)计算: 0 24cos45 8 ( 3) ( 1)     考点:实数的运算;零指数幂;特殊角的三角函数值。 解答:解:原式=4× ﹣2 +1+1=2 ﹣2 +2=2; 15.(2)(2012 成都)解不等式组: 2 0 2 1 13 x x     考点:实解一元一次不等式组。 解答:解: , 解不等式①得,x<2, 解不等式②得,x≥1, 所以不等式组的解集是 1≤x<2. 16.(2012 成都)(本小题满分 6 分) 化简: 2 2(1 )b a a b a b    考点:分式的混合运算。 解答:解:原式= • = • =a﹣b. 17.(2012 成都)(本小题满分 8 分) 如图,在一次测量活动中,小华站在离旗杆底部(B 处)6 米的 D 处,仰望旗杆顶端 A,测得仰角为 60°,眼睛离地面的距离 ED 为 1.5 米.试帮助小华求出旗杆 AB 的高 度.(结果精确到 0.1 米, 3 1.732 ) 考点:解直角三角形的应用-仰角俯角问题。 解答:解:∵BD=CE=6m,∠AEC=60°, ∴AC=CE•tan60°=6× =6 ≈6×1.732≈10.4m, ∴AB=AC+DE=10.4+1.5=11.9m. 答:旗杆 AB 的高度是 11.9 米. 18.(2012 成都)(本小题满分 8 分) 如图,一次函数 2y x b   (b 为常数)的图象与反比例函数 ky x  ( k 为常数,且 k ≠0)的图象交于 A,B 两点,且点 A 的坐标为( 1 ,4). (1)分别求出反比例函数及一次函数的表达式; (2)求点 B 的坐标. 考点:反比例函数与一次函数的交点问题。 解答:解:(1)∵两函数图象相交于点 A(﹣1,4), ∴﹣2×(﹣1)+b=4, =4, 解得 b=2,k=﹣4, ∴反比例函数的表达式为 y=﹣ , 一次函数的表达式为 y=﹣2x+2; (2)联立 , 解得 (舍去), , 所以,点 B 的坐标为(2,﹣2). 19.(2012 成都)(本小题满分 10 分) 某校将举办“心怀感恩·孝敬父母”的活动,为此,校学生会就全校 1 000 名同学 暑假期间平均每天做家务活的时间,随机抽取部分同学进行调查,并绘制成如下条形 统计图. (1)本次调查抽取的人数为_______,估计全校同学在暑假期间平均每天做家务活 的时间在 40 分钟以上(含 40 分钟)的人数为_______; (2)校学生会拟在表现突出的甲、乙、丙、丁四名同学中,随机抽取两名同学向 全校汇报.请用树状图或列表法表示出所有可能的结果,并求恰好抽到甲、乙两名同 学的概率. 考点:频数(率)分布直方图;用样本估计总体;列表法与树状图法。 解答:解:(1)8+10+16+12+4=50 人, 1000× =320 人; (2)列表如下: 共有 12 种情况,恰好抽到甲、乙两名同学的是 2 种, 所以 P(恰好抽到甲、乙两名同学)= = . 20.(2012 成都)(本小题满分 10 分) 如图,△ABC 和△DEF 是两个全等的等腰直角三角形,∠BAC=∠EDF=90°,△DEF 的顶点 E 与△ABC 的斜边 BC 的中点重合.将△DEF 绕点 E 旋转,旋转过程中,线段 DE 与线段 AB 相交于点 P,线段 EF 与射线 CA 相交于点 Q. (1)如图①,当点 Q 在线段 AC 上,且 AP=AQ 时,求证:△BPE≌△CQE; (2)如图②,当点 Q 在线段 CA 的延长线上时,求证:△BPE∽△CEQ;并求当 BP= a ,CQ= 9 2 a 时,P、Q 两点间的距离 (用含 a 的代数式表示). 考点:相似三角形的判定与性质;全等三角形的判定与性质;等腰直角三角形;旋转的性 质。 解答:(1)证明:∵△ABC 是等腰直角三角形, ∴∠B=∠C=45°,AB=AC, ∵AP=AQ, ∴BP=CQ, ∵E 是 BC 的中点, ∴BE=CE, 在△BPE 和△CQE 中, ∵ , ∴△BPE≌△CQE(SAS); (2)解:∵△ABC 和△DEF 是两个全等的等腰直角三角形, ∴∠B=∠C=∠DEF=45°, ∵∠BEQ=∠EQC+∠C, 即∠BEP+∠DEF=∠EQC+∠C, ∴∠BEP+45°=∠EQC+45°, ∴∠BEP=∠EQC, ∴△BPE∽△CEQ, ∴ , ∵BP=a,CQ= a,BE=CE, ∴BE=CE= a, ∴BC=3 a, ∴AB=AC=BC•sin45°=3a, ∴AQ=CQ﹣AC= a,PA=AB﹣BP=2a, 连接 PQ, 在 Rt△APQ 中,PQ= = a. B 卷(共 50 分) 一、填空题(本大题共 5 个小题,每小题 4 分,共 20 分) 21.(2012 成都)已知当 1x  时, 22ax bx 的值为 3,则当 2x  时, 2ax bx 的值 为________. 考点:代数式求值。 解答:解:将 x=1 代入 2ax2+bx=3 得 2a+b=3, 将 x=2 代入 ax2+bx 得 4a+2b=2(2a+b)=2×3=6. 故答案为 6. 22.(2012 成都)一个几何体由圆锥和圆柱组成,其尺寸如图所示,则该几何体的全 面积(即表面积)为________ (结果保留 ) 考点:圆锥的计算;圆柱的计算。 解答:解:圆锥的母线长是: =5. 圆锥的侧面积是: ×8π×5=20π, 圆柱的侧面积是:8π×4=32π. 几何体的下底面面积是:π×42=16π 则该几何体的全面积(即表面积)为:20π+32π+16π=68π. 故答案是:68π. 23.(2012 成都)有七张正面分别标有数字 3 , 2 , 1 ,0,l,2,3 的卡片,它 们除数字不同外其余全部相同.现将它们背面朝上,洗匀后从中随机抽取一张,记卡 片上的数字为 a ,则使关于 x 的一元二次方程 2 2( 1) ( 3) 0x a x a a     有两个不 相等的实数根,且以 x 为自变量的二次函数 2 2( 1) 2y x a x a     的图象不经过...点 (1,O)的概率是________. 考点:二次函数图象上点的坐标特征;根的判别式;概率公式。 解答:解:∵x2﹣2(a﹣1)x+a(a﹣3)=0 有两个不相等的实数根, ∴△>0, ∴[﹣2(a﹣1)]2﹣4a(a﹣3)>0, ∴a>﹣1, 将(1,O)代入 y=x2﹣(a2+1)x﹣a+2 得,a2+a﹣2=0, 解得(a﹣1)(a+2)=0, a1=1,a2=﹣2. 可见,符合要求的点为 0,2,3. ∴P= . 故答案为 . 24.(2012 成都)如图,在平面直角坐标系 xOy 中,直线 AB 与 x 轴、y 轴分别交于点 A,B,与反比例函数 ky x  ( k 为常数,且 0k  )在第一象限的图象交于点 E,F.过点 E 作 EM⊥y 轴于 M,过点 F 作 FN⊥x 轴于 N,直线 EM 与 FN 交于点 C.若 BE 1 BF m  ( m 为 大于 l 的常数).记△CEF 的面积为 1S ,△OEF 的面积为 2S ,则 1 2 S S =________. (用含 m 的代数式表示) 考点:反比例函数综合题。 解答:解:过点 F 作 FD⊥BO 于点 D,EW⊥AO 于点 W, ∵ ,∴ = , 设 E 点坐标为:(x,my),则 F 点坐标为:(mx,y), ∴△CEF 的面积为:S1= (mx﹣x)(my﹣y)= (m﹣1)2xy, ∵△OEF 的面积为:S2=S 矩形 CNOM﹣S1﹣S△MEO﹣S△FON, =MC•CN﹣ (m﹣1)2xy﹣ ME•MO﹣ FN•NO, =mx•my﹣ (m﹣1)2xy﹣ x•my﹣ y•mx, =m2xy﹣ (m﹣1)2xy﹣mxy, = (m2﹣1)xy, = (m+1)(m﹣1)xy, ∴ = = . 故答案为: . 25.(2012 成都)如图,长方形纸片 ABCD 中,AB=8cm,AD=6cm,按下列步骤进行裁 剪和拼图: 第一步:如图①,在线段 AD 上任意取一点 E,沿 EB,EC 剪下一个三角形纸片 EBC(余下部分不再使用); 第二步:如图②,沿三角形 EBC 的中位线 GH 将纸片剪成两部分,并在线段 GH 上 任意取一点 M,线段 BC 上任意取一点 N,沿 MN 将梯形纸片 GBCH 剪成两部分; 第三步:如图③,将 MN 左侧纸片绕 G 点按顺时针方向旋转 180°,使线段 GB 与 GE 重合,将 MN 右侧纸片绕 H 点按逆时针方向旋转 180°,使线段 HC 与 HE 重合,拼成一个 与三角形纸片 EBC 面积相等的四边形纸片. (注:裁剪和拼图过程均无缝且不重叠) 则拼成的这个四边形纸片的周长的最小值为________cm,最大值为________cm. 考点:图形的剪拼;三角形中位线定理;矩形的性质;旋转的性质。 解答:解:画出第三步剪拼之后的四边形 M1N1N2M2 的示意图,如答图 1 所示. 图中,N1N2=EN1+EN2=NB+NC=BC, M1M2=M1G+GM+MH+M2H=2(GM+MH)=2GH=BC(三角形中位线定理), 又∵M1M2∥N1N2,∴四边形 M1N1N2M2 是一个平行四边形, 其周长为 2N1N2+2M1N1=2BC+2MN. ∵BC=6 为定值,∴四边形的周长取决于 MN 的大小. 如答图 2 所示,是剪拼之前的完整示意图. 过 G、H 点作 BC 边的平行线,分别交 AB、CD 于 P 点、Q 点,则四边形 PBCQ 是一个矩 形,这个矩形是矩形 ABCD 的一半. ∵M 是线段 PQ 上的任意一点,N 是线段 BC 上的任意一点, 根据垂线段最短,得到 MN 的最小值为 PQ 与 BC 平行线之间的距离,即 MN 最小值为 4; 而 MN 的最大值等于矩形对角线的长度,即 = = ∵四边形 M1N1N2M2 的周长=2BC+2MN=12+2MN, ∴四边形 M1N1N2M2 周长的最小值为 12+2×4=20, 最大值为 12+2× =12+ . 故答案为:20,12+ . 二、解答题(本大题共 3 个小题,共 30 分) 26.(2012 成都)(本小题满分 8 分) “城市发展 交通先行”,成都市今年在中心城区启动了缓堵保畅的二环路高架桥 快速通道建设工程,建成后将大大提升二环路的通行能力.研究表明,某种情况下, 高架桥上的车流速度 V(单位:千米/时)是车流密度 x (单位:辆/千米)的函数,且当 0< x ≤28 时,V=80;当 28< x ≤188 时,V 是 x 的一次函数. 函数关系如图所示. (1)求当 28< x ≤188 时,V 关于 x 的函数表达式; (2)若车流速度 V 不低于 50 千米/时,求当车流密度 x 为多少时,车流量 P(单 位:辆/时)达到最大,并求出这一最大值. (注:车流量是单位时间内通过观测点的车辆数,计算公式为:车流量=车流速度 ×车流密度) 考点:一次函数的应用。 解答:解:(1)设函数解析式为 V=kx+b, 则 , 解得: , 故 V 关于 x 的函数表达式为:V=﹣ x+94; (2)由题意得,V=﹣ x+94≥50, 解得:x≤88, 又 P=Vx=(﹣ x+94)x=﹣ x2+94x, 当 0<x≤88 时,函数为增函数,即当 x=88 时,P 取得最大, 故 Pmax=﹣ ×882+94×88=4400. 答:当车流密度达到 88 辆/千米时,车流量 P 达到最大,最大值为 4400 辆/时. 27.(2012 成都)(本小题满分 I0 分) 如图,AB 是⊙O 的直径,弦 CD⊥AB 于 H,过 CD 延长线上一点 E 作⊙O 的切线交 AB 的延长线于 F.切点为 G,连接 AG 交 CD 于 K. (1)求证:KE=GE; (2)若 2KG =KD·GE,试判断 AC 与 EF 的位置关系,并说明理由; (3) 在(2)的条件下,若 sinE= 3 5 ,AK= 2 3 ,求 FG 的长. 考点:切线的性质;勾股定理;垂径定理;圆周角定理;相似三角形的判定与性质;解直 角三角形。 解答:解:(1)如答图 1,连接 OG. ∵EG 为切线,∴∠KGE+∠OGA=90°, ∵CD⊥AB,∴∠AKH+∠OAG=90°, 又 OA=OG,∴∠OGA=∠OAG, ∴∠KGE=∠AKH=∠GKE, ∴KE=GE. (2)AC∥EF,理由为: 连接 GD,如答图 2 所示. ∵KG2=KD•GE,即 = , ∴ = ,又∠KGE=∠GKE, ∴△GKD∽△EGK, ∴∠E=∠AGD,又∠C=∠AGD, ∴∠E=∠C, ∴AC∥EF; (3)连接 OG,OC,如答图 3 所示. sinE=sin∠ACH= ,设 AH=3t,则 AC=5t,CH=4t, ∵KE=GE,AC∥EF,∴CK=AC=5t,∴HK=CK﹣CH=t. 在 Rt△AHK 中,根据勾股定理得 AH2+HK2=AK2, 即(3t)2+t2=( )2,解得 t= . 设⊙O 半径为 r,在 Rt△OCH 中,OC=r,OH=r﹣3t,CH=4t, 由勾股定理得:OH2+CH2=OC2, 即(r﹣3t)2+(4t)2=r2,解得 r= t= . ∵EF 为切线,∴△OGF 为直角三角形, 在 Rt△OGF 中,OG=r= ,tan∠OFG=tan∠CAH= = , ∴FG= = = . 28.(2012 成都)(本小题满分 l2 分) 如图,在平面直角坐标系 xOy 中,一次函数 5 4y x m  ( m 为常数)的图象与 x 轴交于点 A( 3 ,0),与 y 轴交于点 C.以直线 x=1 为对称轴的抛物线 2y ax bx c   ( a b c, , 为常数,且 a ≠0)经过 A,C 两点,并与 x 轴的正半轴交于点 B. (1)求 m 的值及抛物线的函数表达式; (2)设 E 是 y 轴右侧抛物线上一点,过点 E 作直线 AC 的平行线交 x 轴于点 F.是 否存在这样的点 E,使得以 A,C,E,F 为顶点的四边形是平行四边形?若存在,求出 点 E 的坐标及相应的平行四边形的面积;若不存在,请说明理由; (3)若 P 是抛物线对称轴上使△ACP 的周长取得最小值的点,过点 P 任意作一条与 y 轴不平行的直线交抛物线于 1 1 1M ( )x y, , 2 2 2M ( )x y, 两点,试探究 21 1 2 P PM M M M  是 否为定值,并写出探究过程. 考点:二次函数综合题。 解答:解:(1)∵ 经过点(﹣3,0), ∴0= +m,解得 m= , ∴直线解析式为 ,C(0, ). ∵抛物线 y=ax2+bx+c 对称轴为 x=1,且与 x 轴交于 A(﹣3,0),∴另一交点为 B(5, 0), 设抛物线解析式为 y=a(x+3)(x﹣5), ∵抛物线经过 C(0, ), ∴ =a•3(﹣5),解得 a= , ∴抛物线解析式为 y= x2+ x+ ; (2)假设存在点 E 使得以 A、C、E、F 为顶点的四边形是平行四边形, 则 AC∥EF 且 AC=EF.如答图 1, (i)当点 E 在点 E 位置时,过点 E 作 EG⊥x 轴于点 G, ∵AC∥EF,∴∠CAO=∠EFG, 又∵ ,∴△CAO≌△EFG, ∴EG=CO= ,即 yE= , ∴ = xE2+ xE+ ,解得 xE=2(xE=0 与 C 点重合,舍去), ∴E(2, ),S▱ ACEF= ; (ii)当点 E 在点 E′位置时,过点 E′作 E′G′⊥x 轴于点 G′, 同理可求得 E′( +1, ),S▱ ACE′F′= . (3)要使△ACP 的周长最小,只需 AP+CP 最小即可. 如答图 2,连接 BC 交 x=1 于 P 点,因为点 A、B 关于 x=1 对称,根据轴对称性质以及两点 之间线段最短,可知此时 AP+CP 最小(AP+CP 最小值为线段 BC 的长度). ∵B(5,0),C(0, ),∴直线 BC 解析式为 y= x+ , ∵xP=1,∴yP=3,即 P(1,3). 令经过点 P(1,3)的直线为 y=kx+3﹣k, ∵y=kx+3﹣k,y= x2+ x+ , 联立化简得:x2+(4k﹣2)x﹣4k﹣3=0, ∴x1+x2=2﹣4k,x1x2=﹣4k﹣3. ∵y1=kx1+3﹣k,y2=kx2+3﹣k,∴y1﹣y2=k(x1﹣x2). 根据两点间距离公式得到: M1M2= = = ∴M1M2= = =4(1+k2). 又 M1P= = = ; 同理 M2P= ∴M1P•M2P=(1+k2)• =(1+k2) • =(1+k2)• =4(1+k2). ∴M1P•M2P=M1M2, ∴ =1 为定值. 2012 年长沙中考数学试卷解析 一、选择题(在下列各题的四个选项中,只有一项是符合题意的. 1.﹣3 相反数是( ) A. B.﹣3 C.﹣ D.3 解 答: 解:﹣3 相反数是 3. 故选 D. 2.下列平面图形中,既是轴对称图形,又是中心对称图形的是( ) A. B. C. D. 解答: 解:A、是轴对称图形,也是中心对称图形,故本选项正确; B、是轴对称图形,不是中心对称图形,故本选项错误; C、不是轴对称图形,是中心对称图形,故本选项错误; D、是轴对称图形,不是中心对称图形,故本选项错误. 故选 A. 3.甲、乙两学生在军训打靶训练中,打靶的总次数相同,且所中环数的平均数也相同,但 甲的成绩比乙的成绩稳定,那么两者的方差的大小关系是( ) A. < B. > C. = D. 不能确定 解 答: 解:根据方差的意义知,射击成绩比较稳定,则方差较小, ∵甲的成绩比乙的成绩稳定, ∴有:S 甲 2<S 乙 2. 故选 A. 4.一个不等式组的解集在数轴上表示出来如图所示,则下列符合条件的不等式组为 ( ) A. B. C. D. 解答:解:由图示可看出,从﹣1 出发向右画出的折线且表示﹣1 的点是实心圆,表示 x≥ ﹣1; 从 2 出发向左画出的折线且表示 2 的点是空心圆,表示 x<2,所以这个不等式组的 解集为﹣1≤x<2,即: . 故选:C. 5.下列四边形中,两条对角线一定不相等的是( ) A.正方形 B.矩形 C.等腰梯形 D.直角梯形 解答: 解:根据正方形、矩形、等腰梯形的性质,它们的两条对角线一定相等,只有直角 梯形的对角线一定不相等. 故选 D. 6.下列四个角中,最有可能与 70°角互补的是( ) A. B. C. D. 解答: 解:70°角的补角=180°﹣70°=110°,是钝角, 结合各选项,只有 D 选项是钝角, 所以,最有可能与 70°角互补的是 D 选项的角. 故选 D. 7.小明骑自行车上学,开始以正常速度匀速行驶,但行至中途时,自行车出了故障,只好 停下来修车,车修好后,因怕耽误上课,他比修车前加快了速度继续匀速行驶,下面是 行驶路程 s(m)关于时间 t(min)的函数图象,那么符合小明行驶情况的大致图象是 ( ) A . B . C . D . 解答: 解:小明骑自行车上学,开始以正常速度匀速行驶,正常匀速行驶的路程、时间图 象是一条过原点 O 的斜线, 修车时自行车没有运动,所以修车时的路程保持不变是一条平行于横坐标的水平 线, 修车后为了赶时间,他比修车前加快了速度继续匀速行驶,此时的路程、时间图象 仍是一条斜线,只是斜线的倾角变大. 因此选项 A、B、D 都不符合要求. 故选 C. 8.已知:菱形 ABCD 中,对角线 AC 与 BD 相交于点 O,OE∥DC 交 BC 于点 E, AD=6cm,则 OE 的长为( ) A.6cm B.4cm C.3cm D.2cm 解答: 解:∵四边形 ABCD 是菱形, ∴OB=OD,CD=AD=6cm, ∵OE∥DC, ∴BE=CE, ∴OE= CD=3cm. 故选 C. 9.某闭合电路中,电源的电压为定值,电流 I(A)与电阻 R(Ω)成反比例.图表示的是 该电路中电流 I 与电阻 R 之间函数关系的图象,则用电阻 R 表示电流 I 的函数解析式为 ( ) A. B. C. D. 解答: 解:设 I= ,那么点(3,2)适合这个函数解析式,则 k=3×2=6, ∴I= . 故选 C. 10.现有 3cm,4cm,7cm,9cm 长的四根木棒,任取其中三根组成一个三角形,那么可以 组成的三角形的个数是( ) A.1 个 B.2 个 C.3 个 D.4 个 解答: 解:四条木棒的所有组合:3,4,7 和 3,4,9 和 3,7,9 和 4,7,9; 只有 3,7,9 和 4,7,9 能组成三角形. 故选 B. 二、填空题(本题共 8 个小题,每小题 3 分,共 24 分) 11.已知函数关系式:y= ,则自变量 x 的取值范围是 x≥1 . 解 答: 解:根据题意得,x﹣1≥0, 解得 x≥1. 故答案为:x≥1. 12.如图,在△ABC 中,∠A=45°,∠B=60°,则外角∠ACD= 105 度. 解答: 解:∵∠A=45°,∠B=60°, ∴∠ACD=∠A+∠B=45°+60°=105°. 故答案为:105. 13.若实数 a、b 满足|3a﹣1|+b2=0,则 ab 的值为 1 . 解答: 解:根据题意得,3a﹣1=0,b=0, 解得 a= ,b=0, ab=( )0=1. 故答案为:1. 14.如果一次函数 y=mx+3 的图象经过第一、二、四象限,则 m 的取值范围是 m<0 . 解答: 解:∵一次函数 y=mx+3 的图象经过第一、二、四象限, ∴m<0. 故答案为:m<0. 15.任意抛掷一枚硬币,则“正面朝上”是 随机 事件. 解答: 解:抛掷 1 枚均匀硬币可能正面朝上,也可能反面朝上, 故抛掷 1 枚均匀硬币正面朝上是随机事件. 故答案为:随机. 16.在半径为 1cm 的圆中,圆心角为 120°的扇形的弧长是 cm. 解答: 解:扇形的弧长 L= = πcm. 故答案为: πcm. 17.如图,AB∥CD∥EF,那么∠BAC+∠ACE+∠CEF= 360 度. 解答: 解:∵AB∥CD, ∴∠BAC+∠ACD=180°…①, ∵CD∥EF, ∴∠CEF+∠ECD=180°…②, ①+②得, ∠BAC+∠ACD+∠CEF+∠ECD=180°+180°=360°, 即∠BAC+∠ACE+∠CEF=360°. 18.如图,等腰梯形 ABCD 中,AD∥BC,AB=AD=2,∠B=60°,则 BC 的长为 4 . 解答: 解:过点 A 作 AE∥CD 交 BC 于点 E, ∵AD∥BC, ∴四边形 AECD 是平行四边形, ∴AE=CD=2,AD=EC=2, ∵∠B=60°, ∴BE=AB=AE=2, ∴BC=BE+CE=2+2=4. 三、解答题:(本题共 2 个小题,每小题 6 分,共 12 分) 19.计算: . 解答: 解:原式=2+2× ﹣3=0. 20.先化简,再求值: ,其中 a=﹣2,b=1. 解答: 解:原式= + = + = , 把 a=﹣2,b=1 代入得:原式= =2. 四.解答题:(本题共 2 个小题,每小题 8 分,共 16 分) 21.某班数学科代表小华对本班上期期末考试数学成绩作了统计分析,绘制成如下频数、 频率统计表和频数分布直方图,请你根据图表提供的信息,解答下列问题: 分组 49.5~59.5 59.5~69.5 69.5~79.5 79.5~89.5 89.5~100.5 合计 频数 2 a 20 16 4 50 频率 0.04 0.16 0.40 0.32 b 1 (1)频数、频率统计表中,a= 8 ;b= 0.08 ; (2)请将频数分布直方图补充完整; (3)小华在班上任选一名同学,该同学成绩不低于 80 分的概率是多少? 解答: 解:(1)a=50﹣2﹣20﹣16﹣4=50﹣42=8, b=1﹣0.04﹣0.16﹣0.40﹣0.32=1﹣0.92=0.08; 故答案为:8,0.08. (2)如图所示; (3)该同学成绩不低于 80 分的概率是:0.32+0.08=0.40=40%. 22.如图,A,P,B,C 是半径为 8 的⊙O 上的四点,且满足∠BAC=∠APC=60°, (1)求证:△ABC 是等边三角形; (2)求圆心 O 到 BC 的距离 OD. 解答: 解:(1)在△ABC 中, ∵∠BAC=∠APC=60°, 又∵∠APC=∠ABC, ∴∠ABC=60°, ∴∠ACB=180°﹣∠BAC﹣∠ABC=180°﹣60°﹣60°=60°, ∴△ABC 是等边三角形; (2)∵△ABC 为等边三角形,⊙O 为其外接圆, ∴O 为△ABC 的外心, ∴BO 平分∠ABC, ∴∠OBD=30°, ∴OD=8× =4. 五、解答题(本题共 2 个小题,每小题 9 分,共 18 分) 23.以“开放崛起,绿色发展”为主题的第七届“中博会”已于 2012 年 5 月 20 日在湖南长沙圆 满落幕,作为东道主的湖南省一共签订了境外与省外境内投资合作项目共 348 个,其中境 外投资合作项目个数的 2 倍比省内境外投资合作项目多 51 个. (1)求湖南省签订的境外,省外境内的投资合作项目分别有多少个? (2)若境外、省内境外投资合作项目平均每个项目引进资金分别为 6 亿元,7.5 亿元,求 在这次“中博会”中,东道湖南省共引进资金多少亿元? 解答: 解:(1)设境外投资合作项目个数为 x 个, 根据题意得出:2x﹣(348﹣x)=51, 解得:x=133, 故省外境内投资合作项目为:348﹣133=215 个. 答:境外投资合作项目为 133 个,省外境内投资合作项目为 215 个. (2)∵境外、省内境外投资合作项目平均每个项目引进资金分别为 6 亿元,7.5 亿 元, ∴湖南省共引进资金:133×6+215×7.5=2410.5 亿元. 答:东道湖南省共引进资金 2410.5 亿元. 24.如图,已知正方形 ABCD 中,BE 平分∠DBC 且交 CD 边于点 E,将△BCE 绕点 C 顺 时针旋转到△DCF 的位置,并延长 BE 交 DF 于点 G. (1)求证:△BDG∽△DEG; (2)若 EG•BG=4,求 BE 的长. 解答: (1)证明:∵将△BCE 绕点 C 顺时针旋转到△DCF 的位置, ∴△BCE≌△DCF, ∴∠FDC=∠EBC, ∵BE 平分∠DBC, ∴∠DBE=∠EBC, ∴∠FDC=∠EBE, ∵∠DGE=∠DGE, ∴△BDG∽△DEG. (2)解:∵△BCE≌△DCF, ∴∠F=∠BEC,∠EBC=∠FDC, ∵四边形 ABCD 是正方形, ∴∠DCB=90°,∠DBC=∠BDC=45°, ∵BE 平分∠DBC, ∴∠DBE=∠EBC=22.5°=∠FDC, ∴∠BDF=45°+22.5°=67.5°, ∠F=90°﹣22.5°=67.5°=∠BDF, ∴BD=BF, ∵△BCE≌△DCF, ∴∠F=∠BEC=67.5°=∠DEG, ∴∠DGB=180°﹣22.5°﹣67.5°=90°, 即 BG⊥DF, ∵BD=BF, ∴DF=2DG, ∵△BDG∽△DEG,BG×EG=4, ∴ = , ∴BG×EG=DG×DG=4, ∴DG=2, ∴BE=DF=2DG=4. 六、解答题(本题共 2 个小题,每小题 10 分,共 20 分) 25.在长株潭建设两型社会的过程中,为推进节能减排,发展低碳经济,我市某公司以 25 万元购得某项节能产品的生产技术后,再投入 100 万元购买生产设备,进行该产品的生产 加工.已知生产这种产品的成本价为每件 20 元.经过市场调研发现,该产品的销售单价定 在 25 元到 30 元之间较为合理,并且该产品的年销售量 y(万件)与销售单价 x(元)之间 的函数关系式为: (年获利=年销售收入﹣生产成本﹣投资成本) (1)当销售单价定为 28 元时,该产品的年销售量为多少万件? (2)求该公司第一年的年获利 W(万元)与销售单价 x(元)之间的函数关系式,并说明 投资的第一年,该公司是盈利还是亏损?若盈利,最大利润是多少?若亏损,最小亏损是 多少? (3)第二年,该公司决定给希望工程捐款 Z 万元,该项捐款由两部分组成:一部分为 10 万元的固定捐款;另一部分则为每销售一件产品,就抽出一元钱作为捐款.若除去第一年 的最大获利(或最小亏损)以及第二年的捐款后,到第二年年底,两年的总盈利不低于 67.5 万元,请你确定此时销售单价的范围. 解答: 解:(1)∵25≤28≤30, , ∴把 28 代入 y=40﹣x 得, ∴y=12(万件), 答:当销售单价定为 28 元时,该产品的年销售量为 12 万件; (2)①当 25≤x≤30 时,W=(40﹣x)(x﹣20)﹣25﹣100=﹣x2+60x﹣925=﹣(x﹣ 30)2﹣25, 故当 x=30 时,W 最大为﹣25,及公司最少亏损 25 万; ②当 30<x≤35 时,W=(25﹣0.5x)(x﹣20)﹣25﹣100 =﹣ x2+35x﹣625=﹣ (x﹣35)2﹣12.5 故当 x=35 时,W 最大为﹣12.5,及公司最少亏损 12.5 万; 对比 1°,2°得,投资的第一年,公司亏损,最少亏损是 12.5 万; 答:投资的第一年,公司亏损,最少亏损是 12.5 万; (3)①当 25≤x≤30 时,W=(40﹣x)(x﹣20﹣1)﹣12.5﹣10=﹣x2+59x﹣782.5 令 W=67.5,则﹣x2+59x﹣782.5=67.5 化简得:x2﹣59x+850=0 x1=25;x2=34, 此时,当两年的总盈利不低于 67.5 万元,25≤x≤30; ②当 30<x≤35 时,W=(25﹣0.5x)(x﹣20﹣1)﹣12.5﹣10=﹣ x2+35.5x﹣547.5, 令 W=67.5,则﹣ x2+35.5x﹣547.5=67.5, 化简得:x2﹣71x+1230=0 x1=30;x2=41, 此时,当两年的总盈利不低于 67.5 万元,30<x≤35, 答:到第二年年底,两年的总盈利不低于 67.5 万元,此时销售单价的范围是 25≤x≤30 或 30<x≤35. 26.如图半径分别为 m,n(0<m<n)的两圆⊙O1 和⊙O2 相交于 P,Q 两点,且点 P (4,1),两圆同时与两坐标轴相切,⊙O1 与 x 轴,y 轴分别切于点 M,点 N,⊙O2 与 x 轴,y 轴分别切于点 R,点 H. (1)求两圆的圆心 O1,O2 所在直线的解析式; (2)求两圆的圆心 O1,O2 之间的距离 d; (3)令四边形 PO1QO2 的面积为 S1,四边形 RMO1O2 的面积为 S2. 试探究:是否存在一条经过 P,Q 两点、开口向下,且在 x 轴上截得的线段长为 的抛物线?若存在,请求出此抛物线的解析式;若不存在,请说明理由. 解答: 解:(1)由题意可知 O1(m,m),O2(n,n), 设过点 O1,O2 的直线解析式为 y=kx+b,则有: (0<m<n),解得 , ∴所求直线的解析式为:y=x. (2)由相交两圆的性质,可知 P、Q 点关于 O1O2 对称. ∵P(4,1),直线 O1O2 解析式为 y=x,∴Q(1,4). 如解答图 1,连接 O1Q. ∵Q(1,4),O1(m,m),根据两点间距离公式得到: O1Q= = 又 O1Q 为小圆半径,即 QO1=m, ∴ =m,化简得:m2﹣10m+17=0 ① 如解答图 1,连接 O2Q,同理可得:n2﹣10n+17=0 ② 由①,②式可知,m、n 是一元二次方程 x2﹣10x+17=0 ③的两个根, 解③得:x=5± ,∵0<m<n,∴m=5﹣ ,n=5+ . ∵O1(m,m),O2(n,n), ∴d=O1O2= =8. (3)假设存在这样的抛物线,其解析式为 y=ax2+bx+c,因为开口向下,所以 a< 0. 如解答图 2,连接 PQ. 由相交两圆性质可知,PQ⊥O1O2. ∵P(4,1),Q(1,4), ∴PQ= = ,又 O1O2=8, ∴S1= PQ•O1O2= × ×8= ; 又 S2= (O2R+O1M)•MR= (n+m)(n﹣m)= ; ∴ = =1,即抛物线在 x 轴上截得的线段长为 1. ∵抛物线过点 P(4,1),Q(1,4), ∴ ,解得 , ∴抛物线解析式为:y=ax2﹣(5a+1)x+5+4a, 令 y=0,则有:ax2﹣(5a+1)x+5+4a=0, 设两根为 x1,x2,则有:x1+x2= ,x1x2= , ∵在 x 轴上截得的线段长为 1,即|x1﹣x2|=1, ∴(x1﹣x2)2=1,∴(x1+x2)2﹣4x1x2=1, 即( )2﹣4( )=1,化简得:8a2﹣10a+1=0, 解得 a= ,可见 a 的两个根均大于 0,这与抛物线开口向下(即 a<0)矛 盾, ∴不存在这样的抛物线. 二○一二年福州市初中毕业会考、高级中等学校招生考试数学试 卷答案解析 一、选择题(共 10 小题,每题 4 分,满分 40 分;每小题只有一个正确的选项,请在 答题卡的相应位置填涂) 1.3 的相反数是 A.-3 B.1 3 C.3 D.-1 3 考点:相反数. 专题:存在型. 分析:根据相反数的定义进行解答. 解答:解:由相反数的定义可知,3 的相反数是-3. 故选 A. 点评:本题考查的是相反数的定义,即只有符号不同的两个数叫做互为相反数. 2.今年参观“5·18”海交会的总人数约为 489000 人,将 489000 用科学记数法表示 为 A.48.9×104 B.4.89×105 C.4.89×104 D.0.489×106 考点:科学记数法—表示较大的数. 分析:科学记数法的表示形式为 a×10n 的形式,其中 1≤|a|<10,n 为整数.确定 n 的值时,要看把原数变成 a 时,小数点移动了多少位,n 的绝对值与小数点移动 的位数相同.当原数绝对值>1 时,n 是正数;当原数的绝对值<1 时,n 是负 数. 解答:解:489000=4.89×105. 故选 B. 点评:此题考查科学记数法的表示方法.科学记数法的表示形式为 a×10n 的形式,其 中 1≤|a|<10,n 为整数,表示时关键要正确确定 a 的值以及 n 的值. 3.如图是由 4 个大小相同的正方体组合而成的几何体,其主视图是 考点:简单组合体的三视图. 分析:从正面看到的图叫做主视图,从左面看到的图叫做左视图,从上面看到的图叫 做俯视图.根据图中正方体摆放的位置判定则可. 解答:解:从正面看,下面一行是横放 3 个正方体,上面一行中间是一个正方体. 故选 C. 点评:本题考查了三种视图中的主视图,比较简单. 4.如图,直线 a∥b,∠1=70°,那么∠2 的度数是 A.50° B.60° C.70° D.80° 考点:平行线的性质. 分析:根据两角的位置关系可知两角是同位角,利用两直线平行同位角相等即可求得 结果. 解答:解:∵ a∥b, ∴ ∠1=∠2, ∵ ∠1=70°, ∴ ∠2=70°. 故选 C. 点评:本题考查了平行线的性质,根据两直线平行同位角相等即可得到答案,比较简 单,属于基础题. 5.下列计算正确的是 A.a+a=2a B.b3·b3=2b3 C.a3÷a=a3 D.(a5)2=a7 正面 第 3 题图 A B C D a 第 4 题图 1 2 b 考点:同底数幂的除法;合并同类项;同底数幂的乘法;幂的乘方与积的乘方. 专题:计算题. 分析:分别根据合并同类项、同底数幂的除法与乘法、幂的乘方与积的乘方法则对各 选项进行逐一计算即可. 解答:解:A、a+a=2a,故本选项正确; B、b3•b3=b6,故本选项错误; C、a3÷a=a2,故本选项错误; D、(a5)2=a10,故本选项错误. 故选 A. 点评:本题考查的是合并同类项、同底数幂的除法与乘法、幂的乘方与积的乘方法 则,熟知以上知识是解答此题的关键. 6.式子 x-1在实数范围内有意义,则 x 的取值范围是 A.x<1 B.x≤1 C.x>1 D.x≥1 考点:二次根式有意义的条件. 分析:根据二次根式有意义的条件列出关于 x 的不等式,求出 x 的取值范围即可. 解答:解:∵ 式子 x-1在实数范围内有意义, ∴ x-1≥0,解得 x≥1. 故选 D. 点评:本题考查的是二次根式有意义的条件,即被开方数大于等于 0. 7.某射击运动员在一次射击练习中,成绩(单位:环)记录如下:8,9,8,7,10.这 组数据的平均数和中位数分别是 A.8,8 B.8.4,8 C.8.4,8.4 D.8,8.4 考点:中位数;算术平均数. 分析:根据平均数公式求解即可,即用所有数据的和除以 5 即可;5 个数据的中位数 是排序后的第三个数. 解答:解:8,9,8,7,10 的平均数为:1 5 ×(8+9+8+7+10)=8.4. 8,9,8,7,10 排序后为 7,8,8,9,10,故中位数为 8. 故选 B. 点评:本题考查了中位数及算术平均数的求法,特别是中位数,首先应该排序,然后 再根据数据的个数确定中位数. 8.⊙O1 和⊙O2 的半径分别是 3cm 和 4cm,如果 O1O2=7cm,则这两圆的位置关系是 A.内含 B.相交 C.外切 D.外离 考点:圆与圆的位置关系. 分析:由⊙O1、⊙O2 的半径分别是 3cm、4cm,若 O1O2=7cm,根据两圆位置关系与 圆心距 d,两圆半径 R,r 的数量关系间的联系即可得出⊙O1 和⊙O2 的位置关 系. 解答:解:∵ ⊙O1、⊙O2 的半径分别是 3cm、4cm,O1O2=7cm, 又∵ 3+4=7, ∴⊙O1 和⊙O2 的位置关系是外切. 故选 C. 点评:此题考查了圆与圆的位置关系.解题的关键是掌握两圆位置关系与圆心距 d, 两圆半径 R,r 的数量关系间的联系.圆和圆的位置与两圆的圆心距、半径的数 量之间的关系:① 两圆外离⇔d>R+r;② 两圆外切⇔d=R+r;③ 两圆相交 ⇔R-r<d<R+r(R≥r);④ 两圆内切⇔d=R-r(R>r);⑤ 两圆内含⇔d<R- r(R>r). 9.如图,从热气球 C 处测得地面 A、B 两点的俯角分别为 30°、45°,如果 此时热气球 C 处的高度 CD 为 100 米,点 A、D、B 在同一直线上,则 AB 两点煌距离是 A.200 米 B.200 3米 C.220 3米 D.100( 3+1)米 考点:解直角三角形的应用-仰角俯角问题. 分析:图中两个直角三角形中,都是知道已知角和对边,根据正切函数求出邻边后, 相加求和即可. 第 9 题图 A B C D 30° 45° 解答:解:由已知,得∠A=30°,∠B=45°,CD=100, ∵ CD⊥AB 于点 D. ∴ 在 Rt△ACD 中,∠CDA=90°,tanA=CD AD , ∴ AD= CD tanA =100 3 3 =100 3 在 Rt△BCD 中,∠CDB=90°,∠B=45°, ∴ DB=CD=100 米, ∴ AB=AD+DB=100 3+100=100( 3+1)米. 故选 D. 点评:本题考查了解直角三角形的应用,解决本题的关键是利用 CD 为直角△ ABC 斜边上的高,将三角形分成两个三角形,然后求解.分别在两三角形中求 出 AD 与 BD 的长. 10.如图,过点 C(1,2)分别作 x 轴、y 轴的平行线,交直线 y=-x+6 于 A、B 两点,若反比例函数 y=k x(x>0)的图像与△ABC 有公共点,则 k 的取值范围 是 A.2≤k≤9 B.2≤k≤8 C.2≤k≤5 D.5≤k≤8 考点:反比例函数综合题. 专题:综合题. 分析:先求出点 A、B 的坐标,根据反比例函数系数的几何意义可知,当反比例函数图 象与△ABC 相交于点 C 时 k 的取值最小,当与线段 AB 相交时,k 能取到最大 值,根据直线 y=-x+6,设交点为(x,-x+6)时 k 值最大,然后列式利用二 次函数的最值问题解答即可得解. 解答:解:∵ 点 C(1,2),BC∥y 轴,AC∥x 轴, ∴ 当 x=1 时,y=-1+6=5, 当 y=2 时,-x+6=2,解得 x=4, A B C O x y 第 10 题图 ∴ 点 A、B 的坐标分别为 A(4,2),B(1,5), 根据反比例函数系数的几何意义,当反比例函数与点 C 相交时,k=1×2= 2 最小, 设与线段 AB 相交于点(x,-x+6)时 k 值最大, 则 k=x(-x+6)=-x2+6x=-(x-3)2+9, ∵ 1≤x≤4, ∴ 当 x=3 时,k 值最大, 此时交点坐标为(3,3), 因此,k 的取值范围是 2≤k≤9. 故选 A. 点评:本题考查了反比例函数系数的几何意义,二次函数的最值问题,本题看似简单 但不容易入手解答,判断出最大最小值的取值情况并考虑到用二次函数的最值 问题解答是解题的关键. 二、填空题(共 5 小题,每题 4 分,满分 20 分;请将正确答案填在答题卡相应位置) 11.分解因式:x2-16=_________________. 考点:因式分解——运用公式法. 分析:运用平方差公式分解因式的式子特点:两项平方项,符号相反.直接运用平方 差公式分解即可.a2-b2=(a+b)(a-b). 解答:解:x2-16=(x+4)(x-4). 点评:本题考查因式分解.当被分解的式子只有两项平方项;符号相反,且没有公因 式时,应首要考虑用平方差公式进行分解. 12.一个袋子中装有 3 个红球和 2 个绿球,这些球除了颜色外都相同,从袋子中随机 摸出一个球,则摸到红球的概率为__________________. 考点:概率公式. 分析:根据概率的求法,找准两点:① 全部情况的总数;② 符合条件的情况数目;二 者的比值就是其发生的概率. 解答:解;布袋中球的总数为:2+3=5, 取到黄球的概率为:3 5 . 故答案为:3 5 . 点评:此题主要考查了概率的求法,如果一个事件有 n 种可能,而且这些事件的可能 性相同,其中事件 A 出现 m 种结果,那么事件 A 的概率 P(A)=m n . 13.若 20n是整数,则正整数 n 的最小值为________________. 考点:二次根式的定义. 专题:存在型. 分析: 20n是正整数,则 20n 一定是一个完全平方数,首先把 20n 分解因数,确定 20n 是完全平方数时,n 的最小值即可. 解答:解:∵ 20n=22×5n. ∴ 整数 n 的最小值为 5. 故答案是:5. 点评:本题考查了二次根式的定义,理解 20n是正整数的条件是解题的关键. 14.计算:x-1 x +1 x =______________. 考点:分式的加减法. 专题:计算题. 分析:直接根据同分母的分数相加减进行计算即可. 解答:解:原式=x-1+1 x =1. 故答案为:1. 点评:本题考查的是分式的加减法,同分母的分式相加减,分母不变,把分子相加 减. 15.如图,已知△ABC,AB=AC=1,∠A=36°,∠ABC 的平分线 BD 交 AC 于点 D,则 AD 的长是______,cosA 的值是______________.(结果保留根号) 考点:黄金分割;相似三角形的判定与性质;锐角三角函数的定义. 分析:可以证明△ABC∽△BDC,设 AD=x,根据相似三角形的对应边的比相等,即可列 出方程,求得 x 的值;过点 D 作 DE⊥AB 于点 E,则 E 为 AB 中点,由余弦定义 可求出 cosA 的值. 解答:解:∵ △ABC,AB=AC=1,∠A=36°, ∴ ∠ABC=∠ACB=180°-∠A 2 =72°. ∵ BD 是∠ABC 的平分线, ∴ ∠ABD=∠DBC=1 2 ∠ABC=36°. ∴ ∠A=∠DBC=36°, 又∵ ∠C=∠C, ∴ △ABC∽△BDC, ∴ AC BC =BC CD , 设 AD=x,则 BD=BC=x.则1 x = x 1-x , 解得:x= 5+1 2 (舍去)或 5-1 2 . 故 x= 5-1 2 . 如右图,过点 D 作 DE⊥AB 于点 E, ∵ AD=BD, ∴E 为 AB 中点,即 AE=1 2AB=1 2 . A B C D 第 15 题图 A B C D E 在 Rt△AED 中,cosA=AE AD = 1 2 5-1 2 = 5+1 4 . 故答案是: 5-1 2 ; 5+1 4 . 点评:△ABC、△BCD 均为黄金三角形,利用相似关系可以求出线段之间的数量关系; 在求 cosA 时,注意构造直角三角形,从而可以利用三角函数定义求解. 三、解答题(满分 90 分;请将正确答案及解答过程填在答题卡相应位置.作图或添辅 助线用铅笔画完,再用黑色签字笔描黑) 16.(每小题 7 分,共 14 分) (1) 计算:|-3|+(π+1)0- 4. (2) 化简:a(1-a)+(a+1)2-1. 考点:整式的混合运算;实数的运算;零指数幂. 专题:计算题. 分析:(1) 原式第一项根据绝对值的代数意义:负数的绝对值等于它的相反数进行化 简,第二项利用零指数公式化简,第三项利用 a2=|a|化简,合并后即可得到 结果; (2) 利用乘法分配律将原式第一项括号外边的 a 乘到括号里边,第二项利用完全 平方数展开,合并同类项后即可得到结果. 解答:解:(1) 解:|-3|+(π+1)0- 4=3+1-2=2. (2) 解:a(1-a)+(a+1)2-1=a-a2+a2+2a+1-1=3a. 点评:此题考查了整式的混合运算,以及实数的运算,涉及的知识有:绝对值的代数 意义,零指数公式,二次根式的化简,完全平方公式,以及合并同类项法则, 熟练掌握公式及法则是解本题的关键. 17.(每小题 7 分,共 14 分) (1) 如图,点 E、F 在 AC 上,AB∥CD,AB=CD,AE=CF.求证:△ABF≌△CDE. (2) 如图,方格纸中的每个小方格是边长为 1 个单位长度的正方形. ① 画出将 Rt△ABC 向右平移 5 个单位长度后的 Rt△A1B1C1; ② 再将 Rt△A1B1C1 绕点 C1 顺时针旋转 90°,画出旋转后的 Rt△A2B2C1,并求出 旋转过程中线段 A1C1 所扫过的面积(结果保留π). 考点:作图——旋转变换;全等三角形的判定;扇形面积的计算;作图——平移变 换. 分析:(1) 由 AB∥CD 可知∠A=∠C,再根据 AE=CF 可得出 AF=CE,由 AB=CD 即可 判断出△ABF≌CDE; (2) 根据图形平移的性质画出平移后的图形,再根据在旋转过程中,线段 A1C1 所扫过的面积等于以点 C1 为圆心,以 A1C1 为半径,圆心角为 90 度的扇形的面 积,再根据扇形的面积公式进行解答即可. 解答:证明:∵ AB∥CD, ∴ ∠A=∠C. ∵ AE=CF, ∴ AE+EF=CF+EF, 即 AF=CE. 又∵ AB=CD, ∴ △ABF≌△CDE. (2) 解:① 如图所示; ② 如图所示; 在旋转过程中,线段 A1C1 所扫过的面积等于90·π·42 360 =4π. A B CD E F 第 17(1)题图 第 17(2)题图 A B C B C A B C A BA 点评:本题考查的是作图-旋转变换、全等三角形的判定及扇形面积的计算,熟知图 形平移及旋转不变性的性质是解答此题的关键. 18.(满分 12 分)省教育厅决定在全省中小学开展“关注校车、关爱学生”为主题的交 通安全教育宣传周活动.某中学为了了解本校学生的上学方式,在全校范围内随 机抽查了部分学生,将收集的数据绘制成如下两幅不完整的统计图(如图所示), 请根据图中提供的信息,解答下列问题. (1) m=_______%,这次共抽取__________名学生进行调查;并补全条形图; (2) 在这次抽样调查中,采用哪种上学方式的人最多? (3) 如果该校共有 1500 名学生,请你估计该校骑自行车上学的学生约有多少名? 考点:条形统计图;用样本估计总体;扇形统计图. 分析:(1) 用 1 减去其他各种情况所占的百分比即可求 m 的值,用乘公交的人数除以 其所占的百分比即可求得抽查的人数; (2) 从扇形统计图或条形统计图中直接可以得到结果; (3) 用学生总数乘以骑自行车所占的百分比即可. 解答:解:(1) 1-14%-20%-40%=26%; 20÷40%=50; 条形图如图所示; 其他 14% 骑自行车 20% 步行 乘公交车 学生上学方式扇形统计图 学生上学方式条形统计图 人数 步行 其他乘公交车 骑自行车 上学方式 25 20 15 10 5 0 13 20 7 学生上学方式条形统计图 人数 步行 其他乘公交车 骑自行车 上学方式 25 20 15 10 5 0 13 20 7 10 (2) 采用乘公交车上学的人数最多; (3) 该校骑自行车上学的人数约为: 150×20%=300(人). 点评:本题考查了条形统计图、扇形统计图及用样本估计总数的知识,解题的关键是 从统计图中整理出进一步解题的信息. 19.(满分 11 分)某次知识竞赛共有 20 道题,每一题答对得 5 分,答错或不答都扣 3 分. (1) 小明考了 68 分,那么小明答对了多少道题? (2) 小亮获得二等奖(70~90 分),请你算算小亮答对了几道题? 考点:一元一次不等式组的应用;一元一次方程的应用. 分析:(1) 设小明答对了 x 道题,则有 20-x 道题答错或不答,根据答对题目的得分减 去答错或不答题目的扣分是 68 分,即可得到一个关于 x 的方程,解方程即可求 解; (2) 小明答对了 x 道题,则有 20-x 道题答错或不答,根据答对题目的得分减去 答错或不答题目的扣分,就是最后的得分,得分满足大于或等于 70 小于或等于 90,据此即可得到关于 x 的不等式组,从而求得 x 的范围,再根据 x 是非负整 数即可求解. 解答:解:(1) 设小明答对了 x 道题, 依题意得:5x-3(20-x)=68. 解得:x=16. 答:小明答对了 16 道题. (2) 设小亮答对了 y 道题, 依题意得: 5y-3(20-y)≥70 5y-3(20-y)≤90. 因此不等式组的解集为 161 4 ≤y≤183 4 . ∵ y 是正整数, A B C D E O 第 20 题图 ∴ y=17 或 18. 答:小亮答对了 17 道题或 18 道题. 点评:本题考查了列方程解应用题,以及列一元一次不等式解决问题,正确列式表示 出最后的得分是关键. 20.(满分 12 分)如图,AB 为⊙O 的直径,C 为⊙O 上一点,AD 和过 C 点的切线互相 垂直,垂足为 D,AD 交⊙O 于点 E. (1) 求证:AC 平分∠DAB; (2) 若∠B=60º,CD=2 3,求 AE 的长. ZxxkCom 考点:切线的性质;圆周角定理;相似三角形的判定与性质;解直角三角形. 专题:几何综合题. 分析:(1) 连接 OC,由 CD 为⊙O 的切线,根据切线的性质得到 OC 垂直于 CD,由 AD 垂直于 CD,可得出 OC 平行于 AD,根据两直线平行内错角相等可得出∠1= ∠2,再由 OA=OC,利用等边对等角得到∠2=∠3,等量代换可得出∠1= ∠3,即 AC 为角平分线; (2) 法 1:由 AB 为圆 O 的直径,根据直径所对的圆周角为直角可得出∠ACB 为 直角,在直角三角形 ABC 中,由∠B 的度数求出∠3 的度数为 30°,可得出∠1 的度数为 30°,在直角三角形 ACD 中,根据 30°角所对的直角边等于斜边的一 半,由 CD 的长求出 AC 的长,在直角三角形 ABC 中,根据 cos30°及 AC 的长, 利用锐角三角函数定义求出 AB 的长,进而得出半径 OE 的长,由∠EAO 为 60°,及 OE=OA,得到三角形 AEO 为等边三角形,可得出 AE=OA=OE,即可 确定出 AE 的长; 法 2:连接 EC,由 AB 为圆 O 的直径,根据直径所对的圆周角为直角可得出 ∠ACB 为直角,在直角三角形 ABC 中,由∠B 的度数求出∠3 的度数为 30°,可 得出∠1 的度数为 30°,在直角三角形 ADC 中,由 CD 及 tan30°,利用锐角三 角函数定义求出 AD 的长,由∠DEC 为圆内接四边形 ABCE 的外角,利用圆内接 四边形的外角等于它的内对角,得到∠DEC=∠B,由∠B 的度数求出∠DEC 的 度数为 60°,在直角三角形 DEC 中,由 tan60°及 DC 的长,求出 DE 的长,最 后由 AD-ED 即可求出 AE 的长. 解答:(1) 证明:如图 1,连接 OC, ∵ CD 为⊙O 的切线, ∴ OC⊥CD, ∴ ∠OCD=90°. ∵ AD⊥CD, ∴ ∠ADC=90°. ∴ ∠OCD+∠ADC=180°, ∴ AD∥OC, ∴ ∠1=∠2, ∵ OA=OC, ∴ ∠2=∠3, ∴ ∠1=∠3, 即 AC 平分∠DAB. (2) 解法一:如图 2, ∵ AB 为⊙O 的直径, ∴ ∠ACB=90°. 又∵ ∠B=60°, ∴ ∠1=∠3=30°. 在 Rt△ACD 中,CD=2 3, ∴ AC=2CD=4 3. 在 Rt△ABC 中,AC=4 3, ∴ AB= AC cos∠CAB = 4 3 cos30° =8. 连接 OE, ∵ ∠EAO=2∠3=60°,OA=OE, A B C D E O 图 2 1 2 3 ∴ △AOE 是等边三角形, ∴ AE=OA=1 2AB=4. 解法二:如图 3,连接 CE ∵ AB 为⊙O 的直径, ∴ ∠ACB=90°. 又∵ ∠B=60°, ∴ ∠1=∠3=30°. 在 Rt△ADC 中,CD=2 3, ∴ AD= CD tan∠DAC = 2 3 tan30° =6. ∵ 四边形 ABCE 是⊙O 的内接四边形, ∴ ∠B+∠AEC=180°. 又∵ ∠AEC+∠DEC=180°, ∴ ∠DEC=∠B=60°. 在 Rt△CDE 中,CD=2 3, ∴ DE= CD tan∠DEC = 2 3 tan60° =2. ∴ AE=AD-DE=4. 点评:此题考查了切线的性质,平行线的性质,等边三角形的判定与性质,锐角三角 函数定义,圆内接四边形的性质,以及圆周角定理,利用了转化及数形结合的 思想,遇到直线与圆相切,常常连接圆心与切点,利用切线的性质得到垂直, 利用直角三角形的性质来解决问题. 21.(满分 13 分)如图①,在 Rt△ABC 中,∠C=90º,AC=6,BC=8,动点 P 从点 A 开 始沿边 AC 向点 C 以每秒 1 个单位长度的速度运动,动点 Q 从点 C 开始沿边 CB 向点 B 以每秒 2 个单位长度的速度运动,过点 P 作 PD∥BC,交 AB 于点 D,连接 PQ.点 P、 Q 分别从点 A、C 同时出发,当其中一点到达端点时,另一点也随之停止运动,设运动 时间为 t 秒(t≥0). A B C D E O 图 3 1 2 3 (1) 直接用含 t 的代数式分别表示:QB=______,PD=______. (2) 是否存在 t 的值,使四边形 PDBQ 为菱形?若存在,求出 t 的值;若不存在, 说明理由.并探究如何改变点 Q 的速度(匀速运动),使四边形 PDBQ 在某一时 刻为菱形,求点 Q 的速度; (3) 如图②,在整个运动过程中,求出线段 PQ 中点 M 所经过的路径长. 考点:相似三角形的判定与性质;一次函数综合题;勾股定理;菱形的判定与性质. 专题:代数几何综合题. 分析:(1) 根据题意得:CQ=2t,PA=t,由 Rt△ABC 中,∠C=90°,AC=6,BC= 8,PD∥BC,即可得 tanA= PD PA =BC AC =4 3 ,则可求得 QB 与 PD 的值; (2) 易得△APD∽△ACB,即可求得 AD 与 BD 的长,由 BQ∥DP,可得当 BQ= DP 时,四边形 PDBQ 是平行四边形,即可求得此时 DP 与 BD 的长,由 DP≠BD,可判定▱PDBQ 不能为菱形;然后设点 Q 的速度为每秒 v 个单位长度, 由要使四边形 PDBQ 为菱形,则 PD=BD=BQ,列方程即可求得答案; (3) 设 E 是 AC 的中点,连接 ME.当 t=4 时,点 Q 与点 B 重合,运动停止.设 此时 PQ 的中点为 F,连接 EF,由△PMN∽△PQC.利用相似三角形的对应边成 比例,即可求得答案. 解答:解:(1) QB=8-2t,PD=4 3t. 第 21 题图① A B C D P Q 第 21 题图② A B C D P Q (2) 不存在. 在 Rt△ABC 中,∠C=90°,AC=6,BC=8, ∴ AB=10. ∵ PD∥BC, ∴ △APD∽△ACB, ∴ AD AB =AP AC ,即:AD 10 =t 6 , ∴ AD=5 3t, ∴ BD=AB-AD=10-5 3t. ∵ BQ∥DP, ∴ 当 BQ=DP 时,四边形 PDBQ 是平行四边形, 即 8-2t=4 3t,解得:t=12 5 . 当 t=12 5 时,PD=4 3 ×12 5 =16 5 ,BD=10-5 3 ×12 5 =6, ∴ DP≠BD, ∴ □PDBQ 不能为菱形. 设点 Q 的速度为每秒 v 个单位长度, 则 BQ=8-vt,PD=4 3t,BD=10-5 3t. 要使四边形 PDBQ 为菱形,则 PD=BD=BQ, 当 PD=BD 时,即 4 3t=10-5 3t,解得:t=10 3 . 当 PD=BQ 时,t=10 3 时,即4 3 ×10 3 =8-10 3 v,解得:v=16 15 . (3) 解法一:如图 2,以 C 为原点,以 AC 所在直线为 x 轴,建立平面直角坐标 系. 依题意,可知 0≤t≤4,当 t=0 时,点 M1 的坐标为(3,0); 图 1 A B C D P Q 当 t=4 时,点 M2 的坐标为(1,4). 设直线 M1M2 的解析式为 y=kx+b, ∴ 3k+b=0 k+b=4 ,解得: k=-2 b=6 . ∴ 直线 M1M2 的解析式为 y=-2x+6. ∵ 点 Q(0,2t),P(6-t,0), ∴ 在运动过程中,线段 PQ 中点 M3 的坐标为(6-t 2 ,t). 把 x=6-t 2 ,代入 y=-2x+6,得 y=-2×6-t 2 +6=t. ∴ 点 M3 在直线 M1M2 上. 过点 M2 作 M2N⊥x 轴于点 N,则 M2N=4,M1N=2. ∴ M1M2=2 5. ∴ 线段 PQ 中点 M 所经过的路径长为 2 5单位长度. 解法二:如图 3,设 E 是 AC 的中点,连接 ME. 当 t=4 时,点 Q 与点 B 重合,运动停止. 设此时 PQ 的中点为 F,连接 EF. 过点 M 作 MN⊥AC,垂足为 N,则 MN∥BC. ∴ △PMN∽△PDC. ∴ MN QC =PN PC =PM PQ ,即:MN 2t = PN 6-t =1 2 . ∴ MN=t,PN=3-1 2t, ∴ CN=PC-PN=(6-t)-(3-1 2t)=3-1 2t. ∴ EN=CE-CN=3-(3-1 2t)= 1 2t. A B C M 1 x y PN Q M 2 M 3 D 图 2 A B C PN Q D 图 3 E M F H ∴ tan∠MEN=MN EN =2. ∵ tan∠MEN 的值不变,∴ 点 M 在直线 EF 上. 过 F 作 FH⊥AC,垂足为 H.则 EH=2,FH=4. ∴ EF=2 5. ∵ 当 t=0 时,点 M 与点 E 重合;当 t=4 时,点 M 与点 F 重合, ∴ 线段 PQ 中点 M 所经过的路径长为 2 5单位长度. 点评:此题考查了相似三角形的判定与性质、平行四边形的判定与性质、菱形的判定 与性质以及一次函数的应用.此题综合性很强,难度较大,解题的关键是注意 数形结合思想的应用. 22.(满分 14 分)如图①,已知抛物线 y=ax2+bx(a≠0)经过 A(3,0)、B(4,4)两点. (1) 求抛物线的解析式; (2) 将直线 OB 向下平移 m 个单位长度后,得到的直线与抛物线只有一个公共点 D,求 m 的值及点 D 的坐标; (3) 如图②,若点 N 在抛物线上,且∠NBO=∠ABO,则在(2)的条件下,求出所有 满足△POD∽△NOB 的点 P 的坐标(点 P、O、D 分别与点 N、O、B 对应). 考点:二次函数综合题. 分析:(1) 利用待定系数法求出二次函数解析式即可; (2) 根据已知条件可求出 OB 的解析式为 y=x,则向下平移 m 个单位长度后的 解析式为:y=x-m.由于抛物线与直线只有一个公共点,意味着联立解析式 后得到的一元二次方程,其根的判别式等于 0,由此可求出 m 的值和 D 点坐 标; (3) 综合利用几何变换和相似关系求解. 方法一:翻折变换,将△NOB 沿 x 轴翻折; 方法二:旋转变换,将△NOB 绕原点顺时针旋转 90°. 特别注意求出 P 点坐标之后,该点关于直线 y=-x 的对称点也满足题意,即满 足题意的 P 点有两个,避免漏解. A B D O x y A B D O x y N 解答:解:(1) ∵ 抛物线 y=ax2+bx(a≠0)经过点 A(3,0)、B(4,4). ∴ 9a+3b=0 16a+4b=4,解得: a=1 b=-3. ∴ 抛物线的解析式是 y=x2-3x. (2) 设直线 OB 的解析式为 y=k1x,由点 B(4,4), 得:4=4k1,解得 k1=1. ∴ 直线 OB 的解析式为 y=x. ∴ 直线 OB 向下平移 m 个单位长度后的解析式为:y=x-m. ∵ 点 D 在抛物线 y=x2-3x 上. ∴ 可设 D(x,x2-3x). 又点 D 在直线 y=x-m 上, ∴ x2-3x =x-m,即 x2-4x+m=0. ∵ 抛物线与直线只有一个公共点, ∴ △=16-4m=0,解得:m=4. 此时 x1=x2=2,y=x2-3x=-2, ∴ D 点坐标为(2,-2). (3) ∵ 直线 OB 的解析式为 y=x,且 A(3,0), ∴ 点 A 关于直线 OB 的对称点 A'的坐标是(0,3). 设直线 A'B 的解析式为 y=k2x+3,过点 B(4,4), ∴ 4k2+3=4,解得:k2=1 4 . ∴ 直线 A'B 的解析式是 y=1 4x+3. ∵ ∠NBO=∠ABO, ∴ 点 N 在直线 A'B 上, ∴ 设点 N(n,1 4n+3),又点 N 在抛物线 y=x2-3x 上, ∴ 1 4n+3=n2-3n, 解得:n1=-3 4 ,n2=4(不合题意,会去), ∴ 点 N 的坐标为(-3 4 ,45 16). 方法一:如图 1,将△NOB 沿 x 轴翻折,得到△N1OB1, 则 N1(-3 4 ,-45 16),B1(4,-4), ∴ O、D、B1 都在直线 y=-x 上. ∵ △P1OD∽△NOB, ∴ △P1OD∽△N1OB1, ∴ OP1 ON1 =OD OB1 =1 2 , ∴ 点 P1 的坐标为(-3 8 ,-45 32). 将△OP1D 沿直线 y=-x 翻折,可得另一个满足条件的点 P2(45 32 ,3 8). D A B O x y N 图 1 A' P1 N1 P2 B1 综上所述,点 P 的坐标是(-3 8 ,-45 32)或(45 32 ,3 8). 方法二:如图 2,将△NOB 绕原点顺时针旋转 90°,得到△N2OB2, 则 N2(45 16 ,3 4),B2(4,-4), ∴ O、D、B2 都在直线 y=-x 上. ∵ △P1OD∽△NOB, ∴ △P1OD∽△N2OB2, ∴ OP1 ON2 =OD OB2 =1 2 , ∴ 点 P1 的坐标为(45 32 ,3 8). 将△OP1D 沿直线 y=-x 翻折,可得另一个满足条件的点 P2(-3 8 ,- 45 32). 综上所述,点 P 的坐标是(-3 8 ,-45 32)或(45 32 ,3 8). 点评:本题是基于二次函数的代数几何综合题,综合考查了待定系数法求抛物线解析 式、一次函数(直线)的平移、一元二次方程根的判别式、翻折变换、旋转变换以 及相似三角形等重要知识点.本题将初中阶段重点代数、几何知识熔于一炉, 难度很大,对学生能力要求极高,具有良好的区分度,是一道非常好的中考压 轴题. 图 2 A' N2P1 P2 B2 A B D O x y N 2012 年沈阳市中考数学试题 *试题满分 150 分 考试时间 120 分钟 参考公式: 抛物线 cbxaxy  2 的顶点是( a b 2  , a bac 4 4 2 ),对称轴是直线 a bx 2  . 一、选择题 (下列各题的备选答案中,只有一个答案是正确的.每小题 3 分,共 24 分) 1.下列各数中比 0 小的数是 A.-3 B. 3 1 1 C.3 D. 3 2.左下图是由四个相同的小立方块搭成的几何体,这个几何体的左视图是 3.沈阳地铁 2 号线的开通,方便了市民的出行.从 2012 年 1 月 9 日到 2 月 7 日的 30 天里,累计客运量约达 3040000 人次,将 3040000 用科学记数法表示为 A.3.04×105 B.3.04×106 C.30.4×105 D.0.304×107 4.计算(2a)3·a2 的结果是 A.2a5 B.2a6 C.8a5 D.8a6 5.在平面直角坐标系中,点 P (-1,2 ) 关于 x 轴的对称点的坐标为 A.(-1,-2 ) B.(1,-2 ) C.(2,-1 ) D.(-2,1 ) 6.气象台预报“本市明天降水概率是 30%” ,对此消息下列说法正确的是 A.本市明天将有 30%的地区降水 B.本市明天将有 30%的时间降水 C.本市明天有可能降水 D.本市明天肯定不降水 7.一次函数 y=-x+2 的图象经过 A.一、二、三象限 B.一、二、四象限 C.一、三、四象限 D.二、三、四象限 8.如图,正方形 ABCD 中,对角线 AC,BD 相交于点 O,则图中的等腰直角三角 形有 A.4 个 B.6 个 C.8 个 D.10 个 二、填空题(每小题 4 分,共 32 分) 9.分解因式:m2-6m+9=____________. 10.一组数据 1,3,3,5,7 的众数是____________. 11.五边形的内角和为____________度. 12.不等式组      021 01 x x 的解集是____________. 13.已知△ABC∽△A′B′C′,相似比为 3∶4,△ABC 的周长为 6,则△A′B′C 的周长为 ____________. 14.已知点 A 为双曲线 y= kx 图象上的点,点 O 为坐标原点过点 A 作 AB⊥x 轴于 点 B,连接 OA.若△AOB 的面积为 5,则 k 的值为____________. 15.有一组多项式:a+b2,a2-b4,a3+b6,a4-b8,…,请观察它们的构成规律,用 你发现的规律写出第 10 个多项式为____________. 16.如图,菱形 ABCD 的边长为 8cm,∠A=60°,DE⊥AB 于点 E,DF⊥BC 于点 F,则四边形 BEDF 的面积为____________cm2. 三、解答题(第 17、18 小题各 8 分,第 19 小题 10 分,共 26 分 ) 17.计算:(-1)2+ |12|  +2sin45° 18.小丁将中国的清华大学、北京大学及英国的剑桥大学的图片分别贴在 3 张完全 相同的不透明的硬纸板上,制成名校卡片,如图.小丁将这 3 张卡片背面朝上洗匀后放 在桌子上,从中随机抽取一张卡片,放回后洗匀,再随机抽取一张卡片. (1) 小丁第一次抽取的卡片上的图片是剑桥大学的概率是多少?(请直接..写出结 果) (2) 请你用列表法或画树状图(树形图) 法,帮助小丁求出两次抽取的卡片上的 图片一个是国内大学、一个是国外大学的概率.(卡片名称可用字母表示) 19.已知,如图,在荀 ABCD 中,延长 DA 到点 E,延长 BC 到点 F,使得 AE= CF,连接 EF,分别交 AB,CD 于点 M,N,连接 DM,BN. (1)求证:△AEM≌△CFN; (2)求证:四边形 BMDN 是平行四边形. 四、( 每小题 10 分,共 20 分 ) 20.为了提高沈城市民的节水意识,有关部门就“你认为最有效的节水措施”随机 对部分市民进行了问卷调查.其中调查问卷设置以下选项(被调查者只能选择其中的一 项 ): A.出台相关法律法规;B.控制用水大户数量;C.推广节水技改和节水器具;D.用水 量越多,水价越高;E.其他. 根据调查结果制作了统计图表的一部分如下: (1)此次抽样调查的人数为 ① 人; (2)结合上述统计图表可得 m= ② ,n= ③ ; (3)请根据以上信息直接..在答题卡中补全条形统计图. 21.甲、乙两人加工同一种机器零件,甲比乙每小时多加工 10 个零件,甲加工 150 个零件所用时间与乙加工 120 个零件所用时间相等,求甲、乙两人每小时各加工 多少个机器零件? 五、(本题 10 分) 22.如图,⊙O 是△ABC 的外接圆,AB 是⊙O 的直径,D 为⊙O 上一点,OD⊥AC, 垂足为 E,连接 BD. (1)求证:BD 平分∠ABC; (2) 当∠ODB=30°时,求证:BC=OD. 六、(本题 12 分) 23.已知,如图,在平面直角坐标系内,点 A 的坐标为(0,24 ),经过原点的 直线 l1 与经过点 A 的直线 l2 相交于点 B,点 B 坐标为(18,6). (1)求直线 l1,l2 的表达式; (2)点 C 为线段 OB 上一动点 (点 C 不与点 O,B 重合),作 CD∥y 轴交直线 l2 于 点 D,过点 C,D 分别向 y 轴作垂线,垂足分别为 F,E,得到矩形 CDEF. ①设点 C 的纵坐标为 a,求点 D 的坐标(用含 a 的代数式表示); ②若矩形 CDEF 的面积为 60,请直接..写出此时点 C 的坐标. 七、(本题 12 分) 24.已知,如图①,∠MON=60°,点 A,B 为射线 OM,ON 上的动点(点 A,B 不与点 O 重合),且 AB= 34 ,在∠MON 的内部、△AOB 的外部有一点 P,且 AP=BP,∠APB=120°. (1)求 AP 的长; (2)求证:点 P 在∠MON 的平分线上; (3) 如图②,点 C,D,E,F 分别是四边形 AOBP 的边 AO,OB,BP,PA 的中 点,连接 CD,DE,EF,FC,OP. ①当 AB⊥OP 时,请直接..写出四边形 CDEF 的周长的值; ②若四边形 CDEF 的周长用 t 表示,请直接..写出 t 的取值范围. 八、(本题 14 分) 25.已知,如图,在平面直角坐标系中,点 A 坐标为(-2,0),点 B 坐标为 (0, 2 ),点 E 为线段 AB 上的动点(点 E 不与点 A,B 重合),以 E 为顶点作∠OET=45°, 射线 ET 交线段 OB 于点 F,C 为 y 轴正半轴上一点,且 OC=AB,抛物线 y= 2 x2+mx+n 的图象经过 A,C 两点. (1) 求此抛物线的函数表达式; (2) 求证:∠BEF=∠AOE; (3) 当△EOF 为等腰三角形时,求此时点 E 的坐标; (4) 在(3)的条件下,当直线 EF 交 x 轴于点 D,P 为(1) 中抛物线上一动 点,直线 PE 交 x 轴于点 G,在直线 EF 上方的抛物线上是否存在一点 P,使得△EPF 的 面积是△EDG 面积的( 122  ) 倍.若存在,请直接..写出点 P 的坐标;若不存在,请 说明理由. 温馨提示:考生可以根据题意,在备用图中补充图形,以便作答. 数学试题 参考答案 一、选择题(每小题 3 分,共 24 分) 1.A 2.D 3.B 4.C 5.A 6.C 7.B 8.C 二、填空题(每小题 4 分,共 32 分) 9. (m-3)2 10.3 11. 540 12.-1<x< 2 1 13.8 14.10 或 -10 15.a10-b20 16. 316 三、解答题 (第 17、 18 小题各 8 分, 第 19 小题 10 分,共 26 分) 17.原式=1+ 2 -1+2× 2 2 =2 2 18.解: (1) 3 1 (2) 列表得 或画树状 (形) 图得 由表格 (或树状图/树形图) 可知, 共有 9 种可能出现的结果, 每种结果出现的 可能性相同,其中两次抽取的卡片上的图片一个是国内大学, 一个是国外大学的结果 有 4 种: (A, C)(B, C)(C, A)(C, B) ∴P(两次抽取的卡片上的图片一个是国内大学一个是国外大学) = 9 4 . 19.证明:(1) ∵四边形 ABCD 是平行四边形∴∠DAB=∠BCD ∴∠EAM=∠FCN 又∵AD∥BC ∴∠E=∠F∵AE=CF ∴△AEM≌△CFN (2) 由(1) 得 AM=CN,又∵四边形 ABCD 是平行四边形∴AB CD ∴BM DN∴四边 形 BMDN 是平行四边形 四、(每小题 10 分,共 20 分) 20.解: (1) 500 (2) 35%, 5% (3) 21.解:设乙每小时加工机器零件 x 个, 则甲每小时加工机器零件(x+10) 个, 根据 题意得: xx 120 10 150  解得 x=40 经检验, x=40 是原方程的解 x+10=40+10=50 答: 甲每小时加工 50 个零件, 乙每小时加工 40 个零件. 五、(本题 10 分) 22.证明: (1) ∵OD⊥AC OD 为半径∴ ∴∠CBD=∠ABD ∴BD 平分∠ABC (2) ∵OB=OD∴∠OBD=∠ODB=30°∴∠AOD=∠OBD+∠ODB=30°+30°=60° 又∵OD⊥AC 于 E ∴∠OEA=90°∴∠A=180°-∠OEA-∠AOD=180°-90°-60°=30° 又∵AB 为⊙O 的直径 ∴∠ACB=90°则在 Rt△ACB 中 BC= 2 1 AB ∵OD= 2 1 AB ∴BC=OD 六、(本题 12 分) 23.解:(1)设直线 l1 的表达式为 y=k1x,它过 B(18, 6) 得 18k1=6 k1= 3 1 ∴y= 3 1 x 设直线 l2 的表达式为 y=k2x+b,它过 A (0, 24), B(18, 6)得      618 24 2 bk b 解得      2 12 b k y=-x+24 (2) ①∵点 C 在直线 l1 上, 且点 C 的纵坐标为 a,∴a= 3 1 x x=3a ∴点 C 的坐标为 (3a, a) ∵CD∥y 轴∴点 D 的横坐标为 3a ∵点 D 在直线 l2 上 ∴y=-3a+24 ∴D(3a, -3a+24) ②C(3, 1) 或 C(15, 5) 七、(本题 12 分) 24.解: (1) 过点 P 作 PQ⊥AB 于点 Q ∵PA=PB, ∠APB=120° AB=4 3 ∴AQ= 2 1 AB= 2 1 ×4 3 =2 3 ∠APQ= 2 1 ∠APB= 2 1 ×120°=60°在 Rt△APQ 中, sin∠ APQ= AP AQ ∴AP= 2 3 32 60sin 32 sin APQ AQ =sin60°=4 (2) 过点 P 分别作 PS⊥OM 于点 S, PT⊥ON 于点 T∴∠OSP=∠OTP=90° 在四边形 OSPT 中,∠SPT=360°-∠OSP-∠SOT-∠OTP=360°-90°-60°-90°=120° ∴∠APB=∠SPT=120° ∴∠APS=∠BPT 又∵∠ASP=∠BTP=90° AP=BP ∴△APS≌△BPT ∴PS=PT ∴点 P 在∠MON 的平分线上 (3) ①8+4 3 ②4+4 3 <t≤8+4 3 八、 (本题 14 分) 25.解:(1) 如答图①, ∵A (-2, 0) B (0, 2) ∴OA=OB=2 ∴AB2=OA2+OB2=22+22=8∴AB=2 2 ∵OC=AB∴OC=2 2 , 即 C (0, 2 2 ) 又∵抛物线 y=- 2 x2+mx+n 的图象经过 A、C 两点 则可得      22 0224 n nm 解 得:      22 2 n m ∴抛物线的表达式为 y=- 2 x2- 2 x+2 2 (2) ∵OA=OB ∠AOB=90° ∴∠BAO=∠ABO=45° 又∵∠BEO=∠BAO+∠AOE=45°+∠AOE ∠BEO=∠OEF+∠BEF=45°+∠BEF ∴∠BEF=∠AOE (3) 当△EOF 为等腰三角形时,分三种情况讨论 ①当 OE=OF 时, ∠OFE=∠OEF=45° 在△EOF 中, ∠EOF=180°-∠OEF-∠OFE=180°-45°-45°=90° 又∵∠AOB=90° 则此时点 E 与点 A 重合, 不符合题意, 此种情况不成立. ②如答图②, 当 FE=FO 时, ∠EOF=∠OEF=45° 在△EOF 中,∠EFO=180°-∠OEF-∠EOF=180°-45°-45°=90° ∴∠AOF+∠EFO=90°+90°=180°∴EF∥AO ∴ ∠BEF=∠BAO=45° 又∵ 由 (2) 可知 ,∠ ABO=45°∴∠BEF=∠ABO ∴BF=EF∴EF=BF=OF= 2 1 OB= 2 1 ×2=1 ∴ E(-1, 1) ③如答图③, 当 EO=EF 时, 过点 E 作 EH⊥y 轴于点 H 在△AOE 和△BEF 中, ∠EAO=∠FBE, EO=EF, ∠AOE=∠BEF ∴△AOE≌△BEF ∴BE=AO=2 ∵EH⊥OB ∴∠EHB=90°∴∠AOB=∠EHB ∴EH∥AO ∴∠BEH=∠BAO=45° 在 Rt△BEH 中, ∵∠BEH=∠ABO=45° ∴EH=BH=BEcos45°=2× 2 2 = 2 ∴OH=OB-BH=2- 2 2 ∴ E(- 2 , 2- 2 ) 综上所述, 当△EOF 为等腰三角形时, 所求 E 点坐标为 E(-1, 1)或 E(- 2 , 2- 2 2 ) (4) P(0, 2 2 )或 P (-1, 2 2 ) 吉林省 2012 年中考数学试题(解析版) 一.单项选择题(每小题 2分,共 12 分) 1.在四个数 0,-2,-1,2 中,最小的数是 (A)0. (B)-2. (C) -1 (D)2 [答案]B。 [考点]有理数大小的比较。 [解析] 根据正数大于负数,负数都小于 0,两个负数之间,绝对值大的这个数反而小可得 正确答案。所以选 B 2. 如图,由 5 个完全相同的小正方形组合成一个立体图形,它的俯视图是 [答案]A。 [考点]三视图 [解析]俯视图是在水平面上由上向下观察物理的图形,所以选 A。 3.下列计算正确的是 (A)3 2a a  ; (B) 2 2 22 3a a a  ; (C) 2 3 6a a a  ; (D) 2 2 2( )a b a b   . [答案] B . [考点] 整式的加减:合并同类项;整式的乘法:同底数幂的乘法;乘法公式:完全平方公 式. [解析] 合并同类项:只把同类项的系数相加,所得的结果作为系数字母和字母的指数不变. 所以 2 2 22 3a a a  是正确的,故选 B . 验 证 : 3 2a a a  ; 同 底 数 的 幂 相 乘 , 底 数 不 变 , 指 数 相 加 , 所 以 , 2 3 2 3 5a a a a   ;完全平方公式:两数和的平方,等于它们的平方和加它们积的 2 倍, 即: 2 2 2( ) 2a b a ab b    .所以, , ,A C D 都是错的. 4.如图,在 ABC 中, 80A   , 40B   , D 、 E 分别是 AB 、 AC 上的点,且 DE BC∥ ,则 AED 的度数为 (A)40° (B)60° (C) 80° (D)120° [答案] B . [考点] 平行线的性质;三角形的内角和. [解析] 由三角形的三个内角和为180 ,可得 60C   ; 又 两 直 线 平 行 , 同 位 角 相 等 , 所 以 , 由 DE BC∥ , 可 得 , AED C   , 所 以 60AED   21 世纪教育网 解:在 ABC 中, 180 180 80 40 60C A B            又 DE BC ∥ , AED C   ,所以 60AED   ,故选 B . 5.如图,菱形 OABC 的顶点 B 在 y 轴上,顶点 C 的坐标为(-3,2).若反比例函数 ky x  ( 0x  )的图像经过点 A ,则 k 的值为 (A) -6. (B) -3. (C) 3. (D) 6. [答案] D . [考点] 菱形的性质.直角坐标系内点的点与曲线方程的关 系,求反比例函数中的待定系数. [解析] 如图,因为菱形 OABC 的两条对角线互相垂直平分,又 OB 在 y 轴上,所以顶点 C 、 A 关于 y 轴对称,已知C 的坐标为(-3,2),所以 A 的坐标为(3,2) 反比例函数 ky x  ( 0x  )的图像经过点 A ,则 3 2 6k    ,故选 D . 6. 某工厂现在平均每天比原计划多生产 50 台机器,现在生产 600 台机器所需时间与原计 划生产 450 台机器所需时间相同.设原计划每天生产 x 台机器,则可列方程为 50 450600)(  xxA . 50 450600)(  xxB . xxC 450 50 600)(  . xxD 450 50 600)(  [答案] C . [考点] 分式方程运用:列分式方程. [解析] 因为原计划每天生产 x 台机器,现在平均每天比原计划多生产 50 台,所以,现在 生产 600 台机器所需时间是 600 50x  天,原计划生产 450 台机器所需时间是 450 x 天,故选 C . 二.填空题(每小题 3 分,共 24 分) 7.计算: 12 3 =_ ____. [答案] 3 . [考点] 二次根式:最简二次根式,根式的运算. [解析] 根式的运算顺序:先把各根式化为最简根式,然后合并同类根式. 解:原式 22 3 3 2 3 3 3      . 8.不等式 2 1x x  的解集为__________. [答案] 1x  . [考点] 不等式:解一元一次不等式. [解析] 解一元一次不等式类似解一元一次方程,即把含未知数的项移到一边,数字项移到 另一边,然后系数化 1,但注意如果在不等式两边同时乘或除以一个负数,要把不等号改 变方向. 解:移项得: 2 1x x  合并得: 1x  所以原不等式的解集为 1x  . 9.若方程 2 0x x  ,的两个根为 1 2 1 2, ( )x x x x ,则 2 1x x =______. [答案]1. [考点] 一元二次方程:解一元二次方程,一元二次方程的根与系数的关系(韦达定理). [解析] 本题给出的一元二次方程较为简单,可直接求解,再求其差;也可利用根与系数的 关系求出所需.常用的关系式有: 1 2 bx x a    , 1 2 cx x a   ,学习中还可由求根公式总结 出: 2 2 1 1 22 4 ( )b acx x x xa    解:[方法一] 2 1 20 ( 1) 0 0, 1x x x x x x        , 2 1 1 0 1x x    . [方法二] 由根与系数的关系得: 2 2 1 2 ( 1) 4 1 0 11x x       10. 若甲,乙两个芭蕾舞团参加演出的女演员人数相同,平均身高相同,身高的方差分别 为 2S甲 =1.5, 2S乙 =2.5,则______芭蕾舞团参加演出的女演员身高更整齐(填“甲”或 “乙”). [答案] 甲. [考点] 数据的分析:数据的波动:方差. [解析] 方差越大,数据的波动性越大;方差越小,数据的波动性越小.两组平均数相同的 数据,方差小的说明身高的整齐度高,所以甲芭蕾舞团参加演出的女演员身高更整齐. 11. 如 图 , , ,A B C 是 O 上 的 三 点 , 25CAO   . 35BCO   , 则 AOB  度. [答案] 120 .[来源:21 世纪教育网] [考点] 等腰三角形的性质;圆:圆 内同弧所对的圆 周角与圆心 角的关系(圆周角定理). [解析] 利用等腰三角形两底角相等,圆内同弧所对的圆周角都 等于这条弧所对的圆心角的一半,即可求解. 解 : 如 图 , 在 AOC 中 , AO CO , 25CAO ACO     , 25CAO ACO     . 25 35 60ACB       又 ACB 是 AC 对的圆周角, AOB 是 AC 对的圆心角 2 2 60 120AOB ACB        12. 如图,在 Rt ABC 中, 90ACB   , 3AC  , 4BC  , 以点 A 为圆心, AC 长为半径画弧,交 AB 于点 D ,则 BD  ______. [答案] 2 . [考点] 圆:圆内半径外外相等;直角三角形:勾股定理. [解析] 如图, AC 、 AD 为半径, 3AC AD   .再由勾股定理:勾三股四弦五得 5AB  , 5 3 2BD AB AD      . 13.如图, AB 是 O 的直径, BC 是 O 的切线, 40ACB   ,点 P 在边 BC 上,则 PAB 的度数可能为 (写出一个符合条件的度 数即可). [答案] 30 . [考点] 圆:圆的切线的性质定理:圆的切线垂直于过切点半 径(或直径),直角三角形:直角三角形的两个锐角互余 . [ 解 析 ] 由 圆 的 切 线 垂 直 于 过 切 点 半 径 ( 或 直 径 ) , 90ABC   ,再由直角三角形的两个锐角互余, 40ACB  ,所以 50CAB   , 故只要写出在 0 到50 间的一个角即可. 14.如图,在 等边 ABC 中, D 是边 AC 上的一点,连接 BD ,将 BCD 绕点 B 逆时针旋 转 60,得到 BAE ,连接 ED ,若 10BC  , 9BD  ,则 AED 的周长是______. [答案] 19 . [考点] 图形的旋转:旋转前、后的图形全等;正三角 形,三角形周长. [解析] 由 BCD BAE CD AE     . 10AE AD AC BC     . 又, 9BD BE  , 60DBE  , DBE 是正三角形 9DE  . ADE 的周长: 9 10 19DE EA AD DE AD       三.解答题(每小题 5 分,共 20 分) 15.先化简,再求值: 2( )( ) 2a b a b a   ,其中 1a  , 2b  . [答案] 1. [考点] 化简求值. . [解析] 利用平方差公式,先作整式乘法运算,合并同类项,将原式化简,然后求值. 解: 2 2 2 2 2 2( )( ) 2 2 3a b a b a a b a a b        , 1a  , 2b  时,原式 2 23 1 ( 2) 1    . 16.如图,在东北大秧歌的踩高跷表演中,已知演员身高是高跷长度的 2 倍,高跷与腿重合 部分的长度是 28cm ,演员踩在高跷上时,头顶距离地面的高 度为 224cm .设演员的身高为 xcm ,高跷的长度为 ycm , 求 x , y 的值. [答案] x 的值为168, y 的值为84 . [考点] 实际问题与二元一次方程组 . [解析] 找出能够表示应用题全部题意的两个相等关系,列出代 数式,从而列出两个方程并组成方程组求解 . 解:依题意得方程组: 2 224 28 x y x y      ,解得: 168 84 x y    所以, x 的值为168 , y 的值为84 . 17.如图,有一游戏棋盘和一个质地均匀的正四面体骰子(各面依次标有1,2,3,4 ,四个数 字).游戏规则是游戏者每投掷一次骰子,棋子按骰子着地一面所示的数字前进相应的 格数.例如;若棋子位于 A 处,游戏者所投掷骰子着地一面所示数字为 3,则棋子由 A 处前进 3 个方格到达 B 处.请用画树形 图法(或列表法)求投掷骰子两次后, 棋子恰好由 A 处前进 6 个方格到达 C 处 的概率. [答案] 3 16 . [考点] 概率初步:随机事件与概率:用列举法(列表法或画树形图法)求概率. [解析] 为不重复不遗漏地列出所有可能的结果,通常采用列表法或用画树形图法求随机事 件发生的概率.在一次试验 n 次所有可能的 结果中,事件 A 件出现 m 次的概率为 ( ) mP A n  [列表法] 在这次游戏中,投掷骰子两次,棋 子恰好由 A 处前进 6 个方格到达 C 处, 即,两次投掷骰子着地一面所示数字和为 6 .而所有可能的结果列表如下: 一 二 三 四 一 2 3 4 5 二 3 4 5 6 三 4 5 6 7 四 5 6 7 8 由表容易看出:投掷骰子两次,所有可能的结果有16 种,而棋子恰好由 A 处前进 6 个 方格到达C 处的结果为3 种,所以: P (棋子恰好由 A 处前进 6 个方格到达C 处) 3 16  . [画树形图法] 在这次游戏中,投掷骰子两次,棋子恰好由 A 处前进 6 个方格到达C 处,即,两次投掷骰子着地一面所示数字和为 6 .而所有可能的结果画树图如下: 由图容易看出:投掷骰子两次,所有可能的结果有16 种,而棋子恰好由 A 处前进 6 个 方格到达C 处的结果为3 种,所以: P (棋子恰好由 A 处前进 6 个方格到达C 处) 3 16  . 18.在如图所示的三个函数图像中,有两个函数图像能近似地刻画如下 a 、b 两个情境: 第二次 二次和 第一次 情境 a :小芳离开家不久,发现把作业本忘在家里,于是返回家里找到了作业本再去学 校; 情境b :小芳从家出发,走了一段路程后,为了赶时间,以更快的速度前进. (1)情境 a ,b 所对应的函数图像分别为 , .(填写序号); 26.请你为剩下的函数图像写出一个适合的情境. [答案](1) �,� ;(2)小芳从家出发,到学校上学,放学回到了家. [考点] 函数的图象表示法. [解析] 从函数的图象能形象直观、清晰地呈现函数的一些性质.(1)情境 a :小芳离开家 不久,发现把作业本忘在家里,于是返回家里找到了作业本再去学校,对应的函数图像为 � ;情境b :小芳从家出发,走了一段路程后,为了赶时间,以更快的速度前进,对应的 函数图像为 � ;(2)函数图像 � 能近似地刻画为:小芳从家出发,到学校上学,放学回 到了家.此问答案不为一,只要注意到是从家里出发,出去后有停留,然后返回到家,满足 了这三条就行。 四.解答题(每小题 7 分,共 28 分) 19.在平面直角坐标系中,点 A 关于 y 轴的对称点为 B , 点 A 关于原点 O 的对称点为点C . (1)若点 A 的坐标为 (1,2) ,请你在给出的坐标系中画出 ABC . 设 AB 与 y 轴 的 交 点 为 D , 则 ADO ABC S S △ △ =________; (2)若点 A 的坐标为 ( , )a b 0ab  ,则 ABC 的形状为 _______.21 世纪教育网 [答案] (1)图形如图19 1 , 1 4 ADO ABC S S △ △ ;(2) ABC 为直角三角形. [考点] 轴对称:用坐标表示轴对称,关于原点对称,相 似三角形的判定、性质.勾股定理的逆定理[来源:Zxxk.Com] [解析] (1)点 A 的坐标为 (1,2) ,关于 y 轴的对称点 B 的坐标为 ( 1,2) ,点 A 关于原点 O 的对称点 C 的坐标为 ( 1, 2)  ,作出点 A 、 B 、 C 、连得 ABC 如图19 1 . 又 AB 与 y 轴的交点为 D ,所以 D 的坐标为 (0,2) ,图中 1 2 AD AO AB AC A A        ADO ABC  , 1 4 ADO ABC S S  △ △ ; (2)由点 A 的坐标为 ( , )a b ,关于 y 轴的对称点 B 的坐标为 ( , )a b ,点 A 关于原点O 的 对称点 C 的坐标为 ( , )a b  ,如图19 2 ,图中: 2AB a 、 2BC b 、 2 22AC a b  , 2 2 2 2 24( )AB BC a b AC    , ABC 为直角三角形。 20.如图,沿 AC 方向开山修一条公路,为了加快施 工进度,要在小山的另一边寻找点 E 同时施工.从 AC 上 的 一 点 B 取 127ABD   , 沿 BD 方 向 前 进 , 取 37BDE   , 测 得 520BD m ,并且 AC 、 BD 和 DE 在同一平面内. (1)施工点 E 离 D 多远正好能使 , ,A C E 成一直线(结果保留整 数); (2)在(1)的条件下,若 80BC m ,求公路 CE 段的长(结果保留整 数) (参考数据:sin37 0.60  , cos37 0.80  , tan37 0.75  ) [答案] (1) 416m ;(2) 232m . [考点] 锐角三角函数:已知一边及一锐角解直角三角形. [解析](1) B 在 AC 上, 127ABD   , 53ABD   , 要使 , ,A C E 成一直线.只要 90BED   .即 BED .为直角 三角形即可,此时,施工点 E 离 D 的距离为 cos37 520 0.80 416( )BD m     . (2)已知一边及一锐角解直角三角形 BED ,得 sin37 520 0.60 80 232( )CE BE BC BD BC m         21.为宣传节约用水,小明随机调查了某小区部分家庭 5 月份的用水情况,并将收集的数据 整理成如下统计图. (1)小明一共调查了多少户家庭? (2)求所调查家庭 5 月份用水量的众数、 平均数; (3)若该小区有 400 户居民,请你估计这 个小区 5 月份的用水量. [答案](1)20 户.(2)4、4.5.(3)1800 吨. [考点] 数据的分析:数据 的代表:平均数、从数;数据的收集、整理与描述:统计调 查,直方图:条形图:. [解析] (1)小明调查的家庭 5 月份用水量 1 吨、2 吨、8 吨的各有 1 户,6 吨、7 吨的各 有 2 户 , 3 吨 的 有 3 户 , 5 吨 的 有 4 户 , 4 吨 的 有 6 户 , 总 户 数 : 3 1 2 2 3 4 6 20       (户) (2)用水量 4 吨的有 6 户家庭,居最多,故众数为 4 吨. 平均数数 1 1 1 2 3 3 6 4 4 5 2 6 2 7 1 8 90 4.520 20                  (吨). ( 3 ) 400 户 居 民 在 5 月 份 用 水 量 约 为 : 400 4.5 1800  (吨). 22.如图,在 ABC 中, AB AC , D 为边 BC 上一点,以 AB 、 BD 为邻边作平行四边 形 ABDE ,连接 AD , EC . (1)求证: ADC ECD   ; (2)若 BD CD ,求证四边形 ADCE 是矩形. [考点] 等腰三角形:等腰三角形两底解相等;四边形:平行四边形的性质:平行四边形对 边平行且相等;特殊平行四边形的判定:矩形的判定;全等三角形:全等三角形的判定 ( SAS ). [解析] (1)如图(第 22 题(1)) 由  D BC AB ED ABDE AB ED B EDC       为边 上一点 ∥ 又,在 ABC 中, AB AC B ACD     , 所以, AC ED , ACD EDC   , 在和 ADC 和 ECD 中, ( ) ( ) ( ) ( ) AC ED ACD EDC ADC ECD SAS DC CD           = 已证 已证 公共边 . (2)如图(第 22 题(2)) 由 AE BDABDE AE BD    ∥ , 又,在 ABC 中, AB AC 、 BD CD 所以, AE CD , AD DC , 故,四边形 ADCE 是矩形.(有一个角是直角的平行四边形叫做矩形) 五.解答题(每小题 8 分,共 16 分) 23.如图,在扇形OAB 中, 90AOB   ,半径 6OA  .将扇形OAB 沿过点 B 的直线折 叠.点 O 恰好落在 AB 上点 D 处,折痕交 OA 于点 C ,求整个阴 影部分的周长和面 积. [答案] 周长:12 3 ;面积:9 12 3  . [考点] 图形的折叠:折叠前、后的图形全等;全等三角形的性质:全等三角形对应 边相 等,全等三角形对应角相等;圆:弧长和扇形面积:弧长 180 n R , 2 360 n RS 扇形 .正三角 形的判定:三边相等的三角形是正三角形.正三角形的性质.锐角三角函数:解直角三角形. [ 解 析 ] 如 图 ( 第 23 题 ) , 由 折 叠 前 、 后 的 图 形 全 等 . 所 以 , BOC BDC   , DB BO , DC CO . 又 在 扇 形 OAB 中 , 90AOB   , 半 径 6OA  . 所 以 , 6DB BO  , 6DC CA OC CA OA     , AB 的长 90 6 3180    .所以, 整个阴影部分的周长 AB 的长 ( ) 3 6 6 12 3BD DC CD          . 如图(第 23 题-1),连接扇形OAB 的半径OD , 由 6OD OB BD    正三角形 30BOD OBC    , 在 Rt BOC 中, 3tan30 6 2 33OC OB      , 12 2 2 3 6 12 32BOCBOCDS S OC OB       四边形 所以,整个阴影部分的面积 290 6 12 3 9 12 3360OAB BOCDS S       扇形 四边形 24.如图 1, , ,A B C 为三个超市,在 A 通往 C 的道路(粗实线部分)上有一 D 点, D 与 B 有道路(细实线部分)相通. A 与 D , D 与 C , D 与 B 之间的路程分别为 25km ,10km ,5km .现计划在 A 通往 C 的道路上建一个配货中心 H ,每天有 一辆货车只为这三个超市送货.该货车每天从 H 出发,单独为 A 送货1次,为 B 送货1次,为 C 送货 2 次.货车每次仅能给一家超市送货, 每次送货后均返回配货中心 H .设 H 到 A 的路 程为 xkm .这辆货车每天行驶的路程为 ykm . (2)用含 x 的代数式填空: 当 0 x≤ ≤25 时,货车从 H 到 A 往返1次的路程为 2xkm . 货车 从 H 到 B 往返1次的路程为_______ km . 货车从 H 到C 往返 2 次的路程为_______ km . 这辆货车每天行驶的路程 y  __________. 当 25 x ≤35 时, 这辆货车每天行驶的路程 y  _________; (2)请在图 2 中画出 y 与 x ( 0 x≤ ≤35 )的函数图象; (3)配货中心 H 建在哪段,这辆货车每天行驶的路程最短? [答案](1) 60 2x , 140 4x , (200 4 )x km , 100km ;(2)如图 2-1;(3) 100km . [考点] 一次函数:一次函数的运用:根据题意列出 一次函数,确定自变量的取舍范围;作一次函数图 象. [解析] 因为 A 与 D 之间的路程为 25km ,当 0 x≤ ≤25 时, H 在 A 与 D 路段上,如图 (第 24 题图 1-1),又, D 与 B 之间的路程为5km ,此时, 货 车 从 H 到 A 往 返 1 次 的 路 程 为 2xkm , 从 H 到 B 往 返 1 次 的 路 程 为 : (25 5) 2 2 60 2 ( )x x km     . 货车从 D 与C 之间的路程为10km , H 到C 往返 2 次的路程为: 2 [(25 10) 2 2 ] 2 (70 2 ) 140 4 ( )x x x km         ; 这 辆 货 车 每 天 行 驶 的 路 程 : 2 (60 2 ) (140 4 ) 200 4 ( )y x x x x km       . 当 25 x ≤35 时, H 在 D 与 C 路段上,如图(第 24 题图 1-2),此时,货车从 H 到 B 往返1次的路程 为: ( 25 5) 2 2 40( )x x km     ,从 H 到 C 往返 2 次的路程还是 (140 4 )x km ; 这 辆 货 车 每 天 行 驶 的 路 程 为 : 2 (2 40) (140 4 ) 100 ( )y x x x km      . (2)由(1)得 y 与 x ( 0 x≤ ≤35 ) 的解析式为: 200 4 (0 ) 100 (25 ) x xy x     ≤ ≤25 ≤35 描点作出相应图象如图(第 24 题图 2-1).; (3)由(1)(2)得知,当 25 x≤ ≤35 时, 100y km ,所以,只要配货中心 H 建在 D 与C 之间(包括 D 、 C )的路段上,这辆货车每天行驶的路程都是100km ,为最短路 程. 六.解答题(每小题 10 分,共 20 分). 25.如图,在 ABC 中, 90A   , 2AB cm , 4AC cm ,动点 P 从点 A 出发,沿 AB 方向以1 /cm s 的速度向点 B 运动,动点Q 从点 B 同时出发,沿 BA 方向以1 /cm s 的速 度向点 A 运动.当点 P 到达点 B 时, P , Q 两 点同时停止运动.以 AP 为一边向上作 正方形 APDE ,过点Q 作QF BC∥ ,交 AC 于点 F .设点 P 的运动时间为ts ,正方形 APDE 和梯形 BCFQ 重合部分的面积为 2Scm . (1)当t  _____s 时,点 P 与点Q 重合; (2)当t  _____s 时,点 D 在QF 上; (3)当点 P 在Q , B 两点之间(不包括Q , B 两点)时,求 S 与t 之间的函数关系式. [答案] (1) 1; (2) 4 5 . (3) 29 4 4 3 4 4 3 3 29 4 4 3 2 (1 ) ( ) 10 8 ( 2) t t t S t t t t            . [考点] 动点问题,一次函数、二次函数综合运用,数学分类讨论思想. [解析] (1) 因为动点 P 从点 A 出发,沿 AB 方向以1 /cm s 的速度向点 B 运动,动点Q 从 点 B 同时出发,沿 BA 方向以1 /cm s 的速度向点 A 运动. P ,Q 同时出发,运动速度都是 1 /cm s , 所 以 P , Q 运 动 到 AB 的 中 点 时 重 合 , 2AB cm , 1 12 AB cm , 此 时 1 11t   . (2) 如图(第 25 题-1),以 A 为直角坐标系的原点, AB 方向为 x 轴的正方向, AC 方向为 y 轴的正方向,建立直角坐 标系,则 (0,0)A 、 (2,0)B 、 (0,4)C . 设 t 时刻时,点 D 在 QF 上,因为正方形 APDE ,所以 ( ,0)P t 、 ( , )D t t 、 (2 ,0)Q t 、又在 ABC 中, 90A   , 2AB cm , 4AC cm , tan 2ACABC AB     . 又 QF BC ∥ , tan tan 2AQF ABC     , 在 Rt AQF 中 , tan (2 ) 2 4 2QF AQ AQF t t        , (0,4 2 )F t  , 得 过 (2 ,0)Q t 、 (0,4 2 )F t 的一次函数的解析式为: 2 4 2 (0 )y x t x    ≤ ≤2 ,由 D 在QF 上,所以 D 的坐标满足QF 的解析式,即: 42 4 2 5t t t t      . (3)因为由(1)知 P , Q 在 1t  时相遇,所以,只有当1 2t  时,点 P 在 Q , B 两点 之间(不包括 Q , B 两点),正方形 APDE 和梯形 BCFQ 重合部分随 D 的位置变化有三 种情况: � D 在 QF BC与 之间; � D 在 BC 上; � D 在 QF BC与 之外. � D 在QF BC与 之 间;如图(第 25 题-2),此时,正方 形 APDE 和 梯 形 BCFQ 重 合 部 分 为 直 角 梯 形 , 由 (2) 得 : ( ,0)P t 、 ( , )D t t 、 (2 ,0)Q t 、过 QF 的一次函数的解析式为: 2 4 2 (0 )y x t x    ≤ ≤2 、设 DE 与 QF 的交点为G , 解 2 4 2 y t y x t       ,得: 3(2 , )2G t t . 所以, (2 ) 2 2QP AP AQ t t t       , 3 5(2 ) 22 2GD ED EG t t t       , 此时: 2 21 1 5 9( ) (2 2 2) 2 ( )2 2 2 4S QP GD PD t t t t t cm          . � D 在 BC 上;如图(第 25 题-3), ( , )D t t 满足过 BC 的一次函数的解析式: 2 4 (0 )y x x   ≤ ≤2 , 即: 42 4 3t t t     , 4 4( , )3 3D , 把 4 3t  代入QF 的一次函数的解析式得: 42 (0 )3y x x   ≤ ≤2 , 4(0, )3F , 所 以 , ,E F G 为 同 一 点 , 所 以 : 4 4 2(2 )3 3 3QP     , 4 3GD  ,此时: 21 1 2 4 4 4( ) ( ) ( )2 2 3 3 3 3S QP GD PD cm       � D 在QF BC与 之外.如图(第 25 题-4),设 PD 与 BC 相交于 M , ED 与 BC 相 交于 N , 解 2 4y x x t      得: ( , 2 4)M t t  ; 解 2 4y x y t      得: 1( 2, )2N t t  . 所以, ( 2 4) 3 4MD PD PM t t t        1 3( 2) 22 2ND ED EN t t t        此时: 2 1 1 2 2AQF MDNAPDES S S S t AQ AF MD ND        正方形 2 2 2 2 21 1 3 3 9(2 ) (4 2 ) (3 4)( 2) (2 ) ( 2) 10 82 2 2 2 4t t t t t t t t t t                 综合 �、 �、 �,得点 P 在 Q , B 两点之间(不包括 Q , B 两点),正方形 APDE 和梯形 BCFQ 重合部分的面积为 2Scm 与t 之间的函数关系式为: 29 4 4 3 4 4 3 3 29 4 4 3 2 (1 ) ( ) 10 8 ( 2) t t t S t t t t            26.问题情境[ 如图,在 x 轴上有两点 ( ,0)A m , ( ,0)B n ( 0n m  ).分别过点 A ,点 B 作 x 轴 的垂线,交抛物线 2y x 于点 C 、点 D .直线 OC 交直线 BD 于点 E ,直线 OD 交直 线 AC 于点 F ,点 E 、点 F 的纵坐标分别记为 .Ey 、 Fy . K] 特 例 探究 填 空: 当 1m  , 2n  时, .Ey =____, Fy =______.当 3m  , 5n  时, .Ey =____, Fy =______.[来源:21 世纪教育网] 归纳证明 对任意 m , n ( 0n m  ),猜想 .Ey 与 Fy 的大小关系,并证明你的猜想 拓展应用. (3) 若将“抛物线 2y x ”改为“抛物线 2 ( 0)y ax a  ”,其它条件不变,请直接 写出 .Ey 与 Fy 的大小关系. (4) 连接 EF , AE .当 . 3 OFEOFEBS S △四边形 时,直接写出 m 和 n 的关系及四边形 OFEA 的形状. [ 答 案 ] 特 例 探 究 2,2 ; 15,15 . 归 纳 证 明 猜 想 E Fy y . 证 明 ( 略 ) 拓 展 应 用 (1) E Fy y .(2)四边形OFEA 是平行四边形. [考点] 一次函数、二次函数综合运用,函数图象上的点与函 数解析式的关系,平行四边形的判定. [解析] 特例探究[来源:Zxxk.Com] 当 1m  , 2n  时, (1,1)C , (2,4)D ,所以直线 OC 的 解析式为: y x ;直线OD 的解析式为: 2y x ;此时 解 2x y x    ,得 (2,2) 2EE y  .解 1 2 x y x    ,得 (1,2) 2FF y  . 所以,此时 1 2 2E Fy y    当 3m  , 5n  时, (3,9)C , (5,25)D ,所以直线OC 的解析式为: 3y x ;直线 OD 的解析式为: 5y x ;此时 解 5 3 x y x    ,得 (5,15) 15EE y  .解 3 5 x y x    ,得 (3,15) 15FF y  . 所以,此时 3 5 15E Fy y    归纳证明 猜想:对任意 m , n ( 0n m  ),都有: E Fy y . 证明:对任意 m , n ( 0n m  )时, 2( , )C m m , 2( , )D n n ,所以直线 OC 的 解析式为: y mx ;直线OD 的解析式为: y nx ;此时 解 x n y mx    ,得 ( , ) EE n mn y mn  .解 x m y nx    ,得 ( , ) FF n mn y mn  . 所以,此时 E Fy y mn  . 拓展应用 (1)若将“抛物线 2y x ”改为“抛物线 2 ( 0)y ax a  ”,其它条件不变,仍然 有: E Fy y . 此时, 2( , )C m am , 2( , )D n an ,所以直线OC 的解析式为: y amx ;直线OD 的 解析式为: y anx ;此时 解 x n y amx    ,得 ( , ) EE n amn y amn  .解 x m y anx    ,得 ( , ) FF n amn y amn  . 哈尔滨市 2012 年初中升学考试 数学试卷 一、选择题(每小题 3 分.共计 30 分) 1.一 2 的绝对值是( ). (A)一 1 2 (B) 1 2 (C)2 (D)-2 2.下列运算中,正确的是( ). (A)a3·a4=a12 (B)(a3)4=a12 (C)a+a4=a5 (D)(a+b)(a—b)=a2+b2 3.下列图形是中心对称图形的是( ). 4.如图所示的几何体是由六个小正方体组合而成的,它的左视图是( ). 5.如图,在 Rt△ABC 中,∠C=900,AC=4,AB=5,则 sinB 的值是( ). (A) 2 3 (B) 3 5 (C) 3 4 (D) 4 5 6.在 10 个外观相同的产品中,有 2 个不合格产品。现从中任意抽取 l 个进行检测, 抽到不合格产品的概率是( ). (A) 1 10 (B) 1 5 (C) 2 5 (D) 4 5 新课 标第一 网 7.如果反比例函数 y= 1k x  的图象经过点(-1,-2),则 k 的值是( ). (A)2 (B)-2 (C)-3 (D)3 8.将抛物线 y=3x2 向左平移 2 个单位,再向下平移 1 个单位,所得抛物线为( ). (A)y=3(x+2)2—1 (B)y=3(x-2)2+1 (C)y=3(x-2)2—1 (D)y=3(x+2)2+l 9.如图,⊙0 是△ABC 的外接圆,∠B=600,0P⊥AC 于点 P,OP=2 3 ,则⊙0 的半径为 ( ). (A)4 3 (B)6 3 (C)8 (D)12 1 0.李大爷要围成一个矩形菜园,菜园的一边利用足够长的墙,用篱笆围成的另外三 边总长应恰好为 24 米.要围成的菜园是如图所示的矩形 ABCD.设 BC 边的长为 x 米, AB 边的长为 y 米,则 y 与 x 之间的函数关系式是( ). (A)y=一 2x+24(00x-1<1 16.一个等腰三角形静的两边长分别为 5 或 6,则这个等腰三角形的周长 是 . 17.一个圆锥的母线长为 4,侧面积为 8 ,则这个圆锥的底面圆的半径 是 . 18.方程 1 3 1 2 3x x   的解是 19.如图,平行四边形 ABCD 绕点 A 逆时针旋转 300,得到平行四边形 AB1C1D1(点 B1 与点 B 是对应点,点 C1 与点 C 是对应点,点 D1 与点 D 是对应点),点 B1 恰好落在 BC 边上,则∠C= 度. 20.如图。四边形 ABCD 是矩形,点 E 在线段 CB 的延长线上,连接 DE 交 AB 于点 F, ∠AED=2∠CED,点 G 是 DF 的中点,若 BE=1,AG=4,则 AB 的长为 [来源:学|科|网] 三 、 解 答 题 (其 中 21 ~ 24 题各 6 分,25~26 题各 8 分,27~28 题各 l0 分,共计 60 分) 21.(本题 6 分) 先化简,再求代数式 2 1 1 2( )x x x x x x     的值,其中 x= cos300+ 1 2 22.(本题 6 分) ? 图 l、图 2 是两张形状、大小完全相同的方格纸,方格纸中的每个小正方形的边长 均为 1.点 A 和点 B 在小正方形的顶点上. (1)在图 1 中画出△ABC(点 C 在小正方形的顶点上),使△ABC 为直角三角形(画一个 即可); (2)在图 2 中画出△ABD(点 D 在小正方形的顶点上),使△ABD 为等腰三角形(画一个 即可); 23.(本题 6 分如图,点 B 在射线 AE 上,∠CAE=∠DAE,∠CBE=∠ADBE. 求证:AC=AD.[来源:Z。xx。k.Com] 24.(本题 6 分) 小磊要制作一个三角形的钢架模型,在这个三角形中,长度为 x(单位:cm)的边与 这条边上的高之和为 40 cm,这个三角形的面积 S(单位:cm2)随 x(单位:cm)的变化而 变化. (1)请直接写出 S 与 x 之间的函数关系式(不要求写出自变量 x 的取值范围); (2)当 x 是多少时,这个三角形面积 S 最大?最大面积是多少?[来源:Zxxk.Com] 25.(本题 8 分) 虹承中学为做好学生“午餐工程”工作,学校工作人员搭配了 A,B,C,D 四种不同 种 类的套餐,学校决定围绕“在 A,B,C,D 四种套餐中,你最喜欢的套餐种类是什么? (必选且只选一种)”的问题,在全校范围内随机抽取部分学生进行问卷调查,将调查 问 适当整理后绘制成如图所示的不完整的条形统计图,其中最喜欢 D 种套餐的学生占被 抽 取人数的 20%. 请你根据以上信息解答下列问题: (1)在这次调查中,一共抽取了多少名学生? (2)通过计算,补全条形统计图; (3)如果全校有 2 000 名学生.请你估计全校学生中最喜欢 B 种套餐的学生有多少 名? [来源:Zxxk.Com] 26.(本题 8 分) 同庆中学为丰富学生的校园生活,准备从军跃体育用品商店一次性购买若干个足 球和篮球(每个足球的价格相同,每个篮球的价格相同),若购买 3 个足球和 2 个篮球 共需 310 元.购买 2 个足球和 5 个篮球共需 500 元. (1)购买一个足球、一个篮球各需多少元? (2)根据同庆中学的实际情况,需从军跃体育用品商店一次性购买足球和篮球共 96 个.要求购买足球和篮球的总费用不超过 5 720 元,这所中学最多可以购买多少个篮 球? 27.(本题 l0 分) 如图,在平面直角坐标系中,点 0 为坐标原点,直线 y=2x+4 交 x 轴于点 A, 交 y 轴于点 B,四边形 ABC0 是平行四边形,直线 y=_x+m 经过点 C,交 x 轴于点 D. (1)求 m 的值; (2)点 P(0,t)是线段 OB 上的一个动点(点 P 不与 0,B 两点重合),过点 P 作 x 轴的平行线,分别交 AB,0c,DC 于点 E,F,G.设线段 EG 的长为 d,求 d 与 t 之间 的函数关系式(直接写出自变量 t 的取值范围); (3)在(2)的条件下,点 H 是线段 OB 上一点,连接 BG 交 OC 于点 M,当以 OG 为 直径 的圆经过点 M 时,恰好使∠BFH=∠AB0.求此时 t 的值及点 H 的坐标. 28.(本题 10 分) 已知:在△ABC 中,∠ACB=900,点 P 是线段 AC 上一点,过点 A 作 AB 的垂线, 交 BP 的延长线于点 M,MN⊥AC 于点 N,PQ⊥AB 于点 Q,A0=MN. (1)如图 l,求证:PC=AN; (2)如图 2,点 E 是 MN 上一点,连接 EP 并延长交 BC 于点 K,点 D 是 AB 上一 点,连接 DK,∠DKE=∠ABC,EF⊥PM 于点 H,交 BC 延长线于点 F,若 NP=2,PC=3,CK: CF=2:3,求DQ 的长. [来源:Zxxk.Com] 哈尔滨市 2012 年初中升学考试 数学试题参考答案及评分标准 一、选择题: l.C; 2.B; 3.A; 4.C; 5.D; 6.B; 7. 8.A; 9.A; 10.B. 二、填空题:11. 1.6×107; 12.x≠5; 13.3; 14.a(a-1)2; 15. 1 2 0y x bx b 的“抛物线三角形”是等腰直角三角形,求b 的值; (3)如图,△OAB 是抛物线  2=- + ' '>0y x bx b 的“抛物线三角形”,是否存在以原点O 为对称中心的矩形 ABCD ?若存在,求出过O C D、 、 三点的抛物线的表达式;若不 存在,说明理由. [来源:学#科#网 Z#X#X#K] 25.(本题满分 12 分) 如图,正三角形 ABC 的边长为3+ 3 . (1)如图①,正方形 EFPN 的顶点 E F、 在边 AB 上,顶点 N 在边 AC 上.在正三角 形 ABC 及其内部,以 A 为位似中心,作正方形 EFPN 的位似正方形 ' ' ' 'EFPN ,且使正 方形 ' ' ' 'EFPN 的面积最大(不要求写作法); (2)求(1)中作出的正方形 ' ' ' 'EFPN 的边长; (3)如图②,在正三角形 ABC 中放入正方形 DEMN 和正方形 EFPH ,使得 DE EF、 在边 AB 上,点 P N、 分别在边CB CA、 上,求这两个正方形面积和的最大 值及最小值,并说明理由. 参考答案 1、【答案】A 【解析】通过题意我们可以联想到数轴,零摄氏度即原点,大于零摄氏度为正方向, 数值为正数, 小于零摄氏度为负数.故选 A. 2、【答案】C 【解析】三视图主要考查学生们的空间想象能力,是近几年中考的必考点,从图中我 们可以知道正 面为三个正方形,(下面两个,上面一个),左视图即从左边观看,上边有一个正方 形,下 面两个正方体重叠,从而看到一个正方形,故选 C. 3、【答案】D 【解析】本题主要考查了数的乘方以及幂的乘方,从整体看,外边是个平方,那么这 个数肯定是正 数,排除 A,C,然后看到 5 的平方,是 25, 3a 的平方是 6a ,积为 625a ,选 D. 4、【答案】C 【解析】统计题目也是年年的必考题,注重学生们的实际应用能力,根据题目规则, 去掉一个最高 分和一个最低分,也就是不算 89 分和 97 分,然后把其余数求平均数,得到 94 分.其实这 种计算有个小技巧,我们看到都是 90 多分,所以我们只需计算其个位数的平均 数,然后再 加上 90 就可以快速算出结果.个位数平均数为 45)62522(  ,所以其 余这些数 的平均数为 94 分.故选 C. 5、【答案】D 【解析】本题主要考查了三角形的中位线的性质,由题意可知, ED 为 ABC 的中位 线,则面积比  ABCEDC SS : 4:1)2 1()( 22  AB ED ,故选 D. 6、【答案】A 【解析】本题考查了一次函数的图象性质以及应用,若干点在同一个正比例函数图像 上,由 kxy  , 可知, y 与 x 的比值是相等的,代进去求解,可知,A 为正确解.选 A. 7、【答案】B 【解析】本题考查了菱形的性质,我们知道菱形的对角线互相平分且垂直,外加 OE AB ,即可得 出  651302 1 2 1 ABCOBEAOE .选 B. 8、【答案】D 【解析】一次函数交点问题可以转化为二元一次方程组求解问题,解得 x=2, y=1.选 D. 9、【答案】C 【解析】本题考查圆的弦与半径之间的边角关系,连接 OB,OD,过 O 作 OH AB ,交 AB 于点 H . 在 OBHRt 中,由勾股定理可知,OH=3,同理可作 ABOE  ,OE=3,且易证 OPHOPE  ,所以 OP= 23 ,选 C. 10、【答案】B 【解析】本题考查了抛物线的平移以及其图像的性质,由 )2)(3(62  xxxxy ,可知其与 x 轴有两个交点,分别为   3 0 -2 0, , , .画图,数形结合,我们得到将抛物线向 右平移 2 个单位,恰好使得抛物线经过原点,且移动距离最小.选 B. 11、【答案】-5 2+1 【解析】原式 2=2 -3 2 2+1=-5 2+12   12、【答案】  2-xy x y 【解析】    23 2 2 3 2 2- 2 -2 -x y x y xy xy x xy y xy x y    13、A【答案】 2 3  【解析】将长度为 4 的线段 AB 绕它的中点 M ,按逆时针方向旋转 30°,则线段 AB 扫过部分的形 状为半径为 2,圆心角度数为 30°的两个扇形,所以其面积为 230 2 22 =360 3   . B【答案】2.47 14、【答案】3 【解析】设小宏能买 x 瓶甲饮料,则买乙饮料 10-x 瓶.根据题意,得  7 +4 10- 50x x  解得 133x  所以小宏最多能买 3 瓶甲饮料. 15、【答案】 18=y x (只要 = ky x 中的 k 满足 9> 2k 即可) 【解析】设这个反比例函数的表达式是 = ky x  0k  . 由 = =-2 +6 ky x y x   , , 得 22 -6 + =0x x k . 因为这个反比例函数与一次函数的图象没有交点,所以方程 22 -6 + =0x x k 无解. 所以  2= -6 -4 2 =36-8 <0k k  ,解得 9> 2k . 16、【答案】 41 【解析】方法一:设这一束光与 x 轴交与点C ,过点C 作 x 轴的垂线CD , 过点 B 作 BE x 轴于点 E . 根据反射的性质,知 ACO BCE   . 所以 Rt ACO Rt BCE  .所以 =AO BE CO CE . 已知 =2AO , =3BE , + =4OC CE ,则 2 3=4-CE CE . 所以 12= 5CE , 8= 5CO . 由勾股定理,得 2= 415AC , 3= 415BC ,所以 = + = 41AB AC BC . 方法二:设设这一束光与 x 轴交与点C ,作点 B 关于 x 轴的对称点 'B ,过 'B 作 'BD y 轴 于点 D . 由反射的性质,知 'A C B, , 这三点在同一条直线上. 再由对称的性质,知 ' =BC BC . 则 = + = ' 'AB AC CB AC CB AB  . 由题意易知 =5AD , ' =4BD ,由勾股定理,得 '= 41AB .所以 = '= 41AB AB . 17、【答案】解:原式= (2 )( ) ( ) ( )( ) 2 a b a b b a b a b a b a b a b        = 2 2 22 2 ( )( 2 ) a ab ab b ab b a b a b        = 22 4 ( )( 2 ) a ab a b a b    = 2 ( 2 ) ( )( 2 ) a a b a b a b    = 2a a b . 18、【答案】解:(1)如图,在 ABCD 中, / /AD BC , ∴ 2 3   . ∵ BF 是 ABC 的平分线, ∴ 1 2   . ∴ 1 3   . ∴ AB AF . (2) 2 3AEF CEB      , , ∴△ AEF ∽△CEB , ∴ 3 5 AE AF EC BC   , ∴ 3 8 AE AC  . 19、【答案】解:(1)如图所示 一周内该校学生从图书馆借出各类图书数量情况统计图 (2)该学校学生最喜欢借阅漫画类图书. (3)漫画类:600×40%=240(本),科普类:600×35%=210(本), 文学类:600×10%=60(本),其它类:600×15%=90(本). 20、【答案】解:如图,作CD AB 交 AB 的延长线于点 D , 则 45 65BCD ACD     , . 在 Rt△ ACD 和 Rt△ BCD中, 设 AC x ,则 sin 65AD x  , cos65BD CD x   . ∴100 cos65 sin65x x    . ∴ 100 207sin 65 cos65x    (米). ∴湖心岛上的迎宾槐C 处与凉亭 A 处之间距离约为 207 米. 21、【答案】解:(1)设 +y kx b ,则有 299, 2000 235. b k b     解之,得 4 125 299. k b      , ∴ 4 299125y x   . (2)当 1200x  时, 4 1200 299 260.6125y      (克/立方米). ∴该山山顶处的空气含氧量约为 260.6 克/立方米. 22、【答案】解:(1)随机掷两枚骰子一次,所有可能出现的结果如右表: 右表中共有 36 种等可能结果,其中点 数和 为 2 的结果只有一种. ∴ P (点数和为 2)= 1 36 . (2)由右表可以看出,点数和大于 7 的 结果 有 15 种. ∴ P (小轩胜小峰)= 15 36 = 5 12 . 23、【答案】解:(1)证明:如图,连接OA ,则OA AP . ∵ MN AP , ∴ //MN OA . ∵ //OM AP , ∴四边形 ANMO 是矩形. ∴ =OM AN . (2)连接OB ,则OB BP . ∵ =OA MN , =OA OB , //OM AP ,[来源:学.科.网] ∴ =OB MN , =OMB NPM  . ∴ Rt OBM Rt MNP   . ∴ =OM MP . 骰子 2 骰子 1 1 2 3 4 5 6 1 2 3 4 5 6 7 2[来源:学科网] 3 4 5 6 7 8 3 4 5 6 7 8 9 4 5 6 7 8 9 1 0 5 6 7 8 9 10 1 1 6 7 8 9 10 11 1 2 设 =OM x ,则 =9-NP x . 在 Rt MNP 中,有  22 2=3 + 9-x x . ∴ =5x .即 =5OM . 24、【答案】解:(1)等腰 (2)∵抛物线  2=- + >0y x bx b 的“抛物线三角形”是等腰直角三角形,[来源:Z.xx.k.Com] ∴该抛物线的顶点 2 2 4 b b     , 满足 2 =2 4 b b  >0b . ∴ =2b . (3)存在. 如图,作△OCD 与△OAB 关于原点O 中心对称, 则四边形 ABCD 为平行四边形. 当 =OA OB 时,平行四边形 ABCD 为矩形. 又∵ =AO AB , ∴△OAB 为等边三角形. 作 AE OB ,垂足为 E . ∴ =AE 3OE . ∴   2' '= 3 '>04 2 b b b . ∴ '=2 3b . ∴  3 3A , ,  2 3 0B , . ∴  - 3 -3C , ,  -2 3 0D , . 设过点O C D、 、 三点的抛物线 2= +y mx nx ,则 12 -2 3 =0 3 - 3 =-3. m n m n   , 解之,得 =1 =2 3. m n  , ∴所求抛物线的表达式为 2= +2 3y x x . 25、【答案】解:(1)如图①,正方形 ' ' ' 'EFPN 即为所求. (2)设正方形 ' ' ' 'EFPN 的边长为 x . ∵△ ABC 为正三角形, ∴ 3'= '= 3AE BF x . ∴ 2 3+ =3+ 33x x . ∴ 9+3 3= 2 3+3 x ,即 =3 3-3x .(没有分母有理化 也对, 2.20x  也正确) (3)如图②,连接 NE EP PN, , ,则 =90NEP  . 设正方形 DEMN 、正方形 EFPH 的边长分别为 m n、  m n , 它们的面积和为 S ,则 = 2NE m , = 2PE n . ∴  2 2 2 2 2 2 2= + =2 +2 =2 +PN NE PE m n m n . ∴ 2 2 21= 2S m n PN  . 延长 PH 交 ND 于点G ,则 PG ND . 在 Rt PGN 中,    2 22 2 2= + = + + -PN PG GN m n m n . ∵ 3 3+ + + = 3+33 3m m n n ,即 + =3m n . ∴ⅰ)当 2- =0m n 时,即 =m n 时, S 最小. ∴ 21 9= 3 =2 2S 最小 . ⅱ)当 2-m n 最大时, S 最大. 即当 m 最大且 n 最小时, S 最大. ∵ + =3m n ,由(2)知, =3 3-3m最大 . ∴  =3- =3- 3 3-3 =6-3 3n m最小 最大 . ∴  21= 9+ -2S m n   最大 最大 最小  21= 9+ 3 3-3-6+3 3 =99-54 32      . [来源:学科网] 2012 年安徽省初中毕业学业考试 数学 本试卷共 8 大题,计 23 小题,满分 150 分,考试时间 120 分钟。 题 号 一 二 三 四 五 六 七 八 总 分 得 分 一、选择题(本大题共 10 小题,每小题 4 分,满分 40 分) 每小题都给出代号为 A、B、C、D 的四个选项,其中只有一个是正确 的,请把正确选项的代号写在题后的括号内,每一小题选对得 4 分,不选、 选错或选出的代号超过一个的(不论是否写在括号内)一律得 0 分. 1.下面的数中,与-3 的和为 0 的是 ………………………….( ) A.3 B.-3 C. 3 1 D. 3 1 2.下面的几何体中,主(正)视图为三角形的是( ) A. B. C. D. 3.计算 32 )2( x 的结果是( ) A. 52x B. 68x C. 62x D. 58x 得 分 评 卷人 4.下面的多项式中,能因式分解的是() A. nm 2 B. 12  mm C. nm 2 D. 122  mm 5.某企业今年 3 月份产值为 a 万元,4 月份比 3 月份减少了 10%,5 月份比 4 月份增加 了 15%,则 5 月份的产值是( ) A.( a -10%)( a +15%)万元 B. a (1-10%)(1+15%)万元 C.( a -10%+15%)万元 D. a (1-10%+15%)万元 6.化简 x x x x  11 2 的结果是( ) A. x +1 B. x -1 C.— x D. x 7.为增加绿化面积,某小区将原来正方形地砖更换为如 图所示的正八边形植草砖,更换后,图中阴影部分为植草 区域,设正八边形与其内部小正方形的边长都为 a ,则阴影部 分的 面积为( ) A.2 2a B. 3 2a C. 4 2a D.5 2a 8.给甲乙丙三人打电话,若打电话的顺序是任意的,则第一个打电话给甲的概率为 ( ) A. 6 1 B. 3 1 C. 2 1 D. 3 2 9.如图,A 点在半径为 2 的⊙O 上,过线段 OA 上的一点 P 作直线  ,与⊙O 过 A 点的 切线交于点 B,且∠APB=60°,设 OP= x ,则△PAB 的面积 y 关于 x 的函数图像大 致是( ) 10.在一张直角三角形纸片的两直角边上各取一点,分别 沿斜边中点与这两点的连线剪去两个三角形,剩下的 部分是如图所示的直角梯形,其中三边长分别为 2、 4、3,则原直角三角形纸片的斜边长是( ) A.10 B. 54 C. 10 或 54 D.10 或 172 二、填空题(本大题共 4 小题,每小题 5 分,满分 20 分) 11.2011 年安徽省棉花产量约 378000 吨,将 378000 用科学计数法表示应是 ______________. 12.甲乙丙三组各有 7 名成员,测得三组成员体重数据的平均数都是 58,方差分别为 362 甲S , 252 乙S , 162 丙S ,则数据波动最小的一组是 ___________________. 13.如图,点 A、B、C、D 在⊙O 上,O 点在∠D 的内部,四边形 OABC 为平行四边 形,则∠OAD+∠OCD=_______________°. 得 分 评 卷人 14.如图,P 是矩形 ABCD 内的任意一点,连接 PA、PB、PC、PD,得到△PAB、 △PBC、△PCD、△PDA,设它们的面积分别是 S1、S2、S3、S4,给出如下结论: ①S1+S2=S3+S4 ② S2+S4= S1+ S3 ③若 S3=2 S1,则 S4=2 S2 ④若 S1= S2,则 P 点在矩形的对角线上 其中正确的结论的序号是_________________(把所有正确结论的序号都填在横线 上). 三、(本大题共 2 小题,每小题 8 分,满分 16 分) 15.计算: )2()1)(3(  aaaa 解: 16.解方程: 1222  xxx 解: 四、(本大题共 2 小题,每小题 8 分,满分 16 分) 17.在由 m×n(m×n>1)个小正方形组成的矩形网格中,研究它的一条对角线所穿过 的小正方形个数 f, (1)当 m、n 互质(m、n 除 1 外无其他公因数)时,观察下列图形并完成下表: 1 2 3 2 1 3 4 3 2 3 5 4 猜想:当 m、n 互质时,在 m×n 的矩形网格中,一条对角线所穿过的小正方形的个数 f 与 m、n 的关系式是______________________________(不需要证明); 解: (2)当 m、n 不互质时,请画图验证你猜想的关系式是否依然成立, 解: 18.如图,在边长为 1 个单位长度的小正方形组成的网格中,给出了格点△ABC(顶点 是网格线的交点)和点 A1. (1)画出一个格点△A1B1C1,并使它与△ABC 全等且 A 与 A1 是对应点; (2)画出点 B 关于直线 AC 的对称点 D,并指出 AD 可以看作由 AB 绕 A 点经过怎样 的旋转而得到的. 2 4 7 3 5 7 解: 五、(本大题共 2 小题,每小题 10 分,满分 20 分) 19.如图,在△ABC 中,∠A=30°,∠B=45°,AC= 32 ,求 AB 的长, 解: 第 19 题 图 20.九(1)班同学为了解 2011 年某小区家庭月均用水情况,随机调查了该小区部分家 庭,并将调查数据进行如下整理, 月均用 水量 x (t) 频 数(户) 频 率 0 5x  6 0 .12 5 10x  0 .24 10 15x  1 6 0 .32 15 20x  1 0 0 .20 20 25x  4 25 30x  2 0 第 20 题 图 .04 请解答以下问题: (1)把上面的频数分布表和频数分布直方图补充完整; (2)若该小区用水量不超过 15t 的家庭占被调查家庭总数的百分比; 解: (3)若该小区有 1000 户家庭,根据调查数据估计,该小区月均用水量超过 20t 的家 庭大约有多少户? 解: 六、(本题满分 12 分) 21.甲、乙两家商场进行促销活动,甲商场采用“慢 200 减 100”的促销方式,即购买商 品的总金额满 200 元但不足 400 元,少付 100 元;满 400 元但不足 600 元,少付 200 元;……,乙商场按顾客购买商品的总金额打 6 折促销。 (1)若顾客在甲商场购买了 510 元的商品,付款时应付多少钱? 解: (2)若顾客在甲商场购买商品的总金额为 x(400≤x<600)元,优惠后得到商家的 优惠率为 p(p= 购买商品的总金额 优惠金额 ),写出 p 与 x 之间的函数关系式,并说明 p 随 x 的变化情况; 解: (3)品牌、质量、规格等都相同的某种商品,在甲乙两商场的标价都是 x(200≤x< 400)元,你认为选择哪家商场购买商品花钱较少?请说明理由。 解: 七、(本题满分 12 分) 22.如图 1,在△ABC 中,D、E、F 分别为三边的中点,G 点在边 AB 上,△BDG 与四 边形 ACDG 的周长相等,设 BC=a、AC=b、AB=c. (1)求线段 BG 的长; 解: (2)求证:DG 平分∠EDF; 证: (3)连接 CG,如图 2,若△BDG 与△DFG 相似,求证:BG⊥CG. 证: 八、(本题满分 14 分) 23.如图,排球运动员站在点 O 处练习发球,将球从 O 点正上方 2m 的 A 处发出,把球 看成点,其运行的高度 y(m)与运行的水平距离 x(m)满足关系式 y=a(x-6)2+h.已 知球网与 O 点的水平距离为 9m,高度为 2.43m,球场的边界距 O 点的水平距离为 18m。 (1)当 h=2.6 时,求 y 与 x 的关系式(不要求写出自变量 x 的取值范围) (2)当 h=2.6 时,球能否越过球网?球会不会出界?请说明理由; (3)若球一定能越过球网,又不出边界,求 h 的取值范围。 第 23 题 图 参考答案 1、A 2、C 3、B 4、D 5、B 6、D 7、A 8、B 9、D 10、C 填空题: 11、3.78*105 12、丙 13、60° 14、②和④ (2) f=m+n-1 (3) 120 户 22、(1)BG= 1 ( )2 b c (2)(3)略 当 y=0 时, 21 ( 6) 2.6 060 x    ,解得: 1 6 2 39 18x    , 2 6 2 39x   (舍去)故会出界 (3) 8 3h  资料由谢老师收集: 了解初中,高中考试信息,做题技巧,解题思路可去谢 老师博客 http://blog.sina.com.cn/xiejunchao1 更多精彩资料请关注谢老师博客 http://blog.sina.com.cn/xiejunchao1 2012 年河南初中学业水平暨高级中等学校招生考试试题 数 学 注意事项: 1. 本试卷共 8 页,三大题,满分 120 分,考试时间 100 分钟请用蓝、黑色钢 笔或圆珠笔直接答在试卷上. 2. 答卷前将密封线内的项目填写清楚. 参考公式:二次函数 2 ( 0)y ax bx c a    图象的顶点坐标为 24( , )2 4 b ac b a a  一、选择题(每小题 3 分,共 24 分) 下列各小题均有四个答案,其中只有一个是正确的,将正确答案的代号字 母填入题后括号内。 1.下列各数中,最小的数是( ) A. -2 B. -0.1 C. 0 D. |-1| 2.如下是一种电子计分牌呈现的数字图形,其中既是轴对称图形又是中心 对称图形的是( ) 3.一种花瓣的花粉颗粒直径约为 0.0000065 米,0.0000065 用科学计数法 表示为( ) A. 6.5×10-5 B. 6.5×10-6 C. 6.5×10-7 D.65×10-6 4.某校九年级 8 位同学一分钟跳绳的次数排序后如下:150,164,168, 168,172,176,183,185.则由这组数据得到的结论中错误的是( ) A. 中位数 B. 众数为 168 C. 极差为 35 D. 平均数为 170 C DBA 5.在平面直角坐标系中,将抛物线 42  xy 先向右平移 2 个单位,再向上 平移 2 个单位,得到的抛物线的解析式是( ) A. 2)2( 2  xy B. 2)2( 2  xy C. 2)2( 2  xy D. 2)2( 2  xy 6.如图所示的几何体的左视图是( ) 7.如图,函数 xy 2 和 4 axy 的图像相交于点 A(m,3),则不等式 2x <ax+4 的解集为( ) A. x< 2 3 B. x<3 C. x> 2 3 D. x>3 8.如图,已知 AB 是⊙O 的直径,且⊙O 于点 A, EC = CB .则下列结论中 不一定正确的是( ) A. BA⊥DA B. OC//AE C. ∠COE=2∠ECA D. OD⊥AC 二、填空题(每小题 3 分,共 21 分) xO y A 第 7 题 A B C D正面 E O C D BA 第 8 题 9.计算:  20 )3()2( _______. 10.如图,在△ABC 中,∠C=90°,∠CAB=50°.按以下步骤作图:①以点 A 为圆心,小于 AC 的长为半径画弧,分别交 AB、AC 于点 E、F; ②分别以点 E、F 为圆心,大于 EF2 1 为半径画弧, 两弧相交于点 G;③作射线 AG 交 BC 边于点 D, 则∠ADC 的度数为_______。 11.母线长为 3,底面圆的直径为 2 的圆锥的侧面积为___________. 12.一个不透明的袋子中装有三个小球,它们除分别标有的数字 1、3、5 不同外,其它完全相同。任意从袋子中摸出一球后放回,再任意摸出一球,则 两次摸出的球所标数字之和为 6 的概率为____________。 13.如图,点 A、B 在反比例函数 )0,0(  xkx ky 的图象上,过点 A、B 作 x 轴的垂线,垂足分别为 M、N,延长线段 AB 交 x 轴于点 C,若 OM=MN=NC,△AOC 的面积为 6,则 k 的值为________。 14.如图,在 Rt△ABC 中,∠C=90°,AC=6,BC=8.把△ABC 绕 AB 边上 的点 D 顺时针旋转 90°得到△A′B′C′,A′C′交 AB 于点 E。若 AD=BE,则△A′DE 的面积是_________. 15.如图,在 Rt△ABC 中,∠ACB=90°,∠B=30°,BC=3,点 D 是 BC 边 上一动点(不与点 B、C 重合),过点 D 作 DE⊥BC 交 AB 边于点 E,将∠B 沿 直线 DE 翻折,点 B 落在射线 BC 上的点 F 处,当△AEF 为直角三角形时,BD 的长为__________. E F C D B G A 第 10 题 xCO M B N y A 第 13 题 E C D B A 第 14 题 A′ B′ C′ E F CDB A 第 15 题 三、解答题(本大题 8 个小题,满分 75 分) 16.(8 分)先化简 )4( 2 44 2 2 xx xx xx    ,然后从 55  x 的范围内选取 一个合适的整数作为 x 的值代入求值。 17.(9 分)5 月 31 日是世界无烟日,某市卫生机构为了了解“导致吸烟人 口比例高的主要原因”,随机抽样调查了该市部分 18 ∼ 65 岁的市民,下图是根 据调查结果绘制的统计图,根据图中信息解答下列问题: m 420 210 240 m 人数 项目 图 1 政 府 对 公 共 场 所 吸 烟 的 监 管 力 度 不 够 对 吸 烟 危 害 健 康 认 识 不 足 人 们 对 吸 烟 的 容 忍 度 大 烟 民 戒 烟 的 毅 力 弱 其 它 政府对公共 场所吸烟的监 管力度不够 28% 图 2 对吸烟危害健 康认识不足 21% 人们对吸 烟的容忍度大 21% 烟民戒烟 的毅力弱 其它 16% (1)这次接受随机抽样调查的市民总人数为_________; (2)图 1 中 m 的值是___________; (3)求图 2 中认为“烟民戒烟的毅力弱”所对应的圆心角的度数; (4)若该市 18 ∼ 65 岁的市民约有 200 万人,请你估算其中认为导致吸烟 人口比例高的最主要原因是“对吸烟危害健康认识不足”的人数。 18.(9 分)如图,在菱形 ABCD 中,AB=2,∠DAB=60°,点 E 是 AD 边 的中点,点 M 是 AB 边上一动点(不与点 A 重合),延长 ME 交射线 CD 于点 N,连接 MD、AN。 (1)求证:四边形 AMDN 是平行四边形; (2)填空:①当 AM 的值为_____时,四边形 AMDN 是矩形; ②当 AM 的值为_______时,四边形 AMDN 是菱形。 19.(9 分)甲、乙两人同时从相距 90 千米的 A 地前往 B 地,甲乘汽车, 乙骑摩托车,甲到达 B 地停留半小时后返回 A 地,如图是他们离 A 地的距离 y(千米)与时间 x(时)之间的函数关系式。 (1)求甲从 B 地返回 A 地的过程中,y 与 x 之间的函数关系式,并写出自 变量 x 的取值范围; (2)若乙出发后 2 小时和甲相遇,求乙从 A 地到 B 地用了多长时间? E CD M B N A 第 18 题 第 19 题 O 3 90 y(千米) x(时)1.51 20.(9 分)某宾馆为庆祝开业,在楼前悬挂了许多宣传条幅。如图所示, 一条幅从楼顶 A 处放下,在楼前点 C 处拉直固定。小明为了测量此条幅的长 度,他先测得楼顶 A 点的仰角为 45°,已知点 C 到大厦的距离 BC=7 米, ∠ABD=90°.请根据以上数据求条幅的长度(结果保留整数。参考数据: tan31°≈0.60,sin31°≈0.52,cos31°≈0.86). 21.(10 分)某中学计划购买 A 型和 B 型课桌共 200 套,经招标,购买一套 A 型课桌比购买一套 B 型课桌少用 40 元,且购买 4 套 A 型和 5 套 B 型课桌共 需 1820 元。 (1)求购买一套 A 型和一套 B 型课桌登各需多少元? (2)学校根据实际情况,要求购买这这两种课桌登总费用不能超过 40880 元,并且购买 A 型课桌的数量不能超过 B 型课桌登数量的 3 2 ,求该校本次购买 A 型和 B 型课桌登共有几种方案?哪种方案的总费用最低? E CD B A 第 20 题 22.(10 分)类比转化、从特殊到一般等思想方法,在数学学习和研究中 经常用到,如下是一个案例,请补充完整。 原题:如图 1,在□ABCD 中,点 E 是 BC 边的中点,点 F 是线段 AE 上一点,BF 的延长线交射线 CD 于点 G。若 3 EF AF ,求 CG CD 的值。 (1)尝试探究 在图 1 中,过点 E 作 EH//AB 交 BG 于点 H,则 AB 和 EH 的数量关系是 _____________,CG 和 EH 的数量关系是______________, CG CD 的值是 __________. (2)类比延伸 如图 2,在原题的条件下,若 )0(  mmEF AF ,则 CG CD 的值是_____________ (用含 m 的代数式表示),试写出解答过程。 E F C D B G A 图 1 E F C D B G A 图 2 (3)拓展迁移 如图 3,梯形 ABCD 中,DC//AB,点 E 是 BC 的延长线上一点,AE 和 BD 相 交于点 F。若 aCD AB  , )0,0(  babBE BC ,则 EF AF 的值是__________(用含 a,b 的代数式表示)。 23.(11 分)如图,在平面直角坐标系中,直线 12 1  xy 与抛物线 32  bxaxy 交于 A、B 两点,点 A 在 x 轴上,点 B 的纵坐标为 3。点 P 是直 线 AB 下方的抛物线上一动点(不与点 A、B 重合),过点 P 作 x 轴的垂线交直 线 AB 于点 C,作 PD⊥AB 于点 C,作 PD⊥AB 于点 D。 (1)求 a、b 及 sin∠ACP 的值; (2)设点 P 的横坐标为 m. ① 用含 m 的代数式表示线段 PD 的长,并求出线段 PD 长的最大值; ②连接 PB,线段 PC 把△PDB 分成两个三角形,是否存在合适的 m 值, 使这两个三角形的面积之比为 9:10?若存在,直接写 m 的值;若不存在,说明 理由。 E F CD BA 图 3 第 23 题 x y A BC D P O 黄冈市2012年初中毕业生学业考试 数学试题 (满分:120 分考试时间:120 分钟) 一、选择题(下列各题A、B、C、D 四个选项中,有且仅有一个是正确的,每小题3 分,共24 分) 1.下列实数中是无理数的是 2.2012 年5 月25 日有700 多位来自全国各地的知名企业家聚首湖北共签约项目投资 总额为909 260 000 000 元,将909 260 000000 用科学记数法表示(保留3 个有效数 字),正确的是 A.909×1010 B.9.09×1011 C.9.09×1010 D.9.0926×1011 3.下列运算正确的是 4. 如图,水平放置的圆柱体的三视图是 5. 若顺次连接四边形ABCD 各边的中点所得四边形是矩形,则四边形ABCD 一定是 A. 矩形B. 菱形 C. 对角线互相垂直的四边形D. 对角线相等的四边形 6.如图,AB 为⊙O 的直径,弦CD⊥AB 于E,已知CD=12,则⊙O 的直径为 A. 8 B. 10 C.16 D.20 7.下列说法中 ①若式子 有意义,则x>1. ②已知∠α=27°,则∠α的补角是153°. ③已知x=2 是方程x2-6x+c=0 的一个实数根,则c 的值为8. ④在反比例函数 中,若x>0 时,y 随x 的增大而增大,则k 的取值范围是k> 2. 其中正确命题有 A. 1 个B. 2 个C. 3 个D. 4 个 8. 如图,在Rt △ ABC 中,∠C=90° ,AC=BC= 6cm,点P 从点A 出发,沿AB 方向以每秒 cm 的速度向终点B 运动;同时,动点Q 从点B 出发 沿BC 方向以每秒1cm 的速度向终点C 运动,将 △PQC 沿BC 翻折,点P 的对应点为点P′.设Q 点运动的时间t 秒,若四边形QPCP′为菱形,则 t 的值为 二、填空题(共8 小题,每小题3 分,共24 分) 9.- 的倒数是__________. 10.分解因式x3-9x=__________. 11.化简 的结果是 . 12.如图,在△ ABC 中,AB=AC,∠A=36° ,AB 的垂直平分线交AC 于点E,垂足为点D,连接 BE,则∠EBC 的度数为________. 13.已知实数x 满足x+ =3,则x2+ 的值为_________. 14.如图,在梯形ABCD 中,AD∥BC ,AD=4,AB= CD=5,∠B=60°,则下底BC 的长为________. 15.在平面直角坐标系中,△ABC 的三个顶点的坐 标分别是A(-2,3),B(-4,-1),C(2,0),将△ABC 平移至△A1B1C1 的位置,点A、B、C 的对应点分别 是A1B1C1,若点A1 的坐标为(3,1).则点C1 的坐标 为__________. 16.某物流公司的快递车和货车同时从甲 地出发,以各自的速度匀速向乙地行驶,快 递车到达乙地后卸完物品再另装货物共用 45 分钟,立即按原路以另一速度匀速返回, 直至与货车相遇.已知货车的速度为60 千 米/ 时,两车之间的距离y(千米)与货车行 驶时间x(小时)之间的函数图象如图所示, 现有以下4 个结论: ①快递车从甲地到乙地的速度为100 千米/时; ②甲、乙两地之间的距离为120 千米; ③图中点B 的坐标为(3 ,75); ④快递车从乙地返回时的速度为90 千米/时. 以上4 个结论中正确的是____________(填序号) 三、解答题(共9 小题,共72 分) 17.(5分)解不等式组 18.(7分)如图,在正方形ABCD 中,对角线 AC、BD 相交于点O,E、F 分别在OD、OC 上, 且DE=CF,连接DF、AE,AE 的延长线交DF 于点M. 求证:AM⊥DF. 19.(6分)在一个口袋中有4个完全相同的小球,把它们分别标 号l、2、3、4.小明先随机地摸出一个小球,小强再随机地摸出一 个小球.记小明摸出球的标号为x,小强摸出的球标号为y.小明和 小强在此基础上共同协商一个游戏规则:当x>y 时小明获胜,否 则小强获胜. ①若小明摸出的球不放回,求小明获胜的概率. ②若小明摸出的球放回后小强再随机摸球,问他们制定的游戏 规则公平吗?请说明理由. 20.(6 分)为了全面了解学生的学习、生活及家庭的基本情况,加 强学校、家庭的联系,梅灿中学积极组织全体教师开展“课外访 万家活动”,王老师对所在班级的全体学生进行实地家访,了解 到每名学生家庭的相关信息,现从中随机抽取15 名学生家庭的 年收入情况,数据如下表: (1)求这15 名学生家庭年收入的平均数、中位数、众数. (2)你认为用(1)中的哪个数据来代表这15 名学生家庭年收入的 一般水平较为合适?请简要说明理由. 21.(6 分)某服装厂设计了一款新式夏装,想尽快制作8800 件投入 市场,服装厂有A、B 两个制衣车间,A 车间每天加工的数量是B 车间的1.2 倍,A、B 两车间共同完成一半后,A 车间出现故障停 产,剩下全部由B 车间单独完成,结果前后共用20 天完成,求A、 B 两车间每天分别能加工多少件. 22.(8 分)如图,在△ABC 中,BA=BC,以AB 为直径作半圆⊙O, 交AC 于点D.连结DB,过点D 作DE⊥BC, 垂足为点E. (1)求证:DE 为⊙O 的切线; (2)求证:DB2=AB·BE. 23.(8 分)新星小学门口有一直线马路,为方便学生过马路,交警在门口设有一定宽 度的斑马线,斑马线的宽度为4 米,为安全 起见,规定车头距斑马线后端的水平距离 不得低于2 米,现有一旅游车在路口遇红 灯刹车停下,汽车里司机与斑马线前后两端的视角分别为 ∠FAE=15° 和∠FAD=30° .司机距车头的水平距离为0.8 米, 试问该旅游车停车是否符合上述安全标准?(E、D、C、B 四点在平 行于斑马线的同一直线上.) (参考数据:tan15°=2- ,sin15°= cos15°= ≈1.732, ≈1.414) 24.(12 分)某科技开发公司研制出一种新型产品,每件产品的成 本为2400 元,销售单价定为3000 元.在该产品的试销期间,为 了促销,鼓励商家购买该新型产品,公司决定商家一次购买这种 新型产品不超过10 件时,每件按3000 元销售;若一次购买该种 产品超过10 件时,每多购买一件,所购买的全部产品的销售单价 均降低10 元,但销售单价均不低于2600 元. (1)商家一次购买这种产品多少件时,销售单价恰好为2600 元? (2)设商家一次购买这种产品x 件,开发公司所获的利润为y 元, 求y(元)与x(件)之间的函数关系式,并写出自变量x 的取值范围. (3)该公司的销售人员发现:当商家一次购买产品的件数超过某 一数量时,会出现随着一次购买的数量的增多,公司所获的利润 反而减少这一情况.为使商家一次购买的数量越多,公司所获的 利润越大,公司应将最低销售单价调整为多少元?(其它销售条件 不变) 25.(14 分)如图,已知抛物线的方程C1: y=- (x+2)(x-m)(m>0)与x 轴相交于 点B、C,与y 轴相交于点E,且点B 在点 C 的左侧. (1)若抛物线C1过点M(2,2),求实数m 的 值. (2)在(1)的条件下,求△BCE 的面积. (3)在(1)的条件下,在抛物线的对称轴上 找一点H,使BH+EH 最小,并求出点H 的坐标. (4)在第四象限内,抛物线C1上是否存在点F,使得以点B、C、F 为 顶点的三角形与△BCE 相似?若存在,求m 的值;若不存在,请说 明理由. 试卷类型:A 2012 年潍坊市初中学业水平考试 数 学 试 题 第 1 卷 (选择题 共 36 分) 一、选择题(本题共 12 个小题,在每个小题给出的四个选项中,只有一个是正确的, 请把正确的选项选出来。每小题选对得 3 分,选错、不选或选出的答案超出一个均记 0 分) 1.计算:2-2=( ). A. 4 1 B. 2 C. 4 1 D.4 2.如果代数式 3 4 x 有意义,则 x 的取值范围是( ).[来源:学&科&网] A.x≠3 B.x<3 C.x>3 D.x≥3 3.某班 6 名同学参加体能测试的成绩如下(单位:分):75,95,75,75,80,80.关 于这组数据的表述错误的是( ). A.众数是 75 B.中位数是 75 C.平均数是 80 D.极差是 20 4.右图空心圆柱体的主视图的画法正确的是( ). 5.不等式组 532 423   x x 的解等于( ). A. 11 C. x<2 D. x<1 或 x>2 6.许多人由于粗心,经常造成水龙头“滴水”或“流水”不断.根据测定,一般情况 下一个水龙头“滴水”1 个小时可以流掉 3.5 千克水.若 1 年按 365 天计算,这个水 龙头 1 年可以流掉( )千克水.(用科学计数法表示,保留 3 个有效数字) A.3.1×104 B.0.31×105 C.3.06×104 D.3.07×104 7.已知两圆半径 r1、r2 分别是方程菇 x2—7x+10=0 的两根,两圆的圆心距为 7,则两 圆 的位置关系是( ). A.相交 B.内切 C.外切 D.外离 8.已知矩形 ABCD 中,AB=1,在 BC 上取一点 E,沿 AE 将ΔABE 向上折叠,使 B 点落在 AD 上的 F 点,若四边形 EFDC 与矩形 ABCD 相似,则 AD=( ). A. 2 15  B. 2 15  C . 3 D.2 9.轮船从 B 处以每小时 50 海里的速度沿南偏东 300 方向匀速航行,在 B 处观测灯塔 A 位于南偏东 750 方向上,轮船航行半小时到达 C 处,在 C 处观测灯塔 A 位于北偏东 600 方向上,则 C 处与灯塔 A 的距离是 ( )海里. A. 325 B. 225 C.50 D.25 10.甲乙两位同学用围棋子做游戏.如图所示,现轮到黑棋下子,黑棋下一子后白棋 再下一子,使黑棋的 5 个棋子组成轴对称图形,白棋的 5 个棋子也成轴对称图形.则 下列下子方法不正确的是( ).[说明:棋子的位置用数对表示,如 A 点在(6,3)] A.黑(3,7);白(5,3) B.黑(4,7);白(6,2) C.黑(2,7);白(5,3) D.黑(3,7);白(2,6) 11.若直线 y=-2x-4 与直线 y=4x+b 的交点在第三象限,则 b 的取值范围是( ). A. -48 D.-4≤6≤8 12.下图是某月的日历表,在此日历表上可以用一个矩形圈出 3×3 个位置相邻的 9 个数(如 6,7,8,l3,14,l5,20,21,22).若圈出的 9 个数中,最大数与最小数的 积为 192,则这 9 个数的和为( ). A.32 B.126 C.135 D.144 第Ⅱ卷 (非选择题 共 84 分) 二、填空题(本大题共 5 个小题,共 15 分,只要求填写最后结果,每小题填对得 3 分) 13.分解因式:x3—4x2—12x= . 14.点 P 在反比例函数 x ky  (k≠0)的图象上,点 Q(2,4)与点 P 关于 y 轴对称,则 反比例函数的解析式为 . 15.方程 060 3 66  xx 的根是 . 16.如图所示,AB=DB,∠ABD=∠CBE,请你添加一个适当的条件 , 使ΔABC≌Δ DBE. (只需添加一个即可) 17.右图中每一个小方格的面积为 l,则可根据面积计算得到如下算式: 1+3+5+7+…+(2n-1)= .(用 n 表示,n 是正整数) 三、解答题(本大题共 7 个小题,共 69 分。解答要写必要的文字说明、证明过程或演 算步骤.) 18.(本题满分 9 分) 如图,三角形 ABC 的两个顶点 B、C 在圆上,顶点 A 在圆外,AB、AC 分别交圆于 E、D 两点,连结 EC、BD. (1)求证:ΔABD∽ΔACE; (2)若ΔBEC 与ΔBDC 的面积相等,试判定三角形 ABC 的形状. 19.(本题满分 9 分)为了援助失学儿童,初三学生李明从 2012 年 1 月份开始,每月一 次将相等数额的零用钱存入已有部分存款的储蓄盒内,准备每 6 个月一次将储蓄盒内 存款一并汇出(汇款手续费不计).已知 2 月份存款后清点储蓄盒内有存款 80 元,5 月 份存款后清点储蓄盒内有存款 125 元. (1)在李明 2012 年 1 月份存款前,储蓄盒内已有存款多少元? (2)为了实现到 2015 年 6 月份存款后存款总数超过 1000 元的目标,李明计划从 2013 年 1 月份开始,每月存款都比 2012 年每月存款多 t 元(t 为整数),求 t 的最小 值. [来源: 学&科&网] 20.(本题满分 l0 分)校车安全是近几年社会关注的重大问题,安全隐患主要是超速和 超载.某中学数学活动小组设计了如下检测公路上行驶的汽车速度的实验:先在公路 旁边选取一点 C,再在笔直的车道l 上确定点 D,使 CD 与l 垂直,测得 CD 的长等于 21 米,在l 上点 D 的同侧取点 A、B,使∠CAD=300,∠CBD=600. (1)求 AB 的长(精确到 0.1 米,参考数据: 41.12,73.13  ); (2)已知本路段对校车限速为 40 千米/小时,若测得某辆校车从 A 到 B 用时 2 秒,这辆校车是否超速?说明理由. 21.(本题满分 l0 分)田忌赛马的故事为我们所熟知.小亮与小齐学习概率初步知识后 设计了如下游戏:小亮手中有方块 l0、8、6 三张扑克牌,小齐手中有方块 9、7、5 三 张扑克牌.每人从各自手中取一张牌进行比较,数字大的为本“局”获胜,每次取的 牌不能放回. (1)若每人随机取手中的一张牌进行比赛,求小齐本“局”获胜的概率;[来源:学.科.网 Z.X.X.K] (2)若比赛采用三局两胜制,即胜 2 局或 3 局者为本次比赛获胜者.当小亮的三张 牌出牌顺序为先出 6,再出 8,最后出 l0 时,小齐随机出牌应对,求小齐本次比赛获 胜的概率. 22.(本题满分 l0 分)如图,已知平行四边形 ABCD,过 A 作 AM⊥BC 于 M,交 BD 于 E, 过 C 作 CN⊥AD 于 N,交 BD 于 F,连结 AF、CE. (1)求证:四边形 AECF 为平行四边形; (2)当 AECF 为菱形,M 点为 BC 的中点时,求 AB:AE 的值. 23.(本题满分 l0 分)许多家庭以燃气作为烧水做饭的燃料,节约用气是我们日常生活 中非常现实的问题.某款燃气灶旋钮位置从 0 度到 90 度(如图),燃气关闭时,燃气灶 旋钮的位置为 0 度,旋钮角度越大,燃气流量越大,燃气开到最大时,旋钮角度为 90 度.为测试燃气灶旋钮在不同位置上的燃气用量,在相同条件下,选择在燃气灶旋钮 的 5 个不同位置上分别烧开一壶水(当旋钮角度太小时,其火力不能够将水烧开,故选 择旋钮角度 x 度的范围是 18≤x≤90),记录相关数据得到下表: 旋钮角度(度) 20 50 70 80 90 所用燃气量(升) 73 67 83 97 115 [来源:Zxxk.Com] (1)请你从所学习过的一次函数、反比例函数和二次函数中确定哪种函数能表示所 用燃气量 y 升与旋钮角度 x 度的变化规律?说明确定是这种函数而不是其它函数的理 由,并求出它的解析式; (2)当旋钮角度为多少时,烧开一壶水所用燃气量最少?最少是多少? (3)某家庭使用此款燃气灶,以前习惯把燃气开到最大,现采用最节省燃气的旋钮 角度,每月平均能节约燃气 10 立方米,求该家庭以前每月的平均燃气用量. 24.(本题满分 11 分)如图,已知抛物线与坐标轴分别交于 A(-2,O)、B(2,0)、 C(0,-l)三点,过坐标原点 0 的直线 y=kx 与抛物线交于 M、N 两点.分别过点 C,D(0,-2)作平行于 x 轴的直线 21 ll 、 . (1)求抛物线对应二次函数的解析式; (2)求证以 ON 为直径的圆与直线 1l 相切; (3)求线段 MN 的长(用 k 表示),并证明 M、N 两点到直线 2l 的距离之和等于线段 MN 的长. 数学试题(A)参考答案及评分标准 一、选择题(本题共 l2 小题,共 36 分.在每小题给出的四个选项中,只有一个是正确 的,请把正确的选项选出来,每小题选对得 3 分,选错、不选或选出的答案超过一 个,均记 0 分.) 题号[来源:学科网ZXXK] l 2 3 4 5 6 7 8 9 10 11 12 答案 A C B C A D C B D C A D 二、填空题(本题共 5 小题,共 l5 分.只要求填写最后结果,每小题填对得 3 分.) 13.X(x+2)(x-6) 14. xy 8 l5.x=30 16.∠BDE=∠BAC 或 BE=BC 或∠ACB=∠DEB 等(写出一个即可.) 17.n2 三.解答题:(本大题共 7 小题,共 69 分解答应写出文字说明、证明过程或演算步 骤.) 18。(本题满分 9 分) (学科网.精校) (1)证明:因为弧 ED 所对的圆周角相等,所以∠EBD=∠ECD,…………………2 分 又因为∠A=∠A,所以△ABD∽ΔACE.……………………………………………4 分 (2)法 1:因为 S△BEC=S△BCD, S△ACE=S△ABC-S△BEC,S△ABD=S△ABC 一 S△BCD,………………………………6 分 所以 S△ACE=S△ABD, 又由(1)知△ABD∽ΔACE, 所以对应边之比等于 1,……………………………………………………………8 分 所以 AB=AC,即三角形 ABC 为等腰三角形.……………………………………9 分 法 2:因为ΔBEC 与ΔBCD 的面积相等,有公共底边 BC,所以高相等, 即 E、D 两点到 BC 距离相等,所以 ED‖BC,……………………………………6 分 所以∠BCE=∠CED, 又因为∠CED=∠CBD, 所以∠BCE=∠CBD,………………………………………………………………8分 由(1)知△ABD∽ΔACE, 所以∠ABD=∠ACE, 所以∠ABC=∠ACB, 即三角形 ABC 为等腰三角形. ……………………………………………………9 分 19.(本题满分 9 分) 解:(1)设李明每月存款 x 元,储蓄盒内原有存款 y 元,依题意得,  802 1255   yx yx ....................................................................................2 分 解得 15 50   x y , 所以,储蓄盒内原有存款 50 元。 …………………………………………………4 分 (2)由(1)得,李明 2012 年共有存款 12×15+50=230(元), …………………5 分 2013 年 1 月份后每月存人 l5+t(元), 2013 年 1 月到 2015 年 6 月共有 30 个月,…………………………………………6 分 依题意得,230+30(15+t)>1000,………………………………………………8 分 解得 3 210t ,所以 t 的最小值为 11. ……………………………………………9 分 20.(本题满分 l0 分) 解:(1)由题意得,在 RtΔADC 中, 33.36321 3 3 21 30tan 0  CDAD ……………………………..2 分 RtΔBDC 中, 11.1237 3 21 60tan 0  CDBD ………………….4 分 所以 AB=AD-BD=36.33-12.1l=24.22≈24.2(米). ………………………6 分. (2)汽车从 A 到 B 用时 2 秒,所以速度为 24.2÷2=12.1(米/秒), 因为 l2.1×3600=43560, 所以该车速度为 43.56 千米/小时,………………………………………………9 分 大于 40 千米/小时, 所以此校车在 AB 路段超速.……………………………………………………l0 分 21.(本题满分 l0 分) 解:(1)每人随机取一张牌共有 9 种情况: [或(10,9),(10,7),(10,5),(8,9),(8,7),(8,5),(6,9),(6,7),(6, 5)] 小齐获胜的情况有(8,9),(6,9)(6,7)共 3 种,…………………………………4 分 所以小齐获胜的概率为 3 1 9 3 1 P ………………………………………………5 分 (2)据题意,小明出牌顺序为 6、8、10 时, 小齐随机出牌的情况有 6 种情况: (9,7,5),(9,5,7),(7,9,5),(7,5,9),(5,9,7),(5,7, 9) ……………7 分 小齐获胜的情况只有(7,9,5)一种,………………………………………………9 分 所以小齐获胜的概率为 6 1 2 P …………………………………………………l0 分 22.(本题满分 l0 分) (1)证明:因为 AE⊥BC,所以∠AMB=900, 因为 CN⊥AD,所以∠CNA=900. 又因为 BC‖AD,所以∠BCN=900. 所以 AE∥CF,………………………………………………………………………2 分 又由平行得∠ADE=∠CBD,又 AD=BC, 所以ΔADE≌△BCF, 所以 AE=CF, 所以四边形 AECF 为平行四边形.…………………………………………………4 分 (2)当 AECF 为菱形时,连结 AC 交 BF 于点 0, 则 AC 与 EF 互相垂直平分,(学.科网精校) 又 BO=OD, 所以 AC 与 BD 互相垂直平分, 所以,四边形 ABCD 是菱形,……………………6 分 所以 AB=BC. 因为 M 是 BC 的中点,AM⊥BC, 所以,ΔABM≌ΔCAM, 所以,AB=AC, ΔABC 为等边三角形, ∠ABC=600,∠CBD=300. ………………………………………………………8 分 在 RtΔBCF 中, CF:BC=tan∠CBF= 3 3 , 又 AE=CF,AB=BC, 所以 AB:AE= 3 …………………………………………………………………l0 分 23.(本题满分 l0 分) 解.(1)若设 y=kx+b(k≠0),由  bk bk   2073 5067 ,解得      5 1 77 k b 所以 775 1  xy ,把 x=70 代人得 y=65≠83,所以不符合;………………l 分 若设 )0(  kx ky , 2073 k ,解得 k=1460, 所以 xy 1460 ,把 x=50 代入得 y=29.2≠67,所以不符合;………………2 分 若设 y=ax2+bx+c,则由       cba cba cba 2040073 70490083 50250067 ,解得       50 1 975 8 a c b 所以 975 8 50 1 2  xxy (18≤x≤90).……………………………………………4 分 把 x=80 代入得 y=97,把 x=90 代入得 y=115,符合题意. 所以选用二次函数能表示所用燃气量 y 升与旋钮角度 x 度的变化规律.………5 分 (2)由(1)得 65)40(50 1975 8 50 1 22  xxxy ,……………………………6 分 所以当 x=40 时,y 取得最小值 65. 即当旋钮角度为 40 度时,烧开一壶水所用燃气量最少,最少为 65 升.…………8 分 (3)由(2)及表格知,采用最节省燃气的旋钮角度 40 度比把燃气开到最大时烧开一 壶水节约用气 ll5—65=50(升),(学科.网精校) 设该家庭以前每月平均用气量为 a 立方米,则由题意得 10115 50 a , 解得 a=23(立方米), 即该家庭以前每月平均用气量为 23 立方米……………………………………10 分 24.(本题满分 ll 分) 解:(1)设抛物线对应二次函数的解析式为 y=ax2+bx+c, 由       cba c cba 240 1 240 解得       4 1 1 0 a c b 所以 14 1 2  xy .………………………………………………………3 分 (2)设 M(x1,y1),N(x2,y2),因为点 M、N 在抛物线上, 所以 ,14 1,14 1 2 22 2 11  xyxy ,所以 x2 2=4(y2+1); 又 ON2=x2 2+y2 2=4(y2+1)+y2 2=(y2+2)2,所以 ON= 22 y ,又因为 y2≥-l, 所以 0N=2+y2.………………………………………5 分 设 ON 的中点 E,分别过点 N、E 向直线 1l 作垂线,垂足为 P、F, 则 2 2 2 2yNPOCEF  , 所以 ON=2EF, 即 ON 的中点到直线 1l ,的距离等于 0N 长度的一半, 所以以 ON 为直径的圆与 1l 相切.………………………………………7 分 (3)过点 M 作 MH⊥NP 交 NP 于点 H,则 MN2=MH2+NH2=(x2-x1)2+(y2-y1), 又 y1=kx1,y2=kx2,所以(y2-y1)2=k2(x2-x1)2 所以 MN2=(1+k2)(x2 一 xl)2; 又因为点 M、N 既在 y=kx 的图象上又在抛物线上, 所以 14 1 2  xkx ,即 x2-4kx-4=0, 所以 2 2 1222 16164 kkkkx  , 所以(x2-x1)2=16(1+k2), 所以 MN2=16(1+k2)2,∴MN=4(1+k2)…9 分 延长 NP 交 2l 于点 Q,过点 M 作 MS⊥ 2l 交 2l 于点 S, 则 MS+NQ=y1+2+y2+2= 2)(4 1414 114 1 2 2 2 1 2 2 2 1  xxxx 又 x1 2+x2 2=2[4k2+4(1+k2)]=16k2+8, 所以 MS+NQ=4k2+2+2=4(1+k2)=MN 即 M、N 两点到 2l 距离之和等于线段 MN 的长(学科网精 校).……………………ll 分 说明:本参考答案给出了一种解题方法,其它正确方法应参考本标准给出相应分 数. 2012 年内蒙古呼和浩特市中考数学试卷 一、选择题(本大题共 10 个小题,每小题 3 分,共 30 分.在每小题给出的四个选项中, 只有一项是符合题目要求的) 1.﹣2 的倒数是( ) A . 2 B . ﹣2 C . D . 2.如图,已知 a∥b,∠1=65°,则∠2 的度数为( ) A . 65° B . 125° C . 115° D . 25° 3.在一个不透明的口袋中,装有 3 个红球,2 个白球,除颜色不同外,其余都相同,则随 机从口袋中摸出一个球为红色的概率是( ) A . B . C . D . 4.下列各因式分解正确的是( ) A . ﹣x2+(﹣2)2= (x﹣2)(x+2) B . x2+2x﹣1=(x﹣ 1)2 C . 4x2﹣4x+1=(2x﹣ 1)2 D . x2﹣4x=x(x+2) (x﹣2) 5.已知:x1,x2 是一元二次方程 x2+2ax+b=0 的两根,且 x1+x2=3,x1x2=1,则 a、b 的值 分别是( ) A . a=﹣3,b=1 B . a=3,b=1 C . ,b=﹣1 D . ,b=1 6.如图,在一长方形内有对角线长分别为 2 和 3 的菱形,边长为 1 的正六边形和半径为 1 的圆,则一点随机落在这三个图形内的概率较大的是( ) A . 落在菱形内 B . 落在圆内 C . 落在正六边形内 D . 一样大 7.下面四条直线,其中直线上每个点的坐标都是二元一次方程 x﹣2y=2 的解是( ) A . B . C . D . 8.已知:在等腰梯形 ABCD 中,AD∥BC,AC⊥BD,AD=3,BC=7,则梯形的面积是 ( ) A . 25 B . 50 C . D . 9.已知:M,N 两点关于 y 轴对称,且点 M 在双曲线 上,点 N 在直线 y=x+3 上,设 点 M 的坐标为(a,b),则二次函数 y=﹣abx2+(a+b)x( ) A . 有最大值,最大值为 B.有最大值,最大值为 C.有最小值,最小值为 D . 有最小值,最小值为 10.下列命题中,真命题的个数有( ) ①一个图形无论经过平移还是旋转,变换后的图形与原来图形的对应线段一定平行 ②函数 图象上的点 P(x,y)一定在第二象限 ③正投影的投影线彼此平行且垂直于投影面 ④使得|x|﹣y=3 和 y+x2=0 同时成立的 x 的取值为 . A . 3 个 B . 1 个 C . 4 个 D . 2 个 二、填空题(本大题共 6 个小题,每小题 3 分,共 18 分,本题要求把正确结果填在答题纸 规定的横线上,不需要解答过程) 11.函数 y= 中,自变量 x 的取值范围是 _________ . 12.太阳的半径约为 696 000 千米,用科学记数法表示为 _________ 千米. 13.如图,在△ABC 中,∠B=47°,三角形的外角∠DAC 和∠ACF 的平分线交于点 E,则 ∠AEC= _________ . 14.实数 a,b 在数轴上的位置如图所示,则 的化简结果为 _________ . 15.一组数据﹣1,0,2,3,x,其中这组数据的极差是 5,那么这组数据的平均数是 _________ . 16.如图是某几何体的三视图及相关数据(单位:cm),则该几何体的侧面积为 _________ cm. 三、解答题(本大题包括 9 个小题,共 72 分,解答应写出必要的演算步骤、证明过程或 文字说明) 17.(1)计算: . (2)先化简,再求值: ,其中 . 18.(1)解不等式:5(x﹣2)+8<6(x﹣1)+7; (2)若(1)中的不等式的最小整数解是方程 2x﹣ax=3 的解,求 a 的值. 19.如图,一次函数 y=kx+b 与反比例函数 的图象交于 A(m,6),B(n, 3)两点. (1)求一次函数的解析式; (2)根据图象直接写出 时 x 的取值范围. 20.如图,四边形 ABCD 是正方形,点 G 是 BC 边上任意一点,DE⊥AG 于 E, BF∥DE,交 AG 于 F. (1)求证:AF﹣BF=EF; (2)将△ABF 绕点 A 逆时针旋转,使得 AB 与 AD 重合,记此时点 F 的对应点为点 F′, 若正方形边长为 3,求点 F′与旋转前的图中点 E 之间的距离. 21.如图是交警在一个路口统计的某个时段来往车辆的车速情况(单位:千米/时) (1)找出该样本数据的众数和中位数; (2)计算这些车的平均速度;(结果精确到 0.1) (3)若某车以 50.5 千米/时的速度经过该路口,能否说该车的速度要比一半以上车的速度 快?并说明判断理由. 22.如图,线段 AB,DC 分别表示甲、乙两建筑物的高.某初三课外兴趣活动小组为了测 量两建筑物的高,用自制测角仪在 B 外测得 D 点的仰角为α,在 A 处测得 D 点的仰角为 β.已知甲、乙两建筑物之间的距离 BC 为 m.请你通过计算用含α、β、m 的式子分别表示 出甲、乙两建筑物的高度. 23.如图,某化工厂与 A,B 两地有公路和铁路相连,这家工厂从 A 地购买一批每吨 1 000 元的原料运回工厂,制成每吨 8 000 元的产品运到 B 地.已知公路运价为 1.5 元/(吨•千 米),铁路运价为 1.2 元/(吨•千米),这两次运输共支出公路运费 15 000 元,铁路运费 97 200 元,请计算这批产品的销售款比原料费和运输费的和多多少元? (1)根据题意,甲、乙两名同学分别列出尚不完整的方程组如下: 甲: 乙: 根据甲,乙两名同学所列方程组,请你分别指出未知数 x,y 表示的意义,然后在等式右边 的方框内补全甲、乙两名同学所列方程组. 甲:x 表示 _________ ,y 表示 _________ 乙:x 表示 _________ ,y 表示 _________ (2)甲同学根据他所列方程组解得 x=300,请你帮他解出 y 的值,并解决该实际问题. 24.如图,已知 AB 为⊙O 的直径,PA 与⊙O 相切于点 A,线段 OP 与弦 AC 垂直并相交 于点 D,OP 与弧 AC 相交于点 E,连接 BC. (1)求证:∠PAC=∠B,且 PA•BC=AB•CD; (2)若 PA=10,sinP= ,求 PE 的长. 25.如图,抛物线 y=ax2+bx+c(a<0)与双曲线 相交于点 A,B,且抛物线经过坐标 原点,点 A 的坐标为(﹣2,2),点 B 在第四象限内,过点 B 作直线 BC∥x 轴,点 C为直 线 BC 与抛物线的另一交点,已知直线 BC 与 x 轴之间的距离是点 B 到 y 轴的距离的 4 倍, 记抛物线顶点为 E. (1)求双曲线和抛物线的解析式; (2)计算△ABC 与△ABE 的面积; (3)在抛物线上是否存在点 D,使△ABD 的面积等于△ABE 的面积的 8 倍?若存在,请 求出点 D 的坐标;若不存在,请说明理由. 2012 年内蒙古呼和浩特市中考数学试卷 参考答案与试题解析 一、选择题(本大题共 10 个小题,每小题 3 分,共 30 分.在每小题给出的四个选项中, 只有一项是符合题目要求的) 1.(3 分)﹣2 的倒数是( ) A . 2 B . ﹣2 C . D . 考 点: 倒数。1444826 分 析:[来 源:Z|xx|k.Com] 根据倒数的定义,若两个数的乘积是 1,我们就称这两个数互为倒数. 解 答: 解:∵﹣2×( )=1,∴﹣2 的倒数是﹣ . 故选 D. 点 评: 主要考查倒数的概念及性质.倒数的定义:若两个数的乘积是 1,我们就称这两个数 互为倒数,属于基础题. 2.(3 分)如图,已知 a∥b,∠1=65°,则∠2 的度数为( ) A . 65° B . 125° C . 115° D . 25° 考 点: 平行线的性质。1444826 分 析: 先根据平行线的性质求出∠3 的度数,再由平角的定义即可得出结论. 解 答: 解:∵a∥b,∠1=65°, ∴∠3=∠1=65°, ∴∠2=180°﹣∠3=180°﹣65°=115°. 故选 C. 点 评: 本题考查的是平行线的性质,用到的知识点为:两直线平行,内错角相等. 3.(3 分)在一个不透明的口袋中,装有 3 个红球,2 个白球,除颜色不同外,其余都相 同,则随机从口袋中摸出一个球为红色的概率是( ) A . B . C . D . 考 点: 概率公式。1444826 分 析: 让红球的个数除以球的总数即为所求的概率. 解 答: 解:袋子中球的总数为 2+3=5,红球有 3 个,则摸出红球的概率为 , 故选 A. 点 评: 本题主要考查概率公式的知识点,解答本题的关键是熟练掌握概率公式:概率=所求 情况数与总情况数之比. 4.(3 分)下列各因式分解正确的是( ) A . ﹣x2+(﹣2)2= (x﹣2)(x+2) B . x2+2x﹣1=(x﹣ 1)2 C . 4x2﹣4x+1=(2x﹣ 1)2 D . x2﹣4x=x(x+2) (x﹣2) 考 点: 因式分解-运用公式法;因式分解-提公因式法。 1444826 分 析: 根据完全平方公式与平方差公式分解因式,提公因式法分解因式,对各选项分析判 断后利用排除法求解. 解 答: 解:A、﹣x2+(﹣2)2=﹣x2+4=(2﹣x)(2+x),故本选项错误; B、x2+2x﹣1 不符合完全平方公式,不能利用公式分解,故本选项错误; C、4x2﹣4x+1=(2x﹣1)2,故本选项正确; D、x2﹣4x=x(x﹣4),故本选项错误. 故选 C. 点 评: 本题考查了公式法分解因式,提公因式法分解因式,熟记平方差公式与完全平方公 式的结构式解题的关键. 5.(3 分)已知:x1,x2 是一元二次方程 x2+2ax+b=0 的两根,且 x1+x2=3,x1x2=1,则 a、 b 的值分别是( ) A . a=﹣3,b=1 B . a=3,b=1 C . ,b=﹣1 D . ,b=1 考 点: 根与系数的关系。 1444826 专 题: 计算题。 分 析: 先根据根与系数的关系可得 x1+x2=﹣2a,x1x2=b,而 x1+x2=3,x1x2=1,那么﹣ 2a=3,b=1,解即可. 解 答: 解:∵x1,x2 是一元二次方程 x2+2ax+b=0 的两根, ∴x1+x2=﹣2a,x1x2=b, ∵x1+x2=3,x1x2=1, ∴﹣2a=3,b=1, 即 a=﹣ ,b=1, 故选 D. 点 评:[来源:学 _科_网 Z_X_X_K] 本题考查了根与系数的关系,解题的关键是掌握根与系数的等量关系的公式. 6.(3 分)如图,在一长方形内有对角线长分别为 2 和 3 的菱形,边长为 1 的正六边形和半 径为 1 的圆,则一点随机落在这三个图形内的概率较大的是( ) A . 落在菱形内 B . 落在圆内 C . 落在正六边形内 D . 一样大 考 点: 几何概率。1444826 分 析: 分别求得三个图形的面积,则面积最大的就是所求的图形. 解 答: 解:菱形的面积是: ×2×3=3; 正六边形的面积是:6× = ; 圆的面积是:π. ∵π> >3, ∴圆的面积最大. ∴一点随机落在这三个图形内的概率较大的是:圆. 故选 B. 点 评: 本题考查了几何概率,正确求得三个图形的面积是关键. 7.(3 分)下面四条直线,其中直线上每个点的坐标都是二元一次方程 x﹣2y=2 的解是 ( ) A . B . C . D . 考 点: 一次函数与二元一次方程(组)。 1444826 分 析: 根据两点确定一条直线,当 x=0,求出 y 的值,再利用 y=0,求出 x 的值,即可得出 一次函数图象与坐标轴交点,即可得出图象. 解 答: 解:∵x﹣2y=2, ∴y= x﹣1, ∴当 x=0,y=﹣1,当 y=0,x=2, ∴一次函数 y= x﹣1,与 y 轴交于点(0,﹣1),与 x 轴交于点(2,0), 即可得出 C 符合要求, 故选:C. 点 评: 此题主要考查了一次函数与二元一次方程的关系,将方程转化为函数关系进而得出 与坐标轴交点坐标是解题关键. 8.(3 分)已知:在等腰梯形 ABCD 中,AD∥BC,AC⊥BD,AD=3,BC=7,则梯形的面 积是( ) A . 25 B . 50 C . D . 考 点: 等腰梯形的性质。 1444826 分 析: 过点 D 作 DE∥AC 交 BC 的延长线于点 E,作 DF⊥BC 于 F,证平行四边形 ADEC,推出 AC=DE=BD,∠BDE=90°,根据等腰三角形性质推出 BF=DF=EF= BE,求出 DF,根据梯形的面积公式求出即可. 解 答: 解:过点 D 作 DE∥AC 交 BC 的延长线于点 E, ∵AD∥BC(已知), 即 AD∥CE, ∴四边形 ACED 是平行四边形, ∴AD=CE=3,AC=DE, 在等腰梯形 ABCD 中,AC=DB, ∴DB=DE(等量代换), ∵AC⊥BD,AC∥DE, ∴DB⊥DE, ∴△BDE 是等腰直角三角形, 作 DF⊥BC 于 F, 则 DF= BE=5, S 梯形 ABCD= (AD+BC)•DF= (3+7)×5=25, 故选 A. 点 评: 本题主要考查对等腰三角形性质,平行四边形的性质和判定,等腰梯形的性质,等 腰直角三角形等知识点的理解和掌握,能求出高 DF 的长是解此题的关键. 9.(3 分)已知:M,N 两点关于 y 轴对称,且点 M 在双曲线 上,点 N 在直线 y=x+3 上,设点 M 的坐标为(a,b),则二次函数 y=﹣abx2+(a+b)x( ) A . 有最大值,最大值为 B.有最大值,最大值为 C.有最小值,最小值为 D . 有最小值,最小值为 考 点: 二次函数的最值;一次函数图象上点的坐标特征;反比例函数图象上点的坐标特 征;关于 x 轴、y 轴对称的点的坐标。 1444826 分 析: 先用待定系数法求出二次函数的解析式,再根据二次函数图象上点的坐标特征求出 其最值即可. 解 答: 解:∵M,N 两点关于 y 轴对称,点 M 的坐标为(a,b), ∴N 点的坐标为(﹣a,b), 又∵点 M 在反比例函数 的图象上,点 N 在一次函数 y=x+3 的图象上, ∴ , 整理得 , 故二次函数 y=﹣abx2+(a+b)x 为 y=﹣ x2+3x, ∴二次项系数为﹣ <0,故函数有最大值,最大值为 y= = , 故选:B. 点 评: 本题考查的是二次函数的最值.求二次函数的最大(小)值有三种方法,第一种可 由图象直接得出,第二种是配方法,第三种是公式法.本题是利用公式法求得的最 值. 10.(3 分)下列命题中,真命题的个数有( ) ①一个图形无论经过平移还是旋转,变换后的图形与原来图形的对应线段一定平行 ②函数 图象上的点 P(x,y)一定在第二象限 ③正投影的投影线彼此平行且垂直于投影面 ④使得|x|﹣y=3 和 y+x2=0 同时成立的 x 的取值为 . A . 3 个 B . 1 个 C . 4 个 D . 2 个 考 点: 命题与定理;非负数的性质:绝对值;二次根式有意义的条件;解一元二次方程-公 式法;二次函数图象上点的坐标特征;平移的性质;旋转的性质;平行投影。 1444826 分 ①根据平移的性质以及旋转的性质得出答案即可;②根据二次根式的性质以及点的 析: 坐标性质,得出答案;③根据正投影的定义得出答案; ④根据使得|x|﹣y=3 和 y+x2=0 同时成立,即 y=|x|﹣3,y=﹣x2,故|x|﹣3=﹣x2,进 而利用绝对值得性质,解方程即可得出答案. 解 答: 解:①平移后对应线段平行;对应线段相等;对应角相等;图形的形状和大小没有 发生变化. 旋转后对应线段不平行;对应线段相等;对应角相等;图形的形状和大小没有发生 变化,故此选项错误; ②根据二次根式的意义得出 x<0,y>0,故函数 图象上的点 P(x, y)一定在第二象限,故此选项正确; ③根据正投影的定义得出,正投影的投影线彼此平行且垂直于投影面,故此选项正 确; ④使得|x|﹣y=3 和 y+x2=0 同时成立,即 y=|x|﹣3,y=﹣x2,故|x|﹣3=﹣x2, x2﹣|x|﹣3=0, 当 x>0,则 x2﹣x﹣3=0, 解得:x1= ,x2= (不合题意舍去), 当 x<0,则 x2+x﹣3=0, 解得:x1= (不合题意舍去),x2= , 故使得|x|﹣y=3 和 y+x2=0 同时成立的 x 的取值为: , ,故此选项 错误, 故正确的有 2 个, 故选:D. 点 评: 此题主要考查了平移的性质以及旋转的性质和二次根式的性质、正投影、解一元二 次方程等知识,熟练根据绝对值性质整理出一元二次方程是解题关键. 二、填空题(本大题共 6 个小题,每小题 3 分,共 18 分,本题要求把正确结果填在答题纸 规定的横线上,不需要解答过程) 11.(3 分)函数 y= 中,自变量 x 的取值范围是 x≠2 . 考 点: 函数自变量的取值范围;分式有意义的条件。 1444826 专 题: 计算题。 分 析: 求函数自变量的取值范围,就是求函数解析式有意义的条件,分式有意义的条件 是:分母不为 0. 解 答: 解:x﹣2≠0,解得 x≠2. 点 评: 本题考查的知识点为:分式有意义,分母不为 0. 12.(3 分)太阳的半径约为 696 000 千米,用科学记数法表示为 6.96×105 千米. 考 科学记数法—表示较大的数。1444826 点: 分 析:[来 源:Z&xx&k.Com] 科学记数法的表示形式为 a×10n 的形式,其中 1≤|a|<10,n 为整数.确定 n 的值时, 要看把原数变成 a 时,小数点移动了多少位,n 的绝对值与小数点移动的位数相 同.当原数绝对值>10 时,n 是正数;当原数的绝对值<1 时,n 是负数. 解 答: 解:将 696 000 千米用科学记数法表示为 6.96×105 千米. 点 评: 此题考查科学记数法的表示方法.科学记数法的表示形式为 a×10n 的形式,其中 1≤|a|<10,n 为整数,表示时关键要正确确定 a 的值以及 n 的值. 13.(3 分)如图,在△ABC 中,∠B=47°,三角形的外角∠DAC 和∠ACF 的平分线交于点 E,则∠AEC= 66.5° . 考 点: 三角形内角和定理;三角形的外角性质。 1444826 分 析: 根据三角形内角和定理、角平分线的定义以及三角形外角定理求得 ∠DAC+ ACF= (∠B+∠B+∠BAC+∠BCA)= ;最后在△AEC 中利用三 角形内角和定理可以求得∠AEC 的度数. 解 答: 解:∵三角形的外角∠DAC 和∠ACF 的平分线交于点 E, ∴∠EAC= ∠DAC,∠ECA= ∠ACF; 又∵∠B=47°(已知),∠B+∠BAC+∠BCA=180°(三角形内角和定理), ∴ ∠DAC+ ACF= (∠B+∠ACB)+ (∠B+∠BAC)= (∠B+∠B+∠BAC+∠BCA)= (外角定理), ∴∠AEC=180°﹣( ∠DAC+ ACF)=66.5°; 故答案是:66.5°. 点 评: 本题考查了三角形内角和定理、三角形外角性质.解题时注意挖掘出隐含在题干中 已知条件“三角形内角和是 180°”. 14.(3 分)实数 a,b 在数轴上的位置如图所示,则 的化简结果为 ﹣ b . 考 点: 二次根式的性质与化简;实数与数轴。1444826 专 计算题。 题: 分 析: 由数轴得出 b<0<a,|b|>|a|,原式化简为|a+b|+a,去掉绝对值符号得出﹣a﹣b+a, 合并同类项即可. 解 答: 解:∵由数轴可知:b<0<a,|b|>|a|, ∴ +a, =|a+b|+a, =﹣a﹣b+a, =﹣b, 故答案为:﹣b. 点 评: 本题考查了二次根式的性质与化简和实数与数轴的应用,解此题的关键是根据数轴 得出 b<0<a 和|b|>|a|,题目比较典型,是一道比较好的题目. 15.(3 分)一组数据﹣1,0,2,3,x,其中这组数据的极差是 5,那么这组数据的平均数 是 1.6 或 0.4 . 考 点: 算术平均数;极差。1444826 分 析: 根据极差的定义求解.分两种情况:x 为最大值或最小值.再根据平均数的公式求解 即可. 解 答: 解:一组数据﹣1,0,2,3,x 的极差是 5, 当 x 为最大值时,x﹣(﹣1)=5,x=4,平均数是:(﹣1+0+2+3+4)÷5=1.6; 当 x 是最小值时,3﹣x=5,解得:x=﹣2,平均数是:(﹣1+0+2+3﹣2)÷5=0.4. 故答案为:1.6 或 0.4. 点 评: 考查了极差的定义和算术平均数,正确理解极差的定义,能够注意到应该分两种情 况讨论是解决本题的关键. 16.(3 分)如图是某几何体的三视图及相关数据(单位:cm),则该几何体的侧面积为 2π cm. 考 点: 圆锥的计算;由三视图判断几何体。 1444826 分 析: 根据三视图易得此几何体为圆锥,再根据圆锥侧面积公式=(底面周长×母线长)÷2 可计算出结果. 解 答: 解:由题意得底面直径为 2,母线长为 2, ∴几何体的侧面积为 ×2×2π=2π, 故答案为:2π. 点 评: 此题主要考查了由三视图判断几何体,以及圆锥的侧面积公式的应用,关键是找到 等量关系里相应的量. 三、解答题(本大题包括 9 个小题,共 72 分,解答应写出必要的演算步骤、证明过程或 文字说明) 17.(10 分)(1)计算: . (2)先化简,再求值: ,其中 . 考 点: 分式的化简求值;实数的运算;负整数指数幂;特殊角的三角函数值。1444826 分 析: (1)根据特殊角的三角函数值,绝对值,负整数指数幂进行计算; (2)先通分,将除法转化为乘法,约分,再代值计算. 解 答: 解:(1)原式= +( ﹣1)+ =2 ﹣ ; (2)原式=(x+1)÷ = , 当 x=﹣ 时,原式=﹣ ×(﹣ +1)÷(﹣ ×3+1)=﹣ . 点 评: 本题考查了分式的化简求值,实数的运算,负整数指数幂,特殊角的三角函数 值.关键是熟练掌握每一个运算法则. 18.(6 分)(1)解不等式:5(x﹣2)+8<6(x﹣1)+7; (2)若(1)中的不等式的最小整数解是方程 2x﹣ax=3 的解,求 a 的值. 考 点: 解一元一次不等式;一元一次方程的解;一元一次不等式的整数解。 1444826 分 析: (1)根据不等式的基本性质先去括号,然后通过移项、合并同类项即可求得原不等 式的解集; (2)根据(1)中的 x 的取值范围来确定 x 的最小整数解;然后将 x 的值代入已知 方程列出关于系数 a 的一元一次方程 2×(﹣2)﹣a×(﹣2)=3,通过解该方程即可 求得 a 的值. 解 答: 解:(1)5(x﹣2)+8<6(x﹣1)+7 5x﹣10+8<6x﹣6+7 5x﹣2<6x+1 ﹣x<3 x>﹣3 (2)由(1)得,最小整数解为 x=﹣2, ∴2×(﹣2)﹣a×(﹣2)=3 ∴a= . 点 评: 本题考查了解一元一次不等式、一元一次方程的解以及一元一次不等式的整数 解.解不等式要依据不等式的基本性质: (1)不等式的两边同时加上或减去同一个数或整式不等号的方向不变; (2)不等式的两边同时乘以或除以同一个正数不等号的方向不变; (3)不等式的两边同时乘以或除以同一个负数不等号的方向改变. 19.(6 分)如图,一次函数 y=kx+b 与反比例函数 的图象交于 A(m,6), B(n,3)两点. (1)求一次函数的解析式; (2)根据图象直接写出 时 x 的取值范围. 考 点: 反比例函数与一次函数的交点问题。 1444826 分 析: (1)先把(m,6)、B(n,3)代入反比例函数,可求 m、n 的值,即可得 A、B 的 坐标,然后把 AB 两点坐标代入一次函数,可得关于 k、b 的二元一次方程组,解可 得 k、b 的值,进而可得一次函数的解析式; (2)根据图象可知当 1<x<2 时,一次函数 y 的值大于反比例函数 y 的值. 解 答: 解:(1)∵点 A(m,6)、B(n,3)在函数 y= 图象上, ∴m=1,n=2, ∴A 点坐标是(1,6),B 点坐标是(2,3), 把(1,6)、(2,3)代入一次函数 y=kx+b 中,得 , 解得 , ∴一次函数的解析式为 y=﹣3x+9; (2)由图象知:1<x<2. 点 评: 本题考查了一次函数与反比例函数交点的问题,解题的关键是先求出 m、n 的值,并 注意待定系数法的使用. 20.(7 分)如图,四边形 ABCD 是正方形,点 G 是 BC 边上任意一点,DE⊥AG 于 E, BF∥DE,交 AG 于 F. (1)求证:AF﹣BF=EF; (2)将△ABF 绕点 A 逆时针旋转,使得 AB 与 AD 重合,记此时点 F 的对应点为点 F′, 若正方形边长为 3,求点 F′与旋转前的图中点 E 之间的距离. 考 点: 正方形的性质;全等三角形的判定与性质;矩形的判定与性质;旋转的性质。 1444826 专 题: 几何综合题。 分 析: (1)由四边形 ABCD 为正方形,可得出∠BAD 为 90°,AB=AD,进而得到∠BAG 与∠EAD 互余,又 DE 垂直于 AG,得到∠EAD 与∠ADE 互余,根据同角的余角相 等可得出∠ADE=∠BAF,利用 AAS 可得出三角形 ABF 与三角形 ADE 全等,利用 全等三角的对应边相等可得出 BF=AE,由 AF﹣AE=EF,等量代换可得证; (2)将△ABF 绕点 A 逆时针旋转,使得 AB 与 AD 重合,记此时点 F 的对应点为点 F′,连接 EF′,如图所示,由旋转的性质可得出∠FAF′为直角,AF=AF′,由第一问的 全等可得出 AF=DE,等量代换可得出 DE=AF′=AF,再利用同旁内角互补两直线平 行得到 AF′与 DE 平行,根据一组对边平行且相等的四边形为平行四边形可得出 AEDF′为平行四边形,再由一个角为直角的平行四边形为矩形可得出 AEDF′为矩 形,根据矩形的对角线相等可得出 EF′=AD,由 AD 的长即可求出 EF′的长. 解 答: (1)证明:如图,∵正方形 ABCD, ∴AB=AD,∠BAD=∠BAG+∠EAD=90°, ∵DE⊥AG, ∴∠AED=90°, ∴∠EAD+∠ADE=90°, ∴∠ADE=∠BAF, 又∵BF∥DE, ∴∠AEB=∠AED=90°, 在△AED 和△BFA 中, ∵ , ∴△AED≌△BDA(AAS), ∴BF=AE, ∵AF﹣AE=EF, ∴AF﹣BF=EF; (2)解:如图, 根据题意知:∠FAF′=90°,DE=AF′=AF, ∴∠F′AE=∠AED=90°,即∠F′AE+∠AED=180°, ∴AF′∥ED, ∴四边形 AEDF′为平行四边形,又∠AED=90°, ∴四边形 AEDF′是矩形, ∴EF′=AD=3. 点 评: 此题考查了正方形的性质,全等三角形的判定与性质,矩形的判定与性质,以及旋 转的性质,熟练掌握判定与性质是解本题的关键. 21.(9 分)如图是交警在一个路口统计的某个时段来往车辆的车速情况(单位:千米/时) (1)找出该样本数据的众数和中位数; (2)计算这些车的平均速度;(结果精确到 0.1) (3)若某车以 50.5 千米/时的速度经过该路口,能否说该车的速度要比一半以上车的速度 快?并说明判断理由. 考 点: 条形统计图;加权平均数;中位数;众数。1444826 专 题: 图表型。 分 析: (1)根据众数的定义,找出车辆数最多的即为众数,先求出车辆数的总数,再根据 中位数的定义解答; (2)根据加权平均数的计算方法列式计算即可得解; (3)与中位数相比较,大于中位数则是比一半以上车的速度快,否则不是. 解 答: 解:(1)该样本数据中车速是 52 的有 8 辆,最多, 所以,该样本数据的众数为 52, 样本容量为:2+5+8+6+4+2=27, 按照车速从小到大的顺序排列,第 13 辆车的车速是 52, 所以,中位数为 52; (2) ≈52.4 千米/时; (3)不能, 因为由(1)知样本的中位数为 52,所以可以估计该路段的车辆大约有一半的车速要 快于 52 千米/时, 该车的速度是 50.5 千米/时,小于 52 千米/时,所以不能说该车的速度要比一半以上 车的速度快. 点 本题考查的是条形统计图的综合运用.读懂统计图,从统计图中得到必要的信息是 评: 解决问题的关键.条形统计图能清楚地表示出每个项目的数据,也考查了平均数、 中位数、众数的认识. 22.(6 分)如图,线段 AB,DC 分别表示甲、乙两建筑物的高.某初三课外兴趣活动小组 为了测量两建筑物的高,用自制测角仪在 B 外测得 D 点的仰角为α,在 A 处测得 D 点的仰 角为β.已知甲、乙两建筑物之间的距离 BC 为 m.请你通过计算用含α、β、m 的式子分别 表示出甲、乙两建筑物的高度. 考 点: 解直角三角形的应用-仰角俯角问题。 1444826 分 析: 首先分析图形,根据题意构造直角三角形.本题涉及到两个直角三角形△ADM、 △DBC,应借助 AE=BC,求出 DC,DM,从而求出 AB 即可. 解 答: 解:过点 A 作 AM⊥CD,垂足为 M, 在 Rt△BCD 中,tanα= , ∴CD=BC•tanα=mtanα, 在 Rt△AMD 中,tanβ= , ∴DM=AM•tanβ=mtanβ, ∴AB=CD﹣DM=m(tanα﹣tanβ). 故甲建筑物的高度为 mtanα,乙建筑物的高度为 m(tanα﹣tanβ). 点 评: 此题主要考查了解直角三角形的应用,借助仰角关系构造直角三角形,并结合图形 利用三角函数解直角三角形是解题关键. 23.(8 分)如图,某化工厂与 A,B 两地有公路和铁路相连,这家工厂从 A 地购买一批每 吨 1 000 元的原料运回工厂,制成每吨 8 000 元的产品运到 B 地.已知公路运价为 1.5 元/ (吨•千米),铁路运价为 1.2 元/(吨•千米),这两次运输共支出公路运费 15 000 元,铁路 运费 97 200 元,请计算这批产品的销售款比原料费和运输费的和多多少元? (1)根据题意,甲、乙两名同学分别列出尚不完整的方程组如下: 甲: 乙: 根据甲,乙两名同学所列方程组,请你分别指出未知数 x,y 表示的意义,然后在等式右边 的方框内补全甲、乙两名同学所列方程组. 甲:x 表示 产品的重量 ,y 表示 原料的重量 乙:x 表示 产品销售额 ,y 表示 原料费 (2)甲同学根据他所列方程组解得 x=300,请你帮他解出 y 的值,并解决该实际问题. 考 点: 二元一次方程组的应用。1444826 分 析: (1)仔细分析题意根据题目中的两个方程表示出 x,y 的值并补全方程组即可; (2)将 x 的值代入方程组即可得到结论. 解 答: 解:(1)甲:x 表示产品的重量,y 表示原料的重量, 乙:x 表示产品销售额,y 表示原料费, 甲方程组右边方框内的数分别为:15000,97200,乙同甲; (2)将 x=300 代入原方程组解得 y=400 ∴产品销售额为 300×8000=2400000 元 原料费为 400×1000=400000 元 又∵运费为 15000+97200=112200 元 ∴这批产品的销售额比原料费和运费的和多 2400000﹣(400000+112200)=1887800 元 点 评: 本题考查了二元一次方程组的应用,解题的关键是从题目中找到等量关系并写出表 示出 x、y 所表示的实际意义. 24.(8 分)如图,已知 AB 为⊙O 的直径,PA 与⊙O 相切于点 A,线段 OP 与弦 AC 垂直 并相交于点 D,OP 与弧 AC 相交于点 E,连接 BC. (1)求证:∠PAC=∠B,且 PA•BC=AB•CD; (2)若 PA=10,sinP= ,求 PE 的长. 考 点: 切线的性质;勾股定理;圆周角定理;相似三角形的判定与性质;解直角三角形。 1444826 专 题: 几何综合题。 分 析: (1)由 PA 为圆 O 的切线,利用切线的性质得到 AP 垂直于 AB,可得出∠PAO 为 直角,得到∠PAD 与∠DAO 互余,再由 AB 为圆 O 的直径,根据直径所对的圆周角 为直角,可得出∠ACB 为直角,得到∠DAO 与∠B 互余,根据同角的余角相等可得 出∠PAC=∠B,再由一对直角相等,利用两对对应角相等的两三角形相似可得出三 角形 APD 与三角形 ABC 相似,由相似得比例,再由 OD 垂直于 AC,利用垂径定理 得到 AD=CD,等量代换可得证; (2)在直角三角形 APD 中,由 PA 及 sinP 的值求出 AD 的长,再利用勾股定理求出 PD 的长,进而确定出 AC 的长,由第一问两三角形相似得到的比例式,将各自的值 代入求出 AB 的上,求出半径 AO 的长,在直角三角形 APO 中,由 AP 及 AO 的 长,利用勾股定理求出 OP 的长,用 OP﹣OE 即可求出 PE 的长. 解 答: (1)证明:∵PA 是⊙O 的切线,AB 是直径, ∴∠PAO=90°,∠C=90°, ∴∠PAC+∠BAC=90°,∠B+∠BAC=90°, ∴∠PAC=∠B, 又∵OP⊥AC, ∴∠ADP=∠C=90°, ∴△PAD∽△ABC, ∴AP:AB=AD:BC, ∵在⊙O 中,AD⊥OD, ∴AD=CD, ∴AP:AB=CD:BC, ∴PA•BC=AB•CD; (2)解:∵sinP= ,且 AP=10, ∴ = , ∴AD=6, ∴AC=2AD=12, ∵在 Rt△ADP 中,PD= =8, 又∵△PAD∽△ABC, ∴AP:AB=PD:AC, ∴AB= =15, ∴A0= , 在 Rt△APO 中,根据勾股定理得:OP= = , ∴PE=OP﹣OE= ﹣ =5. 点 评: 此题考查了切线的性质,相似三角形的判定与性质,圆周角定理,勾股定理,垂径 定理,以及锐角三角函数定义,熟练掌握性质及定理是解本题的关键. 25.(12 分)如图,抛物线 y=ax2+bx+c(a<0)与双曲线 相交于点 A,B,且抛物线经 过坐标原点,点 A 的坐标为(﹣2,2),点 B 在第四象限内,过点 B 作直线 BC∥x 轴,点 C 为直线 BC 与抛物线的另一交点,已知直线 BC 与 x 轴之间的距离是点 B 到 y 轴的距离的 4 倍,记抛物线顶点为 E. (1)求双曲线和抛物线的解析式; (2)计算△ABC 与△ABE 的面积; (3)在抛物线上是否存在点 D,使△ABD 的面积等于△ABE 的面积的 8 倍?若存在,请 求出点 D 的坐标;若不存在,请说明理由. 考 点: 二次函数综合题。 1444826 专 题: 综合题。 分 析: (1)将点 A 的坐标代入双曲线方程即可得出 k 的值,设 B 点坐标为(m,﹣4m) (m>0),根据双曲线方程可得出 m 的值,然后分别得出了 A、B、O 的坐标,利用 待定系数法求解二次函数解析式即可; (2)根据点 B 的坐标,结合抛物线方程可求出点 C 的坐标,继而可得出三角形 ABC 的面积,先求出 AB 的解析式,然后求出点 F 的坐标,及 EF 的长,继而根据 S△ABE=S△AEF+S△BEF 可得出答案. (3)先确定符合题意的三角形 ABD 的面积,继而可得出当点 D 与点 C 重合时,满 足条件,过点 C 作 AB 的平行线 CD,则可求出其解析式,求出其与抛物线的交点坐 标即可得出点 D 的坐标. 解 答: 解:(1)∵点 A(﹣2,2)在双曲线 y= 上, ∴k=﹣4, ∴双曲线的解析式为 y=﹣ , ∵BC 与 x 轴之间的距离是点 B 到 y 轴距离的 4 倍, ∴设 B 点坐标为(m,﹣4m)(m>0)代入双曲线解析式得 m=1, ∴抛物线 y=ax2+bx+c(a<0)过点 A(﹣2,2)、B(1,﹣4)、O(0,0), ∴ , 解得: , 故抛物线的解析式为 y=﹣x2﹣3x; (2)∵抛物线的解析式为 y=﹣x2﹣3x, ∴顶点 E(﹣ , ),对称轴为 x=﹣ , ∵B(1,﹣4), ∴﹣x2﹣3x=﹣4, 解得:x1=1,x2=﹣4, ∴C(﹣4,﹣4), ∴S△ABC=5×6× =15, 由 A、B 两点坐标为(﹣2,2),(1,﹣4)可求得直线 AB 的解析式为:y=﹣2x﹣ 2, 设抛物线的对称轴与 AB 交于点 F,则 F 点的坐标为(﹣ ,1), ∴EF= ﹣1= , ∴S△ABE=S△AEF+S△BEF= × ×3= ; (3)S△ABE= , ∴8S△ABE=15, ∴当点 D 与点 C 重合时,显然满足条件; 当点 D 与点 C 不重合时,过点 C 作 AB 的平行线 CD,其对应的一次函数解析式为 y=﹣2x﹣12,[来源:Zxxk.Com] 令﹣2x﹣12=﹣x2﹣3x, 解得 x1=3,x2=﹣4(舍去), 当 x=3 时,y=﹣18, 故存在另一点 D(3,﹣18)满足条件. 综上可得点 D 的坐标为(3,﹣18)或(﹣4,﹣4). 点 评: 此题属于二次函数的综合题目,第一问的解答关键是掌握待定系数法的运用,求解 第二问需要我们会根据函数解析式求两函数图象的交点坐标,此类综合题目,难度 较大,注意逐步分析. 2012 年贵州省贵阳市中考数学试卷 一、选择题(共 10 小题,每小题 3 分,满分 30 分) 1.(3 分)(2012•贵阳)下列整数中,小于﹣3 的整数是( ) A . ﹣4 B . ﹣2 C . 2 D . 3 2.(3 分)(2012•贵阳)在 5 月份的助残活动中,盲聋哑学校收到社会捐款约 110000 元, 将 110000 元用科学记数法表示为( ) A . 1.1×103 元 B . 1.1×104 元 C . 1.1×105 元 D . 1.1×106 元 3.(3 分)(2012•贵阳)下列四个几何体中,主视图、左视图与俯视图是全等图形的几何 体是( ) A . 圆锥 B . 圆柱 C . 三棱柱 D . 球 4.(3 分)(2012•贵阳)如图,已知点 A、D、C、F 在同一条直线上,AB=DE,BC=EF, 要使△ABC≌△DEF,还需要添加一个条件是( ) A . ∠BCA=∠F B . ∠B=∠E C . BC∥EF D . ∠A=∠EDF 5.(3 分)(2012•贵阳)一个不透明的盒子里有 n 个除颜色外其它完全相同的小球,其中 有 6 个黄球.每次摸球前先将盒子里的球摇匀,任意摸出一个球记下颜色后在放回盒子, 通过大量重复摸球实验后发现,摸到黄球的频率稳定在 30%,那么可以推算出 n 大约是 ( ) A . 6 B . 10 C . 18 D . 20 6.(3 分)(2012•贵阳)下列图案是一副扑克牌的四种花色,其中既是轴对称图形又是中 心对称图形的是( ) A . B . C . D . 7.(3 分)(2012•贵阳)如图,一次函数 y=k1x+b1 的图象 l1 与 y=k2x+b2 的图象 l2 相交于点 P,则方程组 的解是( ) A . B . C . D . 8.(3 分)(2012•贵阳)如图,在 Rt△ABC 中,∠ACB=90°,AB 的垂直平分线 DE 交于 BC 的延长线于 F,若∠F=30°,DE=1,则 EF 的长是( ) A . 3 B . 2 C . D . 1 9.(3 分)(2012•贵阳)为了参加我市组织的“我爱家乡美”系列活动,某校准备从九年级四 个班中选出一个班的 7 名学生组建舞蹈队,要求各班选出的学生身高较为整齐,且平均身 高约为 1.6m.根据各班选出的学生,测量其身高,计算得到的数据如下表所示,学校应选 择( ) 学生平均身高(单位:m) 标准差 九(1)班 1.57 0.3 九(2)班 1.57 0.7 九(3)班 1.6 0.3 九(4)班 1.6 0.7 A . 九(1)班 B . 九(2)班 C . 九(3)班 D . 九(4)班 10.(3 分)(2012•贵阳)已知二次函数 y=ax2+bx+c(a<0)的图象如图所示,当﹣5≤x≤0 时,下列说法正确的是( ) A . 有最小值﹣5、最大值 0 B.有最小值﹣3、最大值 6 C.有最小值 0、最大值 6 D . 有最小值 2、最大值 6 二、填空题(共 5 小题,每小题 4 分,满分 20 分) 11.(4 分)不等式 x﹣2≤0 的解集是 _________ . 12.(4 分)(2012•贵阳)如图,已知∠1=∠2,则图中互相平行的线段是 _________ . 13.(4 分)(2012•贵阳)在正比例函数 y=﹣3mx 中,函数 y 的值随 x 值的增大而增大,则 P(m,5)在第 _________ 象限. 14.(4 分)(2012•贵阳)张老师对同学们的打字能力进行测试,他将全班同学分成五 组.经统计,这五个小组平均每分钟打字个数如下:100,80,x,90,90,已知这组数据 的众数与平均数相等,那么这组数据的中位数是 _________ . 15.(4 分)(2012•贵阳)如图,在△ABA1 中,∠B=20°,AB=A1B,在 A1B 上取一点 C, 延长 AA1 到 A2,使得 A1A2=A1C;在 A2C 上取一点 D,延长 A1A2 到 A3,使得 A2A3=A2D;…,按此做法进行下去,∠An 的度数为 _________ . 三、解答题(共 10 小题,满分 100 分) 16.(8 分)(2012•贵阳)先化简,再求值:2b2+(a+b)(a﹣b)﹣(a﹣b)2,其中 a=﹣ 3,b= . 17.(8 分)(2012•贵阳)为了全面提升中小学教师的综合素质,贵阳市将对教师的专业知 识每三年进行一次考核.某校决定为全校数学教师每人购买一本义务教育《数学课程标准 (2011 年版)》(以下简称《标准》),同时每人配套购买一本《数学课程标准(2011 年版) 解读》(以下简称《解读》),其中《解读》的单价比《标准》的单价多 25 元.若学校购买 《标准》用了 378 元,购买《解读》用了 1053 元,请问《标准》和《解读》的单价各是多 少元? 18.(10 分)(2012•贵阳)林城市对教师试卷讲评课中学生参与的深度和广度进行评价, 其评价项目为主动质疑、独立思考、专注听讲、讲解题目四项.评价组随机抽取了若干名 初中学生的参与情况,绘制了如图两幅不完整的统计图,请根据图中所给信息解答下列问 题: (1)在这次评价中,一共抽查了 _________ 名学生; (2)请将条形统计图补充完整; (3)如果全市有 16 万名初中学生,那么在试卷讲评课中,“独立思考”的学生约有多少万 人? 19.(10 分)(2012•贵阳)小亮想知道亚洲最大的瀑布黄果树夏季洪峰汇成巨瀑时的落 差.如图,他利用测角仪站在 C 处测得∠ACB=68°,再沿 BC 方向走 80m 到达 D 处,测得 ∠ADC=34°,求落差 AB.(测角仪高度忽略不计,结果精确到 1m) 20.(10 分)(2012•贵阳)在一个不透明的口袋里有分别标注 2、4、6 的 3 个小球(小球 除数字不同外,其余都相同),另有 3 张背面完全一样、正面分别写有数字 6、7、8 的卡 片.现从口袋中任意摸出一个小球,再从这 3 张背面朝上的卡片中任意摸出一张卡片. (1)请你用列表或画树状图的方法,表示出所有可能出现的结果; (2)小红和小莉做游戏,制定了两个游戏规则: 规则 1:若两次摸出的数字,至少有一次是“6”,小红赢;否则,小莉赢. 规则 2:若摸出的卡片上的数字是球上数字的整数倍时,小红赢;否则,小莉赢. 小红要想在游戏中获胜,她会选择哪一种规则,并说明理由. 21.(10 分)(2012•贵阳)如图,在正方形 ABCD 中,等边三角形 AEF 的顶点 E、F 分别 在 BC 和 CD 上. (1)求证:CE=CF; (2)若等边三角形 AEF 的边长为 2,求正方形 ABCD 的周长. 22.(10 分)(2012•贵阳)已知一次函数 y= x+2 的图象分别与坐标轴相交于 A、B 两点 (如图所示),与反比例函数 y= (x>0)的图象相交于 C 点. (1)写出 A、B 两点的坐标; (2)作 CD⊥x 轴,垂足为 D,如果 OB 是△ACD 的中位线,求反比例函数 y= (x>0) 的关系式. 23.(10 分)(2012•贵阳)如图,在⊙O 中,直径 AB=2,CA 切⊙O 于 A,BC 交⊙O 于 D,若∠C=45°,则 (1)BD 的长是 _________ ; (2)求阴影部分的面积. 24.(12 分)(2012•贵阳)如果一条直线把一个平面图形的面积分成相等的两部分,我们 把这条直线称为这个平面图形的一条面积等分线. (1)三角形有 _________ 条面积等分线,平行四边形有 _________ 条面积等分线; (2)如图①所示,在矩形中剪去一个小正方形,请画出这个图形的一条面积等分线; (3)如图②,四边形 ABCD 中,AB 与 CD 不平行,AB≠CD,且 S△ABC<S△ACD,过点 A 画出四边形 ABCD 的面积等分线,并写出理由. 25.(12 分)(2012•贵阳)如图,二次函数 y= x2﹣x+c 的图象与 x 轴分别交于 A、B 两 点,顶点 M 关于 x 轴的对称点是 M′. (1)若 A(﹣4,0),求二次函数的关系式; (2)在(1)的条件下,求四边形 AMBM′的面积; (3)是否存在抛物线 y= x2﹣x+c,使得四边形 AMBM′为正方形?若存在,请求出此抛物 线的函数关系式;若不存在,请说明理由. 2012 年贵州省贵阳市中考数学试卷 参考答案与试题解析 一、选择题(共 10 小题,每小题 3 分,满分 30 分) 1.(3 分)(2012•贵阳)下列整数中,小于﹣3 的整数是( ) A . ﹣4 B . ﹣2 C . 2 D . 3 考 点: 有理数大小比较;绝对值。 190187 专 题: 推理填空题。 分 析: 根据正数都大于负数,两个负数比较大小,其绝对值大的反而小,得出 2 和 3 都大 于﹣3,求出|﹣3|=3,|﹣2|=2,|﹣4|=4,比较即可. 解 答: 解:∵﹣4<﹣3<﹣2<2<3, ∴整数﹣4、﹣2、2、3 中,小于﹣3 的整数是﹣4, 故选 A. 点 评: 本题考查了绝对值和有理数的大小比较的应用,有理数的大小比较法则是:正数都 大于 0,正数大于一切负数,负数都小于 0,两个负数比较大小,其绝对值大的反而 小. 2.(3 分)(2012•贵阳)在 5 月份的助残活动中,盲聋哑学校收到社会捐款约 110000 元, 将 110000 元用科学记数法表示为( ) A . 1.1×103 元 B . 1.1×104 元 C . 1.1×105 元 D . 1.1×106 元 考 点: 科学记数法—表示较大的数。190187 分 析: 科学记数法的表示形式为 a×10n 的形式,其中 1≤|a|<10,n 为整数.确定 n 的值时, 要看把原数变成 a 时,小数点移动了多少位,n 的绝对值与小数点移动的位数相 同.当原数绝对值>1 时,n 是正数;当原数的绝对值<1 时,n 是负数. 解 答: 解:将 110000 用科学记数法表示为:1.1×105. 故选:C. 点 评: 此题考查了科学记数法的表示方法.科学记数法的表示形式为 a×10n 的形式,其中 1≤|a|<10,n 为整数,表示时关键要正确确定 a 的值以及 n 的值. 3.(3 分)(2012•贵阳)下列四个几何体中,主视图、左视图与俯视图是全等图形的几何 体是( ) A . 圆锥 B . 圆柱 C . 三棱柱 D . 球 考 简单几何体的三视图。 190187 点: 分 析: 根据几何体的三种视图,进行选择即可. 解 答: 解:A、圆锥的主视图、左视图都是等腰三角形,俯视图是圆形,不符合题意,故此 选项错误; B、圆柱的主视图、左视图可以都是矩形,俯视图是圆形,不符合题意,故此选项错 误; C、三棱柱的主视图、左视图都是矩形,俯视图是三角形,不符合题意,故此选项错 误; D、球的三视图都是圆形,故此选项正确. 故选:D. 点 评: 本题考查了几何体的三种视图,注意所有的看到的棱都应表现在三视图中. 4.(3 分)(2012•贵阳)如图,已知点 A、D、C、F 在同一条直线上,AB=DE,BC=EF, 要使△ABC≌△DEF,还需要添加一个条件是( ) A . ∠BCA=∠F B . ∠B=∠E C . BC∥EF D . ∠A=∠EDF 考 点: 全等三角形的判定。190187 分 析: 全等三角形的判定方法 SAS 是指有两边对应相等,且这两边的夹角相等的两三角形 全等,已知 AB=DE,BC=EF,其两边的夹角是∠B 和∠E,只要求出∠B=∠E 即 可. 解 答: 解:A、根据 AB=DE,BC=EF 和∠BCA=∠F 不能推出△ABC≌△DEF,故本选项 错误; B、∵在△ABC 和△DEF 中 , ∴△ABC≌△DEF(SAS),故本选项正确; C、∵BC∥EF, ∴∠F=∠BCA,根据 AB=DE,BC=EF 和∠F=∠BCA 不能推出△ABC≌△DEF,故 本选项错误; D、根据 AB=DE,BC=EF 和∠A=∠EDF 不能推出△ABC≌△DEF,故本选项错 误. 故选 B. 点 评: 本题考查了对平行线的性质和全等三角形的判定的应用,注意:有两边对应相等, 且这两边的夹角相等的两三角形才全等,题目比较典型,但是一道比较容易出错的 题目. 5.(3 分)(2012•贵阳)一个不透明的盒子里有 n 个除颜色外其它完全相同的小球,其中 有 6 个黄球.每次摸球前先将盒子里的球摇匀,任意摸出一个球记下颜色后在放回盒子, 通过大量重复摸球实验后发现,摸到黄球的频率稳定在 30%,那么可以推算出 n 大约是 ( ) A . 6 B . 10 C . 18 D . 20 考 点: 利用频率估计概率。190187 分 析: 在同样条件下,大量反复试验时,随机事件发生的频率逐渐稳定在概率附近,可以 从比例关系入手,列出方程求解. 解 答: 解:由题意可得, ×100%=30%, 解得,n=20(个). 故估计 n 大约有 20 个. 故选:D. 点 评: 此题主要考查了利用频率估计概率,本题利用了用大量试验得到的频率可以估计事 件的概率.关键是根据黄球的频率得到相应的等量关系. 6.(3 分)(2012•贵阳)下列图案是一副扑克牌的四种花色,其中既是轴对称图形又是中 心对称图形的是( ) A . B . C . D . 考 点: 中心对称图形;轴对称图形。190187 专 题: 推理填空题。 分 析: 根据轴对称图形的定义得出四个图案都是轴对称图形,但是中心对称图形的图形只 有 C,即可得出答案. 解 答: 解:∵根据轴对称图形的定义得出四个图案都是轴对称图形,但是中心对称图形的 图形只有 C, ∴一副扑克牌的四种花色图案中,既是轴对称图形又是中心对称图形的图案是 C, 故选 C. 点 评: 本题考查了对中心对称图形和轴对称图形的理解和运用,注意:中心对称图形是指 一个图形绕一个点旋转 180°后,能和原来的图形完全重合,题目比较典型,但是一 道比较容易出错的题目. 7.(3 分)(2012•贵阳)如图,一次函数 y=k1x+b1 的图象 l1 与 y=k2x+b2 的图象 l2 相交于点 P,则方程组 的解是( ) A . B . C . D.[ 来源:学§科§网 Z§X §X§K] 考 点: 一次函数与二元一次方程(组)。 190187 专 题: 推理填空题。 分 析: 根据图象求出交点 P 的坐标,根据点 P 的坐标即可得出答案. 解 答: 解:∵由图象可知:一次函数 y=k1x+b1 的图象 l1 与 y=k2x+b2 的图象 l2 的交点 P 的坐 标是(﹣2,3), ∴方程组 的解是 , 故选 A. 点 评: 本题考查了对一次函数与二元一次方程组的关系的理解和运用,主要考查学生的观 察图形的能力和理解能力,题目比较典型,但是一道比较容易出错的题目. 8.(3 分)(2012•贵阳)如图,在 Rt△ABC 中,∠ACB=90°,AB 的垂直平分线 DE 交于 BC 的延长线于 F,若∠F=30°,DE=1,则 EF 的长是( ) A . 3 B . 2 C . D . 1 考 点: 线段垂直平分线的性质;角平分线的性质;含 30 度角的直角三角形。190187 专 题: 计算题。 分 析: 连接 AF,求出 AF=BF,求出∠AFD、∠B,得出∠BAC=30°,求出 AE,求出 ∠FAC=∠AFE=30°,推出 AE=EF,代入求出即可. 解 答: 解:连接 AF, ∵DF 是 AB 的垂直平分线, ∴AF=BF, ∵FD⊥AB, ∴∠AFD=∠BFD=30°,∠B=∠FAB=90°﹣30°=60°, ∵∠ACB=90°, ∴∠BAC=30°,∠FAC=60°﹣30°=30°, ∵DE=1, ∴AE=2DE=2, ∵∠FAE=∠AFD=30°, ∴EF=AE=2, 故选 B. 点 评: 本题考查了含 30 度角的直角三角形,线段垂直平分线,角平分线的性质等知识点的 应用,主要考查学生运用性质进行推理和计算的能力,题目综合性比较强. 9.(3 分)(2012•贵阳)为了参加我市组织的“我爱家乡美”系列活动,某校准备从九年级四 个班中选出一个班的 7 名学生组建舞蹈队,要求各班选出的学生身高较为整齐,且平均身 高约为 1.6m.根据各班选出的学生,测量其身高,计算得到的数据如下表所示,学校应选 择( ) 学生平均身高(单位:m) 标准差 九(1)班 1.57 0.3 九(2)班 1.57 0.7 九(3)班 1.6 0.3 九(4)班 1.6 0.7 A . 九(1)班 B . 九(2)班 C . 九(3)班 D . 九(4)班 考 点: 方差;算术平均数;标准差。190187 分 析: 根据标准差的意义,标准差越小数据越稳定,故比较标准差后可以选出身高比较整 齐的班级,再根据平均身高的要求即可作出判断. 解 答: 解:由于选的是学生身高较为整齐的,故要选取标准差小的,应从九(1)和九 (3)里面选,再根据平均身高约为 1.6m 可知只有九(3)符合要求, 故选:C. 点 评: 此题主要考查了差的意义.标准差是反应一组数据离散程度最常用的一种量化形 式,是表示精密确的最要指标.标准差越大,则平均值的离散程度越大,稳定性也 越小;反之,则它与其平均值的离散程度越小,稳定性越好. 10.(3 分)(2012•贵阳)已知二次函数 y=ax2+bx+c(a<0)的图象如图所示,当﹣5≤x≤0 时,下列说法正确的是( ) A . 有最小值﹣5、最大值 0 B. 有最小值﹣3、最大值 6 C.有最小值 0、最大值 6 D . 有最小值 2、最大值 6 考点: 二次函数的最值。 190187 专题:数形结合。 分析:直接根据二次函数的图象进行解答即可. 解答:解:由二次函数的图象可知, ∵﹣5≤x≤0, ∴当 x=﹣2 时函数有最大值,y 最大=6; 当 x=﹣5 时函数值最小,y 最小=﹣3. 故选 B. 点评:本题考查的是二次函数的最值问题,能利用数形结合求出函数的最值是解答此题的 关键. 二、填空题(共 5 小题,每小题 4 分,满分20 分) 11.(4 分)不等式 x﹣2≤0 的解集是 x≤2 . 考 点: 解一元一次不等式。190187 分 析: 利用不等式的基本性质,把不等号右边的 x 移到左边,合并同类项即可求得原不等 式的解集. 解 答: 解:移项得:x≤2. 点 评: 本题考查了解简单不等式的能力,解答这类题学生往往在解题时不注意移项要改变 符号这一点而出错. 解不等式要依据不等式的基本性质: (1)不等式的两边同时加上或减去同一个数或整式不等号的方向不变; (2)不等式的两边同时乘以或除以同一个正数不等号的方向不变; (3)不等式的两边同时乘以或除以同一个负数不等号的方向改变. 12.(4 分)(2012•贵阳)如图,已知∠1=∠2,则图中互相平行的线段是 AB∥CD . 考 点: 平行线的判定。190187 专 题: 探究型。 分 析: 直接根据平行线的判定定理进行解答即可. 解 答: 解:∵∠1=∠2(已知), ∴AB∥CD(内错角相等,两直线平行). 故答案为:AB∥CD. 点 评: 本题考查的是平行线的判定定理,即内错角相等,两直线平行. 13.(4 分)(2012•贵阳)在正比例函数 y=﹣3mx 中,函数 y 的值随 x 值的增大而增大,则 P(m,5) 在第 二 象限. 考 点: 正比例函数的性质;点的坐标。 190187 专 题: 探究型。 分 析: 先根据正比例函数 y=﹣3mx 中,函数 y 的值随 x 值的增大而增大判断出﹣3m 的符 号,求出 m 的取值范围即可判断出 P 点所在象限. 解 答: 解:∵正比例函数 y=﹣3mx 中,函数 y 的值随 x 值的增大而增大, ∴﹣3m>0,解得 m<0, ∴点 P(m,5)在第二象限. 故答案为:二. 点 评: 本题考查的是正比例函数的性质,根据题意判断出 m 的符号是解答此题的关键. 14.(4 分)(2012•贵阳)张老师对同学们的打字能力进行测试,他将全班同学分成五 组.经统计,这五个小组平均每分钟打字个数如下:100,80,x,90,90,已知这组数据 的众数与平均数相等,那么这组数据的中位数是 90 . 考 点: 中位数;算术平均数;众数。190187 专 题: 推理填空题。 分 析: 分别求出当 x=80、x=90、x=100 时的 x 值,再看看这组数据的众数与平均数是否相 等,最后求出这组数据的中位数即可. 解 答: 解:∵100,80,x,90,90, ∴分为 3 种情况:①当众数是 90 时, ∵这组数据的众数与平均数相等, ∴ =90, 解得:x=90; ②当众数是 80 时,即 x=80, ∵这组数据的众数与平均数相等, ∴ ≠80, ∴此时不行; ③当众数是 100 时,即 x=100, ∵这组数据的众数与平均数相等, ∴ ≠100, ∴此时不行; ∵当 x=90 时,数据为 80,90,90,90,100, ∴中位数是 90, 故答案为:90. 点 评: 本题考查了对中位数、平均数、众数的理解和运用,关键是求出符合条件的 x 的 值,题目具有一定的代表性,是一道比较好的题目. 15.(4 分)(2012•贵阳)如图,在△ABA1 中,∠B=20°,AB=A1B,在 A1B 上取一点 C, 延长 AA1 到 A2,使得 A1A2=A1C;在 A2C 上取一点 D,延长 A1A2 到 A3,使得 A2A3=A2D;…,按此做法进行下去,∠An 的度数为 . [来 考 点: 等腰三角形的性质;三角形的外角性质。] 专 题: 规律型。 分 析: 先根据等腰三角形的性质求出∠BA1A 的度数,再根据三角形外角的性质及等腰三角 形的性质分别求出∠CA2A1,∠DA3A2 及∠EA4A3 的度数,找出规律即可得出∠An 的度数. 解 答: 解:∵在△ABA1 中,∠B=20°,AB=A1B, ∴∠BA1A= = =80°, ∵A1A2=A1C,∠BA1A 是△A1A2C 的外角, ∴∠CA2A1= = =40°; 同理可得, ∠DA3A2=20°,∠EA4A3=10°, ∴∠An= . 故答案为: . 点 评: 本题考查的是等腰三角形的性质及三角形外角的性质,根据题意得出∠CA2A1, ∠DA3A2 及∠EA4A3 的度数,找出规律是解答此题的关键. 三、解答题(共 10 小题,满分 100 分) 16.(8 分)(2012•贵阳)先化简,再求值:2b2+(a+b)(a﹣b)﹣(a﹣b)2,其中 a=﹣ 3,b= . 考 点: 整式的混合运算—化简求值。190187 专 题: 探究型。 分 析: 先根据整式混合运算的法则把原式进行化简,再把 a=﹣3,b= 代入进行计算即可. 解 答: 解:原式=2b2+a2﹣b2﹣(a2+b2﹣2ab) =2b2+a2﹣b2﹣a2﹣b2+2ab =2ab, 当 a=﹣3,b= 时,原式=2×(﹣3)× =﹣3. 点 评: 本题考查的是整式的化简求出,熟知整式混合运算的法则是解答此题的关键. 17.(8 分)(2012•贵阳)为了全面提升中小学教师的综合素质,贵阳市将对教师的专业知 识每三年进行一次考核.某校决定为全校数学教师每人购买一本义务教育《数学课程标准 (2011 年版)》(以下简称《标准》),同时每人配套购买一本《数学课程标准(2011 年版) 解读》(以下简称《解读》),其中《解读》的单价比《标准》的单价多 25 元.若学校购买 《标准》用了 378 元,购买《解读》用了 1053 元,请问《标准》和《解读》的单价各是多 少元? 考 点: 分式方程的应用。 190187 分 析: 首先设《标准》的单价为 x 元,根据《解读》的单价比《标准》的单价多 25 元,得 出《解读》的单价是(x+25)元,利用两种书数量相同得出等式方程求出即可. 解 答: 解:设《标准》的单价为 x 元,则《解读》的单价是(x+25)元,由题意得: = , 解得:x=14, 经检验 x=14 是原方程的根, 则 x+25=25+14=39. 答:《标准》和《解读》的单价各是 14 元、39 元. 点 评: 此题主要考查了分式方程的应用,根据已知表示出两种书的数量,进而得出等式方 程是解题关键. 18.(10 分)(2012•贵阳)林城市对教师试卷讲评课中学生参与的深度和广度进行评价, 其评价项目为主动质疑、独立思考、专注听讲、讲解题目四项.评价组随机抽取了若干名 初中学生的参与情况,绘制了如图两幅不完整的统计图,请根据图中所给信息解答下列问 题: (1)在这次评价中,一共抽查了 560 名学生; (2)请将条形统计图补充完整; (3)如果全市有 16 万名初中学生,那么在试卷讲评课中,“独立思考”的学生约有多少万 人? 考 点: 条形统计图;用样本估计总体;扇形统计图。 190187 专 题: 图表型。 分 析: (1)根据扇形统计图专注听讲的百分比与条形统计图中专注听讲的人数,列式计算 即可; (2)用被抽查的学生人数减去主动质疑、独立思考、专注听讲的人数,求出讲解题 目的人数,然后补全统计图即可; (3)用独立思考的学生的百分比乘以 16 万,进行计算即可得解. 解 答: 解:(1)224÷40%=560 名; (2)讲解题目的学生数为:560﹣84﹣168﹣224=560﹣476=84, 补全统计图如图; (3) ×16=4.8 万, 答:在试卷讲评课中,“独立思考”的学生约有 4.8 万人. 点 评: 本题考查的是条形统计图和扇形统计图的综合运用,读懂统计图,从不同的统计图 中得到必要的信息是解决问题的关键.条形统计图能清楚地表示出每个项目的数 据;扇形统计图直接反映部分占总体的百分比大小,本题利用“专注听讲”的人数与 百分比求出总人数是解题的关键. 19.(10 分)(2012•贵阳)小亮想知道亚洲最大的瀑布黄果树夏季洪峰汇成巨瀑时的落 差.如图,他利用测角仪站在 C 处测得∠ACB=68°,再沿 BC 方向走 80m 到达 D 处,测得 ∠ADC=34°,求落差 AB.(测角仪高度忽略不计,结果精确到 1m) 考 点: 解直角三角形的应用-仰角俯角问题。 190187 专 题: 探究型。 分 析: 先根据三角形外角的性质求出∠CAD 的度数,故可得出∠CAD=∠D,所以 AC=CD=80,在 Rt△ABC 中,由 AB=AC×sin68°即可得出结论. 解 答: 解:∵ACB=68°,∠D=34°,∠ACB 是△ACD 的外角, ∴∠CAD=∠ACB﹣∠D=68°﹣34°=34°, ∴∠CAD=∠D, ∴AC=CD=80, 在 Rt△ABC 中,AB=AC×sin68°≈80×0.927≈74(m). 答:落差 AB 为 74m. 点 评: 本题考查的是解直角三角形的应用﹣仰角俯角问题,涉及到三角形外角的性质及等 腰三角形的性质,根据题意得出 AC 的长是解答此题的关键. 20.(10 分)(2012•贵阳)在一个不透明的口袋里有分别标注 2、4、6 的 3 个小球(小球 除数字不同外,其余都相同),另有 3 张背面完全一样、正面分别写有数字 6、7、8 的卡 片.现从口袋中任意摸出一个小球,再从这 3 张背面朝上的卡片中任意摸出一张卡片. (1)请你用列表或画树状图的方法,表示出所有可能出现的结果; (2)小红和小莉做游戏,制定了两个游戏规则: 规则 1:若两次摸出的数字,至少有一次是“6”,小红赢;否则,小莉赢. 规则 2:若摸出的卡片上的数字是球上数字的整数倍时,小红赢;否则,小莉赢. 小红要想在游戏中获胜,她会选择哪一种规则,并说明理由. 考 点: 列表法与树状图法。190187 专 题: 图表型。 分 析: (1)利用列表法或者画出树状图,然后写出所有的可能情况即可; (2)分别求出“至少有一次是“6””和“卡片上的数字是球上数字的整数倍”的概率,小 红选择自己获胜的概率比小莉获胜的概率大的一种规则即可在游戏中获胜. 解 答: 解:(1)列表如下: 画树状图如下: 共有 9 种可能,分别是(2,6),(2,7),(2,8),(4,6),(4,7),(4,8),(6, 6),(6,7),(6,8); (2)从图表或树状图可知,至少有一次是“6”的情况有 5 种, 所以,小红赢的概率是 P(至少有一次是“6”)= , 小莉赢的概率是 , ∵ > , ∴此规则小红获胜的概率大, 卡片上的数字是球上数字的整数倍的有:(2,6)(2,8)(4,8)(6,6)共 4种情 况, 所以,小红赢的概率是 P(卡片上的数字是球上数字的整数倍)= , 小莉赢的概率是 , ∵ > , ∴此规则小莉获胜的概率大, ∴小红要想在游戏中获胜,她应该选择规则 1. 点 评: 本题考查了列表法或树状图法,用到的知识点为:概率=所求情况数与总情况数之 比. ] 21.(10 分)(2012•贵阳)如图,在正方形 ABCD 中,等边三角形 AEF 的顶点 E、F 分别 在 BC 和 CD 上. (1)求证:CE=CF; (2)若等边三角形 AEF 的边长为 2,求正方形 ABCD 的周长. 考 点: 正方形的性质;全等三角形的判定与性质;等边三角形的性质;等腰直角三角形。 190187 分 析: (1)根据正方形可知 AB=AD,由等边三角形可知 AE=AF,于是可以证明出 △ABE≌△ADF,即可得出 CE=CF; (2)连接 AC,交 EF 与 G 点,由三角形 AEF 是等边三角形,三角形 ECF 是等腰直 角三角形,于是可知 AC⊥EF,求出 EG=1,设 BE=x,利用勾股定理求出 x,即可 求出 BC 的上,进而求出正方形的周长. 解 答: (1)证明:∵四边形 ABCD 是正方形, ∴AB=AD, ∵△AEF 是等边三角形, ∴AE=AF, 在 Rt△ABE 和 Rt△ADF 中, ∵ , ∴Rt△ABE≌Rt△ADF, ∴CE=CF, (2)解:连接 AC,交 EF 于 G 点, ∵△AEF 是等边三角形,△ECF 是等腰直角三角形, ∴AC⊥EF, 在 Rt△AGE 中,EG=sin30°AE= ×2=1, ∴EC= , 设 BE=x,则 AB=x+ , 在 Rt△ABE 中,AB2+BE2=AE2,即(x+ )2+x2=4, 解得 x= , ∴AB= + = , ∴正方形 ABCD 的周长为 4AB=2( + ). 点 评: 本题考查了正方形的性质,全等三角形的判定与性质,等边三角形的性质和等腰三 角形的性质,解答本题的关键是对正方形和三角形的性质的熟练运用,此题难度不 大,是一道比较不错的试题. 22.(10 分)(2012•贵阳)已知一次函数 y= x+2 的图象分别与坐标轴相交于 A、B 两点 (如图所示),与反比例函数 y= (x>0)的图象相交于 C 点. (1)写出 A、B 两点的坐标; (2)作 CD⊥x 轴,垂足为 D,如果 OB 是△ACD 的中位线,求反比例函数 y= (x>0) 的关系式. 考 点: 反比例函数与一次函数的交点问题;一次函数图象上点的坐标特征;待定系数法求 反比例函数解析式;三角形中位线定理。 190187 专 题: 计算题。 分 析: (1)分别把 x=0 和 y=0 代入一次函数的解析式,即可求出 A、B 的坐标; (2)根据三角形的中位线求出 OA=OD=3,即可得出 D、C 的横坐标是 3,代入一 次函数的解析式,求出 C 的坐标,代入反比例函数的解析式,求出 k 即可. 解 答: 解:(1)∵y= x+2, ∴当 x=0 时,y=2, 当 y﹣0 时,x=﹣3, ∴A 的坐标是(﹣3,0),B 的坐标是(0,2). (2)∵A(﹣3,0), ∴OA=3, ∵OB 是△ACD 的中位线, ∴OA=OD=3, 即 D 点、C 点的横坐标都是 3, 把 x=3 代入 y= x+2 得:y=2+2=4, 即 C 的坐标是(3,4), ∵把 C 的坐标代入 y= 得:k=3×4=12, ∴反比例函数 y= (x>0)的关系式是 y= . 点 评: 本题考查了一次函数与反比例函数的交点问题,用待定系数法求反比例函数的解析 式,一次函数图象上点的坐标特征等知识点的应用,主要考查学生运用性质进行计 算的能力,题目比较典型,具有一定的代表性. 23.(10 分)(2012•贵阳)如图,在⊙O 中,直径 AB=2,CA 切⊙O 于 A,BC 交⊙O 于 D,若∠C=45°,则(1)BD 的长是 ;(2)求阴影部分的面积. 考 点: 切线的性质;圆周角定理;扇形面积的计算。 190187 分 析: (1)连接 AD,由于 AC 是⊙O 的切线,所以 AB⊥AC,再根据∠C=45°可知 AB=AC=2,由勾股定理可求出 BC 的长,由于 AB 是⊙O 的直径,所以 ∠ADB=90°,故 D 是 BC 的中点,故可求出 BD 的长度; (2)连接 OD,因为 O 是 AB 的中点,D 是 BC 的中点,所以 OD 是△ABC 的中位 线,所以 OD⊥AB,故 = ,所以 与弦 BD 组成的弓 形的面积等于 与弦 AD 组成的弓形的面积,所以 S 阴影=S△ABC ﹣S△ABD,故可得出结理论. 解 答: 解:(1)连接 AD, ∵AC 是⊙O 的切线, ∴AB⊥AC, ∵∠C=45°, ∴AB=AC=2, ∴BC= = =2 , ∵AB 是⊙O 的直径, ∴∠ADB=90°, ∴D 是 BC 的中点, ∴BD= BC= ; (2)连接 OD, ∵O 是 AB 的中点,D 是 BC 的中点, ∴OD 是△ABC 的中位线, ∴OD=1, ∴OD⊥AB, ∴ = , ∴ 与弦 BD 组成的弓形的面积等于 与弦 AD 组成的弓形的面积, ∴S 阴影=S△ABC﹣S△ABD= AB•AC﹣ AB•OD= ×2×2﹣ ×2×1=2﹣1=1. 点 评: 本题考查的是切线的性质,涉及到三角形的面积、等腰三角形的性质及三角形中位 线定理、圆周角定理,根据题意作出辅助线,构造出直角三角形是解答此题的关 键. 24.(12 分)(2012•贵阳)如果一条直线把一个平面图形的面积分成相等的两部分,我们 把这条直线称为这个平面图形的一条面积等分线. (1)三角形有 3 条面积等分线,平行四边形有 无数 条面积等分线; (2)如图①所示,在矩形中剪去一个小正方形,请画出这个图形的一条面积等分线; (3)如图②,四边形 ABCD 中,AB 与 CD 不平行,AB≠CD,且 S△ABC<S△ACD,过点 A 画出四边形 ABCD 的面积等分线,并写出理由. 考 点: 面积及等积变换;平行线之间的距离;三角形的面积;平行四边形的性质;矩形的 性质。190187 分 析: (1)读懂面积等分线的定义,不难得出:一定是三角形的面积等分线的是三角形的 中线所在的直线;平行四边形的一条对角线所在的直线就是平行四边形的一条面积 等分线; (2)由(1)知,矩形的一条对角线所在的直线就是矩形的一条面积等分线; (3)能.过点 B 作 BE∥AC 交 DC 的延长线于点 E,连接 AE.根据“△ABC 和 △AEC 的公共边 AC 上的高也相等”推知 S△ABC=S△AEC;然后由“割补法”可以求得 S 四边形 ABCD=S△ACD+S△ABC=S△ACD+S△AEC=S△AED. 解 答: 解:(1)根据“面积等分线”的定义知,对于三角形,一定是三角形的面积等分线的 是三角形的中线所在的直线;对于平行四边形应该有无数条,只要过两条对角线的 交点的直线都可以把平行四边形的面积分成 2 个相等的部分; 故答案是:6;无数; (2)如图①所示:连接2 个矩形的对角线的交点的直线即把这个图形分成 2 个相等 的部分.即 OO′为这个图形的一条面积等分线; (3)如图②所示.能,过点 B 作 BE∥AC 交 DC 的延长线于点 E,连接 AE. ∵BE∥AC, ∴△ABC 和△AEC 的公共边 AC 上的高也相等, ∴有 S△ABC=S△AEC, ∴S 四边形 ABCD=S△ACD+S△ABC=S△ACD+S△AEC=S△AED; ∵S△ACD>S△ABC, 所以面积等分线必与 CD 相交,取 DE 中点 F,则直线 AF 即为要求作的四边形 ABCD 的面积等分线. 点 评: 本题考查了学生的阅读理解能力、运用作图工具的能力,以及运用三角形、等底等 高性质等基础知识解决问题的能力都有较高的要求.还渗透了由“特殊”到“一般”的数 学思想. 25.(12 分)(2012•贵阳)如图,二次函数 y= x2﹣x+c 的图象与 x 轴分别交于 A、B 两 点,顶点 M 关于 x 轴的对称点是 M′. (1)若 A(﹣4,0),求二次函数的关系式; (2)在(1)的条件下,求四边形 AMBM′的面积; (3)是否存在抛物线 y= x2﹣x+c,使得四边形 AMBM′为正方形?若存在,请求出此抛物 线的函数关系式;若不存在,请说明理由. 考 点: 二次函数综合题。 190187 专 题: 综合题。 分 析: (1)把点 A 的坐标代入二次函数解析式,计算求出 c 的值,即可得解; (2)把二次函数解析式整理成顶点式解析式,根据二次函数的对称性求出点 B 的坐 标,从而求出 AB 的长,再根据顶点坐标求出点 M 到 x 轴的距离,然后求出△ABM 的面积,根据对称性可得 S 四边形 AMBM′=2S△ABM,计算即可得解; (3)令 y=0,得到关于 x 的一元二次方程,利用根与系数的关系求出 AB 的长度, 根据抛物线解析式求出顶点 M 的纵坐标,然后根据正方形的对角线互相垂直平分且 相等列式求解,如果关于 c 的方程有解,则存在,否则不存在. 解 答: 解:(1)∵A(﹣4,0)在二次函数 y= x2﹣x+c 的图象上, ∴ ×(﹣4)2﹣(﹣4)+c=0, 解得 c=﹣12, ∴二次函数的关系式为 y= x2﹣x﹣12; (2)∵y= x2﹣x﹣12, = (x2﹣2x+1)﹣ ﹣12, = (x﹣1)2﹣ , ∴顶点 M 的坐标为(1,﹣ ), ∵A(﹣4,0),对称轴为 x=1, ∴点 B 的坐标为(6,0), ∴AB=6﹣(﹣4)=6+4=10, ∴S△ABM= ×10× = , ∵顶点 M 关于 x 轴的对称点是 M′, ∴S 四边形 AMBM′=2S△ABM=2× =125; (3)存在抛物线 y= x2﹣x﹣ ,使得四边形 AMBM′为正方形. 理由如下:令 y=0,则 x2﹣x+c=0,设点 AB 的坐标分别为 A(x1,0)B(x2,0), 则 x1+x2=﹣ =2,x1•x2= =2c, 所以,AB= = , 点 M 的纵坐标为: = = , ∵顶点 M 关于 x 轴的对称点是 M′,四边形 AMBM′为正方形, ∴ =2× , 整理得,4c2+4c﹣3=0, 解得 c1= ,c2=﹣ , 又抛物线与 x 轴有两个交点, ∴△=b2﹣4ac=(﹣1)2﹣4× c>0, 解得 c< , ∴c 的值为﹣ , 故,存在抛物线 y= x2﹣x﹣ ,使得四边形 AMBM′为正方形. 点 评: 本题综合考查了二次函数的问题,主要利用了待定系数法求函二次数解析式,二次 函数的顶点坐标的求解,二次函数的对称性,以及正方形的对角线互相垂直平分且 相等的性质,综合题,但难度不是很大,(3)中要注意根据抛物线与 x 轴有两个交 点,利用根的判别式求出 c 的取值范围,否则容易多解而导致出错. 2012 年乌鲁木齐市初中毕业生学业水平测试数学试卷分析 注意事项: 1、每到题目之后标有答案解析,蓝色部分为知识点分析 2、题目之前的五角星代表题目难度系数,五角星越多难度等级越高,最高等级为四颗 星 一、选择题(本大题共 10 小题,每小题 4 分,共 40 分)每题的选项中只有一项符合 题目要求. 1、(2012 新疆乌鲁木齐,1,4 分)8 的立方根是( ) A、2 B、-2 C、±2 D、 22 【答案】A 考点:立方根概念 2、(2012 新疆乌鲁木齐,2,4 分)数据 8,7,6,5,7,8,8 的中位数与众数分别 是( ) A、5,7 B、5,8 C、7,7 D、7,8 【答案】D 考点:中位数与众数的概念,注意对数的按从小到大重新排列 3、(2012 新疆乌鲁木齐,3,4 分)如图是某几何体的三视图,其侧面积是( ) A、8 B、4 C、2 D、4 【答案】B 考点:根据三视图确定立体图形的形状以及圆柱侧面积的计算。 4、(2012 新疆乌鲁木齐,4,4 分)在一个不透明的口袋里,装有仅颜色不同的黑 球、白球若干只,某小组做摸球实验:将球搅匀后从中随机摸出一个,记下颜色, 再放入袋中,不断重复,右表是活动中的一组数据,则摸到白球的概率约是( ) A、0.4 B、0.5 C、0.6 D、0.7 【答案】C 考点:概率的估计 5、(2012 新疆乌鲁木齐,5,4 分)图(1)是边长为(a+b)的正方形,将图(1) 中的阴影部分拼成图(2)的形状,由此能验证的式子是( ) A、(a+b)(a-b)=a2-b2 B、(a+b)2-(a2+b2)=2ab C、(a+b)2-(a-b)2=4ab D、(a-b)2+2ab=a2+b2 【答案】B 考点:考察数形结合的思想,由上图可知大正方形的面积为(a+b)2 ,小 正方形的面积为(a2+b2),右边菱形的两条对角线长度分别为 2a,2b,所以菱形的面积为 2ab 6、(2012 新疆乌鲁木齐,6,4 分)函数 x ky 12  (k 为常数)的图象过点(2, y1)和( 5 ,y2),则 y1 与 y2 的大小关系是( ) A、y1y2 D、与 k 的取值有关 【答案】A 考点:反比例函数图像与性质 7、(2012 新疆乌鲁木齐,7,4 分)为使我市冬季“天更蓝、房更暖”、政府决定实 施“煤改气”供暖改造工程,现甲、乙两工程队分别同时开挖两条 600 米长的管 道,所挖管道长度 y(米)与挖掘时间 x(天)之间的关系如图所示,则下列说法 中:①甲队每天挖 100 米;②乙队开挖两天后,每天挖 50 米;③当 x=4 时,甲、 乙两队所挖管道长度相同;④甲队比乙队提前 2 天完成任务. 正确的个数有( ) A、1 个 B、2 个 C、3 个 D、4 个 【答案】D 考点:一次函数图像的应用 8、(2012 新疆乌鲁木齐,8,4 分)如图是一张足够长的矩形纸条 ABCD,以点 A 所 在直线为折痕,折叠纸条,使点 B 落在边 AD 上,折痕与边 BC 交于点 E;然后将其 展平,再以点 E 所在直线为折痕,使点 A 落在边 BC 上,折痕 EF 交边 AD 于点 F.则 ∠AFE 的大小是( ) A、22.50 B、450 C、600 D、67.50 【答案】D 考点:对称轴的性质和平行线的性质,解决本题的关键是能理解题意并 根据题意做出符合条件的图形 9、(2012 新疆乌鲁木齐,9,4 分)古希腊数学家把 1,3,6,10,15,……叫做三 角形数,则第 16 个三角形数与第 14 个三角形数的差是( ) A、30 B、31 C、32 D、33 【答案】B 考点:数字规律探索能力,有关三角形数的性质与特征。 10、(2012 新疆乌鲁木齐,10,4 分)如图,AD∥BC,∠D=900,AD=2,BC=5,DC= 8.若在边 DC 上有点 P,使△PAD 与△PBC 相似,则这样的点 P 有( ) A、1 个 B、2 个 C、3 个 D、4 个 【答案】C 考点:相似三角形的性质与判定以及分类讨论 二、填空题(本大题共 5 小题,每小题 4 分,共 20 分)把答案直接填在答题卡的相应 位置处. 11、(2012 新疆乌鲁木齐,11,4 分)如图,直线 a∥b,则∠ = °. 【答案】153 考点:平行的性质,两直线平行同位角相等 12、(2012 新疆乌鲁木齐,12,4 分)分解因式 x3-x= . 【答案】x(x+1)( x-1) 考点:提公因式法与公式法分解因式的综合应用,先提取公 因式再使用平方差公式继续分解。 13、(2012 新疆乌鲁木齐,13,4 分)如图,在周长为 20 的□ABCD 中,AB S2 B.S1 < S2 C .S1 = S2 D.2S1 = S2 7.不等式组  5x 1 3 x+1 1 3x 7 x2 2 >     的解集是【 】 A .x > 2 B .x≤4 C.x < 2 或 x≥4 D .2 < x≤4 [来源:学科网 ZXXK] 8.圆锥的底面直径是 80cm,母线长 90cm,则它的侧面展开图的圆心角是【 】 A .3200 B.400 C .1600 D.800 9.随机掷一枚质地均匀的正方体骰子,骰子的六个面上分别刻有 1 到 6 的点数,掷两 次骰子,掷得面朝上的点数之和是 5 的概率是【 】 A . 1 6 B. 1 9 C. 1 18 D . 2 15 10 .已知下列命题: ① 若 a≤0 ,则 lal =一 a ;② 若 ma2 > na2 ,则 m > n ; ③ 两组对角分别相等的四边形是平行四边形;④ 垂直于弦的直径平分弦. 其中原命题与逆命题均为真命题的个数是【 】 A.1 个 B .2 个 C.3 个 D .4 个 11.在矩形 ABCD 中,点 O 是 BC 的中点,∠AOD=900,矩形 ABCD 的周长为 20cm,则 AB 的长为【 】 A.1 cm B. 2 cm C. 5 2 cm D . 10 3 cm 12.关于 x 的一元二次方程  2x mx+5 m 5 =0  的两个正实数根分别为 x1,x2,且 2x1+x2=7,则 m 的值是【 】 A.2 B. 6 C. 2 或 6 D . 7。 第 II 卷(非选择题 共 84 分)[来源:Z,xx,k.Com] 二、填空题(本大题共 8 小题,每小题 3 分,共 24 分.请把答案填在题中横线上) 13.计算:  01 8+ 3 1 2+1   = ▲ 。 l4.化简: 2 2 a 2 a 1 a 4 a+2a +2a a +4a+4        ▲ 。 15.某校六个绿化小组一天植树的棵数如下:10 , 11 , 12 , 13 , 8 , x .若这组数据的平 均数是 11,则这组数据的众数是 ▲ 。 16.关于 x 的两个方程 2x x 2 0   与 1 2=x+1 x+a 有一个解相同,则 a= ▲ 。 17.如图,△ABC 内接于⊙O,∠BAC=600,⊙O 的半径为 2 ,则 BC 的长为 ▲ (保留根号)。 18.如图,在平面直角坐标系中,点 A 在 x 上,△ABO 是直角三角形,∠ABO=900,点 B 的坐标为(-1,2),将△ABO 绕原点 O 顺时针旋转 900,得到△Al BlO,则过 A1, B 两点的直线解析式为 ▲ 。[来源:Zxxk.Com] 19.如图,直线 1y= x 22  与 x 轴、y 轴分别交于点 A 和点 B ,点 C 在直线 AB 上,且点 C 的纵坐标为一 1 ,点 D 在反比例函数 ky= x 的图象上 ,CD 平行于 y 轴, OCD 5S 2  则 k 的值为 ▲ 。 20.如图,将△ABC 纸片的一角沿 DE 向下翻折,使点 A 落在 BC 边上的 A ′点处,且 DE ∥BC ,下列结论: ① ∠AED=∠C;② A D A E DB EC   ;③ BC= 2DE ;④ BD A E A CAD A ES S S     四 形边 。 其中正确结论的个数是 ▲ 个。 三、解答题(本大题共 6 小题,共 60 分.解答应写出必要的文字说明、证明过程或演 算步骤) 21.某年级组织学生参加夏令营活动,本次夏令营活动分为甲、乙、丙三组进行.下面 条形统计图和扇形统计图反映了学生参加夏令营活动的报名情况,请你根据图中的信 息回答下列问题: (1)该年级报名参加本次活动的总人数为 人,报名参加乙组的人数为 人: (2)补全条形统计图中乙组的空缺部分; (3)根据实际情况。需从甲组抽调部分学生到丙组,使丙组人数是甲组人数的 3 倍。应从甲组抽调多少名学生到丙组? 22 .如图,拦水坝的横断面为梯形 ABCD ,坝顶宽 AD = 5 米,斜坡 AB 的 坡度 i =1:3 (指坡面的铅直高度 AE 与水平宽度 BE 的比),斜坡 DC 的坡度 i=1:1 . 5 ,已知该拦水坝的高为 6 米。 (1)求斜坡 AB 的长; (2)求拦水坝的横断面梯形 ABCD 的周长。 (注意:本题中的计算过程和结果均保留根号) 23 .某商场用 3600 元购进甲、乙两种商品,销售完后共获利 6000 元.其中甲种商品 每件进价 120 元,售价 138 元;乙种商品每件进价 100 元,售价 120 元。 (1)该商场购进甲、乙两种商品各多少件? (2)商场第二次以原进价购进甲、乙两种商品。购进乙种商品的件数不变,而购进 甲种商品的件数是第一次的 2 倍,甲种商品按原售价出售,而乙种商品打折销售。若 两种商品销售完毕,要使第二次经营活动获利不少于 8160 元,乙种商品最低售价为每 件多少元? 24 .如图,已知 AB 为⊙O 的直径,过⊙O 上的点 C 的切线交 AB 的延长线于点 E , AD⊥ EC 于点 D 且交⊙O 于点 F ,连接 BC , CF , AC 。 (1)求证:BC=CF; (2)若 AD=6 , DE=8 ,求 BE 的长; (3)求证:AF + 2DF = AB。 25 .如图,在 Rt△ABC 中,∠C =900,AC = 4cm , BC = 5 cm,点 D 在 BC 上,且 CD = 3 cm ,现有两个动点 P,Q 分别从点 A 和点 B 同时出发,其中点 P 以 1 厘米/秒的 速度沿 AC 向终点 C 运动;点 Q 以 1 . 25 厘米/秒的速度沿 BC 向终点 C 运动.过点 P 作 PE∥ BC 交 AD 于点 E ,连接 EQ。设动点运动时间为 t 秒(t > 0 )。 (1)连接 DP ,经过 1 秒后,四边形 EQDP 能够成为平行四边形吗?请说明理 由; (2)连接 PQ ,在运动过程中,不论 t 取何值时,总有线段 PQ 与线段 AB 平行。 为什么? (3)当 t 为何值时,△EDQ为直角三角形。 26.已知直线 y = 2x + 4 与 x 轴、y 轴分别交于 A , D 两点,抛物线 21y= x +bx+c2  经 过点 A , D ,点 B 是抛物线与 x 轴的另一个交点。 (1)求这条抛物线的解析式及点 B 的坐标; (2)设点 M 是直线 AD 上一点,且 AOM OMDS : S 1 : 3   ,求点 M 的坐标; (3)如果点 C(2,y)在这条抛物线上,在 y 轴的正半轴上是否存在点 P,使△BCP 为等腰三角形?若存在,请求出点 P 的坐标;若不存在,请说明理由。 2012 年中考数学试题(内蒙古包头) (本试卷满分 150 分,考试时间 120 分钟) 第 I 卷(选择题 共 36 分) 一、选择题(本大题共 12 小题,每小题 3 分,共 36 分.在每小题给出的四个选项中,只 有一项是符合题目要求的) 1. 9 的算术平方根是【 】 A .土 3 B.3 C..一 3 D . 3 【答案】B。 2.联合国人口基金会的报告显示,世界人口总数在 2011 年 10 月 31 日达到 70 亿.将 70 亿 用科学记数法表示为【 】 A .7×109 B . 7×108 C . 70×108 D . 0.7×1010 【答案】A。 3.下列运算中,正确的是【 】 A . 3 2x x =x B . 6 2 3x x =x C. 2+ 3= 5 D . 2 3= 6 【答案】D。 4.在 Rt △ ABC 中,∠C=900,若 AB =2AC ,则 sinA 的值是【 】 A . 3 B . 1 2 C. 3 2 D. 3 3 【答案】C。 5.下列调查中,调查方式选择正确的是【 】 A .为了了解 1000 个灯泡的使用寿命,选择全面调查 B .为了了解某公园全年的游客流量,选择抽样调查 C .为了了解生产的一批炮弹的杀伤半径,选择全面调查 D .为了了解一批袋装食品是否含有防腐剂,选择全面调查 【答案】B。 6.如图,过口 ABCD 的对角线 BD 上一点 M 分别作平行四边形两边的平行线 EF 与 GH ,那么图中的口 AEMG 的面积 S1 与口 HCFG 的面积 S2 的大小关系是【 】 A .S1 > S2 B.S1 < S2 C .S1 = S2 D.2S1 = S2 【答案】C。 7.不等式组  5x 1 3 x+1 1 3x 7 x2 2 >     的解集是【 】 A .x > 2 B .x≤4 C.x < 2 或 x≥4 D .2 < x≤4 【答案】D。[来源:学科网 ZXXK] 8.圆锥的底面直径是 80cm,母线长 90cm,则它的侧面展开图的圆心角是【 】 A .3200 B.400 C .1600 D.800 【答案】C。 9.随机掷一枚质地均匀的正方体骰子,骰子的六个面上分别刻有 1 到 6 的点数,掷两 次骰子,掷得面朝上的点数之和是 5 的概率是【 】 A . 1 6 B. 1 9 C. 1 18 D . 2 15 【答案】B。 10 .已知下列命题: ① 若 a≤0 ,则 lal =一 a ; ② 若 ma2 > na2 ,则 m > n ; ③ 两组对角分别相等的四边形是平行四边形; ④ 垂直于弦的直径平分弦. 其中原命题与逆命题均为真命题的个数是【 】 A.1 个 B .2 个 C.3 个 D .4 个 【答案】B。 11.在矩形 ABCD 中,点 O 是 BC 的中点,∠AOD=900,矩形 ABCD 的周长为 20cm,则 AB 的长为【 】 A.1 cm B. 2 cm C. 5 2 cm D . 10 3 cm 【答案】 D。 12.关于 x 的一元二次方程  2x mx+5 m 5 =0  的两个正实数根分别为 x1,x2,且 2x1+x2=7,则 m 的值是【 】 A.2 B. 6 C. 2 或 6 D . 7 【答案】B。 第 II 卷(非选择题 共 84 分)[来源:Z,xx,k.Com] 二、填空题(本大题共 8 小题,每小题 3 分,共 24 分.请把答案填在题中横线上) 13.计算:  01 8+ 3 1 2+1   = ▲ 。 【答案】 2 。 l4.化简: 2 2 a 2 a 1 a 4 a+2a +2a a +4a+4        ▲ 。 【答案】 2 1 a +2a 。 15.某校六个绿化小组一天植树的棵数如下:10 , 11 , 12 , 13 , 8 , x .若这组数据的平 均数是 11,则这组数据的众数是 ▲ 。 【答案】12。 16.关于 x 的两个方程 2x x 2 0   与 1 2=x+1 x+a 有一个解相同,则 a= ▲ 。 【答案】4。 17.如图,△ABC 内接于⊙O,∠BAC=600,⊙O 的半径为 2 ,则 BC 的长为 ▲ (保留根号)。 【答案】 2 3 。 18.如图,在平面直角坐标系中,点 A 在 x 上,△ABO 是直角三角形,∠ABO=900,点 B 的坐标为(-1,2),将△ABO 绕原点 O 顺时针旋转 900,得到△Al BlO,则过 A1, B 两点的直线解析式为 ▲ 。[来源:Zxxk.Com] 【答案】y=3x+5。 19.如图,直线 1y= x 22  与 x 轴、y 轴分别交于点 A 和点 B ,点 C 在直线 AB 上,且点 C 的纵坐标为一 1 ,点 D 在反比例函数 ky= x 的图象上 ,CD 平行于 y 轴, OCD 5S 2  则 k 的值为 ▲ 。 【答案】3。 20.如图,将△ABC 纸片的一角沿 DE 向下翻折,使点 A 落在 BC 边上的 A ′点处,且 DE ∥BC ,下列结论: ① ∠AED=∠C; ② A D A E DB EC   ; ③ BC= 2DE ; ④ BD A E A CAD A ES S S     四 形边 。 其中正确结论的个数是 ▲ 个。 【答案】4。 三、解答题(本大题共 6 小题,共 60 分.解答应写出必要的文字说明、证明过程或演 算步骤) 21.某年级组织学生参加夏令营活动,本次夏令营活动分为甲、乙、丙三组进行.下面 条形统计图和扇形统计图反映了学生参加夏令营活动的报名情况,请你根据图中的信 息回答下列问题: (1)该年级报名参加本次活动的总人数为 人,报名参加乙组的人数为 人: (2)补全条形统计图中乙组的空缺部分; (3)根据实际情况。需从甲组抽调部分学生到丙组,使丙组人数是甲组人数的 3 倍。应从甲组抽调多少名学生到丙组? 【答案】解:(1)60,12。 (2)补全条形统计图如下: (3)设应从甲组抽调 x 名学生到丙组,可得方程 30 + x = 3 ( 18 一 x ) ,解 得 x = 6 . 答:应从甲组抽调 6 名学生到丙组。 22 .如图,拦水坝的横断面为梯形 ABCD ,坝顶宽 AD = 5 米,斜坡 AB 的 坡度 i =1:3 (指坡面的铅直高度 AE 与水平宽度 BE 的比),斜坡 DC 的坡度 i=1:1 . 5 ,已知该拦水坝的高为 6 米。 (1)求斜坡 AB 的长; (2)求拦水坝的横断面梯形 ABCD 的周长。 (注意:本题中的计算过程和结果均保留根号) 【答案】解:(1)∵ AE 1=BE 3 ,AE=6,∴BE=3AD=18。 在 Rt△ABE 中,根据勾股定理得, 2 2AB AE BE 6 10   。 答:斜坡 AB 的长为 6 10 米。 (2)过点 D 作 DF⊥BC 于点 F, ∴ 四 边 形 AEFD 是 矩 形。 ∴EF=AD。 ∵AD=5,∴EF=5。 又∵ DF 2=CF 3 , DF=AE=6,∴CF= 3 2 DF=9。 ∴BC=BE+EF+CF=18+5+9=32。 在 Rt△DCF 中,根据勾股定理得, 2 2DC DF CF 3 13   。 ∴ 梯 形 ABCD 的 周 长 为 AB + BC + CD + DA= 6 10+32+3 13+5 37+6 10+3 13 。 答:拦水坝的横断面梯形 ABCD 的周长为 37+6 10+3 13 米。 23 .某商场用 3600 元购进甲、乙两种商品,销售完后共获利 6000 元.其中甲种商品 每件进价 120 元,售价 138 元;乙种商品每件进价 100 元,售价 120 元。 (1)该商场购进甲、乙两种商品各多少件? (2)商场第二次以原进价购进甲、乙两种商品。购进乙种商品的件数不变,而购进 甲种商品的件数是第一次的 2 倍,甲种商品按原售价出售,而乙种商品打折销售。若 两种商品销售完毕,要使第二次经营活动获利不少于 8160 元,乙种商品最低售价为每 件多少元? 【答案】解:(1)设商场购进甲种商品 x 件,乙种商品 y 件,根据题意得:     120x 100y 3600 138 120 x 120 100 y 6000       ,解得, x 200 y 120    。 答:该商场购进甲种商品 200 件,乙种商品 120 件。 (2)设乙种商品每件售价 z 元,根据题意,得 120(z-100)+2×200×(138-120)≥8160, 解得:z≥108。 答:乙种商品最低售价为每件 108 元。 24 .如图,已知 AB 为⊙O 的直径,过⊙O 上的点 C 的切线交 AB 的延长线于点 E , AD⊥ EC 于点 D 且交⊙O 于点 F ,连接 BC , CF , AC 。 (1)求证:BC=CF; (2)若 AD=6 , DE=8 ,求 BE 的长; (3)求证:AF + 2DF = AB。 【答案】解:(1)证明:如图,连接 OC, ∵ED 切⊙O 于点 C,∴CO⊥ED。 ∵AD⊥EC,∴CO∥AD。∴∠OCA=∠OCA。 ∴∠OAC=∠CAD。∴  BC CF 。∴BC=CF。 (2)在 Rt△ADE 中,∵AD=6,DE=8, 根据勾股定理得 AE=10。 ∵CO∥AD,∴△EOC∽△EAD。∴ EO OC EA AD  。 设⊙O 的半径为 r,∴OE=10+r,∴10 r r 10 6   。∴r=15 4 。 ∴BE=10-2r= 5 2 。 (3)证明:过 C 作 CG⊥AB 于 G,[来源:Z|xx|k.Com] ∵∠OAC=∠CAD,AD⊥EC,∴CG=CD。 在 Rt△AGC 和 Rt△ADC 中, ∵CG=CD,AC=AC,∴Rt△AGC≌Rt△ADC(HL)。 ∴AG=AD。 在 Rt△CGB 和 Rt△CDF 中, ∵BC=FC ,CG=CD,∴Rt△CGB≌Rt△CDF(HL)。∴GB=DF。 ∵AG+GB=AB,∴AD+DF=AB。∴AF+2DF=AB。 25 .如图,在 Rt△ABC 中,∠C =900,AC = 4cm , BC = 5 cm,点 D 在 BC 上,且 CD = 3 cm ,现有两个动点 P,Q 分别从点 A 和点 B 同时出发,其中点 P 以 1 厘米/秒的 速度沿 AC 向终点 C 运动;点 Q 以 1 . 25 厘米/秒的速度沿 BC 向终点 C 运动.过点 P 作 PE∥ BC 交 AD 于点 E ,连接 EQ。设动点运动时间为 t 秒(t > 0 )。 (1)连接 DP ,经过 1 秒后,四边形 EQDP 能够成为平行四边形吗?请说明理 由; (2)连接 PQ ,在运动过程中,不论 t 取何值时,总有线段 PQ 与线段 AB 平行。 为什么? (3)当 t 为何值时,△EDQ为直角三角形。 【答案】解:(1)不能。理由如下: 假设经过 t 秒时四边形 EQDP 能够成为平行四边形。 ∵点 P 的速度为 1 厘米/秒,点 Q 的速度为 1 . 25 厘米/秒, ∴AP=t 厘米,BQ=1.25t 厘米。 又∵PE∥BC,∴△AEP∽△ADC。∴ EP AP DC AC  。 ∵AC=4 厘米,BC=5 厘米,CD=3 厘米, ∴ EP t 3 4  ,解得,EP=0.75t 厘米。 又∵ 5QD BC BQ DC 5 t 3 2 1.25t4         , ∴由 EP=QD 得 2 1.25t=0.75t ,解得 t=1。 ∴只有 t=1时四边形 EQDP 才能成为平行四边形。 ∴经过 1 秒后,四边形 EQDP 不能成为平行四边形。 (2)∵AP=t 厘米,BQ=1.25t 厘米,AC=4 厘米,BC=5 厘米, ∴ PC 4 t QC 5 1.25t 4 t AC 4 BC 5 4      , 。∴ PC QC AC BC  。 又∵∠C=∠C,∴△PQC∽△ABC。∴∠PQC=∠B。∴PQ∥AB。 ∴在运动过程中,不论 t 取何值时,总有线段 PQ 与线段 AB 平行。 (3)分两种情况讨论: ①当∠EQD=90°时,显然有 EQ=PC=4-t,DQ=1.25t-2 又∵EQ∥AC,∴△EDQ∽△ADC。 ∴ EQ DQ AC DC  ,即 4 t 1.25t 2 4 3   , 解得 t=2.5 。 ②当∠QED=90°时, ∵ ∠ CDA= ∠ EDQ , ∠ QED= ∠ C=90° , ∴ △ EDQ ∽ △ CDA。 ∴ DQ Rt EDQ DA Rt CDA   斜 上的高 斜 上的高 边 边 。 Rt△EDQ 斜边上的高为 4-t,Rt△CDA 斜边上的高为 2.4, ∴1.25t 2 4 t 5 2.4   ,解得 t =3.1。 综上所述,当 t 为 2.5 秒或 3.1 秒时,△EDQ 为直角三角形。 26.已知直线 y = 2x + 4 与 x 轴、y 轴分别交于 A , D 两点,抛物线 21y= x +bx+c2  经 过点 A , D ,点 B 是抛物线与 x 轴的另一个交点。 (1)求这条抛物线的解析式及点 B 的坐标; (2)设点 M 是直线 AD 上一点,且 AOM OMDS : S 1 : 3   ,求点 M 的坐标; (3)如果点 C(2,y)在这条抛物线上,在 y 轴的正半轴上是否存在点 P,使△BCP 为等腰三角形?若存在,请求出点 P 的坐标;若不存在,请说明理由。 【答案】解:(1)在 y = 2x + 4 中,令 y =0,得 x=-2;令 x=0,得 y =4。 ∴A(-2,0),D(0,4)。 将 A(-2,0),D(0,4)代入 21y= x +bx+c2  ,得 1 4 2b+c=02 c=4     ,解得 b=1 c=4    。 ∴这条抛物线的解析式为 21y= x +x+42  。 令 21y= x +x+4=02  ,解得 1 2x = 2 x =4 , 。∴B(4,0)。 (2)设 M(m,2 m + 4),分两种情况: ①当 M 在线段 AD 上时,由 AOM OMDS : S 1 : 3   得    1 12 2m+2 : 4 m 1 : 32 2               , 解得, 3m 2   。∴M1( 3 12  , )。 ②当 M 在线段 DA 延长线上时, 由 AOM OMDS : S 1 : 3   得    1 12 2m+2 : 4 m 1 : 32 2                , 解 得 m 3  。 ∴ M2 ( 3 4  , )。 综上所述,点 M 的坐标为 M1( 3 12  , ),M2( 3 4  , )。 (3)存在。 ∵点 C(2,y)在 21y= x +x+42  上, ∴ 21y= 2 +2+4=42   。∴C(2,4)。 设 P 0, p ,根据勾股定理,得  22 2BC 4 2 +4 20   ,[来源:学科网] 2 2 2 2PB 4 +p 16+p  ,  22 2 2PC 2 + p 4 p 8p+20    。 分三种情况: ①若 PB=BC,则 216+p 20 ,解得, p 2  。 ∵点 P 在 y 轴的正半轴上,∴P1(0,2)。 ②若 PB=PC,则 2 216+p p 8p+20  ,解得, 1p 2  。∴P2(0, 1 2 )。 ③若 BC=PC,则 220 p 8p+20  ,解得, p 0 p 8 或 。 ∵点 P 在 y 轴的正半轴上,∴ p 0 不符合要求。 当 p 8 时,B、C、P 在一直线上,不构成三角形,也不符合要求。 ∴BC=PC 时,在 y 轴的正半轴上是不存在点 P,使△BCP 为等腰三角 形。 综上所述,在 y 轴的正半轴上是存在点 P1(0,2),P2(0 , 1 2 ),使 △BCP 为等腰三角形。 2012 年中考数学精析系列——黄石卷 (本试卷满分 120 分,考试时间 120 分钟) 一、仔细选一选(本题有 10 个小题,每小题 3 分,共 30 分) 3. (2012 湖北黄石 3 分)已知反比例函数 by x  ( b 为常数),当 x 0 时, y 随 x 的增大而增大,则一次函数 y x b  的图像不经过第几象限【 】 A.一 B. 二 C. 三 D. 四 【答案】B。 【考点】一次函数图象与系数的关系,反比例函数的性质。 【分析】∵反比例函数 by x  (b 为常数),当 x>0 时,y 随 x 的增大而增大,∴b< 0。 ∵一次函数 y=x+b 中 k=1>0,b<0,∴此函数的图象经过一、三、四限。 ∴此函数的图象不经过第二象限。故选 B。 4. (2012 湖北黄石 3 分)2012 年 5 月某日我国部分城市的最高气温统计如下表所 示: 城 市 武汉 成都 北京 上海 海南 南京 拉萨 深圳 气温 (℃) 27 27 24 25 28 28 23 26 请问这组数据的平均数是【 】 A.24 B.25 C.26 D.27 【答案】C。 【考点】算术平均数。 【分析】根据算术平均数的求法,求这组数据的算术平均数,用 8 个城市的温度和÷8 即可: (27+27+24+25+28+28+23+26)÷8=208÷8=26(℃)。故选 C。 5. (2012 湖北黄石 3 分)如图所示,该几何体的主视图应为【 】 【答案】C。 【考点】简单组合体的三视图。 【分析】几何体的主视图就是从正面看所得到的图形,从正面看可得到一个大矩形左 上边去掉一个小矩形的图形。故选 C。 6. (2012 湖北黄石 3 分)如图所示,扇形 AOB 的圆心角为 120°,半径为 2,则图中 阴影部分的面积为【 】 A. 4 33   B. 4 2 33   C. 4 3 3 2   D. 4 3  【答案】A。 【考点】扇形面积的计算,等腰三角形的性质,三角形内角和定理,垂径定理,勾股 定理。 【分析】过点 O 作 OD⊥AB, ∵∠AOB=120°,OA=2, ∴ 180 AOB 180 120OAD 302 2           。 ∴OD= 1 2 OA= 1 2 ×2=1, 2 2 2 2AD OA OD 2 1 3     。 ∴ AB 2AD 2 3  , ∴ 2 AOBOAB 120 2 1 4S S S 2 3 1 3360 2 3            扇形影阴 。故选 A。 7. (2012 湖北黄石 3 分)有一根长 40mm 的金属棒,欲将其截成 x 根7mm 长的小 段和 y 根9mm 长的小段,剩余部分作废料处理,若使废料最少,则正整数 x , y 应分别为【 】 A. x 1 , y 3 B. x 3 , y 2 C. x 4 , y 1 D. x 2 , y 3 【答案】B。 【考点】网格问题,一次函数的应用。 【分析】根据金属棒的长度是 40mm,则可以得到 7x+9y≤40,即 7 40y x+9 9   。 如图,在网格中作  7 40y= x+ x 0 y 09 9 > > , 。 则当线段 AB 上有整数点时,是废料为 0,该点即为所求。但从 图中可见,线段 AB 上没有整数点,故在△ABC 区域内离线段 AB 最 近的整数点即为所求,图中可见,点(3,2)离线段 AB 最近。 ∴使废料最少的正整数 x,y 分别为 x=3,y=2。 故选 B。 别解:∵ 7 40y x+9 9   且 x 为正整数,∴x 的值可以是: 1 或 2 或 3 或 4。 当 y 的值最大时,废料最少, ∴当 x=1 时, 33y 9  ,则 y 最大 4,此时,所剩的废料是:40-1×7- 3×9=6mm ; 当 x=2 时, 26y 9  ,则 y 最大 2,此时,所剩的废料是:40-2×7- 2×9=8mm; 当 x=3 时, 19y 9  ,则 y 最大 2,此时,所剩的废料是:40-3×7- 2×9=1mm; 当 x=4 时, 12y 9  ,则 y 最大 1,此时,所剩的废料是:40-4×7- 1×9=3mm。 ∴使废料最少的正整数 x,y 分别为 x=3,y=2。 8. (2012 湖北黄石 3 分)如图所示,矩形纸片 ABCD 中,AB=6cm,BC=8 cm,现 将其沿 EF 对折,使得点 C 与点 A 重合,则 AF 长为【 】 A. 25 cm8 B. 25 cm4 C. 25 cm2 D. 8cm 【答案】B。 【考点】翻折变换(折叠问题),折叠对称的性质,矩形的性质,勾股定理。 【分析】设 AF=xcm,则 DF=(8-x)cm, ∵矩形纸片 ABCD 中,AB=6cm,BC=8cm,现将其沿 EF 对折,使得点 C 与 点 A 重合, ∴DF=D′F, 在 Rt△AD′F 中,∵AF2=AD′2+D′F2,即 x2=62+(8-x)2,解得: x=  25 cm4 。故选 B。 9. (2012 湖北黄石 3 分)如图所示,直线 CD 与线段 AB 为直径的圆相切于点 D,并 交 BA 的延长线于点 C,且 AB=2,AD=1,P 点在切线 CD 上移动.当∠APB 的度数 最大时,则∠ABP 的度数为【 】 A. 15° B. 30° C. 60 ° D. 90° 【答案】 【考点】切线的性质,三角形的外角性质,圆周角定理, 锐角三角函数定义,特殊角的三角函数值。 【分析】连接 BD, ∵直线 CD 与以线段 AB 为直径的圆相切于点 D,∴∠ADB=90°。 ∵当∠APB 的度数最大时,点 P 和 D 重合,∴∠APB=90°。 ∵AB=2,AD=1,∴ AD 1sin DBP =AB 2   。∴∠ABP=30°。 ∴当∠APB 的度数最大时,∠ABP 的度数为 30°。故选 B。 10. (2012 湖北黄石 3 分)如图所示,已知 A 1 1( , y )2 ,B 2(2, y ) 为反比例函数 1y x  图像上的两点,动 点 P(x,0) 在 x 正半轴上运动,当线段 AP 与线段 BP 之差达到最大时,点 P 的坐标是 【 】 A. 1( ,0)2 B. (1,0) C. 3( ,0)2 D. 5( ,0)2 【答案】D。 【考点】反比例函数综合题,待定系数法,曲线上点的坐标与方程的关系,三角形三 边关系。 【分析】∵把 A 1 1( , y )2 ,B 2(2, y ) 分别代入反比例函数 1y x  得:y1=2,y2= 1 2 , ∴A( 1 2 ,2),B(2, 1 2 )。 ∵在△ABP 中,由三角形的三边关系定理得:|AP-BP|< AB, ∴延长 AB 交 x 轴于 P′,当 P 在 P′点时,PA-PB=AB, 即此时线段 AP 与线段 BP 之差达到最大。 设直线 AB 的解析式是 y=kx+b,把 A、B 的坐标代入得: 12= k+b2 1 =2k+b2     ,解得: k= 1 5b= 2   。∴直线 AB 的解析式是 5y x 2    。 当 y=0 时,x= 5 2 ,即 P( 5 2 ,0)。故选 D。 二、认真填一填(本题有 6 个小题,每小题 3 分,共 18 分) 11. (2012 湖北黄石 3 分)分解因式: 2x x 2  = ▲ . 【答案】(x-1)(x+2)。 【考点】十字相乘法因式分解。 【分析】因为(-1)×2=-2,2-1=1,所以利用十字相乘法分解因式即可:x2+x -2=(x-1)(x+2)。 12. (2012 湖北黄石 3 分)若关于 x 的不等式组2x 3x 3 3x a 5     有实数解,则 a 的取值 范围是 ▲ . 【答案】a<4。 【考点】解一元一次不等式组 【分析】分别求出各不等式的解集,再根据不等式组有实数解(同大取大,同小取小,大 小小大中间找,大大小小解不了(无解))即可得到关于 a 的不等式,求出 a 的取值范围 即可: 由 2x>3x-3 得,x<3,由 3x-a>5 得,x> 5 a 3  , ∵此不等式组有实数解,∴ 5 a 3  <3,解得 a<4。 13. (2012 湖北黄石 3 分)某校从参加计算机测试的学生中抽取了 60 名学生的成绩 (40~100 分)进行分析,并将其分成了六段后绘制成如图所示的频数分布直方图 (其中 70~80 段因故看不清),若 60 分以上(含 60 分)为及格,试根据图中信 息来估计这次测试的及格率约为 ▲ . 【答案】75%。 【考点】频数(率)分布直方图,用样本估计总体。 【分析】∵   距距 频数频数 组组 ,∴当 40≤x<50 时,频数=0.6×10=6;当 50≤x<60 时,频数=9; 当 60≤x<70 时,频数=9;当 80≤x<90 时,频数=15;当 90≤x<100 时,频数=3, ∴当 70≤x<80 时,频数=60-6-9-9-15-3=18, ∴这次测试的及格率= 9 18 15 3 100% 75%60      。 14. (2012 湖北黄石 3 分)将下列正确的命题的序号填在横线上 ▲ . ①若 n 大于 2 的正整数,则 n 边形的所有外角之和为 0( 2)n 180 . ②三角形三条中线的交点就是三角形的重心. ③证明两三角形全等的方法有:SSS,SAS,ASA,SSA 及 HL 等. 【答案】②。 【考点】命题与定理,多边形内角和外角性质,三角形的重心,全等三角形的判定。 【分析】分别根据多边形内角和定理、三角形的重心及全等三角形的判定定理得出结 论: 27. 若 n 为大于 2 的正整数,则 n 边形的所有内角之和为(n-2)•180°,所 有外角之和为 3600, 故本命题错误; ② 三角形三条中线的交点就是三角形的重心,符合重心的定义,故本命题正 确; ③ SSA 不能证明两三角形全等,故本命题错误。 15. (2012 湖北黄石 3 分)“数学王子”高斯从小就善于观察和思考.在他读小学时候就 能在课堂上快速 的计算出1 2 3 98 99 100 5050      ,今天我们可以将高斯的做法归纳如 下: 令 1 2 3 98 99 10S 0      ① 100 99 98 3 2 1S       ② ①+②:有 2 (1 100) 100S    解得:S 5050 请类比以上做法,回答下列问题: 若 n 为正整数,3 5 7 (2 1 8n ) 16     ,则 n  ▲ . 【答案】12。 【考点】分类归纳(数学的变化类),有理数的混合运算,解一元二次方程。 【分析】根据题目提供的信息,找出规律,列出方程求解即可: 设 S=3+5+7+…+(2n+1)=168①, 则 S=(2n+1)+…+7+5+3=168②, ①+②得,2S=n(2n+1+3)=2×168, 整理得,n2+2n-168=0,解得 n1=12,n2=-14(舍去)。 ∴n=12。 16. (2012 湖北黄石 3 分)如图所示,已知 A 点从点(1,0)出发,以每秒1个单 位长的速度沿着 x 轴的正方向运动,经过 t 秒后,以 O、A 为顶点作菱形 OABC, 使 B、C 点都在第一象限内,且∠AOC=600,又以 P(0,4)为圆心,PC 为半径 的圆恰好与 OA 所在直线相切,则 t= ▲ . 【答案】 4 3 1 。 【考点】切线的性质,坐标与图形性质,菱形的性质,锐角三角函数定义,特殊角的 三角函数值。 【分析】∵已知 A 点从(1,0)点出发,以每秒 1 个单位长的速度沿着 x 轴的正方向 运动, ∴经过 t 秒后,∴OA=1+t。, ∵四边形 OABC 是菱形,∴OC=1+t。, 当⊙P 与 OA,即与 x 轴相切时,如图所示,则切点为 O,此时 PC=OP。 过点 P 作 PE⊥OC,垂足为点 E。 ∴OE=CE= 1 2 OC,即 OE= 1 2 (1+t)。 在 Rt△OPE 中,OP=4,∠OPE=900- ∠AOC=30°, ∴OE=OP•cos30°= 2 3 ,即 11 t 2 32   。 ∴ t 4 3 1  。 ∴当 PC 为半径的圆恰好与 OA 所在直线相切时, t 4 3 1  。 三、全面答一答(本题有 9 个小题,共 72 分) 17. (2012 湖北黄石 7 分)计算: 0 0( 3 2) 4sin 60 2 2 3    【答案】解:原式 1 2 3 2 3 2 3     。 【考点】实数的运算,零指数幂,特殊角的三角函数值,绝对值。 【分析】针对零指数幂,特殊角的三角函数值,绝对值 3 个考点分别进行计算,然后根据 实数的运算法则求得计算结果。 18. (2012 湖北黄石 7 分)先化简,后计算: 2 2 81 a 9 a 1 a 6a 9 2a 6 a 9       ,其中 a 3 3  . 【答案】解:原式= 2 (9 )(9 ) 2( 3) 1 2=9 9 3( 3) a a a a a aa       。 当a 3 3  时,原式= 2 2 2 3= = 33 3+3 3 。 【考点】分式的化简求值,二次根式化简。 【分析】根据分式混合运算的法则把原式进行化简,再把 a 的值代入进行二次根式化 简即可。 19. (2012 湖北黄石 7 分)如图,已知在平行四边形 ABCD 中,BE=DF. 求证:∠DAE=∠BCF. 【答案】证明:∵四边形 ABCD 为平行四边形, ∴AD∥BC,且 AD=BC。 ∴∠ADE=∠BCF。 又∵BE=DF, ∴BF=DE。 ∴△ADE≌△CBF(SAS)。∴∠DAE=∠BCF 。 【考点】平行四边形的性质,平行线的性质,全等三角形的判定和性质。 【分析】根据平行四边形性质求出 AD∥BC,且 AD=BC,推出∠ADE=∠CBF,求出 DE=BF,由 SAS 证△ADE≌△CBF,推出∠DAE=∠BCF 即可。 20. (2012 湖北黄石 8 分)解方程组: 2 2 2x y 2 yx 14      【答案】解:依题意: 2 2 y 2(x 1) y 4x 4    - - ① ② 将①代入②中化简得:x2+2x-3=0 ,解得:x=-3 或 x=1。 当 x=-3 时, y 2( 3 1)= 3 2 - - - ;当 x=1 时,y=0。 ∴原方程组的解为: x 3 y 4 2   - - 或 x 1 y 0    。 【考点】解高次方程组,因式分解法一元二次方程。 【分析】把方程①变形成 y 2x 2  ,代入方程②,即可消去 y,得到关于 x 的方 程,解得 x 的值,从而求得 y 的值。 21. (2012 湖北黄石 8 分)已知甲同学手中藏有三张分别标有数字 1 2 , 1 4 ,1的卡 片,乙同学手中藏有三张分别标有数字1,3,2 的卡片,卡片外形相同.现从甲乙 两人手中各任取一张卡片,并将它们的数字分别记为 a,b. (1)请你用树形图或列表法列出所有可能的结果. (2)现制定这样一个游戏规则:若所选出的 a,b 能使得 2ax bx 1 0   有两个不 相等的实数根,则甲获胜;否则乙获胜.请问这样的游戏规则公平吗?请你用 概率知识解释。 【答案】解:(1)画树状图如下: (a,b)的可能结果有( 1 2 ,1)、( 1 2 ,2)、( 1 2 ,3)、( 1 ,14 )、 ( 1 24 ,)、( 1 ,34 )、(1,1)、(1,2)及(1,3), ∴(a,b)取值结果共有 9 种 。 (2)∵Δ=b2-4a 与对应(1)中的结果为:-1、2、7、0、3、8、-3、 0、5 ∴P(甲获胜)= P(Δ>0)= 5 9 , P(乙获胜) = 5 41 =9 9  。 ∴P(甲获胜)>P(乙获胜) 。∴这样的游戏规则对甲有利,不公平。 【考点】列表法或树状图法,概率,游戏公平性,一元二次方程根根的判别式。 【分析】(1)根据题意画出树状图或列表,然后根据图或表即可求得所有等可能的结 果。 (2)判断游戏公平性就要计算每个事件的概率,概率相等就公平,否则就不 公平。因此,利用一元二次方程根的判别式,即可判定各种情况下根的情况,然后利 用概率公式求解即可求得甲、乙获胜的概率,比较概率大小,即可确定这样的游戏规 是否公平。 22. (2012 湖北黄石 8 分)如图所示(左图为实景侧视图,右图为安装示意图),在 屋顶的斜坡面上安装太阳能热水器:先安装支架 AB 和 CD(均与水平面垂直), 再将集热板安装在 AD 上.为使集热板吸热率更高,公司规定:AD 与水平面夹角为 θ1,且在水平线上的射影 AF 为 1.4m.现已测量出屋顶斜面与水平面夹角为θ2,并 已知 1tan 1.082  , 2tan 0.412  。如果安装工人确定支架 AB 高为 25cm,求支 架 CD 的高(结果精确到 1cm)。 【答案】解:如图所示,过点 A 作 AE∥BC,则 2EAF CBG     ,且 AF AB 25cm  。 在 Rt△ADF 中: 1DF AF tan  ,在 Rt△EAF 中, 2EF AF tan  , ∴ 1 2DE DF EF AF(tan tan )     。 又∵ AF 140cm , tan 1 1.082  , 2tan 0.412  , ∴  DE 140(1.082 0.412) 93.8 cm   。 ∴CD DE CE 93.8 25 118.8 119(cm)      。 答:支架 CD 的高约为 119cm 。 【考点】解直角三角形的应用,锐角三角函数定义。 【分析】过 A 作 AE∥BC,则∠EAF=∠CBG=θ2,EC=AB=25cm,再根据锐角三角函 数的定义用θ1、θ2 表示出 DF、EF 的值,再根据 DC=DE+EC 进行解答即可。 23. (2012 湖北黄石 8 分)某楼盘一楼是车库(暂不销售),二楼至二十三楼均为商 品房(对外销售).商品房售价方案如下:第八层售价为 3000 元/米 2,从第八层起 每上升一层,每平方米的售价增加 40 元;反之,楼层每下降一层,每平方米的售 价减少 20 元.已知商品房每套面积均为 120 平方米.开发商为购买者制定了两种购 房方案: 方案一:购买者先交纳首付金额(商品房总价的 30%),再办理分期付款(即贷 款). 方案二:购买者若一次付清所有房款,则享受 8%的优惠,并免收五年物业管理费 (已知每月物业管理费为 a 元) (1)请写出每平方米售价 y(元/米 2)与楼层 x(2≤x≤23,x 是正整数)之间的函数 解析式; (2)小张已筹到 120000 元,若用方案一购房,他可以购买哪些楼层的商品房呢? (3)有人建议老王使用方案二购买第十六层,但他认为此方案还不如不免收物业管理 费而直接享受 9%的优惠划算.你认为老王的说法一定正确吗?请用具体的数据阐明你 的看法。 【答案】解:(1)当 2≤x≤8 时,每平方米的售价应为:3000-(8-x)×20=20x+ 2840 ; 当 9≤x≤23 时,每平方米的售价应为:3000+(x-8)·40=40x+ 2680。 ∴ 20x 2840(2 x 8,x )y 40x 2680(8 x 23,x )        为正整数 为正整数 。 (2)由(1)知: ∵当 2≤x≤8 时,小张首付款为 (20x+2840)·120·30%=36(20x+2840)≤36(20·8+2840) =108000 元<120000 元 ∴2~8 层可任选。 ∵当 9≤x≤23 时,小张首付款为(40x+2680)·120·30%=36 (40x+2680)元 由 36(40x+2680)≤120000,解得:x≤ 116 3 。 ∵x 为正整数,∴9≤x≤16。 综上所述,小张用方案一可以购买二至十六层的任何一层。 (3)若按方案二购买第十六层,则老王要实交房款为: y1=(40·16+2680) ·120·92%-60a(元) 若按老王的想法则要交房款为:y2=(40·16+2680) ·120·91% (元) ∵y1-y2=3984-60a , 当 y1>y2 即 y1-y2>0 时,解得 0<a<66.4。此时老王想法正确; 当 y1≤y2 即 y1-y2≤0 时,解得 a≥66.4。此时老王想法不正确。 【考点】一次函数和一元一次不等式的应用。 【分析】(1)根据题意分别求出当 2≤x≤8 时,每平方米的售价应为 3000-(8- x)×20 元,当 9≤x≤23 时,每平方米的售价应为 3000+(x-8)•40 元。 (2)由(1)知:当 2≤x≤8 时,小张首付款为 108000 元<120000 元,即 可得出 2~8 层可任选, 当 9≤x≤23 时,小张首付款为 36(40x+2680)≤120000,9≤x≤16,即可得出小张用 方案一可以购买二至十六层的任何一层。 (3)分别求出若按方案二购买第十六层,则老王要实交房款为 y1 按老王的想 法则要交房款为 y2,然后根据即 y1-y2>0 时,解得 0<a<66.4,y1-y2≤0 时,解得 a≥66.4,即可得出答案。 24. (2012 湖北黄石 9 分)如图 1 所示:等边△ABC 中,线段 AD 为其内角平分线, 过 D 点的直线 B1C1⊥AC 于 C1 交 AB 的延长线于 B1. (1)请你探究: AC CD AB DB  , 1 1 1 1 AC C D AB DB  是否成立? (2)请你继续探究:若△ABC 为任意三角形,线段 AD 为其内角平分线,请问 AC CD AB DB  一定成立吗?并证明你的判断. (3)如图 2 所示 Rt△ABC 中,∠ACB=900,AC=8, AB 40 3  ,E 为 AB 上一点且 AE=5,CE 交其内角角平分线 AD 与 F.试求 DF FA 的值. 【答案】解:(1)∵线段 AD 为等边△ABC 内角平分线,∴根据三线合一,得 CD=DB。 ∴ AC CD1AB DB   。 过点 D 作 DN⊥AB 于点 H。 ∵线段 AD 为等边△ABC 内角平分线,∴C1D=ND。 ∵等边△ABC 中,B1C1⊥AC,∴∠B1=300。 ∴ 1 1 1 1 AC C D1 AB 2 DB   。 ∴ AC CD AB DB  , 1 1 1 1 AC C D AB DB  都成立。 (2)结论仍然成立。证明如下: 如图,ΔABC 为任意三角形,过 B 点作 BE∥AC 交 AD 的延长线于点 G 。 ∵∠G=∠CAD=∠BAD, ∴BG=AB。 又ΔGBD∽ΔACD , ∴ AC CD BG DB  ,即 AC CD AB DB  。 ∴ AC CD AB DB  对任意三角形结论仍然成立。 ﹙3﹚如图,连接 ED。 ∵AD 为ΔABC 的内角角平分线, AC=8, 40AB 3  , ∴由(2)得, CD AC 8 3 40DB AB 5 3    。 又∵AE=5,∴EB=AB-AE= 40 2553 3   。∴ AE 5 3 25EB 5 3   。 ∴ CD AE DB EB  。∴DE∥AC。 ∴ΔDEF∽ΔACF。 ∴ DF EF AE 5 FA FC AC 8    。 25. (2012 湖北黄石 10 分)已知抛物线 C1 的函数解析式为 2y ax bx 3a(b 0)    ,若抛物线 C1 经过点(0, 3) ,方程 2ax bx 3a 0   的两 根为 1x , 2x ,且 1 2x x 4  。 (1)求抛物线 C1 的顶点坐标. (2)已知实数 x 0 ,请证明: 1x x  ≥ 2 ,并说明 x 为何值时才会有 1x 2x   . (3)若抛物线先向上平移 4 个单位,再向左平移 1 个单位后得到抛物线 C2,设 1A(m, y ) , 2B(n, y ) 是 C2 上的两个不同点,且满足: 00AOB 9  , m 0 , n 0 .请你用含有 m 的表达式表示出△AOB 的面积 S,并求出 S 的最小值及 S 取最 小值时一次函数 OA 的函数解析式。 (参考公式:在平面直角坐标系中,若 1 1P(x , y ) , 2 2Q(x , y ) ,则 P,Q 两点间的距离 2 2 2 1 2 1(x x ) (y y )   ) 【答案】解:(1)∵抛物线过(0,-3)点,∴-3a=-3。∴a=1 。 ∴y=x2+bx-3 ∵x2+bx-3=0的两根为 x1,x2 且 1 2x x 4  , ∴ 2 2 1 2 1 2 1 2x x (x x ) 4x x = b +12    =4且 b<0。∴b=- 2。 ∴  22x x x   y= 2 3= 1 4。 ∴抛物线C1的顶点坐标为(1,-4)。 【考点】二次函数综合题,曲线上点的坐标与方程的关系,一元二次方程根与系数的 关系,二次函数的性质,不等式的知识。 【分析】(1)求抛物线的顶点坐标,即要先求出抛物线的解析式,即确定待定系数 a、b 的值.已知抛物线图象与 y 轴交点,可确定解析式中的常数项(由此得到 a 的 值);然后从方程入手求 b 的值,题目给出了两根差的绝对值,将其进行适当变形 (转化为两根和、两根积的形式),结合根与系数的关系即可求出 b 的值。 (2)将 1x x  配成完全平方式,然后根据平方的非负性即可得证。 (3)结合(1)的抛物线的解析式以及函数的平移规律,可得出抛物线 C2 的 解析式;在 Rt△OAB 中,由勾股定理可确定 m、n 的关系 式,然后用 m 列出△AOB 的面积表达式,结合不等式的相关知识可确定△OAB 的最小 面积值以及此时 m 的值,从而由待定系数法确定一次函数 OA 的解析式。 别解:由题意可求抛物线 C2 的解析式为:y=x2。 ∴A(m,m2),B(n,n2)。 过点 A、B 作 x 轴的垂线,垂足分别为 C、D, 则 AOC BODACDBS S S S   梯形 2 2 2 21 1 1(m n )(m n) m m n n2 2 21 mn(m n)2           由 BOD△ ∽ OAC△ 得 BD OD OC AC  ,即 2 2 n n m m  。 ∴ mn 1  。 ∴ 1 1 1 1S mn(m n)= m+ 2 12 2 m 2          。 ∴SΔAOB 的最小值为1,此时 m=1,A(1,1)。 ∴直线 OA 的一次函数解析式为y=x。
查看更多

相关文章

您可能关注的文档